You are on page 1of 372

Function 1

1
FUNCTION

Exercise-1 : Single Choice Problems

1. f( x) = log 2 (2 - 2 log 2
(16 sin 2 x + 1))

0 £ log 2
(16 sin 2 x + 1) £ log 2 17 Þ 2 - 2 log 2 17 £ 2 - 2 log 2
(16 sin 2 x + 1) £ 2

Þ 0 < 2 - 2 log 2
(16 sin 2 x + 1) £ 2 Þ f( x) £ 1

2. For any b Î R e| x -b| is


e|x–b|
|||||
||||| ||||||| |e|x–b|–3|
|||
|||

||||
|||||

||||

1
|

b
min
–2
|e|x -b| - a| has four distinct solutions a > 3 so a Î (3 , ¥)
3. Domain = [-1, 1] and both are increasing functions.
\ x = -1, we get minimum value & x = 1, we get maximum value.
é p p p pù é p pù
êë - 4 - 4 , 4 + 4 úû = êë - 2 , 2 úû
2 2 2 2 2
4. æç 2 2 x + 2 y - 2 2 x + 2 y ö÷ = 1 - 2 2 x + 2 y + 2 x + 2 y +1 ³ 0
è ø
2 2
æ 1ö æ 1ö
Þ çx + ÷ +çy + ÷ £0
è 2ø è 2ø
2 Solution of Advanced Problems in Mathematics for JEE

5. O
x

(0,–a)

æ 5 2 ö
7. sec -1 ç - + ÷ x2 + 2 ³ 2
ç 2 2( x 2 + 2) ÷
è ø
æ 5 1 ö æ 1 1ö
= sec -1 ç - + ÷ ç 2 £ ÷
ç 2 ( x 2 + 2) ÷ è x +2 2 ø
è ø
æ 5 1 5 1 ö
£ sec -1 ( -2) = p - sec -1 (2) ç- + 2 £ - + = -2 ÷
è 2 x +2 2 2 ø
2p
=
3
8. f ¢( x) = x 2 + ax + b is injective if D £ 0
a 2 - 4b £ 0
If a = 1, b = 1, 2 , 3 , 4 , 5 Number of pair = 5
a = 2 , b = 1, 2 , 3 , 4 , 5 Number of pair = 5
a = 3, b = 3, 4, 5 Number of pair = 3
a = 4, b = 4, 5 Number of pair = 2
a =5 b has no value
9. f( x) = log x [ x] Þ f( x) Î [0 , 1]
g( x) =|sin x| + |cos x|
Þ g( x) Î [1, 2 ]
10. f( x) = 2 x 3 - 3 x 2 + 6
f ¢( x) = 6 x 2 - 6 x ³ 0 Þ x Î [1, ¥)
and f( x) Î [5, ¥)
11. 0 £ { x} < 1
{ x}({ x} - 1)({ x} + 2) ³ 0
Þ { x} = 0 Þ x Î z
14. 1 + sin 2 x Î [1, 2]
1 é1 ù
Î ê , 1ú
2
1 + sin x ë 2 û
æ 1 ö ép pù
sin -1 ç ÷Îê , ú
2
è 1 + sin x ø ë 6 2 û
Function 3

Kp é p p ù
Îê , ú K Î[1, 3]
6 ë6 2 û
15. f( x - y) = f( x) f( y) - f( a - x) f( a + y)
Put x = y = 0
f(0) = [ f(0)]2 - f( a) f( a)
Þ f( a) = 0 [Q f(0) = 1]
Put x = a and y = x
f( a - x) = f( a) f( x) - f(0) f( a + x)
Þ - f ( a - x) = f ( a + x) Þ f (2 a - x) = - f ( x)
18. f( x) = 4 x - x 2 = y
x2 - 4x + y = 0
f -1 ( x) = 2 - 4 - x
19. [5 sin x] + [cos x] = -6
Þ -1 £ cos x < 0 and -5 £ 5 sin x < -4
4
-1 £ sin x < -
5
20. f( x) = ax + cos x
f ¢( x) = a - sin x
if f( x) is invertible, then
f ¢( x) ³ 0 or f ¢( x) £ 0
Þ a ³ 1 or a £ -1
é xù é xù é xù
21. f( x) = [1 + sin x] + ê2 + sin ú + ê3 + sin ú +¼ + ên + sin ú
ë 2û ë 3û ë nû
é xù é xù é xù
= (1 + 2 + 3 +¼ + n) + [sin x] + êsin ú + êsin ú +¼ + êsin ú
ë 2 û ë 3 û ë n û
x 2 + ax + 1
22. y =
x2 + x + 1
( y - 1) x 2 + ( y - a) x + ( y - 1) = 0
D³0
( y - a) 2 - 4( y - 1) 2 ³ 0
-3 y 2 + y(8 - 2 a) + a 2 - 4 ³ 0 " y Î R
Not possible
23. f( x) = [ x] + [- x]
4 Solution of Advanced Problems in Mathematics for JEE

f( x) = ìí 0 x Î I
î-1 x Ï I
g( x) = { x}
h( x) = f[ g( x)] = f({ x}) { x} = 0 xÎI
{ x} = { x} xÏI
f ( 0) xÎI
h( x) = ìí Þ h( x) = ìí 0 xÎI
î f({ x}) xÏI î-1 xÏI
Hence, the option (b).
é x ù é 15 ù
24. f( x) = ê ú ê - ú x Î(0 , 90)
ë 15 û ë x û
0 £ x < 15 f( x) = 0
15 £ x < 30 f( x) = -1
30 £ x < 45 f( x) = -2
45 £ x < 60 f( x) = -3
60 £ x < 75 f( x) = -4
75 £ x < 90 f( x) = -5
Total integers in range f( x) = {0 , - 1, - 2 , - 3 , - 4 , - 5}
1
25. g( x) =
f(| x|)
g( x) Þ even functions Þ symmetric about y-axis
Þ x ® ¥ f( x) ® 0
at x = x 1 f( x) = 0 Þ g( x 1 ) ® ¥
26. Homogeneous function Þ f(tx , ty) = t n f( x , y)
2x + 3 x £ 1
27. f( x) = é 2
êë a x + 1 x > 1
For x £ 1 f( x) £ 5
So for range of f( x) to be R.
Þ a 2 + 1 £ 5 and a ¹ 0
Þ a Î [-2 , 2]
Hence, a = {-2 , - 1, 1, 2}
28. log 1/ 3 (log 4 ( x - 5)) > 0
0 < log 4 ( x - 5) < 1
1< x - 5 < 4
6< x<9
æ 4 ö
29. f( x) = log 2 ç ÷;-2 £ x £ 2
è 2+x + 2-xø
Function 5

2+x + 2-x =y

4 + 2 4 - x2 = y2
y Î[2 , 2 2 ]
é 4 4ù
Range f( x) = êlog 2 , log 2 ú
ë 2 2 2û
é1 ù
f( x) lies between ê , 1ú
ë2 û
30. | x 2 + 5 x| + | x - x 2| =|6 x| Þ | x 2 + 5 x| + | x - x 2| =|( x 2 + 5 x) + ( x - x 2 )|
|a| + |b| =|a + b| Þ ab ³ 0
( x 2 + 5 x)( x - x 2 ) ³ 0
x( x + 5) × x( x - 1) £ 0 Þ -5 £ x £ 1
æ 1ö æ 1ö
31. f( x) + f ç ÷ = f( x) f ç ÷
x
è ø è xø
Þ f( x) = 1 ± x n
f(2) = 33 Þ n = 5
Hence, f( x) = 1 + x 5
Here, f ( x ) + f ( - x ) ¹ 0.
Hence not an odd function.
sin x + sin 7 x 2 sin 4 x cos 3 x
32. g( x) = + |sin x| = + |sin x|
cos x + cos 7 x 2 cos 4 x cos 3 x
= tan 4 x + |sin x|
g( x) period = p
éx -1
ê x = odd
33. f( x) = ê 2 f( x) : N ® Z
x
ê - x = even
ë 2
Let x = odd = (2n + 1); n > 0
2n + 1 - 1
f( x) = = n Þ +ve integer
2
Let x = even = 2m ; m > 0
2m
f( x) = - = -m Þ –ve integer
2
Þ Range = codomains Þ onto and clearly f( x) is one-one function.
Hence, bijective.
2 x + 1 - 2 1- x 2 2 x +1 - 2
34. y = =
2 x + 2 -x 2 2x + 1
6 Solution of Advanced Problems in Mathematics for JEE

y(2 2 x + 1) = (2 2 x - 1) 2
2 2x × y + y = 2 2x × 2 - 2
2 2 x ( y - 2) = - ( 2 + y )
( y + 2)
2 2x =
(2 - y)
(2 + y)
2 x = log 2
(2 - y)
1 (2 + y)
x= log 2
2 (2 - y)
1 (2 + x)
f -1 ( x) = log 2
2 (2 - x)
35. | y| = x (Q x > 0)
2
Þ | y| = x
Þ y = x2 y³0
2
y = -x y<0
36. f( x) = log [ x] (9 - x 2 )

Domains = 9 - x2 > 0 ü
Þ x Î[2 , 3) Þ f( x) = log 2 (9 - x 2 )
[ x] > 0 and [ x] ¹ 1ýþ
Range = ( -¥, log 2 5]
37. Gives e x + e f( x) = e
e f( x) = e - e x
Þ f( x) = log e ( e - e x )
Domain e - e x > 0
x < 1 Þ x Î ( -¥, 1)
Range ( -¥, 1)
38. Gives y + | y| = x + | x|
If x > 0, y > 0 Þ 2 y = 2 x Þ y = x
x < 0, y > 0 Þ 2 y = 0 Þ y = 0
x > 0, y < 0 Þ 0 = 2 x Þ x = 0
x < 0, y < 0 Þ 0 = 0 Þ whole region of III quadrant.
For person to be safe there should not be point common to the given curves and the voltage
field graph. Only y = m +| x| does not have any point of intersection with the curve.
39. Gives | f( x) + 6 - x 2| =| f( x)| + |4 - x 2| + 2
Þ | f( x) + 2 + ( 4 - x 2 )| =| f( x)| + |4 - x 2| + 2
Function 7

Since |a + b + c| =|a| + |b| + |c|


If a ³ 0 , b ³ 0 , c ³ 0 or a £ 0 , b £ 0 , c £ 0
Þ f( x) ³ 0 and 4 - x 2 ³ 0 Þ -2 £ x £ 2 and f( x) ³ 0
40. f( x) = cos px + sin x
æ 2p 2p ö
Period : L.C.M. of çç , ÷÷
è p 1 ø
For period to exist p should be a rational number.
41. y = f( e x ) + f(ln| x|)
Domain f( x) = (0 , 1)
Þ 0 < ex < 1 Þ x<0 …(1)
and 0 < ln| x|< 1 Þ 1 < | x| < e Þ x Î ( -e, - 1) È (1, e) …(2)
Taking intersection x Î ( -e, - 1)
42. Givens f(1) = 2 , f(2) = 3 , f(3) = 4 , f( 4) = 1, g(1) = 3 and f [ g( x)] = g [ f( x)]
at x = 1 f [ g(1)] = g [ f(1)] Þ f(3) = g(2) Þ g(2) = 4
at x = 2 f [ g(2)] = g [ f(2)] Þ f( 4) = g(3) Þ g(3) = 1
at x = 3 f [ g(3)] = g [ f(3)] Þ f(1) = g( 4) Þ g( 4) = 2
43. Gives [ y + [ y]] = 2 cos x Þ [ y] + [ y] = 2 cos x Þ 2 [ y] = 2 cos x; [ y] = cos x …(1)
1
where y = [sin x + [sin x + [sin x]]]
3
1
y = [sin x + [sin x] + [sin x]]
3
1
y = (3 [sin x])
3
y = [sin x] …(2)
From eqn. (1) & (2),
[sin x] = cos x
Þ cos x = 0 , 1, - 1
Hence, no solution.
é 1 -
1 ù
x 2n êex - e x ú
44. f( x) = ê ú x ¹ 0 and f(0) = 1
( x 2n sgn x) 2n +1 ê 1 -
1
ú
ëe + e x
x
û
é 1 -
1 ù
2n
(x ) êex - e x ú
when f( x) = ê 1 ú; x>0
( x 2n ) 2n +1 ê x -
1
ú
ëe + e x û
8 Solution of Advanced Problems in Mathematics for JEE

é 1 -
1 ù
2n
x êe - e x
x ú
f( x) = ê 1 ú; x<0
- ( x 2n ) 2n +1 ê x -
1
ú
e + e x
ë û
Clearly, f( x) = f( - x). Hence, f( x) is even function.
45. f(n) = 2( f(1) + f(2) ¼¼ + f(n - 1))
f(2) = 2 f(1)
é f ( 2) ù
f(3) = 2 [ f(1) + f(2)] = 2 ê + f ( 2) ú = 3 f ( 2)
ë 2 û
é f ( 3) ù
f( 4) = 2 [ f(1) + f(2) + f(3)] = 2 ê + f ( 3) ú = 3 f ( 3) = 3 2 f ( 2)
ë 2 û
M
M
m
å f( r) = f(1) + f(2)¼¼ + f(m) = f(1) + f(2) + 3 f(2)¼¼ +
r =1

= f(1) + f(2)[1 + 3 + 3 2 ¼¼ 3 m -2 ]
(3 m -1 - 1)
= f(1) + 2 × = 3 m -1
(3 - 1)
x
46. Gives f( x) =
1+ x2
x
f( f( x)) =
1 + 2x2
x
f( f( f( x))) =
1 + 3x2
M
M
x x
fofo¼¼ fof( x) = =
1442443
1 + nx 2 æ n ö
n times
1 + ç 1÷ x 2
å
ç ÷
è r =1 ø
47. f( x) = 2 x + |cos x|
Range f( x) = R = codomain Þ onto.
Clearly, f( x) is increasing function Þ one-one function.
48. Gives f( x) = x 3 + x 2 + 3 x + sin x
Since, f( x) is continuous function.
and f( x) = ¥ as x ® ¥
f( x) = -¥ as x ® -¥
Function 9

Range f( x) = R = codomains Þ onto function


2
æ 1ö 8
and f ¢( x) = 3 x 2 + 2 x + 3 + cos x = 3 ç x + ÷ + + cos x Þ f ¢( x) > 0
è 3ø 3
Hence, f( x) is one-one.
49. f( x) = { x} + { x + 1} +¼ { x + 99}
Since { x} = { x + I } where I = integer
f( x) = { x} + { x} ¼ { x}
1442443
100 times

f( x) = 100{ x} Þ f( 2 ) = 100{ 2 } = 100 ´ 0.414 = 41.4


[ f( 2 )] = 41
50. |cot x + cosec x| =|cot x| + |cosec x|; x Î [0 , 2p]
Þ cot x ³ 0 and cosec x ³ 0 Þ 1st quadrant
or cot x £ 0 and cosec x £ 0 Þ 4 th quadrant
æ p ù é3p ö
Hence, x Î ç 0, ú È ê , 2p ÷
è 2û ë 2 ø
51. If f( 4 + x) = f( 4 - x)
Þ f( x) is symmetric about x = 4.
Roots of f( x) = 0 are of the form
4 - a , 4 + a , 4 - b, 4 + b, 4 - g , 4 + g , 4 - d, 4 + d
52. f( x) + x - 6 = ( x - 1)( x - 2)( x - 3)( x - 4)( x - 5)
Þ f(6) = 120
53. f( x) = x - 2 + 4 - x
f(x)

2
2

x
1 2 3 4

éxù é x ù
54. ê ú = ê ú
ë 9 û ë 11û
x Î [1, 9) È [11, 18) È [22 , 27) È [33 , 36) È [44 , 45)
55. log é 1ù (2 x 2 + x - 1)
êx+ 2ú
ë û

é 1ù é 1ù 2
êë x + 2 úû > 0 , êë x + 2 úû ¹ 1 & 2 x + x - 1 > 0
10 Solution of Advanced Problems in Mathematics for JEE

1
Þ x+ ³ 2 & (2 x - 1)( x + 1) > 0
2
3 æ1 ö
x ³ & x( -¥, - 1) È ç , ¥ ÷
2 è2 ø
é3 ö
Þ x Î ê , ¥÷
ë2 ø
56. [ x 2 ] + [ x] - 2 = 0
Let [ x] = t
Þ t2 + t -2 =0
Þ (t + 2)(t - 1) = 0
Þ t = -2 or t = 1
Þ [ x] = -2 or [ x] = 1
Þ x Î [-2 , - 1) È [1, 2)
58. f( x) is many one function.
59. f( f( x)) = 2 + f( x) f( x) ³ 0
= 2 - f( x) f( x) < 0
f( f( x)) = 4 + x x³0
= 4 - ( x) x<0
2
7(3 x - 2 x + 3)
60. f ¢( x) = > 0 Þ f( x) ­
(3 + 3 x - 4 x 2 ) 2
y

61. O
x

62.

x
–2 a –1 3b4

[a ] = -2
[b] = 3
Function 11

63. f( x) = sin(log 7 (cos(sin x)))


cos(sin x) £ 1 Þ cos(sin x) = 1 Þ f( x) = 0
64. - 3 £|[ x]| £ 2 Þ - 2 £ [ x] £ 2 Þ -2 £ x < 3
p
65. f( x) = + cot -1 {- x}
2
p p
0 £ {- x} < 1 Þ < cot -1 {- x} £
4 2
66. f( f( x)) = x
3x + 5 2x2 + 5
f2008 ( x) + f2009 ( x) = x + f( x) = x + =
2x - 3 2x - 3
æ 1 öæ 1 ö 1 1
67. f( x) = ç x + + 1÷ ç x 2 + 2
÷ ; x + 2 ³ 2 ; x + + 1 ³ 3 Þ f( x) ³ 6
x 2 x
è øè x ø x
3
-3 x 2 -9 x + 2
68. f( x) = e x
3
-3 x 2 -9 x + 2)
f ¢( x) = e( x 3 ( x - 3)( x + 1)
Þ f( x) is many one.
at x = -1, f( x) = e7
at x ® -¥, f( x) ® 0
Range of f( x) is (0 , e7 ].
69. D f : ( -2 , 1)
æ 4 - x2 ö
-¥ < log ç ÷<¥
ç 1- x ÷
è ø
æ æ 4 - x2 öö
-1 £ sin ç log ç ÷÷ £ 1
ç ç 1- x ÷÷
è è øø
70. f ¢( x) ³ 0 " x Î R Þ 3 x 2 + 2 ( a + 2) x + 3 a ³ 0 " x Î R
Þ D£0
Þ 4 ( a + 2) 2 - 4 × 9 a £ 0
Þ a 2 - 5a + 4 £ 0 Þ ( a - 1)( a - 4) £ 0
Þ a Î[1, 4]
2
71. Min. value of 3 x + bx + c = 0
Þ D =0
p
72. f( x) = sin -1 x - cos -1 x = 2 sin -1 x -
2
12 Solution of Advanced Problems in Mathematics for JEE

3p/2

p/2

x
–1 1
73. is one-one when
2 3 = ln 1 + b 2 - 3 b + 10
8
Þ b2 - 3b + 2 = 0
Þ b = 1, 2 x=1
–1

80. We have, [ x]2 - 7 [ x] + 10 < 0


Þ ([ x] - 5)([ x] - 2) < 0
Þ 2 < [ x] < 5
Þ [ x] = 3 or 4
Þ x Î[3 , 5)
and 4[ y]2 - 16 [ y] + 7 < 0
(2 [ y] - 7)(2 [ y] - 1) < 0
1 7
Þ < [ y] <
2 2
Þ [ y] = 1 or 2 or 3
Þ y Î[1, 4)
Therefore, x + y Î[4 , 9)
[ x + y ] Î { 4 , 5, 6 , 7 , 8}
Hence, [ x + y] cannot be 9.
e| x| - e - x
81. f : R ® R f( x) = y
e x + e-x 1
ì e x - e-x
ï x if x ³ 0
ï -x
f( x) = í e- x + e - x x
ïe - e if x < 0
O
ïî e x + e - x
Many one into function.
Function 13

82. f( x) such f(1 - x) + 2 f( x) = 3 x " x Î R


æ1 ö
x ® ç + x÷
è 2 ø
æ1 ö æ1 ö æ1 ö
f ç - x÷ + 2 f ç + x÷ =3ç + x÷ …(1)
è2 ø è2 ø è2 ø
æ1 ö
x ® ç - x÷
è 2 ø
æ1 ö æ1 ö æ1 ö
f ç + x÷ + 2 f ç - x÷ =3ç - x÷ …(2)
è2 ø è2 ø è2 ø
(1) + (2)
æ æ1 ö æ1 öö æ1 ö æ1 ö
3ç f ç + x÷ + f ç - x÷÷ = 3 ; f ç - x÷ = 1 - f ç + x÷
è è 2 ø è 2 øø è 2 ø è 2 ø
æ1 ö 3 æ1 ö 1
1 + f ç + x÷ = + 3x ; f ç + x÷ = + 3x
è 2 ø 2 è 2 ø 2
1 1 3
x = - Þ f(0) = - = -1
2 2 2
83. f : [0 , 5] ® [0 , 5]
f( x) = ax 2 + bx + c a , b, c Î R , abc ¹ 0

or
0 5 0 5
c¹0

25a + 5b + c = 0
f(5) = 0
c
ax 2 + bx + c = 0 (a ) = 5 ´b
a
æ 1ö 1
cx 2 + bx + a = 0 ç ÷ b=
èaø a
æ 1ö
So, roots are ç a, ÷ .
è 5ø
84. f( x) = x 2 + lx + m cos x
f( x) = x
85. f( k) = odd
f( k + 1) = even k = 1, 2 , 3
14 Solution of Advanced Problems in Mathematics for JEE

f(1) Þ odd ü
f(2) Þ even ï
f(3) Þ odd ý Hence , 4 functions.
ï
f( 4) Þ even þ
f(1) Þ even ü
f(2) Þ odd ï
f(3) Þ even ý Hence , 4 functions.
ï
f( 4) Þ odd þ
f(1) Þ odd ü
f(2) Þ even ï
f(3) Þ even ý Hence , 4 functions.
ï
f( 4) Þ odd þ
86. y = tan(sin x).
Here function is continuous and differentiable and y max = tan(1); y min = - tan 1.
2x
87. f( x) = y
x -1 (0,2) (1,2)
2
y =2 + x
( x - 1) (1,0)

( y - 2)( x - 1) = 2
88. R f = [-2 , 4]
R g = [-1, 2]
1
89. f( x) = ( x 4 + 1) +
2
x + x +1
y
90. 0 £ f( x) £ 1
0 £ 7 f( x) £ 7 1

-1 £ sin(7 f( x)) £ 1 x
O 1 2 3

91. ln|ln| x|| ³ 0 Ç | x|2 - 7| x| + 10 £ 0


|ln| x|| ³ 1 (| x| - 2)(| x| - 5) £ 0
ln| x|Î ( -¥, - 1) È [1, ¥) 2 £ | x| £ 5
æ 1ù
| x|Î ç 0 , ú È [e, ¥) x Î ( -5, - 2] È [2 , 5]
è eû
é 1 ö æ 1ù
x Î ( -¥, - e] È ê - , 0 ÷ È ç 0 , ú È [e, ¥)
ë e ø è eû
Function 15

ææ 2
5ö 3ö
92. log [ x] + 3{ x} ç ç [ x] - ÷ + ÷ ³ 0 Þ [ x] + 3 { x} > 1
çè 2ø 4÷
è ø
93. x - 3 = X | X | + |Y | = 5
y -1=Y
x + y -4=X +Y |X + Y|= 5
number of pairs of ( x , y) = 12

(0, 5)

(5, 0)
(–5, 0) |x+y| = 5
|x| + |y| = 5

(0, –5)

f(1) = 1

95. f(2) = 3 f(2) = 4


f(4) = 2
f(3) = 4

f(3) = 2 f(3) = 3
f(4) = 3 f(4) = 2

96. x 2 - x ¹ 0 Þ x ¹ 0 , 1
97. Total one-one function – (at least one get right place) + (at least two get right place)
– (at least three get right place) + (at least four get right place)
= 6 C 4 ´ 4 ! - 4 C 1 ´ 5 C 3 ´ 3 ! + 4 C 2 ´ 4 C 2 ´ 2 ! - 4 C 3 ´ 3 C 1 + 4 C 4 = 181

98. f( x) = x 2 - 2 x - 3

g( x) = f -1 ( x) = 1 + x+4 x ³ -4

f( x) = g( x) = f -1 ( x) Þ f( x) = x
3 + 21
Þ x2 - 3x - 3 = 0 Þx=
2
16 Solution of Advanced Problems in Mathematics for JEE

Exercise-2 : One or More than One Answer is/are Correct

1. f( -4) = f( 4) = 40
f( -13) = f(13) = f(3) = 19
f( -11) = f(11) = f(1) = 2

2. 2

1
x
0 1 2 3 4

æ 1 - tan 2 ( x 2) ö
3. f( x) = cos -1 ç ÷ is defined when
ç 1 + tan 2 ( x 2) ÷
è ø
x p
¹ (2n - 1)
2 2
Þ x ¹ (2n - 1)p
Domain = R - {(2n - 1)p : n Î I }
\ Range = [0 , p)
f( x) = cos -1 (cos x)
f( x) is even function.
when x Î( p , 2 p), then f( x) = 2p - x is differentiable.

2
4.
–e2 –1 1 e2

0 < |k - 1| - 3 < 2
Þ k Î ( -4 , - 2) È ( 4 , 6)
5. (a) D f Î R
(b) D f Î R
Function 17

1
(c) f( x) = 2 cos 2 x + cos x +
8
Df ÎR
(d) ln(1 + | x|) ³ 0
ì(2n + 1) p ü
Df Îí ý
î 2 þ
3
æ ö 9
6. f ç ÷ =
è2ø 4
æ æ 3 öö 3
f ç f ç ÷÷ =
è è 2 øø 2
æ æ æ 3 ö öù 9
f çç f ç f ç ÷ ÷ ú =
è è è 2 ø øû 4
æ 5ö
f ç ÷ =2
è2ø
æ æ 5 öö
f ç f ç ÷÷ = 1
è è 2 øø
æ æ æ 5 ööö
f çç f ç f ç ÷ ÷ ÷÷ = 1
è è è 2 øøø
8. f -1 ( f( x)) = f( f -1 ( x)) = x
if f( f -1 ( x)) = f -1 ( x) Þ x = f -1 ( x)
if f( f -1 ( x)) = f -1 ( x) Þ f( f -1 ( f( x))) = f -1 ( f( x)) Þ f( x) = f -1 ( f( x)) = x
æx 3 × 1- x2 ö
9. f( x) = cos -1 x + cos -1 ç + ÷
ç2 2 ÷
è ø
Let x = cos q
æ1 3 ö
f( x) = cos -1 (cos q) + cos -1 çç cos q + sin q ÷÷
è2 2 ø
æ æ p öö
= cos -1 (cos q) + cos -1 ç cos ç q - ÷ ÷
è è 3 øø
p p
= 0£q£
3 3
p p
= 2q - <q£ p
3 3
18 Solution of Advanced Problems in Mathematics for JEE

10. f( x) = cos -1 ( - {- x})


ép ö
- {- x} Î ( -1, 0] Þ cos -1 ( - {- x}) Î ê , p ÷
ë2 ø
12. h( x) = [ln x - 1] + [1 - ln x]
-1, ln x - 1 Ï I
Þ h( x) = é
êë 0 , ln x - 1 Î I
1
14. f( x) = , f( x) is periodic & constant function.
2
e-x
16. f( x) =
1+ x
- e - x ( x + 2) 1
f ¢( x) =
(1 + x) 2 –2
–e–2

2 x{ x} 2{ x}([ x] + { x})
17. [ x] = =
x + { x} [ x] + 2 { x}
Þ [ x ] 2 = 2{ x } 2
1
Þ x =1+ (Q x Î R + )
2
18. || x - 1| + a| = 4
if a > 0 if a < 0
y y

–a
a

x x
1 1

(a) if eq. has three distinct real root then a < 0 and a = -4
(b) 4 distinct roots for a Î ( -¥, - 4)
(c) if -4 < a < 4 , there are two distinct real roots
(d) if a > 4, no real root.
19. (a) f2 ( x) = (sin x) 1/ 2 + (cos x) 1/ 2
sin x > sin 2 x; cos x > cos 2 x Þ sin x + cos x > 1
1/ 2 1/ 2
(b) f2 ( x) = (sin x) + (cos x) Þ f2 ( x) = 1 at x = 2 kp
(c) f2 ( x) = (sin x) 1/ 2 + (cos x) 1/ 2 ; f3 ( x) = (sin x) 1/ 3 + (cos x) 1/ 3
if x Î (2 kp , 2 kp + p / 2) 0 < sin x < 1 and 0 < cos x < 1
As power increases, value of function decreases.
Function 19

Þ f2 ( x) < f3 ( x)
(d) f3 ( x) = (sin x) 1/ 3 + (cos x) 1/ 3
f5 ( x) = (sin x) 1/ 5 + (cos x) 1/ 5
Þ f3 ( x) < f5 ( x)
æ x2 ö 2
20. -1 £ log 3 ç ÷£1 Þ 1£ x £3
ç 3 ÷ 3 3
è ø
Range is [0 , 1].
3x - 1
21. =n
2
é 4n + 5 ù é 4n + 5 1 ù
êë 9 úû + êë 9 + 2 úû = n

n =2 ü
M ï
ý
M ï
n = 10 þ
22. sin 6 q + cos 6 q = 1 - 3 sin 2 q cos 2 q
3 æ 1 - cos 4q ö 5 3
=1- ç ÷ = + cos( 4q)
4è 2 ø 8 8
5 3
= + cos( x)
8 8
23. (a) g( f( x)) = ln(sin x)
(b) x 2 + ( a - 1) x + 9 > 0 " x Î R
( a - 1) 2 - 36 < 0 Þ -5 < a < 7
(c) f( f( x)) = (2011 - (2011 - x 2012 )) 1 2012 = x
é 1 150 ù é 1 151 ù é 1 199 ù
24. ê + ú +ê + ú +¼ + ê + = 50
ë 4 200 û ë 4 200 û ë 4 200 úû

Exercise-3 : Comprehension Type Problems

Paragraph for Question Nos. 4 to 6


Sol. f( x) = qx 2 - 2 (q 2 - 3) x - 12q
g( x) = ln( x 2 - 49)
if domain of f + g is same as domain of g. Then
qx 2 - 2 (q 2 - 3) x - 12q ³ 0 " x Î ( -¥, - 7) È (7 , ¥)
20 Solution of Advanced Problems in Mathematics for JEE

é6 7 ù
Þ qÎ ê , ú
ë7 2 û
éq ù
h(q) = ln ê 4 cos 2 t dt - q 2 ú = ln[2q + sin 2q - q 2 ]
ò
êë 0 úû

Paragraph for Question Nos. 7 to 8


7. For x Î[5 4 , 5 5 ]
é x ù
f( x) = a 4 ê2 - -3 ú
4
ë 5 û
a =2
f( x) max = 32
8. a = 5
é x ù
f( x) = 5 4 ê2 - -3 ú
4
ë 5 û
é 2007 ù
f(2007) = 5 4 ê2 - + 3 ú = 1118
ë 625 û

Paragraph for Question Nos. 9 to 10


x 0£x£2
9. f(x)
–x –2 £ x < 0
f ( x ) = f ( x + 4)
{ f(5.12)} = { f(1.12)} = 0.12
{ f(7.88)} = { f(3.88)} = { f( -0.12)} = 0.12
10.
2

–2p –6 –4 –p –2 O p/2 2 p 4 6 2p

Paragraph for Question Nos. 13 to 14


13. f( x) = 3
3 + ln b1 , 3 + ln b2 , 3 + ln b3 are in A.P.
14. y = 3 x 2
Let slope of tangent be m.
Function 21

Þ y = m( x - 2)
Þ m( x 1 - 2) = 3 x 12
Also, m = 6 x1
Þ 6 x 1 ( x 1 - 2) = 3 x 12
x1 = 4
m = 24
Paragraph for Question Nos. 15 to 16
4
-4 x 2
15. y = 2 x Þ x 4 - 4 x 2 = log 2 y
4 + 16 - 4 log 2 y
x2 = Þ x = 2 + 4 - log 2 y
2
6
16. g( x) = 1 + Þ Range [-5, - 2]
sin x - 2

Exercise-4 : Matching Type Problems

1. [ x] + { x} + [ y] + {z } = 12.7 …(i)
[ x] + { y} + [z ] + {z } = 4.1 …(ii)
{ x } + [ y ] + { y } + [z ] = 2 …(iii)
Adding (i), (ii) & (iii),
Þ [ x] + { x} + [ y] + { y} + [z ] + {z } = 9.4
Þ { y} + [z ] = -3.3, { x} + [ y] = 5.3, [ x] + {z } = 7.4
Þ { y} = 0.7, [z ] = -4 , { x} = 0.3, [ y] = 5
[ x] = 7 , {z } = 0.4
4. (A) f( x) = sin 2 2 x - 2 sin 2 x = 2 sin 2 x cos 2 x
Function is even, hence many one, function is also periodic.
2
1 æ 1ö
f( x) = (1 - cos 2 x) cos 2 x = - ç cos 2 x - ÷
4 è 2ø
é 1ù
Range of function is ê -2 , ú .
ë 4 û
(B) f( x) = 4 x
(C) f( x) = ln(cos(sin x))
ln(cos(sin x)) ³ 0
Þ cos(sin x) = 1
Þ f( x) = 0
22 Solution of Advanced Problems in Mathematics for JEE

æ x2 + 1 ö
(D) f( x) = tan -1 ç ÷
ç x2 + 3 ÷
è ø
f( x) is even & hence many one.
ép p ö
Range is ê , ÷.
ë6 4 ø
7. (A) Domain of g( x) is [0, 3].
(B) Range of g( x) is [0, 3].
(C) f( f( f(2))) = 1
f( f( f(3))) = 2
(D) m = 3

Exercise-5 : Subjective Type Problems

1. f( x) - 2 x + 1 = ( x - 1)( x - 2)( x - 3)( x - 4)( x - 5)(2009 x - a )


2. f( x) = x 3 - 3 x + 1
f( f( x)) = 0
(0,3)
Let f( x) = t
(0,1)
Þ f (t ) = 0 b
Þ t = a ,b, g –2 a 0 1 g 2
–1
Þ f( x) = a , a Î ( -2 , - 1)
No. of solution = 1 –1

f( x) = b , b Î (0 , 1)
No. of solution = 3
f( x) = g , g Î (1, 2)
No. of solution = 3
3. Put x = y = 0 f(1) = 4
Put x = 0 , y = 1 f(2) = 9
2x -3 3
4. -1 £ £1 Þ £ x£
3 2 2
27
12 - 3 x - ³ 0 Þ (3 x - 3)(3 x - 9) £ 0 Þ 1 £ x £ 2
x
3
4
5. sin (0) + cos -1 ( -1) = p
-1
0 £ x2 <
9
-1 -1 4 2 13
sin (1) + cos (0) = p £x <
9 9
Function 23

8. Let P( x) = ax 4 + bx 3 + cx 2 + dx + 2
P(1) = a + b + c + d + 2 = 5 …(1)
P( -1) = a - b + c - d + 2 = 5 …(2)
Þ b + d = 0 and a + c = 3
P(2) = 16 a + 8 b + 4 c + 2 d + 2 = 2 …(3)
P( -2) = 16 a - 8 b + 4 c - 2 d + 2 = 2 …(4)
Þ 4 a + c = 0 and 4 b + d = 0
Þ b= d =0 and a = -1, c = 4
Þ P( x ) = - x 4 + 4 x 2 + 2
9. ( x + 1) 2 + y 2 = 1 (Q y > 0) y

x + y =k
k+1 x
(–2,0) (–1,0) 0
<1
2
- 2 - 1< k < 2 - 1
Þ 0 < k< 2 -1 (Q k > 0)
éxù éxù
10. [ x] + ê ú + ê ú = 3
ë2 û ë3 û
éxù éxù
\ when ê ú is an integer then definitely [ x] + ê ú is also an integer.
3
ë û ë2 û
éxù éxù
So, [ x] + ê ú = 2 and ê ú = 1 (and check like this)
ë2 û ë3 û
éxù éxù
[ x] + ê ú = 4 , ê ú = 1 Þ x Î[3 , 6)
2
ë û ë3 û
when x Î[3 , 4)
éxù
[ x] = 3 , ê ú = 1
ë2 û
So, x Î[3 , 4) satisfies.
éxù éxù
when x Î [4 , 5) [ x] = 4 ê ú = 2 Þ [ x] + ê ú = 6 ¹ 4 not satisfies, similarly on checking all
ë2 û ë2 û
possibilities we have only x Î[3 , 4).
\ a = 3, b = 4
2011
1 1 - x 2011
11. f( f( x)) = =
1 -x
2011 1 -
1 - x 2011
24 Solution of Advanced Problems in Mathematics for JEE

æ -1 ö -x
2011 ç 1 - ÷
2011 2011
è 1- x ø 1 - x 2011
f( f( f( x))) = = =x
-1 -1
2011 2011
1 - x 2011 1 - x 2011
\ f2013 ( x) = x = {- x}
12. f( x) = 0 0< x<6
= -1 6 £ x < 12
= -2 12 £ x < 18
= -3 18 £ x < 24
= -4 24 £ x < 30
= -5 x = 30
1
13. ( f( x , y)) 2 - ( g( x , y)) 2 =
2
3
f( x , y) × g( x , y) =
4
3
Þ f( x , y) = x 2 - y 2 = ±
2
1
g( x , y) = 2 xy = ±
2
x +5
14. f( x) =
x2 + 1
y

26
5

1
x
–5 1/5
–1

æ 1ù
15. f( x) is injective for x Î ç -¥, ú
è 5û
é 1ù
[l] = ê ú = 0
ë5û
x3
16. f : R ® R f( x) = + (m - 1) x 2 + (m + 5) x + n
3
f ¢( x) = x 2 + 2(m - 1) x + (m + 5) ³ 0
D£0
Function 25

4(m - 1) 2 - 4(m + 5) £ 0
m 2 - 3m - 4 £ 0
(m - 4)(m + 1) £ 0
-1 £ m £ 4
( x - 1)( x - 3)
17. f( x) = - ex
( x - 2)( x - 4)
f( x) = 0 has three solutions.
y

1
x
0 1 2 3 4

( x + 1)( x + 3)
f( - x) = - e - x = 0 has three solutions.
( x + 2)( x + 4)
x 3 = cos x

one solution
–p/2 0 p/2

there are total 7 solutions.


æ 2 ö
18. cos -1 ç - 1÷ = p (1 - { x}) y
ç (1 + x) 2 ÷
è ø
there are total 76 solutions. p

x
0 1 2 3 4

p1
19. f( x) = x 2 - bx + c = 0
p2
p1 + p 2 = b (odd no.)
Þ p1 = 2
26 Solution of Advanced Problems in Mathematics for JEE

p1 p 2 = c
b + c = ( p 2 + 2) + 2 p 2 = 35
Þ p 2 = 11
Þ f( x) = x 2 - 13 x + 22
81
l = f( x) min = -
4
æ xö
f( x) - f ç ÷
è7ø 1
20. f ¢( x) = lim =
x ®0 x 6
x-
7
x
Þ f( x) = + 1 Þ f( 42) = 8
6
1
21. g( x) = f( x) 0£ x< y
2 2
1 1
= £ x£1
4 2 1

=3 - x 1< x £ 2 1/4
x
0 1/2 1
100
10 æ 1 1 ö 10 æ 1 1 1 1 1 1 1 ö
22. x = å ç - ÷= ç1 + + + -
4 r =3 è r - 2 r + 2 ø 4 è
- - - ÷
2 3 4 102 101 100 99 ø
æ 1 1 1 1 ö
= 5 ´ 49 ç + + + ÷
è 99 200 303 408 ø
[ x] = 5
23. f( x) = x has two real roots.
7
cx 2 + ( d - a) x - b = 0
11
a-d -b
= 18 and = 77
c c
if f( f( x)) = x " x Î R Þ ( ac + cd) x 2 + ( d 2 - a 2 ) x - ( a + d)b = 0
Þ a + d = 0 Þ a = -d
a
f( x) will not attain the value = 9.
c
24. A = (1, 3)
p £ -2 1-1 , p £ -2 1-3
1 - 2( p + 7) + 5 £ 0 and 9 - 6 ( p + 7) + 5 £ 0 Þ p Î [-4 , - 1]
Function 27

1
x-
x 1
25. y = Let t = x - > 0 for x > 1
1 x
x3 - +2
x3
t 1
y= x3 - = t(t 2 + 3)
2 3
t(t + 3) + 2 x
t
=
3
t + 3t + 2
æ 2 2 2 1 1
1 ç t + =t + + ³ 3
= ç t t t
2 2 2 2
çç\t + + 3 ³ 6 ( AM ³ GM)
t + +3
t è t
1 1
y max = =
æ 2 2 ö 6
çt + + 3÷
è t ø min
p = 1, q = 6
28. a + ar + ar 2 = 1
a 2 r + a 2 r 2 + a 2 r 3 = b = ar ( a + ar + ar 2 ) = ar
a 3 r 3 = -g
29. m = 6 C 4 ´ 1 = 15
6! 6!
n= ´ 4 !+ ´ 4 ! = 1560
3 !1!1!1!3 ! (2 !) 4
n
30. å[log 2 r] = 0 + 1 + 1 + (2 + 2 + 2 + 2) + (13 4
+ 3 +¼ + 3) +¼
42443
r =1 8 times

= 2 × 1 + 4 × 2 + 8 × 3 +¼ +
32. 3
|( x - 2 y)( y + x)( x + 3 y)| = f( x , y)
No rain, then f( x , y) = 0 hence 3 lines.
33. Cubic = ( x 2 - 5 x + 6)( x + a ) + 2 ( Bx + 100 - 4a )
( x 2 - 5 x + 4)( x + a ) + Bx + 100 - 4a
Both identical B = -2
a = 50
2
Cubic = ( x - 5 x + 6)( x + 50) - 4 x - 200
28 Solution of Advanced Problems in Mathematics for JEE

34. f(q) = 0 Þ q = -5 ± 5
Þ f( f( f( x))) = -5 ± 5
Since f( x) = ( x + 5) 2 - 5
f( f( f( x))) = -5 ± 5
(( f( f( f( x)))) + 5) 2 = -5 ± 5
( f( f ) + 5) 2 = 5
f ( f ) + 5 = ± 51/ 4
f( f ) = -5 ± 51/ 4
( f + 5) 2 - 5 = -5 ± 51/ 4
( f + 5) 2 = 51/ 4
f + 5 = ± 51/ 8
35. Let ln x = t
2t 2 + 3t + 3
y= Þ ( y - 2) t 2 + ( 2 y - 3) t + ( 2 y - 3) > 0
2
t + 2t + 2
3 5
D ³ 0 Þ (2 y - 3)(2 y - 5) £ 0 Þ £ y<
2 2
36. P( x) = ( x - 3)Q 1 ( x) + 6 Þ P(3) = 6 …(1)
P( x) = ( x 2 - 9) Q ( x) + ( ax + b)
P ( 3) = 3 a + b = 6
If equation of odd degree polynomial, then b = 0 , a = 2.
37. f( x) = 2 x 3 - 3 x 2 + P
1
f ¢( x) = 6 x 2 - 6 x = 6 x( x - 1) 0
x

f(0) ³ 0 Ç f(1) £ 0
Þ P ³ 0 Ç P - 1£ 0
1
38. f( x) =
ln(cos -1 x)
ln(cos -1 x) > 0 Þ cos -1 x > 1

❑❑❑
Limit 29

2
LIMIT

Exercise-1 : Single Choice Problems

æ x - tan x ö æ tan x + x ö
2 sin ç ÷ sin ç ÷
è 2 ø è 2 ø ´ æ x - tan x ö æ x + tan x ö ´ 1
1. lim ç ÷ç ÷
x ®0 æ x - tan x ö æ tan x + x ö è x3 øè x ø 4
ç ÷ç ÷
è 2 øè 2 ø
1 æ 1ö 1
= ´ ç - ÷ ×2 = - (use expansions)
2 è 3ø 3
æ ln(1 + cos 2 x - 1) ö (cos 2 x - 1) 2
3. a = lim ç ÷ =-
x ®0 è cos 2 x - 1 2 3
ø 3x
æ sin 2 2 x ö 4x2
b = lim ç ÷ = -4
x ®0 ç 4 x 2 ÷ x
è ø x 2 æç 1 - e ö
÷
ç x ÷
è ø
x (1 - x) -1
c = lim =
x ®1 æ ln(1 +x - 1) ö 2
ç ÷ ( x - 1)( x + 1)
è x -1 ø
p
4. f( x) = - 3 tan -1 x
2
g( x) = 2 tan -1 x
f( x) - f( a) f ¢( a) 3
lim = =-
x ®0 g( x) - g( a) g¢( a) 2
30 Solution of Advanced Problems in Mathematics for JEE

æ æ ln(1 + x ) ö ö
4 4 ç 2 çè x -1 ÷ø ÷
lim çe -1 ÷
æ 2 ln (1+ x ) - 2 ö sin x x ® 0 sin x ç ÷
5. lim çex ÷ = e è ø
x ®0 ç ÷
è ø
æ 2 é ln (1 + x ) -1 ù ö
ç ê ú ÷
4 çe ë x û -1
÷ ´ 2 é ln(1+ x ) -1ù
lim
x ® 0 sin x ç æ ln(1+ x ) ö ÷ êë x úû
ç 2ç -1 ÷ ÷
è è x ø ø
= e
æ 2 ö
ç x- x +¼ ÷
2 1
lim 8 ç -1 ÷´
x ®0 ç x ÷ sin x
ç ÷
= e è ø

8
-
=e 2 = e -4
3 æ x ì x üö 3 3
6. lim ç - í ý÷ = - 0 =
x ®¥ x çè 4 î 4 þ ÷ø 4 4
Þ p + q=7
æ n
æ p ö ö÷
x ç1 + ç ÷
ç è 3 x ø ÷ø p
7. f( x) = lim è = x; x >
n ®¥ n -1 3
æ p ö
1+ ç ÷
è 3x ø
n
p æç æ 3 x ö ö
ç ÷ + 1÷
3 çè p ø ÷
= lim è ø = p;x< p
n ®¥ æ n -1 ö 3 3 x
ç æç 3 x ö÷ + 1÷ p/3
çè p ø ÷
è ø
p p p/3
= x=
3 3
p
\ f( x) = x ; x ³
3
p p
= ; x<
3 3
\ Option (d) is wrong.
2
sin( p - p cos 2 (tan(sin x)) sin[p sin 2 (tan(sin x))] æ sin(tan(sin x)) ö
8. lim = lim ´ pç ÷ =p
x ®0 x2 x ®0 p sin 2 (tan(sin x)) è x ø
( x + 3) x
(27) 27 -9 1 - cos( x - 3)
9. lim f( x) = lim f( x) Þ lim = lim l
x
x ®3 -
x ®3 +
x ®3 -
3 - 27 x ®3 +
( x - 3) 2
Limit 31

æ x 2+ 3 x ö
-2
2ç 9
÷
3 ç3 - 1÷
ç ÷ l
Þ lim è ø=
x ® 3- 3 x -3 2
3 (3 - 1)
1 x 2 + 3 x - 18 l 1 l 2
Þ lim = Þ ×9 = Þ l=
x ®3 3 9( x - 3) 2 27 2 3
æp ö æp ö æp ö æp ö
ç - x÷ ç - x÷ ç - x÷ ç - x÷
2 sin ç 3 ÷ cos ç 3 ÷ sin ç 3 ÷ cos ç 3 ÷
ç 2 ÷ ç 2 ÷ ç 2 ÷ ç 2 ÷
ç ÷ ç ÷ ç ÷ ç ÷
10. lim è ø è ø = lim è ø è ø

p æ p ö x®
p æ p ö æ p ö
3 2 ç cos x - cos ÷ 3 ç - x÷ ç + x÷
è 3 ø 2 sin ç 3 ÷ sin ç 3 ÷
ç 2 ÷ ç 2 ÷
ç ÷ ç ÷
è ø è ø
1 1 1
= =
2 3 3
2
p
sin
sin x 2 = 1 =2
11. lim Þ
p cos -1 [sin 3 x ] -1
cos (0) p 2 p

2

e{ x} - { x} - 1
13. lim { x} = lim x - [ x] = 1 ; lim = e -2
x ®I - x ®I - x ®I - { x} 2
é c ù
5c -1 æ 7 2 ö
16. lim x ê x ç1 + + ÷ - 1ú = l
x ®¥ ê è x x5 ø úû
ë
Case-I: 5c - 1 > 0 , then l ® ¥
Case-II: 5c - 1 < 0 , then l ® -¥
1
Since limit is finite and non-zero so 5c - 1 = 0 Þ c =
5
éæ 1/ 5 ù
7 2 ö
\ l = lim x êç 1 + + ÷ - 1ú
x ®¥ êè x x5 ø úû
ë
é æ 1öæ 7 2 ö ù
= lim x ê1 + ç ÷ ç + ÷ +¼¼ - 1ú (by binomial approximation)
5
x ®¥ ë è 5øè x x ø û
7
=
5
32 Solution of Advanced Problems in Mathematics for JEE

cos x - 1 æç cos x - 1 ( e x - 1) ö÷
17. lim - = 0 Þ n = 1, 2 , 3
x ®0 x 2 çè x n -2 x n -2 ÷ø
1 æ sin x - x ö
lim ç ÷
¥
18. 1 (form) = e x ® 0 1- cos x è x ø = e 2 ´ -1/ 6 = e -1/ 3
19. lim [ x 2 - x + 1 - ( ax + b)] = 0
x ®¥

So a > 0, on rationalizing
é( x 2 - x + 1) - [a 2 x 2 + b 2 + (2 ab) x] ù
lim ê ú =0
x ®¥ ê
ë x 2 - x + 1 + ax + b úû
So, 1 - a 2 = 0 -1 - 2 ab = 0
a =1
2
lim sec [k ! p( - 1 2)] = 1 = a
n ®¥

20. f( x + T ) = f( x + 2T ) = ¼¼ = f( x + nT ) = f( x)
æ n(n + 1) ö
nç ÷
nf( x)(1 + 2 + 3 +¼ + n) è 2 ø =3
lim = lim
n ®¥ f ( x )(1 + 2 2 + 3 2 +¼ + n 2 ) n ®¥ n(n + 1)(2n + 1) 2
6
é ù
ê ú
h2 + 3 3 53 ´ 3
21. 265 ê lim ú = -265 ´ =-
ê h ® 0 æ f(1 - h) - f(1) ö æ sin 5h ö ú f ¢(1) × 5 f ¢(1)
ê ç ÷ç ÷ú
ë è - h øè h øû
53 ´ 3
=- [Q f ¢(1) = -53]
-53
=3
2
cos x - 1
22. lim
x ® 0 cos x × x 2 × ( x + 1)
æ sin 2 x ö -1
lim - ç ÷ = -1
ç 2 ÷
x ®0
è x ø cos x( x + 1)
23. f( x + y) = f( x) × f( y)
f( x + h) - f( x) f( x) f( h) - f( x) æ f( h) - 1 ö
f ¢( x) = lim = lim = f( x) ç lim ÷
h ®0 h h ®0 h è h ®0 h ø
hP( h) + h 2 Q ( h)
If f( h) = 1 + hP( h) + h 2 Q ( h) Þ f ¢( x) = f( x) lim = P ( 0) f ( x )
h ®0 h
Limit 33

æ xö
ç 1 - tan ÷ (1 - sin x)
2
24. lim è ø
pæ x
x ® ç 1 + tan ö 3
2 ÷ ( p - 2 x)
è 2ø
æ p x öæ æp öö
tan ç - ÷ çç 1 - cos ç - x ÷ ÷÷
è 4 2 øè è 2 øø
= lim
p 3
x® ( p - 2 x)
2
p
Let x = +h
2
æ hö
tan ç - ÷ (1 - cos h)
lim è 2ø =
1
x ®0 3 32
( -2 h)
x æ -5 ö
lim x ç ÷
æ x -3 ö
25. lim ç ÷ =e
x ®¥ è x + 2 ø
= e -5
x ®¥ è x + 2 ø

27. ln c = I , (I Î integer)
I
Þ c=e
c is rational when I = 0
1/ x 1 é a sin bx ù
lim ê1+ cos x -1úû
æ a sin bx ö
28. lim ç 1 + ÷ = e x ®0 x ë = e ab
x ®0 è cos x ø
æ x 1 ö x2 -1 x + 1 x -1
30. a = lim ç - ÷ = lim = lim × =2
x ®1 è ln x x ln x ø x ®1 x ln x x ®1 x ln x
b = -4 , c = 1, d = -2
32. f( x) = x 2 -1 < x < 0
=1 x =0
1
= 0< x<1
x2
lim { f( x)} + lim { f( x)} + lim { f( x)} = 0
x ® 0- x ®1- x ®-1-

33. Let sin -1 x = q


p
2q -
cos -1 sin 2q 2
Þ lim = lim =2 2
p+ p p+ æ pö æ pö
q® sin q - sin q® çq - ÷ çq + ÷
4 4 4 4 4÷
2 sin ç ÷ cos ç
ç 2 ÷ ç 2 ÷
ç ÷ ç ÷
è ø è ø
34 Solution of Advanced Problems in Mathematics for JEE

cos -1 sin 2q
lim = -2 2
p- p
q® sin q - sin
4 4
n n
æ p p ö æ p p ö
34. lim
n ®¥
å ç sin
è 2k
- sin ÷ + lim å
ç cos
2( k + 2) ø n ®¥ k =1 è 2( k + 2)
- cos ÷
2k ø
k =1
æ p p p p p p p p ö
= lim ç sin - sin + sin - sin + sin - sin +¼ + sin - sin ÷
n ®¥ è 2 6 4 8 6 10 2n 2(n + 2) ø
æ p p p p p p p p ö
+ lim ç cos - cos + cos - cos + cos - cos +¼ + cos - cos ÷
n ®¥ è 6 2 8 4 10 6 2(n + 2) 2n ø
1 1
=1+ +2- =3
2 2
1 1
-
1 1 æ 2 xöm n
- ç 1 - 2 sin ÷ -1
(cos x) m n -1 è 2ø
36. lim = lim
x ®0 x2 x ®0 x2
x
sin 2
æ 1 1ö 2 = m -n
= lim - 2 ç - ÷
2
x ®0 èm nø x 2mn
x + xa cos x - b sin x
37. lim =1
x ®0 x3
Using expansion,
æ x 2 ö÷ æ x 3 ö÷ ax 3 bx 3
x + xa ç 1 - - bç x - x + ax - - bx +
ç 2 ! ÷ ç 3 ! ÷
Þ lim è ø è ø Þ lim 2! 3!
x ®0 3 x 0 3
x ® x
Clearly, 1 + a - b = 0 for limit to be finite …(1)
3
æ b aöx b a
Þ lim ç - ÷ = 1 Þ - = 1 Þ b - 3a = 6 …(2)
2 !ø x 3
x ®0 è 3 ! 6 2
5 3
Þ From (1) and (2), a = - , b = -
2 2
e x (cos x - sin x)
a cos ax -
38. lim e x × cos x =
1
x ®0 sin bx + bx × cos bx 2
cos 2 x - cos x + sin x 1
Þ lim = (Q a = 1)
x ® 0 cos x (sin bx + bx cos bx ) 2
2
æ n(n + 1) ö n (2n + 1)(n + 1)
ç ÷ -
( 13 + 2 3 + 3 3 ¼ + n 3 ) - ( 12 + 2 2 ¼ + n 2 ) 2 6
39. a = lim = lim è ø
n ®¥ 4 n ®¥ 4
n n
Limit 35

é1æ 2
1ö (2n + 1)(n + 1) ù 1
Þlim ê ç 1 + ÷ - ú=
n ®¥ ê 4 è nø 3
ë 6 n úû 4
æ sin x + x ö æ x - sin x ö
2 sin ç ÷ sin ç ÷
cos(sin x) - cos x è 2 ø è 2 ø
40. lim Þ lim
x® 0 4 x 0 4
x ® x
2 (sin x + x) ( x - sin x) ì sin x + x x - sin x ü
Þ lim
4
× íQ ® 0; ® 0ý
x® 0 x 2 2 î 2 2 þ
1æ sin x ö æ x - sin x ö 1
Þ lim ç1 + ÷ç ÷Þ
x® 0 2 è x øè x 3 6
ø
1 2 3 n
42. un = + + ¼+ …(1)
2 22 23 2n
1 1 2 n -1 n
u n =¼ + + ¼+ + …(2)
2 3 n
2 2 2 2 2 +1
n

Substracting equation (1) and (2),


1æ 1 ö
ç1 - ÷
un 1 1 1 1 n un 2 è 2n ø n
= + + +¼ + - Þ = -
2 3 n
2 2 2 2 2 2 +1
n 2 æ 1ö 2 +1
n
ç1 - ÷
è 2ø
æ 1 ö n
Þ un = 2 ç 1 - ÷- ; lim u = 2
n n +1 n ® 0 n
è 2 ø 2
sin 2 x tan -1 3 x
lim
(cos x -1) ×2x + 6x × + 3x2
sin 2 x 2 x 3 x 1
43. e x ®0 + lim = +2
x ® 0 ln(1 + 3 x + sin 2 x ) e
2 x
× (3 x + sin x) + xe
3 x + sin 2 x
x
44. tan (1 + sec x) = tan x
2
x
fn ( x) = tan (1 + sec x)(1 + sec 2 x) ¼(1 + sec 2 n x) = tan 2 n x
2
1 1
ln (tan x ) ln (tan x )
45. lim (1 + [ x]) = lim (1) =1
p p
x® x®
4 4
{( a - n) nx - tan x} sin nx
46. lim xn = 0
x ®0 x2 nx
1
Þ {( a - n) n - 1}n = 0 Þ a = n +
n
36 Solution of Advanced Problems in Mathematics for JEE

3n 3 + 4
æ n ! ö 4n 4 -1
47. y = lim ç ÷
n ®¥ è n n ø

n 1
3n 3 + 4
æ ö r 3 -3
ln y = lim å ln ç ÷ = ò ln x dx = Þ y = e -3/ 4
4
n ®¥ 4n - 1 ènø 4 r =1 0
4
2
ax + bx + c ax + b + ( c x) æa bö
48. lim = lim = lim ç x + ÷
x ®¥ dx + e x ®¥ d + ( e x) x ®¥ èd dø
æ aö
= + ¥ if ç ÷ is positive.
è dø
æ aö
= -¥ if ç ÷ is negative.
è dø
Alternate solution :
ax 2 + bx + c a + ( b x) + ( c x 2 )
lim = lim
x ®¥ dx + e x ®¥ ( d x ) + ( e x 2 )

e d
Here << . Therefore,
2 x
x
ax 2 + bx + c a
lim = lim
x ®¥ dx + e x ®¥ d x
ì a ì+ ¥ if a > 0 and d > 0
ïï + if d > 0 í -¥ if
î a < 0 and d > 0
= í0
a ì -¥ if a > 0 and d < 0
ï if d < 0 í+ ¥ if
ïî 0 - î a < 0 and d < 0
æ 2 ö
ç æ x ö ÷
ç sin ÷ 2
æ x ö -1 ç 2n ÷ × x ÷
49. f( x) = lim tan -1 ç 4n 2 × 2 sin 2 2
÷ = lim tan ç 8n ç
-1 2
÷ = tan (2 x )
n ®¥ è 2n ø n ®¥ ç x ÷ 4n 2
çç ç ÷ ÷÷
è è 2n ø ø
æ æ 2x öö
ç ln ç 1 + cos 2 - 1÷ ÷
n2 ç è n ø ÷ æ cos 2 x - 1ö = x 2
g( x) = lim
n ®¥ 2 ç 2 2x ÷ çè n
÷
ø
ç cos - 1 ÷
è n ø
sin 2 x 1
50. lim = Þ f( x) = x 2 ( ax + 3) ; a ¹ 0
x ® 0 f( x) 3
(2 e 2 sin x - e sin x - 1) (2 e sin x + 1)( e sin x - 1) 3
51. lim = lim =
x ®0 2 sin x x ®0 sin x 2
( x + 2 x) e x ( x + 2) e
Limit 37

52. x n + ax + b = ( x - x 1 )( x - x 2 )( x - x 3 ) ¼( x - x n )
x n + ax + b
lim = ( x 1 - x 2 )( x 1 - x 3 ) ¼( x 1 - x n )
x ® x1 x - x1
æ 1 2 ö æ 1 ö
ç 1 + sin x +¼ ÷ - ç 1 - (2 tan x) +¼ ÷
è 3 ø è 4 ø 1
53. lim =
x ®0 2 2
sin x + tan x
2 sin x
cos x tan x
x 1 2 0
f( x)
54. lim = lim 1 1 1 = 1 1 1 = -1
x ®0 x 2 x ®0
1 2 1
1 2 1

Exercise-2 : One or More than One Answer is/are Correct


1
lim ( p tan qx 2 - 3 cos2 x + 3)
x ®0 3 x 2
1. e
pq 3 (1- cos2 x )
lim +
e x ®0 3 3x2

pq 5
Þ + 1= ; pq = 2
3 3
3. a ³ e > 2
1/ x
æ x x
æ2ö æ eö ö
(a) L = a lim ç 1 + ç ÷ + ç ÷ ÷
x ®¥ ç è aø è a ø ÷ø
è
æ2ö æ eö 1
\ x ® a, ç ÷ ® 0 , ç ÷ ® 0 , ® 0
è aø è aø x
So, L = a
(b) If a = 2 e > 2
1/ x
x x x 1/ x
éæ 1 ö x x
æ 1ö ù
L = lim (2 + (2 e) + e ) = 2 e lim êç ÷ + 1 + ç ÷ ú = 2 e(1) = 2 e
x ®¥ x ®¥ êè e ø è 2 ø úû
ë
(c) If 0 < a £ e
æ 1/ x ö
ç ææ 2 ö x æ a ö x ö ÷
ç
L = e ç lim ç ÷ + ç ÷ + 1 ÷
÷=e
ç x ®¥ çè è e ø è eø ÷
ø ÷
è ø
38 Solution of Advanced Problems in Mathematics for JEE

e
(d) a > >1
2
1/ x 1/ x
é x ù ææ 1ö x æ 1ö x æ e ö x ö
æ 2a ö
L = lim ê2 x + ç x
÷ +e ú = 2 a lim ç ç ÷ + ç ÷ + ç ÷ ÷ =0
x ®¥ ê è 2 ø úû x ®¥ ç è a ø è 2ø è 2a ø ÷
ë è ø
5. f( x) = cos(sin x)
Range is [cos 1, 1].
æ3 3 ö
8. f( x) = x ç + [cos x]÷
è2 2 ø
1 1
9. If x ¹ then f( x) = 0 but if x = then lim f( x) = lim ( -1) n , hence does not exist.
2n 2n x ®0 n ®¥
2 2
1 1
Also, if x = then 2 x ¹ Þ f (2 x) = 0
2n n
2 22
æ sin -1 2 x - x 2 ö
ç ÷ 2 x - x 2 × sin -1 (1 - x)
ç ÷
cos -1 (1 - x) sin -1 (1 - x) è 2x - x2 ø p
11. lim = lim =
x ® 0+ 2 x (1 - x) x ® 0+ 2 x (1 - x) 2
cos -1 ( - x) sin -1 ( - x) p
lim =
-
x ®0 2( x + 1) ( - x) 2 2
æ sin x - x ö æ sin x + x ö
2 sin ç ÷ × cos ç ÷
è 2 ø è 2 ø = -1
12. lim
x ®0 3 5 12
ax + bx + c
æ æ sin x - x ö ö
ç sin ç ÷÷
ç è 2 ø ÷ æ sin x - x ö × cos æ sin x + x ö
2
ç sin x - x ÷ çè 2
÷
ø
ç
è 2
÷
ø
ç ÷
2 -1
lim è ø =
x ®0 3 5 12
ax + bx + c
æ pö
14. cos 2 ç np + ÷
è 3ø
sin b 1
15. sin a + sin b = - Þ sin a = sin b = -
sin a 2
16. lim [5 - 2 x] = 0
x ® 2+
lim [| x - 2| + a 2 - 6 a + 9] = 0 Þ ( a - 3) 2 < 1
x ® 2-
Limit 39

Exercise-3 : Comprehension Type Problems

Paragraph for Question Nos. 1 to 2


1. S 1 = 1, S 2 = 7 , S 3 = 19
Þ S n = 1 + 3n(n - 1)
S
lim n = 3
n ®¥ n 2

1 1
2. r1 = 1, r2 = , r3 =
3 5
1
or rn =
2n - 1
1 1
lim n ´ =
n ®¥ 2n - 1 2
Paragraph for Question Nos. 3 to 4
3. x > 0, x < tan x
x < 0, x > tan x Þ x - tan x > 0
\ [ x - tan x] = 0
\ lim f([ x - tan x]) = f(0) = 4
x ® 0-

4. x > 0 x < tan x


x
<1
tan x
ì x ü x
lim í ý= ® 1-
+ tan x
x ®0 î þ tan x
æ ì x üö æ x ö -
\ lim çç f í ý ÷÷ = lim+ f ç ÷ = f (1 ) = 2 + 5 = 7
x ® 0 è îtan x þ ø x ® 0
+
è tan x ø
Paragraph for Question Nos. 5 to 6
5. f( x) = 1 -| x - 2|
x ® 2 + , f( x) ® 1- and x ® 2 - , f( x) ® 1-
1 f ( x )-1
lim
æ px ö x ® 2+ æ px ö
sin ç ÷ sin ç ÷
R.H.L. = lim ( f( x)) è 2 ø =e è 2 ø
x ® 2+
40 Solution of Advanced Problems in Mathematics for JEE

1-( x - 2)-1
lim
x ® 2+ sin p æ 1- x ö lim
ç ÷ ( x - 2) p
=e è 2ø +
= e x ®2 ´ (2 - x)
æ p ö 2
ç sin (2 - x) ÷
ç 2 ÷
ç p ÷
ç (2 - x) ÷
è 2 ø
= e 2/p
1 f ( x ) -1
lim
æ px ö x ® 2- sin æ px ö
sin xç ÷ ç ÷
L.H.L. = lim ( f( x)) è 2 ø =e è 2 ø
x ® 2-
lim lim
- 1 + ( x - 2) - 1 - x -2
= e x ®2 = e x ®2
p p
sin (2 - x) (2 - x)
2 2
= e -2/p
\ Limit does not exist.
6. [1, 3]
as f(3 x) = af( x)
x Î[1, 3] ; f( x) Î [0 , 1] 1
3 x Î[3 , 9] ; f(3 x) = af( x) Î [0 , a ]
9 x Î[9 , 27] ; f(9 x) = af(3 x) Î [0 , a 2 ]
1 3 9
1
area between [1, 3] is D 1 = ´2 ´1=1
2
1
area between [3 , 9] is D 2 = ´ 6 ´ a = 3a
2
1
area between [9 , 27] is D 3 = ´ 18 ´ a 2 = 9a 2
2
æ 1 1ö
\ 1, 3a , 9a 2 , ¼¼ is converges when (g.p.) |3a| < 1 a Î ç - , ÷
è 3 3ø
Paragraph for Question Nos. 7 to 9
æ x x 2 x 3 ö÷
(1 + bx) - (1 + ax) ç 1 + - +
[(1 + bx) - (1 + ax) 1 + x ] ç 2 8 16 ÷ø
7. lim = lim è
x ®0 x3 x ®0 x3
x x2 x3 ax 2 ax 3
bx - + - - ax - +
= lim 2 8 16 2 8
x ®0 x3
Limit 41

1 a 1
Þ coefficient of x and x 2 = 0 Þ b - a = and =
2 2 8
1 3
Þ a= ,b=
4 4
8. a + b = 1
1 3
9. l = - ; b=
32 4
Paragraph for Question Nos. 10 to 11
Sol. sin x + sin y = 1
- cos x
y¢ =
2 sin x - sin 2 x
sin 2 x - sin x + 1
Þ y ¢¢ =
(2 sin x - sin 2 x) 3/ 2

Exercise-5 : Subjective Type Problems

æp ö éæ 1 - tan bx ö ù
ln tan ç - bx ÷ ln êçç - 1÷÷ + 1ú
1. lim - è4 ø = - lim ëè 1 + tan bx ø û
x ®0 tan ax x ®0 tan ax
æ bö
= -1ç -2 ÷ = 1
è aø
a
Þ =2
b
3. a( x 3 - 1) + ( x - 1) = 0
( x - 1)( ax 2 + ax + a + 1) = 0
a , b ¹ 1 so, a , b are roots of ax 2 + ax + a + 1 = 0
a+1
a + b = -1, ab =
a
(1 + a) x 3 - x 2 - a ( x 3 - x 2 ) + a( x 3 - 1)
lim = lim

1 ( e1-ax - 1)( x - 1) x®
1 ( e1-ax - 1)( x - 1)
a a

[ x 2 + a( x 2 + x + 1)] (1 + a) x 2 + ax + a
= lim = lim

1 ( e1-ax - 1) x®
1 æ e1-ax - 1 ö
a a ç ÷ (1 - ax)
ç 1 - ax ÷
è ø
42 Solution of Advanced Problems in Mathematics for JEE

éæ 1 + a ö 2 ù
êç a ÷ x + (1) x + 1ú 2
= lim a ë
è ø û = lim a (abx - (a + b) x + 1)

1 (1 - ax) x®
1 (1 - ax)
a a
(1 - (a ) x)(1 - (b) x) a(a - b)
= lim a =

1 (1 - ax) a
a
x x x
(4 - 1)(5 - 1)(7 - 1)
4. lim = 2 ln 2 ln 5 ln 7
x ®0 2
x sin x
ax cos x + b sin x 1
5. lim =
x ®0 2 3
x sin x
æ x 2 x 4 ö÷ æ x 3 x 5 ö÷
ax ç 1 - + ¼ + bç x - + ¼
ç 2! 4 ! ÷ø ç 3! 5 ! ÷ø 1
lim è è =
x ®0 x 2 sin x 3
a b 1
a + b = 0 and - - =
2 6 3
Þ b = 1, a = -1
é sin x ù
7. lim ê =0
+ x - 1ú
x ®a ë û
x
y=

1
x–
y=

0 1 a2 3 p

❑❑❑
Continuity, Differentiability and Differentiation 43

3
CONTINUITY, DIFFERENTIABILITY
AND DIFFERENTIATION

Exercise-1 : Single Choice Problems

f( x + h) - f( x) f( x) + f( h) + 3 hx( h + x) - f( x)
1. f ¢( x) = lim = lim
h ®0 h h ®0 h
f ¢( x) = 3 x 2 + f ¢(0) Þ f ¢¢( x) = 6 x

2. 0 1
–2 –1 2
–1

f( x) is non-differentiable at five points.


x
3. is integer at 21 points in [0 , 100]
5
x
is integer at 51 points in [0 , 100]
2
\ But when x is a multiple of 10 then f( x) is continuous.
So that respective points should be subtract from both i . e. , multiple of 10 are 11 points in
[0 , 100].
21 + 51 - 11 - 11 = 72 - 22 = 50
4. f( x) has isolated point of discontinuity but| f( x)|is continuous at
x =a
So, lim f( x) and f( a) has opposite sign, with same magnitude.
x ®a
So, lim f( x) = - f( a)
x ®a
lim f( x) + f( a) = 0
x ®a
f ( 4 x) - 3 f (3 x) + 3 f (2 x) - f ( x)
5. lim = 12
x® 0 x3
44 Solution of Advanced Problems in Mathematics for JEE

4 f ¢( 4 x) - 9 f ¢(3 x) + 6 f ¢(2 x) - f ¢( x)
lim = 12
x® 0 3x2
4 2 f ¢¢( 4 x) - 27 f ¢¢(3 x) + 12 f ¢¢(2 x) - f ¢¢( x)
lim = 12
x® 0 6x
4 3 f ¢¢¢( 4 x) - 81 f ¢¢¢(3 x) + 24 f ¢¢¢(2 x) - f ¢¢¢( x)
lim = 12
x® 0 6
\ ( 4 3 - 81 + 24 - 1) f ¢¢¢(0) = 12 ´ 6
6 f ¢¢¢(0) = 12 ´ 6
f ¢¢¢(0) = 12
1 1
6. y = +
sin q - cos q cot q - cos q cos q - sin q
1 + (tan q) + (tan q) 1 + (tan q) + (tan q) cot q - sin q
1
+
1 + (tan q) cos q - cot q + (tan q) sin q - cot q
(tan q) cos q (tan q) sin q
y= +
(tan q) cos q + (tan q) sin q + (tan q) cot q (tan q) cos q + (tan q) sin q + (tan q) cot q
(tan q) cot q
+
(tan q) cos q + (tan q) sin q + (tan q) cot q
y =1
dy
=0
dx 0 = p/ 3

7. f ¢( x) = sin( x 2 )
y = f( x 2 + 1)
dy
= f ¢( x 2 + 1)2 x
dx
dy
= 2 × f ¢(2) = 2 sin 4
dx x =1
7p
8. Clearly sin x , cos x are negative at x =
6
So, f( x) = - (sin x + cos x)
f ¢( x) = (sin x - cos x)
9. 2 sin x cos y = 1
cos x cos y - sin x sin y × y ¢ = 0 Þ y (¢ p/ 4, p/ 4) = 1
y ¢ = cot x cot y
y ¢¢ = - cot x cosec 2 y ´ y ¢ - cot y cosec 2 x
y (¢¢p/ 4, p/ 4) = - (1 ´ 2 ´ 1) - (1 ´ 2) = 0
Continuity, Differentiability and Differentiation 45

dx dy
10. = 2t f ¢(t 2 ), = 3t 2 f ¢(t 3 )
dt dt
dy 3 tf ¢(t 3 )
=
dx 2 f ¢(t 2 )

d2 y 3 æç f ¢(t 2 )( f ¢(t 3 ) + 3t 3 f ¢¢(t 3 )) - 2t 2 f ¢(t 3 ) × f ¢¢(t 2 ) ö÷ dt


=
dx 2 2 çè ( f ¢(t 2 )) 2 ÷ dx
ø
d2 y 3 æç f ¢(1)( f ¢(1) + 3 f ¢¢(1)) - 2 f ¢(1) × f ¢¢(1) ö÷ 1 3 æ f ¢¢(1) + f ¢(1) ö÷
= = ç
2 2è ç 2 ÷ ç 2 ÷
dx t =1
( f ¢(1)) ø 2 f ¢(1) 4 è ( f ¢(1)) ø
11. L.H.L. = a + 1
R.H.L. = b + 1
\ they are continuous L.H.L. = R.H.L.
1 b g

12. y = + x x + x2
1 1 1 1 1 1
- a æç - a ö÷ æç - b ö÷ æç - a ö÷ æç - b ö÷ æç - g ö÷
x èx øè x ø èx øè x øè x ø
1 g 1
2 2
= x + x = x3
æ1 öæ 1 ö æ1 öæ 1 öæ 1 ö æ1 öæ 1 öæ 1 ö
ç - a ÷ç -b÷ ç - a ÷ç -b÷ç - g÷ ç - a ÷ç -b÷ç - g÷
è x øè x ø è x øè x øè x ø è x øè x øè x ø
æ1 ö æ1 ö æ1 ö
log y = -3 ln x - ln ç - a ÷ - ln ç - b ÷ - ln ç - g ÷
èx ø èx ø èx ø
1 1 1
1 -3 2 2 2
y¢ = + x + x + x
y x æ1 ö æ1 ö æ1 ö
ç - a ÷ ç -b÷ ç - g÷
è x ø è x ø è x ø
æ 1 1 1 ö÷
ç
y x x x
y ¢ = ç -3 + + + ÷
xç æ1 ö æ1 ö æ1 ö÷
ç ç - a ÷ ç -b÷ ç - g÷ ÷
è èx ø èx ø èx øø
yæ a b g ö
y¢ = ç + + ÷
x çè 1 x - a 1 x - b 1 x - g ÷ø

1 + sin -1 x
13. f( x) =
1 - tan -1 x
1
ln f( x) = [ln(1 + sin -1 x) - ln(1 - tan -1 x)]
2
46 Solution of Advanced Problems in Mathematics for JEE

f ¢( x) 1 é 1 1 ù
= ê + ú
f( x) 2 ê(1 + sin -1 x) 1 - x 2 (1 - tan -1 x)(1 + x 2 ) ú
ë û
\ f ¢(0) = 1
14. sin x = - sin 2 x Þ 2 sin 2 x = 0 Þ x = np
2

tan x tan x < cot x


15. f(x)
cot x
tan x ³ cot x
p 3 p 5p 7 p
Points of non-derivability = , , ,
4 4 4 4
16. g( x) =||| x - 1| - 1| - 1|
= x -3 x>3
= - ( x - 3) 2< x<3
d2 x 1 d2 y
18. =-
dy 2 æ dy ö
3
dx 2
ç ÷
è dx ø
dy d2 y
=1+ ex , = ex
dx dx 2
dy d2 y
at x = ln 2 , = 3, =2
dx dx 2
d2 x -2
=
2 27
dy
1
19. g¢( f( x)) =
f ¢( x)
f( x) = -4 at x = -2
1 1
Þ g¢( -4) = =
f ¢( -2) 2
20. f( x) = 2 - x x³1
=x 0£ x<1
= -x -1 £ x < 0
=x +2 x < -1
21. f( x) = cos x 2
f ¢( x) = -2 x sin x 2
1
22. f( g( x)) = x Þ f ¢( g( x)) g¢( x) = 1 Þ g¢( x) = = 1 + ( g( x)) 5
f ¢( g( x))
g¢¢( x) = 5( g( x)) 4 g¢( x)
Continuity, Differentiability and Differentiation 47

23. f( x) = x 2 x³1
=x 0£ x£1
= 2x -1 £ x £ 0
= x -1 x £ -1
Clearly it is non-differentiable at x = 0 , - 1 and 1.
æ x x x x ö sin x sin x
24. f( x) = lim ç cos × cos × cos ¼¼ cos ÷ = lim =
2 3
n ®¥ è 2 2 2 2 n ø n ®¥ æ x ö x
2 n sin ç ÷
n
è2 ø
æ p- ö æ p+ ö
25. f ç ÷ = f æç p ö÷ = fç ÷
ç 4 ÷ è4ø ç 4 ÷
è ø è ø
æp ö
tan ç - x ÷ × (1 + tan x)
æ 1 - tan x ö è 4 ø 1
lim ç ÷ = lim =-
x® è 4 x - p ø x® pö 2
p p æ
4 4 4ç x - ÷
è 4ø
1
-
h2 1
e sin
f(0 + h) - f(0) h
26. f ¢(0) = lim = lim
h ®0 h h ®0 h
dy
27. = 2 y + 10
dx
dy
òy +5
= 2 dx ò
ln( y + 5) = 2 x + c
y = 5 ( e 2 x - 1) (Q c = ln 5)
f( x) + 5 sec 2 x = 0 Þ e 2 x + tan 2 x = 0
æ p- ö
28. f ç ÷ = lim sin{cos x} = lim sin(cos x) = -1
ç 2 ÷ - p p- p
è ø x® p x- x® x-
2 2 2 2
æ p+ ö
fç ÷ = lim sin{cos x} = lim sin(cos x + 1)
ç 2 ÷ + p p+ p
è ø x® p x- x® x-
2 2 2 2
29. Let g( x) = f( e x )
g¢( x) = f ¢( e x ) × e x
g¢¢( x) = f ¢¢( e x ) × e 2 x + f ¢( e x )e x
x
30. e f ( x ) = ln x Þ f( x) = ln(ln x) Þ g( x) = f -1 ( x) = e e
x x
g¢( x) = e e × e x = e e +x
48 Solution of Advanced Problems in Mathematics for JEE

32. ln f( x) = 4 ln( x - 1) + 3 ln( x - 2) + 2 ln( x - 3)


f ¢( x) 4 3 2
= + +
f( x) x - 1 x - 2 x - 3
æ 4 3 2 ö
f ¢( x) = f( x) ç + + ÷
è x -1 x -2 x -3ø
34. f(2 + ) = 0 Þ c = 0
b sin{- x}
f (2 - ) = = f (2 + ) = 0 Þ b = 0
{- x }
e tan x - e x + ln(sec x + tan x) - x
35. f(0) = lim
x® 0 tan x - x
( e tan x - x - 1) ln(sec x + tan x) - x sec x - 1 1 3
= lim e x + lim = 1 + lim =1+ =
x® 0 tan x - x x 0 tan x - x x 0 2 2 2
® ® sec x - 1
36. f(0 - ) = e a
f ( 0) = b
c =1
2 2
f(0 + ) = Þ b = ea =
3 3
37. x+ y+ y - x =5
x + y =5 - y-x
Sq. both sides,
Þ x + y = 25 + y - x - 10 y - x
Þ 25 - 2 x = 10 y - x
10 ( y ¢ - 1)
Þ -2 =
2 y-x
Þ -2 y - x = 5 ( y ¢ - 1)
æ 2x ö
Þ -ç5 - ÷ = 5 ( y ¢ - 1)
è 5 ø
2x
-5 + = 5 ( y ¢ - 1)
5
2
Þ y ¢¢ =
25
38. g( x) = f -1 ( x)
Þ f( g( x)) = x
Þ f ¢( g( x)) g¢( x) = 1
Continuity, Differentiability and Differentiation 49

1
Þ g¢(2) =
f ¢( g(2))
f(1) = 2
Þ g(2) = 1
1
Þ g¢(2) =
f ¢(1)
1
f ¢( x) = 3 x 2 + 4 x 3 +
x
f ¢(1) = 8
1
Þ g¢(2) =
8
39. f( x) =| x| x Î ( -¥, - 1)
2
=x x Î [-1, 1)
= 2x - 1 x Î [1, ¥)
Function is not differentiable at x = -1.
40. g( x) = ( f( x)) 2 + ( f ¢( x)) 2 Þ g¢( x) = 2 f( x) f ¢( x) + 2 f ¢( x) f ¢¢( x)
or g¢( x) = 2 f( x) f ¢( x) - 2 f( x) f ¢( x) = 0 Þ g( x) = c Þ g(8) = 8
æ æ a ö ö
ç fç ÷-1 ÷
xø ÷
lim ç è
x ®¥ ç 1 ÷
x çç ÷÷
æ æ a öö è x ø
41. l = lim çç f çç ÷÷ ÷÷ = e
x ®¥ è è x øø
Using L’ Hospital’s rule, we get
a2 a2
f ¢¢( 0) -
l= e2 =e 2

d d d
42. fn ( x) = e fn-1 ( x ) fn -1 ( x) = fn ( x) fn -1 ( x)
dx dx dx
= fn ( x) fn -1 ( x) ¼¼ f2 ( x) f1 ( x)
-1
43. y = tan ( x 1/ 3 ) - tan -1 ( a 1/ 3 )
4k - 1 4k + 1
44. f( x) is continuous at x = 0 then =
3 5
q
45. Put x = sin q then y = tan -1 tan
2
e x cos x - ln(1 + x) - 1
46. lim
x ®0 x
( e x - 1) ln(1 + x) æ cos x - 1 ö
lim cos x - +ç ÷ =0
x ®0 x x è x ø
50 Solution of Advanced Problems in Mathematics for JEE

y = 1–cos x
2
y = cos x
1
47. y = sin x

p/2 p 3p/2 2p

Clearly 3 sharp points.


48. g( x) = f -1 ( x) f( 4) = 2 Þ g(2) = 4
1 1
G( x) = f ¢( 4) = Þ g¢(2) = 16
g( x) 16
-1 -1 -1
G ¢( x) = × g¢( x) Þ G ¢(2) = × g¢(2) = × 16 = -1
2 2 16
( g( x)) ( g( x))
y

49. 1/81
x
O 1 1
9
æ 1ö 1 1
f( x) = maximum ç x 4 , x 2 , ÷ = x£
è 81 ø 81 9
1
= x2 < x<1
9
= x4 x³1
1
f( x) is non-differentiable at x = , 1
9
ln( f(2 + h 2 )) - ln( f(2 - h 2 ))
50. lim
h ®0 h2
Apply L’ Hospital rule,
2 hf ¢(2 + h 2 ) 2 hf ¢(2 - h 2 )
+
f (2 + h 2 ) f (2 - h 2 )
lim =4
h ®0 2h
æ pö
51. f( x) = ( x 2 - 3 x + 2)|( x - 1)( x - 2)( x - 3)| + sin ç x + ÷
è 4ø
3p 7p
Not differentiable at x = 3 , ,
4 4
52. h( x) = f(2 x g( x) + cos px - 3)
h¢( x) = f ¢(2 x g( x) + cos px - 3)[2 g( x) + 2 xg¢( x) - p sin px]
h¢(1) = f ¢(2 g(1) - 4)[2 g(1) + 2 g¢(1)] = 32
Continuity, Differentiability and Differentiation 51

( x + 1) 7 1 + x 2
53. f( x) = ( f(0) = 1)
( x 2 - x + 1) 6
1
ln(1 + x 2 ) - 6 ln( x 2 - x + 1)
ln f( x) = 7 ln(1 + x) +
2
f ¢( x) 7 x 6 (2 x - 1)
= + -
2
f( x) 1 + x 1 + x x2 - x + 1
f ¢(0) = 13
ì
ï - sin 2 x ; x>1
ï
54. f( x) í ln(1 + x) ; x<1 ; f(1+ ) ¹ f(1- ) ¹ f(1)
ïln 2 - sin 2
ïî ; x =1
2
f( x) ; if f( x) is rational
55. f( f( x)) é
ëê1 - f( x) ; if f( x) is irrational
é x; if x is rational
f( f( x))
êë1 - (1 - x) ; if x is irrational
x ; if x is rational
f( f( x)) é
êë x ; if x is irrational

y = |x2–x–2|

y = x2–3x
56.
0 1 2 3

y = ln(–x)

y = ex
a
1

So, graph of f( x) is
2
ln(–x) ex

a
1 2

Clearly, 3 non-differentiability points.


52 Solution of Advanced Problems in Mathematics for JEE

57. g( f( x)) = x
ln(2 - cos 2 x) e sin 2 x - 1
58. lim = K = lim
ln 2 (1 + sin 3 x)
x ® 0- x ® 0+ ln(1 + tan 9 x )

1 - cos 2 x sin 2 x
lim = K = lim
-
x ® 0 sin 3 x2 x ®0 + tan 9 x

dx 3 2 -3 - 2t
59. =- - =
dt t4 t3 t4
dy -3 2 -3 - 2t
= - =
dt t 3 t 2 t3
dy
=t
dx
3
dy æ dy ö æ1+ t ö 3
- xç ÷ =t -ç ÷ × t = -1
dx è ødx è t3 ø
2
60. - y ¢ = 2 2( -2 sin 2 x)
y3
2
( y ¢) 2 2
æ 1 ö
= 8 - (2 2 cos x) = 8 - ç - 1÷
y6 ç y2 ÷
è ø
( y ¢) 2 8 y 2 - (1 - y 2 ) 2
=
y6 y4
( y ¢) 2 = 8 y 4 - y 2 (1 - y 2 ) 2 then diff.
61. f( x) = x satisfy the equation.
\ f(5) = 5
é x x£0
62. f( x) ê x 2 0< x<1
ê2 x - 1 x³1
ë
é1 x£0
f ¢( x) ê2 x 0 < x < 1
êë 2 x³1
f( x) is not derivable at x = 0.
63. y = ( x + 1 + x 2 ) n
dy ny
=
dx 1+ x2
Continuity, Differentiability and Differentiation 53

é 2 yx ù
2 ê 1 + x y¢ - ú 2
d y 1+ x2 ú Þ (1 + x 2 ) d y + x dy = n 2 y
=nê
2 2
dx ê 1+ x ú dx 2 dx
ê ú
ë û
æ ö æ 2ö æ 2 ö
x ÷ = ç 1 - æç x - 1 - x 2 ö÷ ÷ × ç x + 1 - x ÷
64. g¢( x) = f ¢( x - 1 - x 2 ) × ç 1 +
ç ÷ ç è ø ÷ø ç ÷
è 1- x2 ø è è 1- x2 ø

= 2 x æç x + 1 - x 2 ö÷
è ø
f( x + h) - f( x) æ f( h) - 1 ö
66. f ¢( x) = lim = lim f( x) ç ÷ = f( x) × f ¢(0) = 3 f( x) (Q f ¢(0) = 3)
h ®0 h h ®0 è h ø
log e (2 + x) - x 2n sin x
67. f( x) = lim
n ®¥ 1 + x 2n
ìln(2 + x) | x|< 1
ï- sin x | x|> 1
ïïln 3 - sin 1
f( x) = í x =1
ï 2
ïsin 1 x = -1
ïî 2

x - e x + 1 - {1 - cos 2 x} x - e x + 1 - 1 + cos 2 x
68. lim Þ lim
x® 0 x2 x® 0 x2
x - e x + cos 2 x 1+ x - ex (cos 2 x - 1) 5
Þ lim = lim + =-
x® 0 2 x® 0 2 2 2
x x x
69.
y = |x2–10x+21|

(0,21)

(0,4)
x
0 (3,0) (7,0)

71. xy = const.
y
y + xy ¢ = 0 Þ y ¢ = -
x
72. f( x) = -1 + | x - 2| is a continuous function.
54 Solution of Advanced Problems in Mathematics for JEE

g( x) = 1 -| x| is a continuous function.
Þ f( g( x)) is a continuous function.
f( k + h) - f( k)
73. f ¢( K + ) = lim
h ®0 h
K tan( pk + ph) - k tan kp
= lim
h ®0 h
æ tan ph ö
= lim k ç ÷ = kp
h ®0 è h ø
ae sin x + be - sin x - c
74. lim =2
x ®0 x2
a + b- c =0 …(1)
Applying L Hospital Rule,
ae sin x × cos x - be - sin x × cos x
lim =2 Þ a = b
x ®0 2x
75. tan x = sec a × tan y
sec 2 x = sec a × sec 2 y × y ¢
æp pö
y ¢ = 1 at ç , ÷
è4 4ø
2 sec 2 x tan x = sec a (sec 2 y × y ¢¢ + 2 sec 2 y × tan y × ( y ¢) 2 ) Þ y ¢¢ = 0

76. We gave, y = ( x 2 - 9)( x 2 - 4)( x 2 - 1) x

= { x 6 - 14 x 4 + x 2 ( 49) - 36} x

= x 7 - 14 x 5 + 49 x 3 - 36 x
dy
Therefore, = 7 x 5 - 70 x 4 + 147 x 2 - 36
dx
d2 y
Thus, = 42 x 5 - 280 x 3 + 294 x
2
dx
2
d y
= 42 - 280 + 294 = 56
dx 2 x =1

f( x + h) - f( x) f( x) f( h) - f( x) æ f( h) - 1 ö
77. f ¢( x) = lim = lim = lim f( x) ç ÷
h ®0 h h ®0 h h ®0 è h ø
Þ f ¢( x) = f ¢(0), f( x) Þ f( x) = e kx (where k = f ¢(0))
Continuity, Differentiability and Differentiation 55

1 1
78. f( g( x)) = x ; f ¢( g( x)) g¢( x) = 1 Þ g¢(6) = =
f ¢( g(6)) f ¢(0)
dy dy dx f ¢( x)
79. = =
dz dz dx g¢( x)
d æ dy ö
2 ç ÷
d y d æ dy ö dx è dz ø g¢f ¢¢ - f ¢g¢¢
= ç ÷ = =
dz 2 dz è dz ø dz ( g¢) 3
dx
f(x)

80.

ì f( x) + 1 = x + 2 , x Î ( -¥, - 1) x = -1, 1non differentiable


ï
g( f( x)) = í( f( x) - 1) 2 = ( x + 1 - 1) 2 = x 2 , x Î ( -1, 0)
ï 2
î(| x - 1| - 1) , x ³ 0
81. f( x) = [sin x] + [cos x]

O p/2 p 3p/2 2p

ìcos x , x Î [0 , p]
82. g( x) = í
îsin x - 1 , x > p
g( p - ) = g( p) = g( p + ) = -1
but not differentiable at x = p.
¥
f r (0) f(0) f ¢(0) f ¢¢(0)
83. å r!
=
0!
+
1!
+
2!

r =0

n × 4 n -1 n(n - 1) × 4 n -2
= 4n + + +¼
1! 2!
= n C 0 4 n + n C 1 4 n -1 + n C 2 4 n -2 +¼

= ( 4 + 1) n = 5 n
56 Solution of Advanced Problems in Mathematics for JEE

x
84. f( x) = x £ -1
1- x
x
= -1 < x < 0
1+ x
x
= 0£ x<1
1- x
x
= x³1
1+ x
Function is discontinuous at x = -1, 1
f( x) is not differentiable at x = -1, 1
85. f( g( x)) = x
æ -7 ö 1 1
f ¢( g( x)) g¢( x) = 1 Þ g¢ ç ÷= =
è 6 ø æ æ -7 ö ö f ¢(1)
f ¢ç gç ÷÷
è è 6 øø
86. f( x) = 0 x³0
2 2
= 4 x (1 - 2 x) x<0
Differentiable everywhere.
88. f( x) is discontinuous at x = 1, 2
x=1
Þ g( x) = x 2 - ax + b = 0
x=2

89. f -1 ( f( x)) = x

( f -1 ( f( x))) ¢ f ¢( x) = 1

( f -1 ( f(9))) ¢ f ¢( g) = 1
1 1
( f -1 (3)) ¢ = =
f ( 9) 5
¢
1
90. f(0 + ) = lim h n sin =0 Þ n > 0
h ®0 h
æ 1ö
f(0 - ) = lim ( -h) n sin ç - ÷ = 0 Þ n > 0
h ®0 è hø
1 1
f ¢( x) = n × x n -1 × sin - x n -2 × cos = finite Þ n = 2
x x
Continuity, Differentiability and Differentiation 57

Exercise-2 : One or More than One Answer is/are Correct

1. f( x) has exactly one point of discontinuity so that sgn ( x 2 - lx + 1) is equal to zero for some
values of l.
\ D =0
2
Þ l - 4 =0
Þ l = ±2
2. Answer from the graph.
y

2–x x–2
–1
x
1 2

2(x+1)

x(3 e1/ x + 4) æ4ö


3. (a) L.H.L. = lim =0ç ÷ =0
1/ x
x ® 0- 2-e è2ø
x(3 e1/ x + 4) æ 3 + 4 e -1/ x ö
÷ = 0 æç 3 ö÷ = 0
R.H.L. = lim = lim x ç
x ® 0+ 2 - e 1/ x + ç -1 / x
- 1 ÷ø è -1 ø
x ®0 è 2e
f(0) = 0
\ f( x) is continuous at x = 0.
æ 3 e 1/ x + 4 ö
ç ÷
f ( x ) - f ( 0) ç 2 - e 1/ x ÷
+ è ø
(b) f ¢(0 ) = lim = lim x
x ®0 + x - 0 x ®0 + x
3 + 4 e -1/ x
= lim = -3
x ® 0+ 2 e -1/ x - 1
f ( x ) - f ( 0) 3 e 1/ x + 4 4
f ¢(0 - ) = lim = lim = =2
x ® 0- x -0 x ® 0- 2 - e 1/ x 2
f ¢(0 + ) ¹ f ¢(0 - )
(c) f ¢(0 + ) = -3
(d) f ¢(0 - ) = 2 exist
4. Given | f( x)| £ sin 2 x
Clearly | f(0)| £ 0 Þ f(0) = 0
58 Solution of Advanced Problems in Mathematics for JEE

lim | f( x)| = lim f( x) = 0


x® 0 x® 0

f ( x ) - f ( 0) f( x)
| f ¢(0)| = lim = lim £0
x® 0 x x ® 0 x
é æ x 3 ö÷ ù æ x 2 x 4 ö÷
a ê1 - x ç x - ¼ ú + bç 1 - + ¼ +5
ç 3 ! ÷ ç 2 ! 4 ! ÷
ê
ë è ø ú
û è ø
5. f(0 - ) = lim = f ( 0)
- 2
x® 0 x
æ bö
( a + b + 5) - ç a + ÷ x 2 +¼
è 2ø
= lim =3
- 2
x® 0 x
a + b+ 5 =0 …(1)
æ bö
-ç a + ÷ = 3
è 2ø
2a + b = 6 …(2)
On solving (1) and (2),
a + b+ 5 =0
2a + b + 6 = 0
– – –
-a -1=0
a = -1
b = -4
\ a + b = -5
+
f ¢(0 ) is exist when c = 0
lim (1 + dx) 1/ x = 3
x® 0
1
lim ( dx )
x®0 x
e =3
d
Þ e =3
d = ln 3
2
7. (a) f( x) = 3 x | x| - 1 -| x|

But x 2| x| =| x|3
So, f( x) =| x| - 1 -| x| = -1 is every where differentiable.
So, no where non-differentiable.
Continuity, Differentiability and Differentiation 59

(b) lim [ x(tan -1 ( x + 1) - x tan -1 ( x + 1))] + [5 tan -1 ( x + 1) - tan -1 ( x + 1)]


x ®¥
æpö
= lim 4 tan -1 ( x + 1) = 4 ç ÷ = 2 p
x ®¥ è2ø
æ 1 ö
(c) f( - x) = sin æç ln æç - x + x 2 + 1 ö÷ ö÷ = sin ç ln ÷
è è øø ç 2 ÷
è x + x + 1ø
= sin æç - ln æç x + x 2 + 1 ö÷ ö÷ = - sin æç ln æç x + x 2 + 1 ö÷ ö÷
è è øø è è øø
= - f( x)
So, f( x) is an odd function.
4 - x2
(d) f( x) = is discontinuous at where denominator is zero, 4 x - x 3 = 0
4x - x3
Þ x = 0, x = ± 2
\ a, b, c only correct.
8. g¢( x) = ae ax + f ¢( x) Þ g¢(0) = a - 5
g¢¢( x) = a 2 e ax + f ¢¢( x)
g¢¢(0) = a 2 + 3
Þ a 2 + a - 2 = 0 ; a = -2 , 1
10. f(0 + ) = f(0) = f(0 - ) = 0
11. ò f ¢( x) dx = ò f ¢( - x) dx
Þ f( x) + f( - x) = c
12. | f( x)| £ x 4n
Þ f(0) = 0
lim ( -h 4n ) £ lim f(0 + h) £ lim ( h) 4n Þ f(0 + h) = 0
h® 0 h® 0 h® 0

lim ( - ( -h) 4n ) £ lim f(0 - h) £ lim ( -h) 4n Þ f(0 - h) = 0


h® 0 h® 0 h® 0

Þ f( x) is continuous at x = 0.
f(0 + h) - f(0) é -h 4 n f(0 + h) - f(0) h 4n ù
f ¢(0 + ) = lim =0 ê lim £ lim £ lim ú
h® 0 h ëh ® 0 h h® 0 h h® 0 h
û
Þ f( x) is differentiable at x = 0.
13. g( x) = 0 xÎI
2
=x xÏI
gof( x) = 0 for x Î R
60 Solution of Advanced Problems in Mathematics for JEE

14. If f( x) is continuous at x = 2 then 3 p + 10 q = 4


f( x) is differentiable at x = 2 then 2 p + 11q = 4
16. f( x) = x 2 -2 £ x £ 0
=x 0< x<1
3
=x 1£ x £ 2
f( x + h)
-1
f( x + h) - f( x) f( x)
17. f ¢( x) = lim = f( x) lim
h® 0 h h® 0 h
æ hö
f ç1 + ÷ - 1
è xø
= f( x) lim
h® 0 h
f( x)
So, f ¢( x) = × f ¢(1)
x
ln( f( x)) = 3 ln x + ln c
f( x) = cx 3
f(1) = 1 so c = 1
f( x) = x 3
So, we can check options.
18. f( x) = ( x - 1)( x - 2)( x + 1)( x + 2) = ( x 2 - 1)( x 2 - 4)
f ¢( x) = ( x 2 - 1) 2 x + ( x 2 - 4)(2 x) = 2 x(2 x 2 - 5) = 0
5
x = 0, ±
2
19. If f( x) is continuous at x = 2 then 3 p + 10 q = 4
f( x) is differentiable at x = 2 then 2 p + 11q = 4
3
20. y = e x sin x + e x ln (tan x )
dy 3 æ 1 ö
= e x sin( x ) [ x cos( x 3 )3 x 2 + sin( x 3 )] + e x ln(tan x ) ç ln(tan x) + x sec 2 x ÷
dx è tan x ø
3
y ¢ = e x sin( x ) [3 x 3 cos( x 3 ) + sin( x 3 )] + (tan x) x (ln(tan x) + 2 x cosec 2 x)
21. f( x) = 1 - (1 - x) + (1 - x) x 2 + (1 - x)(1 - x 2 ) x 3 +¼ + (1 - x)(1 - x 2 )¼(1 - x n -1 ) x n
n
= 1 - (1 - x)(1 - x 2 )(1 - x 3 ) ¼(1 - x n ) = 1 - Õ(1 - x r )
r =1
n
( f( x) - 1) = - Õ(1 - x r )
r =1
Continuity, Differentiability and Differentiation 61

22. \ f and g must be continuous.


1+ a =2 + b
Þ a =1+ b
3 + b = 1 Þ b = -2
a = -1
ax3+b ; 0£x£1
23. f(x)
2cos px + tan–1x ; 1<x£2
is must be continuous and differentiable at x = 1.
p
\ a + b = -2 + …(1) (continuity)
4
1
3a = 0 + …(2) (By differentiable)
2
We get, a and b
24. f( f( x)) = 2 + x 0£ x£1
=2 - x 1< x £ 2
=4-x 2< x£3
25. ln( f( x)) = ln( x + 1) + ln( x + 2) +¼¼ + ln( x + 100)
f ¢( x) 1 1 1
= + +¼¼ +
f( x) x + 1 x + 2 x + 100
f( x) f ¢¢( x) - ( f ¢( x)) 2 æ 1 1 1 1 ö
= -ç + + +¼¼ + ÷
( f( x)) 2 ç ( x + 1) 2 ( x + 2) 2 ( x + 3) 2 ( x + 100) 2 ÷
è ø
if g( x) = f( x) f ¢¢( x) - ( f ¢( x)) 2 = 0
1 1 1
Þ + +¼¼ + =0
2 2
( x + 1) ( x + 2) ( x + 100) 2
Þ g( x) = 0 has no solution.
26. h( x) = -1 x<1
=| x - 2| + a + 2 -| x| 1£ x < 2
=| x - 2| + a + 1 - b x³2
if h( x) is continuous at x = 1, then a = -3
if h( x) is continuous at x = 2, then b = 1
27. lim f( x) = 1 = lim f( x)
x ® 0- x ® 0+
Clearly, C = 1 and use L’ Hospital’s rule.
28. Differentiable w.r.t. ‘ x’
2 f( x) f ¢( x) + 2 y = 2 f( x + y) f ¢( x + y)
62 Solution of Advanced Problems in Mathematics for JEE

put x = 0
k + y = f ¢( y) f( y)
integrate on both sides,
y2 f 2 ( y)
ky + = +c …(1)
2 2
put x = y = 0 in given equation, we get
f 2 ( 0) = 2
f(0) = 2 as ( f( x) > 0)
put y = 0 in (1)
1 + c = 0 Þ c = -1
also put y = 2
4
k 2 + 1= -1
2
k 2 =0
k =0
2
y f 2 ( y)
\ = -1
2 2
f 2 ( y) = y 2 + 2

f( y) = y2 + 2

f( x) = x 2 + 2
Hence, we can answer.
f( x + h) - f( x) f( x + h) - f( x)
30. f ¢( x) = lim = lim
h ®0 h h ® 0 h
f( h)
= lim + x = x + f ¢(0) ; f ¢( x) = x 2 - 1
2 2
h ®0 h
1
32. f(1- ) = f(1+ ) = f(1) =
2
f ¢( x) = x 0£ x<1
= 4x - 3 1£ x £ 2
f ¢¢( x) = 1 0£ x<1
=4 1£ x £ 2
34. gof( x) = 0
fog( x) = 0 xÎI
2
= [x ] xÏI
Continuity, Differentiability and Differentiation 63

35. f( g( x)) = x
f ¢( g( x)) g¢( x) = 1
1
g¢( x) =
f ¢( g( x))
1 1 1
g¢( e) = = =
f ¢( g( e)) f ¢(1) e + 1
-1
g¢¢( x) = f ¢¢( g( x)) × g¢( x)
( f ¢( g( x))) 2
-1
g¢¢( e) = f ¢¢(1) × g¢( e)
( f ¢(1)) 2
36. f(2 + ) = lim [ x - 1] = 1
x ® 2+

3x - x2
f(2 - ) = lim =1
x ® 2- 2
f(3 - ) = lim [ x - 1] = 1
x ® 3-

f(3 ) = lim ( x 2 - 8 x + 17) = 2


+
x ® 3+

Exercise-3 : Comprehension Type Problems

Paragraph for Question Nos. 1 to 2


2
tan(ln(sec( x n))) (ln(sec( x n) - 1) + 1) sec( x n) - 1 æ x ö
2. f( x) = lim n 2 ´ ´ ´ç ÷
n ®¥ ln(sec( x n)) sec( x n) - 1 ( x n) 2 ènø
x2
f( x) =
2

1
|
||||

g(x)
|||

|||| ||
||||||||||||||||
–1 1 2

Paragraph for Question Nos. 3 to 4


4. f ¢( x) = 2 x + g¢(1)
f ¢¢( x) = 2 g¢(1) = 2 f(1) + 2 + f ¢(1)
f ¢(1) = 2 - 3 = -1 Þ f(1) = -2
64 Solution of Advanced Problems in Mathematics for JEE

f( x) = x 2 - 3 x g¢¢(2) = 2( -2) + 2(2) = 0


-2 = 1 + g¢(1)
2
g( x) = -2 x + x(2 x - 3) + 2 g¢(1) = -3
= -3 x + 2
f(1) + g( -1) = -2 + (3 + 2) = 3

Paragraph for Question Nos. 5 to 6


5. Clearly, 3 is non-repeated root where as 1 is repeats and also ( x - 2) 1/ 3 is not diff. at x = 2.
\ at 3, 2 is non-diff. and sum is 5.
6. h( x) is continuous.
x -1= x2 - x -2
x2 - 2x - 1 = 0
( x - 1) 2 = 2
x =1± 2
3p æpö
tan = 1 + 2 , tan ç ÷ = 2 - 1
8 è8ø
7p
tan =1- 2
8
2 - 1 is not differentiable.

Paragraph for Question Nos. 7 to 8

Sol.
–2 –1 1 2

Paragraph for Question Nos. 9 to 10

10. 0
x
p/2 p 3p/2 2p
–1
Continuity, Differentiability and Differentiation 65

Paragraph for Question Nos. 11 to 13


é 0 0£ x<1
ê x 1£ x < 2
11. f( x) = 2( x - 1) 2 £ x < 3
ê
êë3( x - 1) x =3
No. of values where f( x) is discontinuous = 2
12. f( x) is non-differentiable at x = 1, 2 , 3.
13. No. of integers in the range of f( x) = 5

Paragraph for Question Nos. 14 to 16


f( x) f( h) - f( x)
Sol. f ¢( x) = lim = f ¢(0) f( x)
h® 0 h
Þ f( x) = e 2 x ( f ¢(0) = 2)
2
g( x) = x

Paragraph for Question Nos. 17 to 18


(1 - cos x)(l - f( x))
Sol. g¢( x) = l sec 2 x + (1 - l) cos x - 1 =
f( x)

Paragraph for Question Nos. 19 to 21


x2
Sol. f( x) = x<0 f(x)
x2 + 1 2
=1 x =0 1
x
=2 x>0 0

Paragraph for Question Nos. 22 to 24


Sol. f( x) = g¢(1) sin x + ( g¢¢(2) - 1) x
æpö
Þ f ¢( x) = g¢(1) cos x + g¢¢(2) - 1 Þ f ¢ ç ÷ = g¢¢(2) - 1
è2ø
æpö
f ¢¢( x) = - g¢(1) sin x Þ f ¢¢ ç ÷ = - g¢(1)
è2ø
æpö æ pö æpö
g( x) = x 2 - f ¢ ç ÷ × x + f ¢¢ ç - ÷ Þ g¢( x) = 2 x - f ¢ ç ÷
è2ø è 2ø è2ø
g¢¢( x) = 2 Þ g¢¢(2) = 2
2
f( x) = sin x + x and g( x) = x - x + 1
66 Solution of Advanced Problems in Mathematics for JEE

Paragraph for Question Nos. 25 to 26


f( x) + f( h)
- f( x)
f( x + h) - f( x) 1 + f( x) f( h)
Sol. f ( x) = lim
¢ = lim
h ®0 h h ®0 h
(1 - ( f( x)) 2 )
f ¢( x) = lim f( h)
h ®0 h
f ¢( x) = lim f ¢(0)(1 - f( x) 2 ) ( f ( 0) = 0)
h ®0

e2x - 1
Þ f( x) =
e2x + 1
f ¢( x) ³ 0 " x Î R
-2 x
lim
x x ® 0 ( e 2x + 1 )
lim ( f( x)) = e =1
x ®0

Paragraph for Question Nos. 27 to 28


Sol. f( x) = 3( x + 6)( x + 1)( x - 2)( x - 3) + x 2 + 1 then k why!
3( x + 1)( x - 2)( x - 3)( x + 6) 6!
27. lim =-
x ®-6 x +6 2
1
28. g( x) =
-3( x + 6)( x + 1)( x - 2)( x - 3)

Paragraph for Question Nos. 29 to 30


Sol. f( x) = g( x)
x ln x = e 2 x
(ln x) 2 = 2 + ln x
1
x = a = , b = e2
e
f( x) - cb f ¢( x)
29. lim = =4
x ® e g( x ) - b 2
2 g¢( x)
c = e2
g(a ) f ¢(a ) - g¢(a ) f(a ) e( -2 e 2 ) - e 2 - e
30. h¢(a ) = = = -3e
g 2 (a ) ( e) 2
Continuity, Differentiability and Differentiation 67

Exercise-4 : Matching Type Problems


p
log(sin x)
1. (A) Let I = ò dx
0 cos 2 x
p/ 2 é p/ 2 ù é p/ 2 ù
log(sin x) 2 p/ 2 cos x
=2 ò dx = 2 ê log(sin x) sec x dx ú = 2 êlog(sin x) tan x|0 -
ò ò tan x dx ú
0 cos 2 x êë 0 úû êë 0
sin x úû
p/ 2
æpö
= 2(0 - 0) - 2 ò dx = 0 - 2 çè 2 ÷ø = -p = -k
0

Þ k=p
3k
\ = 3 > 0 , 1, 2
p
(B) e x + y + e y - x = 1
e x + e-x = e-y
e x - e - x = e - y ( - y ¢)
e x + e - x = e - y ( - y ¢¢) + e - y ( y ¢) 2
e - y = e - y ( - y ¢¢) + e - y ( y ¢) 2 Þ y ¢ - ( y ¢) 2 + 1 = 0
\ k =1
-1
(C) Let f =g
g{ f( x)} = x Þ ( g¢f( x)) f ¢( x) = 1
g¢(2 ln 2) f ¢(2) = 1
1
g¢2(ln 2) =
1 + ln 2
2
2( f -1 ) ¢(ln 4) = > 0, 1
1 + ln 2
1
2
(D) l = lim ( x ln x) x +1
x® ¥
1 1 1
+ ×
ln x + ln(ln x)
ln l = lim = lim x ln x x = 0
x® ¥ x2 + 1 x® ¥ 2x
ln(l) = 0 Þ l = 1
sec 2, –2 £ x < –1
2. g(f(x)) sec 1, –1 £ x < 0
sec x, 0£x£2
68 Solution of Advanced Problems in Mathematics for JEE

f(g(x)) 2

–4p/3 –p –2p/3 –p/3 p/3 2p/3 p 4p/3


–1

–2

3. (A) f(1+ ) = f(1- ) = -1


3 3 3
æ x2 ö -3
(B) ò ([ x] × { x} -| x|) × dx = (2( x - 2) - x) dx = ç
ò - 4x÷ =
ç 2 ÷
2 2 è ø2 2
(C) [ x] × { x} = -1 x£0
1 1
x = -3 + , - 2 +
3 2
(D) l = lim f( x) = lim ([ x]{ x} -| x|) = -4
x ® 4+ x ® 4+

2 x +1
æ x 2 + 2 x - 1 ö 2 x -1 1
4. (A) lim ç ÷ =
x ®¥ ç 2 x 2 - 3 x - 2 ÷ 2
è ø
2
log sec x / 2 cos x æ ln cos x ö
(B) lim = lim (log x cos x) 2 = lim çç ÷ =2
x ®0 x x ®0 sec x ® 0è ln sec x / 2 ÷
ø
log sec x cos 2
2

p 3p/2
(C) x
0 p/2

1 2 3
(D) sin x ¹ , ,
3 3 3
5. f(1+ ) = f(1- ) = f(1) Þ b = 0
f ( 3 - ) = f ( 3 + ) = f ( 3)
3 = 9 p + 3q + 2 Þ 3 p + q = 0
f ¢( x) = 2 ax - a x<1
=1 1£ x < 3
= 2 px + q x > 3
Continuity, Differentiability and Differentiation 69

f ¢(3 + ) = f ¢(3 - ) = f ¢(3)


1
6p + q = 1 Þ p = , q = -1
3
f ¢(1+ ) ¹ f ¢(1- )
a¹1

Exercise-5 : Subjective Type Problems

1. f( x) is discontinuous at x = 1,| f( x)| is diff. every where


f(1) = - f(1+ ) = - f(1- )
Þ 3 = - (0 + b)
Þ b = -3
f ¢(1+ ) = - f ¢(1- ) (as | f( x)| is differentiable every where)
\ 1 = - (2 a - a) Þ a = -1
Continuous at x = 3,
So, 5 = 9 p + 3q + 2
Þ 3p + q = 1 …(1)
f ¢( x) is continuous at x = 3
So, f ¢(3 - ) = f ¢(3 + )
Þ 6p + q = 1 …(2)
On solving (1) & (2) we get, p = 0 , q = 1
So, |a + b + p + q| =|- 1 - 3 + 0 + 1| = 3
2. sin -1 y = 8 sin -1 x
y¢ 8
=
2
1- y 1- x2
(1 - x 2 )( y ¢) 2 = 64(1 - y 2 )
(1 - x 2 ) y ¢¢ - xy ¢ = -64 y
3. yy ¢ = 4 a
( y ¢) 2 + yy ¢¢ = 0
4. f( x) is discontinuous at x = - 3 , - 2 , 2 , 3 , 5 , 6 , 7 , 8
sin px = 0 at x = -2 , - 1, 0 , 1, 2 , 3
So, continuous at these points.
5. Let f ¢( x) = K
Þ f( x) = Kx + c
70 Solution of Advanced Problems in Mathematics for JEE

Þ f(9) - f( -3) = 12 K
Maximum value of f(9) - f( -3) = 96
6. g( x) = sin x 3 - x 3 + 1 x³1
3 3
= sin x + x - 1 0£ x<1
= - sin x 3 - x 3 - 1 -1 £ x < 0
3 3
= - sin x + x + 1 x £ -1
Function is not differentiable at x = -1, 1
7. F ( x) = g( x) x>1
f( x) + g( x)
= x =1
2
= f( x) -1 < x < 1
f( x) + g( x)
= x = -1
2
= g( x) x < -1
If F ( x) is continuous at x = 1
F (1+ ) = F (1) = F (1- )
b=a+3
If F ( x) is continuous at x = -1
F ( -1- ) = F ( -1) = F ( -1+ )
a + b=5
8. f -1 ( x) = 2 - x 2£ x£5
=2 + x -2 < x < 2
9. f( x) + 2 f(1 - x) = x 2 + 2
( x - 2) 2
f (1 - x) + 2 f ( x) = (1 - x) 2 + 2 Þ f ( x) =
3
10. g( x) = x( x - 3)( x - 7)
f( g( x)) = sgn( x( x - 3)( x - 7))
d2
11. (sin 2 x - sin x + 1) = -4 sin 2 x + sin x + 2
dx 2
12. f( x) = a cos( px) + b
f ¢( x) = -ap sin( px)
3/ 2
2a 2
ò f( x) dx = -
p
+ b = + 1 Þ a = -1, b = 1
p
1/ 2

13. a ¢( x) = f ¢( x) - 2 f ¢(2 x) ; b ¢( x) = f ¢( x) - 4 f ¢( 4 x)
a ¢(1) = f ¢(1) - 2 f ¢(2) = 5
Continuity, Differentiability and Differentiation 71

a ¢(2) = f ¢(2) - 2 f ¢( 4) = 7
b ¢(1) = f ¢(1) - 4 f ¢( 4) = a ¢(1) + 2a ¢(2) = 5 + (2 ´ 7) = 19
b ¢(1) - 10 = 19 - 10 = 9
14. g( f( x)) = x
g¢( f( x)) f ¢( x) = 1
f(1) = - 7 6
\ x =1
æ 7ö
g¢ ç - ÷ f ¢(1) = 1
è 6ø
æ 7ö 1
g¢ ç - ÷ =
è 6 ø f ¢(1)
1- x
2 æ 1ö 2
f ¢( x) = -4 × e ç- ÷ + x + x + 1
è 2ø
f ¢(1) = 2 + 1 + 1 + 1 = 5
5 1
-
h( x) = ax 4 + bx 4
-9 -3
5a 4 b 4
h¢( x) = - x + x
4 4
5a -9/ 4 b -3/ 4
h¢(5) = 0 Þ - ×5 + ×5 =0
4 4
Þ 5a × 5 -3/ 2 = b
a
Þ = 51/ 2
b
2
æ aö
ç ÷ =5
è bø
a2 5
= =5
2 æ -7 ö 1
5b g ç
¢ ÷ 5 ´
è 6 ø 5
æ x ö
16. Let lim ç f( x) + ò f(t) dt ÷÷ = l …(1)
x® ¥ ç
è 0 ø
æ x ö
ç e x f(t ) dt ÷
ç ò ÷
è 0 ø
lim =l
x® ¥ x
(e )
72 Solution of Advanced Problems in Mathematics for JEE

x
ex ò f(t) dt
0
Þ lim =l
x® ¥ ex
x
Þ lim ò f(t) dt = l …(2)
x® ¥
0

From (1) and (2) we get, lim f( x) = 0


x® ¥

17. f(0) = 0 , f ¢(0) = 1, f ¢¢(0) = 1, f ¢¢¢(0) = 2


g( f( x)) = x Þ g¢( f( x)) f ¢( x) = 1
- f ¢¢( x)
Þ g¢¢( f( x)) =
( f ¢( x)) 3
é( f ¢( x)) 3 × f ¢¢¢( x) - 3 ( f ¢¢( x)) 2 ( f ¢( x)) 2 ù
Þ g¢¢¢( f( x)) f ¢( x) = - ê ú
êë ( f ¢( x)) 6 úû
Put x = 0
é 1 ´ 2 - 3 ´ 1ù
g¢¢¢(0) 1 = - ê úû = 1
ë 1
æ æ h öö
f ç x ç 1 + ÷ ÷ - f( x)
f( x + h) - f( x) è è x øø
19. f ¢( x) = lim = lim
h ®0 h h ®0 h
æ hö
f ç1 + ÷
f( x) è xø æ hö æ hö
+ - f( x) f( x) ç - ÷ f ç 1 + ÷
1+ h x x è x ø è xø
= lim = lim +
h ®0 h h ®0 æ hö hx
hç1 + ÷
è x ø
æ hö
f ç 1 + ÷ - f(1)
- f( x) è xø
= + lim (as f(1) = 0)
x h ®0 2æ h ö
x ç ÷
è xø
- f( x) f ¢(1)
f ¢( x) = +
x x2
1
xf ¢( x) + f( x) =
x
d 1
( xf( x)) =
dx x
1
xf( x) = ò dx
x
xf( x) = ln x + k
Continuity, Differentiability and Differentiation 73

Put x = 1, we get k = 0
ln x
\ f( x) =
x
1 x
H( x) = =
f( x) ln x
ln x × 1 - 1 – +
H ¢( x) = H( x) ³ e
2 0 e
(ln x)
H( e) = e
é 1 ù
\ lim ê ú =2
x ® e ë f( x) û

2x2
21. f ¢( x) = tan -1 ( x 2 ) + + 4x3
4
1+ x
æ (1 + x 4 ) × 2 x - x 2 ( 4 x 3 ) ö
2x
+ 2ç
f ¢¢( x) = ÷ + 12 x 2
1+ x4 ç ( 1 + x 4 2
) ÷
è ø
dy dy dq 3
22. = = -3 sin q cos q = - sin 2q
dx dx dq 2
d2 y -3 cos 2q
=
2 sin q
dx
t 2 + 16 + 8t t 2 + 16 - 8t |t + 4| + |t - 4|
23. Let 8 x - 16 = t 2 Þ + =
8 8 2 2
24. f( x) = [ x] 0< x<1
5
= { x} 1£ x <
4
3 5
= x- £ x<2
2 4
No. of points where f( x) is non-differentiable are three.
5 3
x = 1, ,
4 2

❑❑❑
74 Solution of Advanced Problems in Mathematics for JEE

4
APPLICATION OF DERIVATIVES

Exercise-1 : Single Choice Problems

1. Maximum value of f( x) = 3
Minimum value of f( x) = -1
2. f ¢¢( x) = 6 x - 6
f ¢( x) = 3 x 2 - 6 x + 3 (Q f ¢(2) = 3)
f( x) = x 3 - 3 x 2 + 3 x - 1 (Q f(2) = 1)
4 4
5. V = p(10 + T ) 3 - p(10) 3
3 3
dV dT
= 4 p(10 + T ) 2
dt dt
dT 1
Þ = (QT = 5 cm)
dt 18p
(| x| - 1)(| x| - 2)
6. g( x) =
(| x| - 3)(| x| - 4)
g( x) is an even function so there is an extrema at x = 0.
Also number of extrema for x > 0 will be equal to number of extrema for x < 0
for x > 0
( x - 1)( x - 2)
g( x) =
( x - 3)( x - 4)
Number of extrema = 2
Þ Total extrema = 5
Application of Derivatives 75

2
æ 45 ö 75 2
7. A ¢B ¢ = ç 700 - t÷ + t B
è 2 ø 4

/s

m
( A ¢B ¢) min at t = 30 sec

5
60° A¢ 20 m/s
O A
- 700 m
8. f(0 ) ³ f(0) Þ a ³ 3
3+k–x, x£k
9. f(x) Þ f ¢( k + ) > f( k), f ¢( k - ) > f( k)
sin(x–k) ,
a2–2+ x>k
x–k
sin( x - k)
So, lim ( a 2 - 2) + = a2 - 1> 3
x ®k + ( x - k)
2
a >4
|a| > 2
dy
10. = 3 x 2 - 4 x + C1
dx
y = x 3 - 2 x 2 + C1 x + C 2
dy ù
Also, = 0 and y]at x = 1 = 5
dx úû at x = 1
dy ù
11. m 1 = = 2a + b
dx úû at (1, 2)
dy ù 1
m 2 = g¢( x) = = 2 Þ 2a + b = -
dx úû at ( -2, 2) 2
7
Also, 2 =a+ b+
2
dy
12. 18 y = 3x2
dx
dy 3 x 2
=
dx 18 y
a2
Þ = 1 Þ a 2 = 6b
6b
Also, 9 b 2 = a 2
dy
13. = 3x2 - 4x + c
dx
dy
at x = 1, =0 Þ c =1
dx
76 Solution of Advanced Problems in Mathematics for JEE

dy
= 3x2 - 4x + 1
dx
y = x3 - 2x2 + x + d
at x = 1, y = 5 Þ 5 =1-2 + 1+ d
Þ d =5
14. A(0 , 2)
dy dy -15a 2 x 2 - 20ax - 1
5a 2 (3 x 2 ) + 10a(2 x) + 1 + 2 =0 Þ =
dx dx 2
dy 1
Þ at A = -
dx 2
Equation of normal at A is y = 2 x + 2
Let normal meets the curve at B
5a 2 x 3 + 10ax 2 + x + 4 x + 4 - 4 = 0
5 x(ax + 1) 2 = 0
1
x =-
a
æ -1 -2 ö
So, B ç , + 2÷
è a a ø
-15 + 20 - 1
\ Slope of tangent at B = =2
2
1 1
15. f( x) = cos x + cos 2 x - cos 3 x
2 3
f ¢( x) = - sin x - sin 2 x + sin 3 x = 2 sin x(2 cos x + 1)(cos x - 1) = 0
16. Closest distance exist always alone the normal
1 - x dy
\ ´ = -1 A(2,1)
2 - x dx
1- x 1 P(x, x )
´ = -1
2-x 2 x
Let x =t
2+ 3
x=
2
17. Let x = 2 sin q
æ 2 + 2 cos q ö æ 2 cos 2 q 2 ö
y = ln ç ÷ - 2 cos q = ln ç ÷ - 2 cos q
è 2 - 2 cos q ø ç 2 sin 2 q 2 ÷
è ø
æ qö
= 2 ln ç cot ÷ - 2 cos q
è 2ø
Application of Derivatives 77

dy 1 æ 2 qö
= ç -cosec ÷ + 2 sin q
dq cot q 2 è 2ø
dy -2 -2 cos 2 q
= + 2 sin q =
dq sin q sin q
dx dy
= 2 cos q; = -cot q
dq dx
æ æ qö ö
çç y - 2 ln ç cot ÷ + 2 cos q ÷÷ = - cot q ( x - 2 sin q)
è è 2ø ø
æ æ qö ö
\ T = çç 0 , 2 ln ç cot ÷ - 2 cos q + 2 cos q ÷÷
è è 2ø ø
æ q ö
P = ç 2 sin q , 2 ln cot - 2 cos q ÷
è 2 ø
PT 2 = ( 4 sin 2 q + 4 cos 2 q ) = 4
18. g¢( x) = (2 x 2 - ln x) f( x)
1 1
f ¢( x) = 3x2 - 2x
3
ln x ln x 2
x2 - x
f ¢( x) =
ln x
x( x - 1)
f ¢( x) = > 0 " x > 1; f( x) > f(1) Þ f( x) > 0 " x > 1
ln x
For g( x) is increasing
g¢( x) > 0 Þ 2 x 2 - ln x > 0 as ( f( x) > 0)
Let H( x) = 2 x 2 - ln x
1 4x2 - 1
H ¢( x) = 4 x - = > 0 when x > 1
x x
H( x) > H(1) Þ H( x) > 2
\ g¢( x) > 0 " x Î (1, ¥)
\ g( x) is increasing on (1, ¥).
19. f ¢( x) = 3 x 2 + 12 x + a
f ¢( x) < 0 in ( -3 , - 1)
a
Product of the roots = =3 Þ a =9
3
æ 1- x ö
20. f( x) = tan -1 ç ÷
è1+ x ø
1 æ -2 ö 2 1
f ¢( x) = ç ÷= = >0
2 ç 2 ÷ 2 (1 + x 2 ) 1 + x 2
æ 1 - x ö è (1 + x) ø
1+ ç ÷
è1+ x ø
78 Solution of Advanced Problems in Mathematics for JEE

f ¢( x) is decreasing " x Î R
p
So, in [0 , 1] f(0) = tan -1 (1) = (max)
4
f(1) = 0 (min)
21. f ¢( x) = 3 x 2 + 2( a + 2) x + 3 a
\ D£0
a 2 - 5a + 4 £ 0
\ a Î[1, 4]
22. f ¢( x) = 0
1
cos 2 x - 3 x + x 1/ 3 - =0
2
1
cos 2 x =
2
1
cos x = ±
2
\ total number is 12.
y

23. – 2,0
3
x
O 1 (1,0)
–1, 0 ,0
2 2

y
2

24. O
x
–3 –2

b2 + 1 ³ 2
25. f( x) is continuous and differentiable in [-1, 1].
cos x 1
26. = - sin x 1 Þ x 1 = -cot x 1
x1
1 1
Point ( x 1 , cos x 1 ) always lie on = +1
2
y x2
Application of Derivatives 79

a
27. x + > 2 " x Î (0 , ¥)
x2
f( x) = x 3 - 2 x 2 + a > 0 " x Î (0 , ¥)
æ 4ö
f ¢( x) = 3 x 2 - 4 x = 3 x ç x - ÷
è 3 ø
4
Minimum value at x =
3
64 æ 16 ö 32
-2ç ÷ + a > 0 Þ a >
27 è 9 ø 27
29. f ¢( x) = cos 2 x + cos x + 2 > 0
f( x) min = f(0) = 0
f( x) max = f(2 p) = 5p
31. f( x) = x 3 - 3 x + c = 0
f ¢( x) = 3 ( x 2 - 1)
–1 1
Þ f(1) f( -1) < 0
( c - 2)(2 + c) < 0
32. f ¢( x) = e x ( x - 1)( x - 2) < 0
dy
33. = 3 ax 2 + 2 bx + c = 0 has one root Þ D = b 2 - 3 ac = 0 Þ b 2 = 6
dx
34. Let x = tan q then y = cos 2 q
dy
=|2 sin q cos 3 q|
dx
dy p
at q =
dx max 6
35. h( x) = f( x) - g( x) = 2 x - 3 sin x + x cos x
h(0) = 0
h¢( x) = 2 - 2 cos x - x sin x
h¢(0) = 0
h¢¢( x) = sin x - x cos x
h¢¢(0) = 0
æ pö
h¢¢¢( x) = x sin x > 0 " x Î ç 0 , ÷
è 2ø
36. f( x) = 2 tan -1 ( g( x)) | g( x)| £ 1
-1
= p - 2 tan g( x) g( x) > 1
= - p - 2 tan -1 g( x) g( x) < -1
80 Solution of Advanced Problems in Mathematics for JEE

2 g¢( x)
f ¢( x) = | g( x)| < 1
1 + ( g( x)) 2
2 g¢( x)
=- | g( x)| > 1
1 + ( g( x)) 2
é 7 é ln(1 + 7 f( x)) ù 1 æ sin f( x) ö ù
37. lim ê ê ú - 3 çç f( x) ÷÷ ú = 2
a 3 7 f( x)
x® e ë ë û è øû
38. If f( x) is strictly decreasing for all x,
f ¢( x) = log 1/ 3 (log 3 (sin x + a)) £ 0
Þ sin x + a ³ 3 " x Î R
Þ a³ 4
39. f( x) = a ln| x| + bx 2 + x
a 2 bx 2 + x + a
f ¢( x) = + 2 bx + 1 =
x x
if x = 1 and x = 3 are point of extrema.
1 a
Þ - = 4 and =3
2b 2b
y

–3p
40. –2p –p
x
O 1

f( x) has local maximum at x = 0.


x
41. f( x) = (t - a) 2n (t - b) 2m +1 dt
ò
1

f ¢( x) = ( x - a) 2n ( x - b) 2m +1
No sign change of f ¢( x) about x = a.
f ¢( x) will change sign from negative to positive at x = b Þ Point of minima.
43. Let point P on the curve y 2 = x 3 is P(t 12 , t 13 ).
Equation of tangent at P(t 12 , t 13 ) is
3
y - t 13 = t 1 ( x - t 12 )
2
If this intersect the curve again at Q (t 22 , t 23 )
Application of Derivatives 81

t1
Þ t2 = -
2
tan a (3t 1 2)
= = -2
tan b (3t 2 2)
44. y 2 = ax 3 - b
if (2 , 3) is lie on the curve
8a - b = 9 …(1)
Slope of normal at (2 , 3)
1 1
- =- Þ a =2
4 2a
45. Equation of tangent at (0 , 1) to the curve y - 1 = kx meet x-axis at ( a , 0) then
1 é1 ù
-2 £ - £ -1 Þ k Î ê , 1ú
k ë2 û
x -u 2 1
2
46. f( x) = ò e x du = x e -t dt
ò
0 0
u
where t =
x
Þ f( x) = K x , K > 0
47. f ¢¢(a ) = 0 Þ x = a is the point where concavity changes.

a b c

48. f( x) = x 6 - x - 1
f ¢( x) = 6 x 5 - 1 > 0 " x Î [1, 2]
If f(1) = -1 < 0 and f(2) = 2 6 - 3 > 0 then f( x) has one root in [1, 2].
49. Every line passing from ( a , b) is normal to the circle ( x - a) 2 + ( y - b) 2 = k
50. f ¢( x) = cos x(3 sin 2 x - m ) = 0
m m
sin 2 x = Þ 0< <1
3 3
0<m<3
1/ x
51. Let y = x
y ¢ = x (1/ x )-2 (1 - ln x)
f( x) is increasing (0 , e)
and f( x) is decreasing ( e, ¥)
82 Solution of Advanced Problems in Mathematics for JEE

52. Let y = mx
Point of tangency be ( x 1 , y 1 )
Þ mx 1 = x 13 + x 1 + 16 & m = 3 x 12 + 1
Þ x 1 (3 x 12 + 1) = x 13 + x 1 + 16
x1 = 2
m = 13
53. y ¢ = 3 x 2 - 6 x + 6
y ¢¢ = 6 x - 6 = 0
x =1
y¢ = 3
54. Let H( x) = ln( f( x) + f ¢( x) +¼ f n ( x)) - x
Þ H( a) = H( b)
Þ H ¢( c) = 0 (by L.M.V.T.)
n +1
f ¢( c) + f ¢¢( c) +¼ + f ( c)
Þ -1=0
n
f( c) + f ¢( c) +¼ + f ( c)
Þ f n +1 ( c) = f( c)
55. h( x) = g( x) + x
Þ h¢( x) = g¢( x) + 1
Þ g¢( x) = h¢( x) - 1
Þ g¢¢( x) = h¢¢( x)
Þ h¢¢( x) - 3( h¢( x) - 1) > 3
Þ h¢¢( x) - 3 h¢( x) > 0
d -3 x
Þ (e h¢( x)) > 0
dx
Let P( x) = e -3 x h¢( x)
Þ P ¢( x) > 0
Þ P( x) is an increasing function.
P(0) = h¢(0) = 0
Þ P( x ) > 0 " x > 0
Þ h¢( x) > 0 " x > 0
Þ h( x) is an increasing function " x > 0
dy c
56. =- = -1 Þ ( x + 1) 2 = c
dx ( x + 1) 2
Point ( c - 1, c ) lie on the line x + y = 3 Þ c = 2
Application of Derivatives 83

2
57. |sin x| = e - x
y

x
–p/2 O p/2 p 3p/2
60. x n + ax + b = 0
x is even.
nx n -1 + a = f ¢( x)

f¢(x)
–a 1/(n–1)
n

f(x)

2
61. f( b) = sin x + +b " xÎR
3 + sin x max

sin x = t
2
g(t ) = t + t Î [-1, 1]
3+t
2
g¢(t ) = 1 - >0
(3 + t ) 2
(3 + t ) 2 - 2 > 0
(3 + t - 2 )(3 + t + 2 ) > 0
–3 – 2 –3 + 2 –1 1
2
g(t ) = t + increasing " Î [-1, 1]
3+1
3
g(t ) max =
2
g(t ) min = 0
3
+b 3
f( b) = 2 if b ³ -
-b 4 –3/2 0

3
b< -
4
æ -3 ö 3
min. of f( b) = - ç ÷=
è 4 ø 4
x dy 1- x2
62. y = Þ =
1+ x2 dx (1 + x 2 ) 2
84 Solution of Advanced Problems in Mathematics for JEE

æ xö
63. f -1 ( x) = 2 cos -1 ç ÷
è3ø
d -1 -2 æ 1 ö
f ( x) = ç ÷
dx x2 è3ø
1-
9
64. f( x) = sin x + tan x - 2 x
f ¢( x) = cos x + sec 2 x - 2 = 0
cos 3 x - 2 cos 2 x + 1 = 0 Þ (cos x - 1)(cos 2 x - cos x - 1) = 0
1- 5
cos x = 1,
2
a + 2c 4
65. + =0 Û 3 a + 4 b + 6 c + 12 b = 0
b + 3b 3
1 b
Û a + + d =0
4 3
ax 4 bx 3 cx 2
Consider f( x) = + + + dx
4 3 2
then f(0) = 0 = f(1)
\ f( x) satisfies the conditions of Rolle’s theorem in [0, 1].
Hence, f ¢( x) = 0 has at least one solution in (0, 1).
66. f ¢( x) = f ( x) × ( x - 2) 2
f(2) > 0 Þ f ¢( x) > 0 Þ f( x) ­
f(2) < 0 Þ f ¢( x) < 0 Þ f( x) ¯
67. f(1) = f(3) Þ a + b - 5 = 3 a + b - 27 Þ a = 11
f ¢( c) = 3 c 2 - 12 c + a = 0 Þ b Î R
3 at
70. Let x = , y = at 3/ 2
2 2/ 3
æ 9 a 2t 2 ö
2ç ÷
dy ç 2 4/ 3 ÷
è ø
= = - cot a Þ t = -2 1/ 3 cot a
dx 2 3/ 2
9a t
æ 3 at at 3/ 2 ö÷
and P = cos a ç -
ç 2 2/ 3 - cot a ÷
è ø
P
Þ = cos a cot 2 a
a
Application of Derivatives 85

Exercise-2 : One or More than One Answer is/are Correct

1. Equation of tangent to y = x 3
y - x 13 = 3 x 12 ( x - x 1 )
Equation of tangent to y = x 1/ 3 is
1
y - x 12/ 3 = (x - x2)
3 x 22/ 3
If these tangents represent same line
2 2/ 3
1 9 x1 x 2 -2 x 13 1
= = Þ x1 = ±
1 1 2 1/ 3 3
x
3 2
f ( 4) - f ( 0) 1
2. (a) f ¢(C 1 ) = = ; C 1 Î(0 , 4)
4 -0 4
f ( 8) - f ( 0) 1
(c) f ¢(C 1 ) = = ; C 1 Î(0 , 8)
8 -0 8
f ( C 2 ) = f ( 8) = 1
x3
(d) Let g( x) = ò f(t) dt
0
8
Þ g(0) = 0 , g(2) = ò f(t) dt
0
g(2) - g(0)
g¢(a ) = 3a 2 f(a 3 ) = a Î(0 , 2)
2
g(2) - g(0)
and g¢(b) = 3b 2 f(b 3 ) = b Î(0 , 2)
2
8
g¢(a ) + g¢(b) = g(2) - g(0) = ò f(t) dt
0
1
4. f( x) = 2 x 4 + x 4 sin x ¹0
x
=0 x =0
1 1
f ¢( x) = 8 x 3 + 4 x 3 sin - x 2 cos
x x
5. -1 £ f ( x) £ 1
¢¢
- x £ f ¢( x) £ x (Q f ¢(0) = 0)
x2 x2
- £ f( x) < (Q f(0) = 0)
2 2
86 Solution of Advanced Problems in Mathematics for JEE

6. f ¢¢( x) > 0 " x Î [-3 , 4]


Þ f ¢( x) is increasing for x Î [-3 , 4]
7. f ¢¢( x) > 0 " x Î [0 , 2]
Þ f ¢( x) ­
f(1) - f(0) f(2) - f(1)
f ¢(C 1 ) = , C 1 Î (0 , 1) and f ¢(C 2 ) = , C 2 Î(1, 2)
1-0 2 -1
f ¢(C 1 ) < f ¢(C 2 ) Þ f(0) + f(2) > 2 f(1)
é 2ù é2 ù
Similarly applying LMVT between ê0 , ú and ê , 2 ú
ë 3û ë3 û
æ2ö æ2ö
f ( 2) - f ç ÷ f ç ÷ - f ( 0)
è3ø è3ø æ2ö
> Þ 2 f ( 0) + f ( 2) > 3 f ç ÷
4 2 è3ø
3 3
8. Let g¢¢( x) = a( x - 1)
ax 2
g¢( x) = - ax + b
2
3a
g¢( -1) = 0 Þ b = -
2
ax 3 ax 2
g( x) = - + bx + c Þ g( x) = x 3 - 3 x 2 - 9 x + 5 (Q g( -1) = 10 , g(3) = -22)
6 2
9. f( x) = 2 x 3 - 3 (l + 2) x 2 + 2lx + 5
f ¢( x) = 6 x 2 - 6 (l + 2) x + 2l = 0 has two real roots, then
D > 0 Þ 3l2 + 8l + 12 > 0
Þ lÎR
10. f( x) = 1 + x ln( x + 1 + x 2 ) - 1 + x 2

f ¢( x) = ln( x + 1 + x 2 )
f ¢( x) ³ 0 for " x Î [0 , ¥)
f ¢( x) £ 0 for " x Î ( -¥, 0]
11. f( x , y) = x m ( k - x) n
f ¢( x , y) = mx m -1 ( k - x) n - x m n × ( k - x) n -1 = 0
mk
Þ x=
m +n
k m+n × m m × n n
Maximum value =
( m + n) m + n
Application of Derivatives 87

12. Let line is tangent at (3t 12 , 2t 13 ) and normal at (3t 22 , 2t 23 )


dy
Þ = t1
dx 3t 2, 2t 3
1 1
1
So, slope of normal at (3t 22 , 2t 23 ) = -
t2
1
Þ t1 = -
t2
1
t2 = -
t1
2
2t 13 +
t 13
Þ t1 =
æ 1ö
3 ç t 12 - ÷
ç t 12 ÷ø
è
Þ t 12 (t 14 - 3) = 2 or t 1 = ± 2
dy
13. m =
dx
|my| y
= then solve it.
x+ y x
1
14. First draw the graph f( x) =
2
x -4

–2 2

While drawing diff. possibilities of y = ax 2 + bx + c


We get possible intersections.
15. y ¢ = 3 x 12
x 13 - 8
3 x 12 = Þ x 1 = -1 or 2
x1 - 2
y ¢ = 3 or y ¢ = 12
16. Let f( x) = x + cos x - a
f ¢( x) = 1 - sin x ³ 0 " x Î R
Þ f( x) is increasing.
Þ x + cos x - a = 0 for one positive value of x , a Î (1, ¥)
88 Solution of Advanced Problems in Mathematics for JEE

17. Let y = ax + b = f( x) a ¹ 0
x -b
f -1 ( x) =
a
1
(1) m 1 = a , m 2 = Þ m 1m 2 = 1
a
-1
(2) m 1 = a , m 2 = Þ m 1 m 2 = -1
a
1
(3) m 1 = - a , m 2 = Þ m 1 m 2 = -1
a
-1
(4) m 1 = - a , m 2 = Þ m 1m 2 = 1
a
18. f ¢( x) = e x ( x 2 - 1) x 2 ( x + 1) 2011 ( x - 2) 2012
= e x x 2 ( x + 1) 2012 ( x - 1)( x - 2) 2012
+ +
– – –
–1 0 1 2
x = -1, 0 , 2 are points of inflections and might be more points in (1, 2).
x = 1 is point of minima (Answer can be given either d, ad, bd or abd)
19. f( x) = sin x + ax + b
f ¢( x) = cos x + a
if a > 1 then f( x) is increasing.
So, only one real root, which is positive if b > 0 and negative if b > 0
if a < -1
f( x) is decreasing so only one real root, which is negative if b < 0.
20. f ¢¢( c) = 0 for c Î(0 , 1)
f ¢¢( x) > 0 for x Î(0 , c)
f ¢¢( x) < 0 for x Î( c , 1)
21. f ¢( x) = 5 sin x cos x(sin x - cos x)(1 + sin x cos x)
æp pö
Clearly, f ¢( x) > 0 " x Î ç , ÷
è4 2ø
æ pö
f ¢( x) < 0 " x Î ç 0 , ÷
è 4ø
æpö
f ( 0) = 0 , f ç ÷ = 0
è2ø
æ pö
By Rolle’s theorem $ c Î ç 0 , ÷ Þ f ¢( c) = 0
è 2ø
æpö
Clearly, f( x) ³ f ç ÷
è4ø
Application of Derivatives 89

5
æ 1 ö æ 1 ö 1 æ pö
f( x) ³ 2 çç ÷÷ - 1 = 2 çç ÷÷ - 1 = - 1 and f( x) < 0 " x Î ç 0 , ÷
è 2ø è4 2ø 2 2 è 2ø
22. f( x) = x 2a +1 ln x x>0
=0 x =0
1
f( x) is not continuous at a = - ,-1
2
cos x
23. f ¢( x) =
x
æ pö æ p pö
Clearly, f ¢( x) > 0 Þ x Î ç 0 , ÷ È ç ( 4n - 1) ,( 4n + 1) ÷ " n Î N
è 2 ø è 2 2 ø
æ p pö
and f ¢( x) < 0 Þ x Î ç ( 4n + 1) ,( 4n + 3) ÷ " n Î N
è 2 2ø
p
\ f( x) has a local minima at x = ( 4n - 1) " nÎ N
2
p p
and f( x) has a local maxima at x = and ( 4n + 1) " n Î N
2 2
x ( - sin x) - cos x
Also, f ¢¢( x) = = 0 Þ x tan x + 1 = 0
x2
\ All the points of inflection of f( x) lie on the curve x tan x + 1 = 0
p
Also, f ¢( x) = 0 Þ x = (2n + 1) " n Î N
2
\ Number of values of x in (0 , 10p) in which f ¢( x) = 0 are 20.
24. | f( x)| £ 1
Applying L.M.V.T. in x Î(0 , 1)
Þ | f ¢( x)| =| f(1) - f(0)|
| f(1) - f(0)| £ 2
Þ | f ¢( x)| £ 2 for atleast one x in (0 , 1)
Similarly | f ¢( x)| £ 2 for atleast one x is ( -1, 0)
F ( x) = ( f( x)) 2 + ( f ¢( x)) 2
For atleast one x in (0 , 1) & ( -1, 0)
| f ¢( x)| £ 2 & | f( x)| £ 1
Þ ( f ¢( x)) 2 £ 4 & ( f( x)) 2 £ 1
Þ ( f ¢( x)) 2 + ( f( x)) 2 £ 5
Þ F ( x) £ 5, for atleast one x in ( -1, 0) & (0 , 1)
90 Solution of Advanced Problems in Mathematics for JEE

æpö æ p+ ö æpö æ p- ö
25. f ç ÷ > fç ÷ and fç ÷> fç ÷
è2ø ç 2 ÷ è2ø ç 2 ÷
è ø è ø
Also, absolute maximum occurs at x = -1
26. Symmetric about y = x
dy
=1
dx
1
2x = 1 Þ x =
2
æ1 5ö
Point = ç , ÷
è2 4ø
- (1 + x) e - x - e - x
27. f ¢( x) > 0 Þ >0
(1 + x) 2
for x < -2 increasing.

–2

– e2

x = –1

28. Point (1, 2) lies on y = mx + 5 Þ m = -3 …(1)


Point (1, 2) lies on x 3 y 3 = ax 3 + by 3 Þ 8b + a = 8 …(2)
2 3 3 2 1 2 2 1
3 x y + x × 3 y y = 3 ax + 3 by y Þ a - 12 b = -4 …(3)
16 3
Þ a= , b=
5 5
f( x) - 1 x4 + x2 + 1 x2 - x + 1
29. = =
f( x) + 1 ( x 2 + x + 1) 2 x 2 + x + 1

x2 + 1
Þ f( x) =
x
Application of Derivatives 91

Exercise-3 : Comprehension Type Problems

Paragraph for Question Nos. 1 to 2


x +1
1. f( x) =
x -1
x( x + 1) 2
g( x) = =x +2+
x -1 x -1
2
g¢( x) = 1 - =0
( x - 1) 2
x =1+ 2,1- 2
4
g¢¢( x) =
( x - 1) 3
g¢¢(1 + 2 ) > 0
Minimum value of g( x) is 3 + 2 2 .
1 2 1
2. g¢( x) = Þ 1- = Þ x = 3, - 1
2 2 2
( x - 1)

Paragraph for Question Nos. 3 to 5


1 1 1
t2 1
3. g(1) = ò f(t ) dt = (1 - t ) dt = t -
ò =
2 2
0 0 0

4. For x Î(2 , 3]
1 2 x
g( x) = ò f(t) dt + ò f(t) dt + ò f(t) dt
0 1 2
3
1 ( x - 2)
g( x) = +
2 3
5 æ 5 ö 13
at x = , g ç ÷ =
2 è 2 ø 24
g¢( x) = f( x)
æ 5ö 1
g¢ ç ÷ =
è2ø 4
13 1 æ 5ö
y- = çx- ÷
24 4 è 2ø
12 y = 3 x - 1
92 Solution of Advanced Problems in Mathematics for JEE

1
5. Slope of tangent at P =
4
2
Slope of tangent at R =
3
5
tan q =
14

Paragraph for Question Nos. 6 to 8


y

–1
6. 0
x

7. –1
x
0

Paragraph for Question Nos. 9 to 11


9. By putting x = 1, 2 , - 1, 0 we get a , b, c , d clearly other roots product is 1.
10. P( x) + k = 0 has 4 distinct real roots.
P( x) = -k , where - k Î (1, 2) Þ k Î ( -2 , - 1)
\ pull the graph more than 1 and less than 2, now the graph intersect the x-axis
in ( -2 , - 1),( -1, 0),(0 , 1),(2 , 3)
\ - 2 + ( -1) + 0 + 2 = -1
11. P( x) = 0 has two roots.
P ¢( x) = 0 has three root
P( x) = 0 has atleast 5 roots.
( P ¢( x)) 2 + P( x)P ¢¢( x) = 0 Þ ( P ¢( x)P( x)) ¢ = 0 has atleast four roots.

Paragraph for Question Nos. 12 to 14


f( x 2 ) - f( x 1 )
Sol. The equation of chord AB will be y - f( x 1 ) = ( x - x1 )
x 2 - x1
This line passes through (0 , 2 x 1 x 2 )
f( x 2 ) - f( x 1 )
\ 2 x 1 x 2 - f( x 1 ) = ( -x1 )
x 2 - x1
Application of Derivatives 93

( x 2 - x 1 ) f( x 1 ) - x 1 f( x 2 ) + x 1 f( x 1 )
Þ 2 x1 x 2 =
x 2 - x1
Þ 2 x 1 x 2 ( x 2 - x 1 ) = x 2 f( x 1 ) - x 1 f( x 2 )
f( x 1 ) f( x 2 ) f( x 1 ) f( x 2 )
Þ - = 2 ( x 2 - x1 ) Þ + 2 x1 = + 2x2 = k
x1 x2 x1 x2
f( x)
\ + 2x = k
x
\ f( x) = kx - 2 x 2
Given that f(1) = -1
\ -1 = k - 2 Þ k = 1
\ f( x) = x - 2 x 2
1/ 2
1/ 2 é x2 2x3 ù 1 2 1 1 1 1
12. \ ò f( x) dx = ê - ú = - × = - =
0 êë 2 3 ûú
0
8 3 8 8 12 24
1
13. f ¢( x) = 1 - 4 x ³ 0 Þ x £
4
14. F ( x) = f( x) + x = 2 x - 2 x 2
Clearly, F (0) = F (1) = 0
\ Rolle’s theorem is applicable in [0 , 1].
Paragraph for Question Nos. 15 to 16
1 1
15. f( x) = 1 + x e y f( y) dy + e x
ò ò y f( y) dy
0 0
1 1
Let A = e y f( y) dy ,
ò B= ò y f( y) dy
0 0
1
Þ f( x) = 1 + Ax + Be x Þ A = e x (1 + Ay + Be y ) dy
ò
0
2 3
B =- ,A=-
e+1 2
f ¢( x) + 3 > 0
3 2e x 3( e + 1) é4 ù
Þ -- + 3 > 0 Þ ex < Þ ê e x ú = [e + 1] = 3
2 ( e + 1) 4 ë3 û
3x
16. Ax 1 + Be x1 = - 1 - 2 Þ Be x1 = -2
2
f ¢( x 1 ) = A + Be x1 = m 1
94 Solution of Advanced Problems in Mathematics for JEE

3 7
m1 = - -2 = -
2 2
3
m2 = -
2
8
tan q =
25

Exercise-4 : Matching Type Problems

3.
y = f(x)
y = g(x)

0
x
1 –1
– 2 1 2
x
–2 –1 1 2

y = h(x) = f(–x)

1
x
–2 –1 0 1 2

x3
4. (A) y =
( x - a )( x - b)( x - g)
8
y=
(2 - a )(2 - b)(2 - g)
x 3 - 3 x 2 + 2 x + 4 = ( x - a )( x - b)( x - g)
Application of Derivatives 95

Put x = 2
8 - 12 + 4 + 4 = (2 - a )(2 - b)(2 - g) = 4
8
\ y|at x = 2 = = 2
4
(B) x 3 + ax + 1 = 0 , x 4 + ax + 1 = 0 …(1)
4 2
x + ax + x = 0 …(2)
2
(2) – (1) gives ax - ax + x - 1 = 0
ax( x - 1) + ( x - 1) = 0
( x - 1)( ax + 1) = 0
1
x = 1 or x = -
a
put x = 1 in (1) we get, 1 + a + 1 = 0 Þ a = -2
|a| = 2
(C) f( x) = x 2 + 4 cos x + 5 x/2

f ¢( x) = 2 x - 4 sin x = 2 ( x - 2 sin x) = 0
x
sin x = –p –2 b
2 O a2 p

(>0) when x > a


<0 when 0 < x < a
f ¢(x) – +
>0 when b < x < 0
b 0 a
<0 when x < b
x = 0 only a point of maxima
So, number of local maxima is 1.
(D) Let | x| = t Î [0 , 2]
\ f( x) = 2t 3 + 3t 2 - 12t + 1 = g(t ) – +
2 2
g¢(t ) = 6t + 6t - 12t = 6 (t + t - 2) = 6 (t + 2)(t - 1) –2 0 1

g(1) is min. of f( x) i . e. , fmin = 2 + 3 - 12 + 1 = -6


g(0) = 1, g(2) = 16 + 12 - 24 + 1 = 5
max. of f( x) is 5 > 4 , 3 , 2 , 0
1
6. f ¢ = - a + 2 x > 0 since x > 0
8x
1 - 8 ax + 16 x 2 > 0
96 Solution of Advanced Problems in Mathematics for JEE

See D = 0
D<0
D>0
7. Let g( x) = ax 3 + bx 2 + cx + d
f( x) = g( x)
f( x) has local maxima and local maxima at x = -2 and x = 2.
Þ g( x) has same local minima and maxima and x = -2 and 2.
Þ a < 0 ; a = -2
3 ax 2 + 2 bx + c
f ¢( x) = =0
2 ax 3 + bx 2 + cx + d
f ¢( -2) = 0 and f ¢(2) = 0
Þ b = 0 , c = 24
Also, g(2) > 0 and g( -2) > 0
Þ -16 + 48 + d > 0 and 16 - 48 + d > 0
d > -32 and d > 32
Þ d > 32
æ h 2 ö÷ æ 2 ö
çQ R 2 = r 2 + h ÷
8. (A) V = pr 2 h = ph × ç R 2 -
ç 4 ÷ø ç 4 ÷ø
è è
dV æ 3 h 2 ö÷ 2R 2
= pçR 2 - =0 Þ h = and r = R
dh ç 4 ø ÷ 3 3
è
pr 2 h ph 2
(B) V = = {R - ( h - R ) 2 } {Q R 2 = r 2 + ( h - R ) 2 }
3 3
dV p 4R 2 2R
= ( 4 hR - 3 h 2 ) = 0 Þ h= and r =
dh 3 3 3
r R h2r 2
(C) sin q = = Þ R2 =
h-r R + h2
2 h 2 - 2 hr
p 2 p æ h 2 r 2 ö÷
Volume of cone = R h= ç
3 3 çè h - 2 r ÷ø

dV p 2 æç ( h - 2 r) 2 h - h 2
ö
÷ = 0 Þ h = 4r
= r
dh 3 çè ÷
( h - 2 r) 2
ø
æ 2x 2x 2x ö æ 3 y 3 y 3 y 3 y ö
ç + + ÷+ç + + + ÷ 1/7
è 3 3 3 ø è 4 4 4 4 ø æç 8 x 3 81 y 4 ö 32
(D) ³ × ÷ Þ x3 y4 £
7 ç 27 256 ÷ 3
è ø
Application of Derivatives 97

Exercise-5 : Subjective Type Problems

A2 pr 2 2p
1. = =
A1 æ 2 p - q ö 2 2p - q
ç ÷ × pr
è 2p ø O
1 q
2 2
pæ q ö æ q ö
V = ç ÷ 12 - ç ÷
3 è 2p ø è 2p ø
dV 8
=0 Þ q = p
dq 3
A2 3
= =3 + 6
A1 3- 2
2. f( x) = x 2 ln x
Þ f ¢( x) = x (1 + 2 ln x)
and f ¢( x) > 0 for Î[1, e]
\ f( x) is continuously increasing on [1, e] with the least value zero at x = 1 and the greatest
value e 2 at x = e.
x2
3. f( x) = px e - x - +x
2
f ¢( x) = (1 - x)[ pe - x + 1] £ 0
Þ p £ -1
ì ax e ax + e ax ; x £ 0
4. f ¢( x) = í 2
î1 + 2 ax - 3 x ; x > 0
Clearly, f ¢( x) is continuous at x = 0
ì 2 ax ax
Þ f ¢¢( x) = ía x e + 2 ae ; x £ 0 ; f ¢( x) increasing if ( ax + 2) ae ax ³ 0 and 2 a - 6 x ³ 0
î 2a - 6 x ; x>0
5. f( x) = x 2 - 2 bx + 1
Case I : b > 1
Þ f(0) - f(1) = 4
Þ 1 - (2 - 2 b) = 4
5
Þ b=
2
1
Case II : 0 < b <
2
Þ f(1) - f( b) = 4
Þ b = 3, - 1 (Not possible)
98 Solution of Advanced Problems in Mathematics for JEE

1
Case III : < b< 1
2
Þ f(0) - f( b) = 4
b = ±2 (Not possible)
Case IV : b < 0
Þ f(1) - f(0) = 4
3
Þ b=-
2
6. x 2 + 9 y 2 = 36 x 2 + y 2 = 12
12 - y 2 + 9 y 2 = 36
8 y 2 = 24 Þ y 2 = 3 Þ y = 3 , - 3
when y = ± 3 , x 2 = 12 - 3 = 9
x = ±3
\ point of intersections are ( ± 3 , ± 3 )
Let one of point of intersect is (3 , 3 )
2x 2 y æ 2x ö æ 4 ö
Now, + y¢ = 0 Þ y¢ = ç - ÷ ´ çç ÷÷
36 4 è 36 ø è 2 y ø
1
( y ¢) ( 3, 3)
=- = (m 1 )
3 3
x 3
2x + 2 y y¢ = 0 Þ y¢ = - Þ ( y ¢) ( 3, 3)
=- = - 3 =m2
y 3
1
- + 3
m1 - m 2 3 3 8 2
tan q = = = =
1 + m 2m 1 1 4 3 3
1+
3
2 æ 2 ö
tan q = Þ q = tan -1 çç ÷÷
3 è 3ø
2x
7. f ¢( x) = 2 e - (l + 1) e x + 2 ³ 0 " x Î R
i . e. , 2 e 2 x + 2 - (l + 1) e x
l + 1 £ 2( e x + e - x )
l+1
£ (e x + e-x ) " x Î R
2
l+1
Þ £ ( e x + e - x ) min " x Î R
2
Application of Derivatives 99

l+1
So, £2
2
l + 1£ 4
l£3
l Î ( -¥, 3]
\ k =3
8
9. f( x) = x 2 , g( x) = -
x
8
Þ q = p2 Þ s=-
r
s-q 8
Also, = 2p & 2p =
r-p r2
pr 2 = 4 …(1)
s-q
= 2p
r-p
8
- - p2
r 8
Þ = 2 p Þ - - p 2 = 2 pr - 2 p 2
r-p r
8
p 2 = + 2 pr
r
Þ p 2 r = 16 …(2)
Multiply (1) & (2),
Þ pr = 4
Þ r = 1, p = 4
é| x + 2|, x ³ 0
10. f( x) = ê
ë| x - 2|, x < 0
Minimum value of f( x) is 2.
x
11. f( x) = [( a - 1)(t 2 + t + 1) 2 - ( a + 1)(t 4 + t 2 + 1)] dt
ò
0
x
2 (t 2 + t + 1)( at - t 2 - 1) dt
ò
0

f ¢( x) = 2 ( x 2 + x + 1)( ax - x 2 - 1) = 0
D < 0 Þ a2 - 4 < 0

12. f( x) = x 2013 + e 2014 x


f ¢( x) = 2013 x 2012 + 2014 e 2014 x > 0
Þ f( x) is increasing function.
100 Solution of Advanced Problems in Mathematics for JEE

14. P = ( x 1 , x 13 - ax 1 )
Q = ( x 2 , x 23 - ax 2 )
Q
y = x 3 - ax P
dy
= 3x2 - a
dx
Slope at P = slope of PQ
æ x 3 - ax 2 - x 13 + ax 1 ö
\ (3 x 12 - a) = ç 2 ÷ (Q x 1 ¹ x 2 )
ç x 2 - x1 ÷
è ø
( x 2 - x 1 )( x 2 + 2 x 1 ) = 0
Þ x 2 = -2 x 1 …(1)
Slope at P ´ Slope at Q = -1
(3 x 12 - a)(3 x 22 - a) = -1 …(2)
Put (1) in (2),
36 x 14 - 15ax 12 + ( a 2 + 1) = 0
D³0 (Q x 1 Î R )
2 4
9 a ³ 16 Þ a ³
3
b
x3
15. I(t ) = ( x 2 + 2 x - t 2 ) dx =
ò + x 2 - t 2 x|ba (max)
3
a

b3 -a3
I( t ) = + (b 2 - a 2 ) - t 2 (b - a )
3 O

I ¢(t ) = -2t (b - a ) = 0
I ( t ) £ I ( 0)
0
2 4 p 4
\ I ( 0) = ò(x + 2 x) dx = - Þ =|I(0)|=
3 q 3
-2
dy 3y
16. =1- when t = 0 , y = 0
dt 100 - 2t
dy æ 3 ö
+ç ÷ y =1
dt è 100 - 2t ø
y(100 - 2t ) -3/ 2 = + (100 - 2t ) -1/ 2 + c
as when t = 0 , y = 0
1
c=-
10
1
y = (100 - 2t ) - (100 - 2t ) 3/ 2
10
Application of Derivatives 101

dy 1 æ3ö 1/ 2
= -2 - ç ÷ (100 - 2t ) ( -2) = 0
dt 10 è 2 ø
250 7
t= = 27 +
9 9
17. Let f ¢( x) = K
Þ f( x) = Kx + c
Þ f(9) - f( -3) = 12 K
Maximum value of f(9) - f( -3) = 96
æ3 ö -9 y 12 æ 3 3ö
19. Equation of normal at P ç y 13 , y 1 ÷ is y - y 1 = ç x - y1 ÷
è4 ø 4 è 4 ø
If it passes from (0, 1) then 27 y 15 + 16 y 1 - 16 = 0 has only one real root.
æ x2 ö
20. e - x ç + x + 1÷ = a
ç 2 ÷
è ø
æ x2 ö
Let f( x) = e - x ç + x + 1÷
ç 2 ÷
è ø
æ x2 ö
f ¢( x) = e - x ç - ÷<0
ç 2 ÷
è ø
21. f ¢( x) = a - 2 sin 2 x + cos x - sin x
Let g( x) = -2 sin 2 x + cos x - sin x
= -2 {(cos x - sin x) 2 - 1} + cos x - sin x
where cos x - sin x = t
-2t 2 + t + 2 " t Î [- 2 , 2 ]
17 17
-2 - 2 £ g ( x) £ Þ a³
8 8
22. Let x = 6 cos 3 q , y = 6 sin 3 q
dy 6 (3 sin 2 q cos q)
= = - tan q
dx -6 (3 cos 2 q sin q)
Equ. of tangent
y - 6 sin 3 q = - tan q ( x - 6 cos 3 q) Þ p1 = 6 sin q cos q
Equ. of normal
y - 6 sin 3 q = cot q ( x - 6 cos 3 q) Þ p 2 = 6 (cos 2 q - sin 2 q)

4 p12 + p 22 = 6 4 sin 2 q cos 2 q + cos 4 q + sin 4 q - 2 sin 2 q cos 2 q = 6


❑❑❑
102 Solution of Advanced Problems in Mathematics for JEE

5
INDEFINITE AND DEFINITE
INTEGRATION

Exercise-1 : Single Choice Problems

é ù
1. ò ê a x ln x + a x ln a × x (ln x - 1) ú dx
ê 1
424 3 14243 ú
ë II I û
é é 1 ù ù
= a x × ln x dx + ê x (ln x - 1) a x - ê x × + (ln x - 1)ú a x ú dx
ò ò
ë ë x û û
= a x × ln x dx + [ x (ln x e)] a x - (ln x) a x dx
ò ò
1
1 n 1 dx
2. lim
n® ¥ n
å r
= ò x +1
= 2 ( 2 - 1)
r =1 0
1+
n
sin x
3. ò sin( x - a) dx
Let x - a = t Þ dx = dt
sin(t + a )
ò sin t
dt = t cos a + sin a log sin t + C = x cos a + sin a log sin( x - a ) + C

2 22
log( x 2 + 2) æ - log ( x 2 + 2) ö
ç ÷ 2 x dx
4. ò dx = + ò
( x + 2) 2 ç x +2 ÷ 2
0 è ø 0 0 ( x + 2)( x + 2)
2 5 1
= tan -1 2 - log 2 + log 3
3 12 12
5. For 0 < x < 1
1+ x9 < 1+ x8 < 1+ x4 < 1+ x3
x
6. Let g( x) = ò 1 - ( f( s)) 2 ds
0
Indefinite and Definite Integration 103

æ g(t ) - g( x) ö
lim çç ÷ = f( x) Þ g¢( x) = f( x) f ¢( x) = 1 - ( f( x)) 2
t ® x è f (t ) - f ( x) ÷
ø
y dy æ 1 3ö
1- y2 =1- x çQ f æç ö÷ = ÷
ò 1- y2
= dxò Þ ç
è è 2 ø 2 ÷ø
1 1
1 n æ rö 2
7. lim å fç ÷ = ò f( x) dx = ò 1 - cos 6 x - sin 6 x dx
n® ¥ n ènø 0 3
r =1 0
1
1 - cos 2
= sin 2 x dx =
ò 2
0
æ 1 ö
1 1 2ç1 - ÷
(x 6 - x 3) 1 è x3 ø 1 æ 1 ö
8. ò (2 x 3 + 1) 3 dx = ò dx = - ç Put 2 x + 2 = t ÷
2 æ 3 36
0 0 1 ö è x ø
ç2x + 2 ÷
è x ø
1/ 2 1
sin -1 x tan -1 x
9. 2 ò dx - ò dx
x x
0 0
p/ 4 p/ 4 p/ 4 p/ 4
q cos q q sec 2 q p
2 ò dq - ò dq = - ln 2 - 2 ò ln sin q dq + ò ln tan q dq
sin q tan q 4
0 0 0 0
p/ 4
p
=- ò ln sin 2q dq = 4 ln 2
0
x 0
10. f( x) = x 2 + e -t f( x - t ) dt = x 2 - e - x eu f(u) du
ò ò (Let x - t = u)
0 x
0
f ¢( x) = 2 x + e - x eu f(u) du + f( x)
ò Þ f ¢( x) = x 2 + 2 x
x

x3
Þ f( x) = + x2
3
æ 1 ö
11. f ¢( x) = f( x) + k1 çk = f( x) dx ÷
ç 1 ò ÷
è 0 ø
Þ y = ke x - k1
if f(0) = 1 Þ k - k1 = 1
1
2 e -1
k1 = ( ke x - k1 ) dx Þ 2 k1 = k ( e - 1) Þ k =
ò and k1 =
3-e 3-e
0
104 Solution of Advanced Problems in Mathematics for JEE

1+ cos2 x 1+ cos2 x 1+ cos2 x


12. I 1 = ò tf(t(2 - t )) dt = 2 ò f (t(2 - t )) dt - ò tf(t(2 - t )) dt
2 2 2
sin x sin x sin x

I1 = 2I 2 - I1 Þ I1 = I 2
5 sin x dx cos x + 2 sin x
13. ò = dx + 2 ò ò dx = x + 2 ln|sin x - 2 cos x| + C
sin x - 2 cos x sin x - 2 cos x
2 1
+
(2 + x ) dx 2 3/ 2
14. x x
ò
( x + 1 + x)2
=
æ1 1
ò ö
2
dx
ç + + 1÷
èx x ø
1 1 æ 1 1 ö
Let + + 1=t Þ ç- - ÷ dx = dt
2
x x è x 2 x 3/ 2 ø
æ 2 - x2 - x ö
æ3 2 öæ6 2 ö
3
x + (2 - x 2 ) 3 ç ÷
ç x + 2 - x ÷ ç 1 - x 2 - x ÷ dx ç 2 ÷
è øè ø è ø
15. ò = ò dx
3 3
1- x2 1- x2
= 2 1/ 6 dx = 2 1/ 6 x + C
ò
dx
16. ò 1 - tan 2 x
cos x dx 1
ò = sin -1 ( 2 sin x) + C
2 2
1 - 2 sin x
dx
17. I = ò x 5/ 6 ×( x + 1)7 / 6
x dx
=t = dt
x +1 ( x + 1) 2
( x + 1) 2 dt -5/ 6
I= ò [t( x + 1)]5/ 6 ( x + 1)7 / 6 = ò t dt = 6t 1/ 6 + C

18. I n = sin n x dx = sin n -2 x(1 - cos 2 x) dx


ò ò
n -2
I n = I n -2 - ò 1sin442x 44
× cos x × cos x dx
3 123
II I
æ cos x × sin n -1 x sin n -1 x ö÷
I n = I n -2 -ç - - sin x ×
ò dx
ç n -1 n -1 ÷
è ø
cos x × sin n -1 x 1
I n = I n -2 - - In
n -1 n -1
nI n - (n - 1) I n -2 = - cos x × sin n -1 x
Indefinite and Definite Integration 105

2 1 dt
19. òx dx Let a + bx = t then dx =
2 b
( a + bx)
2
1 æt - aö 1 dt 1 æ t 2 - 2 at + a 2 ö
\ ò x2 dx = ç ò
÷ × 2 × = ò ç ÷ dt
( a + bx) 2
è b ø t b b3 ç t2 ÷
è ø
æ 2 ö é 2ù
1 ç 1 - 2 a + a ÷ dt = 1 êt - 2 a ln|t| - a ú + C
=
b3
ò ç
è t t 2 ÷ø b3 ë t û

1 é a2 ù
= ê a + bx - 2 a ln|a + bx| - ú +C
b3 ë a + bx û

8 x 43 + 13 x 38
20. ò ( x 13 + x 5 + 1) 4 dx
8 x -9 + 13 x -14
ò (1 + x -8 + x -13 ) 4 dx
Let 1 + x -8 + x -13 = t
( -8 x -9 - 13 x -14 ) dx = dt
dt 1 1 x 39
\ ò -
t4
=+
3t 3
+C =
3 (1 + x -8 + x -13 ) 3
+C =
3 ( x 13 + x 5 + 1) 13
+C

æ (cos 6 x + cos 4 x) + 5 (cos 4 x + cos 2 x) + 10 (cos 2 x + 1) ö


21. ò çè ÷ dx
10 cos 2 x + 5 cos x cos 3 x + cos x cos 5 x ø
2 cos 5 x cos x + 5 (2 cos 3 x cos x) + 10 (2 cos 2 x)
= ò 10 cos 2 x + 5 cos x cos 3 x + cos x cos 5 x
dx

= 2 dx = 2 x + C
ò
f( x) = 2 x Þ f(10) = 20
-1 -1 -1
-1
22. ò (1 + x - x ) e x + x dx = e x + x ò × 1 dx + ( x + x -1 ) e x + x dx
ò
-1 -1 æ 1 ö x + x -1
= e x+ x × x - e x+ x ç 1 -
ò
-1
÷ x dx + ( x - x ) e
ò dx + C
2
è x ø
-1
+C = xe x + x
é ù
ê 2 tan x 1 ú 2 tan x 2
23. ex ê ú dx = e x é ù
ò +
ê 1 + tan x æ 1 - cos( p 2 + 2 x) ö ú ê 1 + tan x + (1 + sin 2 x) ú dx
ë
ò û
ê ç ÷ú
ë è 2 øû
é sin x 1 ù
=2 ex ê
ò + dt

ë sin x + cos x (sin x + cos x) û
106 Solution of Advanced Problems in Mathematics for JEE

sin x 1
Let f( x) = , f ¢( x) =
sin x + cos x (sin x + cos x) 2
æ sin x ö
=2 ×exç ÷+C
è sin x + cos x ø
æ 5p ö
gç ÷ =1
è 4 ø
d
24. ( x sin x + cos x) = x cos x f ¢( x) = x cos x
dx
Let f( x) = x sin x + cos x f ¢¢( x) = - x sin x + cos x
æ f ¢¢( x) ö÷ f ¢¢( x)
ò e f ( x ) ç xf ¢( x) + dx = xe f ( x ) f ¢( x) dx + e f ( x ) ×
ò ò dx
ç 2 ÷
( f ¢( x) ) ø ( f ¢( x) 2 )
è
-1 æ -1 ö
= xe f ( x ) - e f ( x ) dx + e f ( x )
ò - e f ( x ) × f ¢ ( x) çç
ò ÷÷ dx
f ¢ ( x) è f ¢ ( x) ø
e f( x) æ 1 ö
= xe f ( x ) - + C = e f ( x ) çç x - ÷+C
f ¢ ( x) è f ¢ ( x) ÷ø
æ 1 ö
= e x sin x + cos x ç x - ÷+C
è x cos xø
1
æ 1 ö
25. ò çè x+ ÷ dx
0
x + 1+ x ø
1 1 1
2 2
ò ( x + ( 1 + x - x ) dx = ( 1 + x dx = (1 + x) 3/ 2
ò = (2 3/ 2 - 1)
3 0 3
0 0
2
x
26. òx x(2 ln x + 1) dx
2
xx =t
x 2 ln x = ln t
æ 2 1 ö 1
çx + (ln x) 2 x ÷ dx = dt
è x ø t
dt 2
\ t× = dt = t + C = x x + C = ( x x ) x + C
ò ò
t
27. = sec 2010 x cosec 2 x dx - 2010 sec 2010 x dx
ò ò
2010
= ò sec x( - cot x) - ò 2010 sec 2010 x × tan x × ( - cot x) - ò 2010 sec 2010 x dx
cot x - cot x
=- + 2010 ò sec 2010 x dx - 2010 ò sec 2010 x dx + C = +C
2010
(cos x) (cos x) 2010
f( x) 1
\ = = { x} no solution.
g( x) sin x
Indefinite and Definite Integration 107

28. Let x x ln x = t
æ x x ö÷
Þ ç x x ln x (1 + ln x) + dx = dt
ç x ÷ø
è
æ 1ö
Þ x x ç ln x + (ln x) 2 + ÷ dx = dt
è xø
x
ò dt = t + C = x ln x + C

æ 1 1 ö
ç 3 - 5 ÷ dx
èx x ø
29. I = ò 2 1
2- +
2
x x4
2 1 1 dt
Let 2 - + =t Þ I = ò
2 4 4
x x t
30. Put ln x = t
2
æ t -1 ö æ 1 2t ö
I = et ç
ò ÷ dt = et çç - ò ÷ dt
2 2 2 2 ÷
è t + 1ø è t + 1 (t + 1) ø
dx dx
31. I = ò = ò
3/ 4 5/ 4 3/ 4
( x - 1) ( x + 2) æ x -1ö 2
ç ÷ ( x + 2)
è x +2ø
x -1 3 dx
Let = t Þ dt =
x +2 ( x + 2) 2
2 - (1 + x 7 ) dx 2 7x6 2 7
32. ò dx = - ò + ò x 7 (1 + x 7 ) dx = - ln| x| + 7 ln|1 + x | + C
7 x 7
x(1 + x )
(sin 4 x - cos 4 x)(sin 4 x + cos 4 x)
33. I = ò dx = (sin 2 x - cos 2 x) dx = - cos 2 x dx
ò ò
2 2
1 - 2 sin x cos x
(tan x) 1/ 3 d ((tan x) 1/ 3 )
34. I = 2 1/ 3 ò (tan x) 2/ 3 + 1
Let (tan x) 1/ 3 = t Þ d((tan x) 1/ 3 ) = dt
2 1/ 3 2t
I= ò t 2 + 1 dt
2
(2012) x -1
( 2012)x
35. ò × (2012) sin dx
1 - (2012) 2 x
-1 x
-1 x 1 (2012) sin ( 2012)
Let sin (2012) = t Þ (2012) t dt = ò +C
ln 2012 ln 2 (2012)
108 Solution of Advanced Problems in Mathematics for JEE

36. Let x + 1 = t 2 Þ dx = 2t dt
æ 1ö
ç 1 + 2 ÷ dt
(t 2 + 1) dt è t ø
2 ò t 4 + t 2 + 1 = 2ò æ 2

çt - ÷ + 3
è tø
æ f( x) g¢ ( x) - f ¢ ( x) g( x) ö æ g( x) ö
37. ò ççè ÷÷ ln çç ÷÷ dx
f( x) g( x) ø è f( x) ø
g( x) f( x) g¢( x) - g( x) f ¢( x) ln t (ln t ) 2
Let =t Þ dx = dt; ò dt = +C
f( x) ( f( x)) 2 t 2
æ xæ 2 1 ö ö
38. ò ççè ò e ç ln x + - 2 ÷ dx ÷÷ dx
è x x ø ø
æ xæ 1ö xæ 1 1 ö ö æ xæ 1ö ö x
ò ççè ò e ç ln x + ÷ dx + e ç - 2 ÷ dx ÷÷ dx = ç e ç ln x + ÷ + C 1 ÷ dx = e ln x + C 1 x + C 2
è xø ò
èx x ø ø è è xø ø
ò
æ -t cos( x + pt ) 1 1
1 × cos( x + pt ) ö÷
39. f( x) = p 2 ç + ò dt = p cos x - 2 sin x
ç p 0 p ÷
è 0 ø
2 é 2 ö
40. £ 5 Þ xÎê , 1÷
x ë 5 ø
1 2/ 5 1 1
æ 2 ö 2
\ ò f( x) dx = ò f( x) dx + ò f( x) dx £ 5 ç
è 5
- 0÷ +
ø 2/
ò x
dx
0 0 2/ 5 5
1
\ ò0 f( x) dx £ 2 + 2 [ln x]1
2/ 5

é 5ù
\ a = 2 + 2 ln ê ú
ë 2 û
42. f(0) = 0 , f(2 p) = 2 p
2p 2p 2p
\ ò f( x) dx + ò f -1 ( x) dx = ò 2 p dx = 4 p
2

0 0 0
2p 2p
éx2 ù
Þ ê - cos x ú + I = 4 p 2 Þ I= òf
-1
( x) dx = 2 p 2
ë 2 û0 0

é 1 1ù é é 1 ù 1 ù
4 4 4 4 4
43. = 2 ê2 e - x dx - 8 x 4 e - x dx ú = 2 ê2 ê( xe - x ) 10 + 4 x 4 e - x dx ú - 8 x 4 e - x dx ú
ò ò ò ò
êë 0 0 úû êë êë 0 úû 0 úû
4
=
e
Indefinite and Definite Integration 109

46. Put y - 2 = z
2
z2 +1
I= ò sin( z ) dz = 0
2
-2 2 z +3
4
3 sin x 3
47. òxe dx
1

Let x 3 = t Þ 3 x 2 dx = dt
64 sin t
e
ò t
dt = F (64) - F (1)
1
x
2
x × et dt
ò
x
2
-x2
51. lim x et ò dt = lim 0
2
x® ¥ x ®¥
0 ex
Apply L’ Hospital’s rule,
x æ x ö
2 2 2
x × ( e x ) + et dt × 1 ç et dt ÷
ò ç1 ò ÷ 1
0
lim 2
= lim ç + 0 2
÷=
x ® ¥ç 2
x® ¥
e x ×2x 2x ex ÷ 2
ç ÷
ç ÷
è ø
n 1
2 ×r + n (2 x + 1) dx
52. L = å r 2 + n × r + n 2 ò x 2 + x + 1 = ln( x 2 + x + 1)|10
=
r =1 0

L = ln 3
3
53. Let x 2 + 2 x = y = f( x)
x = -1 + ( y 3 + 1) 1/ 2
2
I = ( f -1 ( x) + f( x) + 1) dx
ò
0
2 2
Consider ò f -1 ( x) = tf ¢(t ) dt
ò Let f -1 ( x) = t ; x = f(t ); dx = f ¢(t ) dt
0 0
2
= tf(t )|20 - dx = 6
ò
0
p/ 2
æ ln 2 + ln tan q ö
54. Put x = 2 tan q then I = ò çç ÷÷ 2 sec 2 q dq then solve it.
2
0 è 4 sec q ø
110 Solution of Advanced Problems in Mathematics for JEE

55. Put x - 5 = t
x = 0, t = - 5
x = 10 , t = 5
5 5
2 3 t3 250
ò (t + t + t ) dt =
3
=
3
-5 -5
¥
dx
56. Let I= ò (1 + x 9 )(1 + x 2 ) …(1)
0
1 1
Put x = Þ dx = - dt
t t2
dt
0 - ¥
t2 t 9 dt
I= ò = ò (t 9 + 1)(1 + t 2 ) …(2)
æ 9 öæ
¥ ç t + 1÷ ç 1 + t
2 ö 0
÷
ç t9 ÷ç t2 ÷
è øè ø
On adding (1) & (2),
¥
dt -1
¥
2I = ò (1 + t 2 ) = tan t
0
0
p p
2I = Þ I=
2 4
p/ 2 p/ 2
æ 1 + sin 3 x ö 1 + 3 sin x - 4 sin 3 x
57. I = ò ç ÷ dx = ò dx
0
è 1 + 2 sin x ø 0
1 + 2 sin x
p/ 2
(1 + 2 sin x)( -2 sin 2 x + sin x + 1) æ1 pö p
= ò (1 + 2 sin x)
= -2 ç ÷ + 1+ =1
è2 2ø 2
0
-1 2
(tan x)
58. lim
x® ¥ 1
2x
2 x2 + 1
1+ x2 p2
lim (tan -1 x) 2 =
x® ¥ x 4
2013
59. Let t = Õ( x 2 + r 2 )
r =1
2013
ln t = å ln( x 2 + r 2 )
r =1
2013 æ 2013 x ö
1 2x
t
dt = å
x 2
+ r 2
dx Þ dt = 2 ç
ç x 2
+ r 2
÷ t dx
÷ å
r =1 è r =1 ø
Indefinite and Definite Integration 111

1
dt t 1 æç 2013 2 ö 1 æ 2013 2013 ö
\ ò = = ( x + r 2 )÷ = ç
Õ (1 + r 2 ) - r2 ÷Õ Õ
ç
2 2 2 è r =1 ÷ ç ÷
ø 0 2 è r =1 r =1 ø
1 1 æ 2x ö
60. f ¢( x) = 2 - - ç1 + ÷ =2 - 1 - 1
2 2 ç ÷ 2
1+ x x + 1+ x è 2 1+ x2 ø 1+ x 1+ x2
æç (1 + x 2 ) - 1 + x 2 ö÷ + x 2
2(1 + x 2 ) - 1 - 1 + x 2 è ø
= = >0" xÎR
1+ x2 1+ x 2

p/ 2 p/ 2
sin x cos x dx xæ x xö
61. I = ò = ò sin ç cos - sin ÷ dx
1 + sin x + cos x 2è 2 2ø
0 0
p/ 2
1 p
=
2 ò [sin x + cos x - 1] dx = 1 - 4
0
7
cos x 2 dx
62. I = ò cos x 2 + cos(10 - x) 2
3
7
cos(10 - x) 2 dx
I= ò cos(10 - x) 2 + cos x 2
3

Þ 2I = 4 Þ I = 2
1 e2 1 e2
ln x ln x é(ln x) 2 ù é(ln x) 2 ù 1 5
63. - ò dx + ò dx = - ê ú +ê ú =2 + =
-1
x x ë 2 û e -1 ë 2 û 1 2 2
e 1
cosec 2 x

ò tg(t ) dt
cosec 2 x g(cosec 2 x) 2cosec x( -cosec x) cot x -16
2
64. lim = lim = g(2)

p p2 x®
p 2x p
4 x2 - 4
16
n
n-k 4k
65. lim
n® ¥ n
å 2
cos
n
k =1
æ kö
n ç1 - ÷ 1
è nø æ 4k ö
lim
n® ¥
å
n
cos ç
è n ø
÷ = (1 - x) cos 4 x dx ò
k =1 0
1 1 1
sin 4 x sin 4 x 1
= (1 - x) + ò dx = - cos 4 x
4 0 4 16 0
0
1 1
= - (cos 4 - 1) = (1 - cos 4)
16 16
112 Solution of Advanced Problems in Mathematics for JEE

é1/ n p
2/ n
2p
1
np ù
66. lim ê sin ò dx + sin
ò dx +¼¼ + sin dx ú
ò
n® ¥ê 2n 2n 2n ú
ë 0 1/ n 1-1/n û
1é p 2p np ù
lim sin + sin +¼¼ + sin ú
n® ¥ n êë 2n 2n 2n û
æ np ö
sin ç ÷
è 4n ø æ (n + 1) p ö 2
lim sin ç ÷=
n® ¥ p è 4n ø p
n sin
4n
n +1
1 1 1 1 n+3
67. ò min.{| x - 1|,| x - 2|,| x - 3|, ¼¼| x - n|} dx = 2 (1) + 2 ´ 2 ´ (n - 1) + 2 ´ (1) = 4
0

1/2

1 2 3 n–1 n n+1

1 æ kp ö
68. S k = k sin ç ÷
2 è 2n ø
n 1
1 1 kp 1 æ px ö 2
lim
n® ¥ n 2
å 2
k sin
2 n
=
2
x sin ç ÷ dx =
ò 2
è ø p2
k =1 0
g( x)
dt
71. f( x) = ò g¢( x) = - sin x × (1 + sin(cos x)) 2
3
0 1+ t
1
f ¢( x) = g¢( x) ×
1 + ( g( x)) 3
æpö æpö p
f ¢ ç ÷ = g¢ ç ÷ = - sin = -1
è2ø è2ø 2
x
72. x 2 f( x) = ( 4t 2 - 2 f ¢(t )) dt
ò
4

x f ¢( x) + 2 x f( x) = 4 x 2 - 2 f ¢( x)
2

16 f ¢( 4) + 8 f( 4) = 64 - 2 f ¢( 4)
18 f ¢( 4) = 64
9 f ¢( 4) = 32
Indefinite and Definite Integration 113

2
æ rö
2n 2n ç ÷ 2 2
r2 1 ènø x 2 dx 1 1
73. lim å n 3 + r 3 = nlim å = ò 1+ x3 = ln|1 + x 3| = ln 9
n® ¥ ®¥ n 3 3 3
r =1 r =1 æ rö 0 0
1+ ç ÷
ènø
2p p p
-1 x2 -1
74. ò cos (cos x) dx = 2 cos (cos x) dx = 2 ×ò = p2
2
0 0 0
x
75. 2 f( x) = ( x 2 - 2 xt + t 2 ) g(t ) dt
ò
0
x x x
2 f( x) = x 2 g(t ) dt - 2 x t × g(t ) dt + t 2 g(t ) dt
ò ò ò
0 0 0
x x
2 f ¢( x) = x 2 × g( x) + ò g(t ) dt × 2 x - 2 x( x g( x)) - t × g(t ) dt × 2 + x 2 g( x)
ò
0 0
x x
2 f ¢( x) = 2 x g(t ) dt - 2 tg(t ) dt
ò ò
0 0
x
f ¢¢( x) = x × g( x) + ò g(t) dt - xg( x)
0
x
f ¢¢( x) = g(t ) dt
ò
0

f ¢¢¢( x) = g( x)
p p/ 2
x 3 cos 4 x sin 2 x 2
76. I = ò p 2 - 3 px + 3 x 2 dx = l ò sin x dx
0 0
p 3 4 2 p
( p - x) cos x sin x
I= ò dx Þ 2 I = p cos 4 x sin 2 x dx
ò
0 p 2 - 3 px + 3 x 2 0
3
1 æ 2x ö 3 p p
77. × tan -1 ç ÷ = tan -1 x = -0 =
2 è1- x2 ø 0
0 3 3
3 3 1 2 3
[ x] [ x]
78. ò { x} dx = ( x - [ x])
ò dx = 1 × dx + ( x - 1) dx + ( x - 2) 2 dx
ò ò ò
0 0 0 1 2
1 -1
tan x
79. I = ò dx
x
0

x = tan q
114 Solution of Advanced Problems in Mathematics for JEE

p/ 4 p/ 4 p/ 2
q 2q 1 t
I= ò × sec 2 q dq = ò dq = ò dt
tan q sin 2q 2 sin t
0 0 0
4/ p 4/ p 4/ p 4p 4/ p
1 1 1 1 æ -1 ö 3 1
80. ò 3 x 2 sin dx - ò x cos dx = sin × x 3 - ò cos ×ç ÷ × x dx - ò x cos dx
123 x x x x è x2 ø x
II 123
0 0 0 0 0
I

64 1 æ 1 ö 32 2
× - lim ç x 3 sin ÷ =
3
p 2 x ® 0 è xø p3
3x
x +1
æ 2 28 t ö 2
81. ç 8t +
ò + 4 ÷ dt =
è 3 ø log ( x +1) x + 1
-1

x
3x
+1
8t 3 14t 2
+ + 4t = 2
3 3 1
-1
2
8 x 3 14 x 2 æ -8 14 ö
+ + 4x - ç + - 4÷ = 3x + 2
3 3 è 3 3 ø
8 x 3 + 14 x 2 + 12 x + 8 - 14 + 12 = 9 x + 6
8 x 3 + 14 x 2 + 3 x = 0
x(8 x 2 + 14 x + 3) = 0
x(2 x + 3)( 4 x + 1) = 0
3 1
x = 0, - ,-
2 4
But x > -1 & x ¹ 0
1
So, x = -
4
4 x 4
85. f( x) = e| x -t|dt = e( x -t ) dt + e(t - x ) dt = e x + e 4- x - 2 ³ 2 e 2 - 2
ò ò ò
0 0 x
¥ ¥ ¥ ¥
1 4 cos 3 x 1 cos 3 x + 3 cos x 1 cos 3 x 3 cos x
86. ò dx = ò dx = ò dx + ò dx
4 x 4 x 4 x 4 x
0 0 0 0

æ x x öæ x xö æ 2 x xö
87. ò çè sin 2 + cos 2 ÷ø çè cos 2 - sin 2 ÷ø dx = ò çè cos - sin 2 ÷ dx = cos x dx ò
2 2ø
Indefinite and Definite Integration 115

p p p
sin(2nx + x) sin 2nx × cos x cos 2nx × sin x
88. I 1 = ò dx = dx + ò ò dx
n+ sin 2 x sin 2 x sin 2 x
2 0 0 0
p p
1 sin 2nx 1 cos 2nx
=
2 ò
sin x
dx +
2 cos x
dx ò
0 0
1
89. f ¢( x) = 1 + ln 2 x + 2 ln x = 0 Þ x =
e
1/ e
æ 1ö 1 2
f ç ÷ =1+ + ò (ln t + 2 ln t ) dt
è eø e
1
1/ e
2
Let I = ò (ln t + 2 ln t ) dt
1
-1
1
ln t = x Þ t = e ; dt = e dx = ( x 2 + 2 x) e x dx = [e x × x 2 ]-01 =
x x
ò e
0
x
90. f( x) = x 2 + e -t f( x - t ) dt
ò
0
x x
x 2 + et - x f(t ) dt = x 2 + e - x et f(t ) dt
ò ò
0 0
x
Þ f ¢( x) = 2 x - e - x òe
t
f(t ) dt + f( x)
0

x3
Þ f ¢( x) = 2 x + x 2 Þ f( x) = + x2
3
1
Þ y= ( -2 x 2 + 6 x - 1)
4
p/ 2 p/ 2
cos 2 x cos 2 x
91. I= ò dx = ò dx
-p/ 2 1+ 5x -p/ 2 1 + 5-x
p/ 2
2
Þ 2I = ò cos x dx
-p/ 2
p/ 2
2 p
Þ I= ò cos x dx =
4
0
116 Solution of Advanced Problems in Mathematics for JEE

æ x 2 - x + 1 ö cot -1 x
92. ò ççè x 2 + 1 ÷ø
÷e dx

-1
Let cot -1 x = 1 Þ dx = dt
1+ x2
t 2
ò e (cot t - cosec t ) dt = et × cot t + c
-1
= x × e cot x
+c
2 1 1
1 n æç r n 2 + r 2 ö
÷ = lim 1
n
r æ rö é( 1 + x 2 ) 3 / 2 ù
93. lim å 1 + ç ÷ = x 1 + x 2 dx = ê
å ò ú
x ®¥ n ç
r =1 è n2 ÷ x ®¥ n
ø r =1 n ènø 0 ë 3 û0

( x 3 - 1) x3 1 1 4
94. ò ( x 4 + 1)( x + 1) dx = ò x 4 + 1 dx - ò x + 1 dx = 4 ln(1 + x ) - ln(1 + x) + c

(cos -1 cos x)( - sin x) - x sin x 1


95. lim = lim =-
x ®0 + 2 - 2 cos 2 x +
x ® 0 4 sin x2 4
0 x > tan1

96. f(x) cos x 0 < x < tan1


cos x
x = tan1
2
¥ tan 1 ¥

ò f( x) dx = ò cos x + ò 0 dx = sin(tan 1)
0 0 tan 1
æ k ö
n ç ÷ 1
æ k ö 1 n ç n ÷ = x dx
97. lim
n ®¥
ç 2 å ÷ = lim
n ®¥ n ç å 1 2k ÷ ò
k =1 è n + n + 2 k ø k =1 1 + + 0
ç n n 2 ÷ø
è
y

ò|t - 1|dt y -1
1
98. lim Þ lim =0 (Applying L’ Hospital Rule)
+ tan( y - 1) + 2
y ®1 y ®1 sec ( y - 1)
1
1 dx é x ù 5 1 dx
99. ò =ê ú + ò0 (1 + x 2 ) 3
2 4 2 4 -1
0 (1 + x ) êë 2 ( 4 - 1)(1 + x ) úû 0 6
1
æ 1 ö 5é x ù 5 3 1 dx
=ç - 0÷ + ê + × ò0 (1 + x 2 ) 2
ç 6 ( 2) 3 ÷ 6 ê 2 (2)(1 + x 2 ) 2 úú
è ø ë û0 6 4
Indefinite and Definite Integration 117

1
1 æ 5ö é 1 ù 5é x ù 5 1 1 dx
= +ç ÷ê - 0ú + ê ú + ´ ò0 1 + x 2
48 è 6 ø ë 16 2
û 8 ëê 2 (1)(1 + x ) úû 0 8 2
1 5 5æ 1 ö 5
= + + ç - 0÷ + [tan -1 x]10
48 6 ´ 16 8 è 4 ø 16
7 5 5 ép ù
= + + - 0ú
6 ´ 16 8 ´ 4 16 êë 4 û
22 5p
= +
6 ´ 16 64
11 5p
= +
48 64
Alternate solution :
1 dx
I= ò0 (1 + x 2 ) 4
Put x = tan q; therefore, dx = sec 2 q dq.
p / 4 sec 2 q dq
I= ò0 (sec q) 8
That is,
p/ 4
I= ò0 (cos q) 6 dq
2
p / 4 æ 3 cos q + cos 3q ö
= ò0 ç
è 4
÷ dq
ø
9 p/ 4 1 p/ 4 3 p/4
= ò0 cos 2 q dq + ò0 (cos 3q) 2 dq + ò0 cos q cos 3q dq
16 16 8
9 p/ 4 1 + cos 2q 1 p/ 4 1 + cos 6q 3 p / 4 cos 4q + cos 2q
=
16 ò0 2
dq +
16 ò0 2
dq +
8 ò0 2
dq
p/ 4 p/ 4 p/ 4
9 é sin 2q ù 1 é sin 6q ù 3 é sin 4q sin 2q ù
= q+ + q+ + +
32 êë 2 úû 0 16 ´ 2 êë 6 úû 0 8 ´ 2 êë 4 2 úû 0
æ 9 ö é p 1ù 1 é p 1ù 3 é 1 ù
=ç ÷ê + ú + ê - ú+ ê0 + - 0ú
è 32 ø ë 4 2 û 16 ´ 2 ë 4 6 û 8 ´ 2 ë 2 û
5 11
= +
64 p 48
p/ 4
4
100. We have, I= ò (sin x) dx …(1)
0
1 - cos 2 x
We know that, sin 2 x =
2
118 Solution of Advanced Problems in Mathematics for JEE

Therefore,
2
æ 1 - cos 2 x ö
sin 4 x = (sin x) 4 = ç ÷
è 2 ø
1
= [1 - 2 cos 2 x + (cos 2 x) 2 ]
4
1æ 1 + cos 4 x ö
= ç 1 - 2 cos 2 x + ÷
4è 2 ø
1æ3 cos 4 x ö
= ç - 2 cos 2 x + ÷
4è2 2 ø
Substituting this value of sin 4 x in Eq. (1), we get
p/ 4
æ3 1 1 ö
I= ò ç - cos 2 x + cos 4 x ÷ dx
0
è8 2 8 ø
p/ 4
é3 ù 1 1
=ê xú - [sin 2 x]p0/ 4 + [sin 4 x]p0/ 4
ë8 û0 4 32
æ3 pö 1 1
= ç × ÷ - ( 1 - 0) + ( 0 - 0)
è8 4ø 4 32
3p 1
= -
32 4
Alternate solution : We have,
p/ 4
4
I= ò (sin x) dx
0

which can be written as


J = (sin 2 x)(1 - cos 2 x) dx
ò
1
= ò sin 2 x dx - ò 4 sin 2 x cos 2 x dx
4
1 - cos 2 x 1
= ò dx - (sin 2 x) 2 dx
ò
2 4
1 1 1 1 - cos 4 x
= x - sin 2 x - ò dx
2 4 4 2
x sin 2 x 1 1
= - - x+ sin 4 x + c
2 4 8 32
3 sin 2 x sin 4 x
= x- + +c
8 4 32
Indefinite and Definite Integration 119

Using the given limits, the above equation becomes


p/ 4 p/ 4 p/ 4
é3 ù é sin 2 x ù é sin 4 x ù
I = [ J ]p0/ 4 = ê x ú -ê ú +ê
ë8 û 0 ë 4 û0 ë 32 úû 0
3p 1
= -
32 4
(cos 9 x + cos 6 x) sin 5 x 5x 3x
101. ò dx = 2 cos cos
ò = (cos 4 x + cos x)
ò
sin 10 x - sin 5 x 2 2
sin 4 x
= + sin x + C
4
1
A = ,B =1
4
dx
x 2014 1 æ x 2013 ö
102. ò = ln ç ÷+C
1 2013 çè 1 + x 2013 ÷ø
1+
x 2013
1 1 2 1é 2 1 2 ù
103. x × (2 x × e - x ) dx = êæç - xe - x ö÷ + e - x dx ú
ò ò
2 0 2 ëè ø 0 û
1é 1 ù
= ê - + aú
2ë e û
é 1 2 3 ù 4 5
104. 2 ê ò0 f( x) dx + ò1 f( x) dx + ò2 f( x) dx úû + ò3 f( x) dx + ò4 f( x) dx
ë
é 0 2 12 2 2 ù 3 2 4 2 35
=2ê + + ú+ + =
ë 2 2 2 û 2 2 2

1 3x2
105. ò x 6 (1 + x 3 ) 2 dx
3
Let 1 + x 3 = t Þ 3 x 2 dx = dt
1 dt 1 æç 2 1 2 1 ö
÷ dt
Þ
2 ò
3 t (t - 1) 2
= + - +ò
3 çè t t 2 t - 1 (t - 1) 2 ÷
ø
3n 3
1 1
106. lim å = ò dx = (2 1 + x ) 30 = 2
n ®¥
r =1 r 0
1+ x
n 1+
n
2 2 2 2 2
éx2 ù x x2
107. x f( x) dx = ê
ò f ( x)ú - f ¢( x) dx = 0 + ò ò2 dx
0 ë 2 û0 0 2 0 1+ x3
120 Solution of Advanced Problems in Mathematics for JEE

p/ 3
108. ò (ln(cos x + 3 sin x) - ln cos x) dx
0
p/ 3
ì æ æ p öö ü p
= ò ílnç 2 cos ç x - ÷ ÷ - ln cos x ý dx = ln 2
î è è 3 øø þ 3
0
100 1 1 1 1 1
109. åò f( r - 1 + x) dx = ò f( x) dx + ò f( x + 1) dx + ò f( x + 2) dx +¼ + ò f( x + 99) dx
r =1 0 0 0 0 0
1 2 3 100 100
= ò f( x) dx + ò f( x) dx + ò f( x) dx +¼ + ò f( x) dx = ò f( x) dx = a
0 1 2 99 0
n k 1 2
æ (2 x) ö e -1
110. lim
n ®¥
å x 2 çç k !
÷ = x 2 × e2x Þ
÷ òx
2 2x
e dx =
4
k =0 è ø 0
5
111. òx 1 + x 3 dx

Let 1 + x 3 = t 2
3 x 2 dx = 2t dt
2 2 2 2 æ t 5 t 3 ö÷
t (t - 1) dt = ç
ò - +c
3 3 çè 5 3 ÷ø
sin x
112. f ¢( x) =
x
f ¢( x) > 0 " x Î (0 , p)
f ¢( x) < 0 " x Î ( p , 2 p)
x( x 2 + 1) + 3 ( x 2 + 3)
113. ò dx
( x 2 + 1)( x 2 + 3)
æ x 3 ö
ò çè x 2 + 3 + x 2 + 1 ÷ø dx
1
ln| x 2 + 3| + 3 tan -1 x + c
2
sec 5 x sec 4 x
114. ò dx = ò dx
sin 3 x tan 3 x
Let tan x = t 2
sec 2 x dx = 2t dt
(1 + t 4 ) × 2t × dt æ 1 ö
ò =2 ç + t 2 ÷ dt
ò
3 2
t èt ø
Indefinite and Definite Integration 121

115. Let tx = y Þ x dt = dy
x2
sin y
òe dy 2
e sin x 2 x
0
lim = lim =1
x ®0 x2 x ®0 2x
p/ 2 p/ 2 p/ 2 p
cos 2 x æ sin 2 x ö sin 2 x sin q
116. ò dx = ç ÷ +2 ò dx = -1 + ò dq (Q Let 2 x = q)
2
0
x è 2x ø0 0 (2 x) 0 q2

Exercise-2 : One or More than One Answer is/are Correct

dx
1. ò (1 + Let x = t 2 Þ dx = 2t dt
x)8

2t dt é dt dt ù é 1 1 ù
ò (1 + t) 8 = 2 êë ò (t + 1)7 - ò (t + 1) 8 úû = 2 êêë - 6 (1 + t) 6 + 7(1 + t)7 úúû + C
a a
2. é e x + cos x ln( x + 1 + x 2 )ù dx = 2 e x dx = 2 ( e a - 1) Þ e a > 7
ò ëê ûú ò 4
-a 0

x
3. I = ò dx Let x 3/ 2 = a 3/ 2 cos q
a - x3
3

x 2 a 3/ 2 cos q
= ò ( a 3/ 2 ) 2 - ( x 3/ 2 ) 2
dx =
3 ò a 3 - a 3 sin 2 q
dq

2 2 2 æ x 3/ 2 ö
= dq = q + C = sin -1 ç
ò ÷+C
3 3 3 ç a 3/ 2 ÷
è ø
æ ö tan 2 x tan 2 x
4. ò x sin x sec
3
x ç tan x sec 2 x ÷ dx = x
x dx = { ò - dx ò
ç 14 4244 3÷ 2 2
I è II ø
tan 2 x (sec 2 x - 1) tan 2 x 1
=x - ò dx = x - (tan x - x) + C
2 2 2 2
1
= ( x sec 2 x - tan x) + C
2
\ f( x) = sec 2 x , g( x) = tan x
(a) Clear f( x) Ï( -1, 1)
(b) tan x = sin x
122 Solution of Advanced Problems in Mathematics for JEE

Þ cos x = 1 Þ tan x is not defined.


no solution
p
(c) g¢( x) = f( x) " x Î R except (2n - 1)
2
(d) sec 2 x = tan x
1 + tan 2 x - tan x = 0 has no solution.
sin q
5. ò (sin 3q + sin q) cos q e dq = ( 4 sin q - 4 sin 3 q) e sin q cos q dq
ò
Let t = sin q
dt = cos q dq = 4 [-t 3 + 3t 2 - 5 t + 5] et + C
Compare it
A = -4 , B = -12 , C = -20
q q
2 x dx 2 (q - x) dx
7. I = ò = ò
0
(3q - 2 x)(q + 2 x) 0
(3q - 2 x)(q + 2 x)
q
q dx
Þ I= ò
2 q 2 - ( x - q 2) 2
0

8. Let f( x) = a x , F ( x) = F ( - x)
0 2
é -1 æ 1 ö -1 ù æ -1 æ1ö -1 ö
9. J = ò êcot ç x ÷ + cot ( x)ú dx + ò çè cot ç ÷ + cot x ÷ dx
-1 ë
è ø û 0
è xø ø
0 2
æ pö p
= ò çè p + 2 ÷ø dx + ò 2 dx
-1 0
p
K = dx = p
ò (As 2p is period)
0

x - cos 2 x
11. l1 = lim =1
x® ¥ x + sin x
1
h dx -1 1
l 2 = lim ò h 2 + x 2 = hlim 2 tan =p
h® 0 + ®0 h
-1

dx sec 4 x
13. ò (1 + sin 2 x) cos 2 x ò 1 + 2 tan 2 x dx
=
Indefinite and Definite Integration 123

(1 + tan 2 x) sec 2 x 1 1 sec 2 x dx


= ò dx = òsec 2 x dx + ò
(1 + 2 tan 2 x) 2 2 1 + 2 tan 2 x
1 1
= tan x + tan -1 ( 2 tan x) + c
2 2 2

(1 + sin 2015 x) - 1 + sin 4030 x


14. ò (Rationalise)
2 sin 2015 x
2014
1
ò 2
dx ò odd = 0
-2014

p
15. tan -1 (nx)|¥
a = - tan -1 (na)
2
a > 0, a = 0, a < 0
16. Let x = cos 2q
dx = -sin 4q dq
p/ 4 p/ 4
I= ò cot q sin 4q dq and J = ò tan q sin 4q dq
0 0

Exercise-3 : Comprehension Type Problems

Paragraph for Question Nos. 1 to 2


2. f( x) = (2 x cos x + 6 x 2 sin x cos x - 2 x 3 sin 2 x) dx
ò
3 2

æ ö
= ç 2 x 3 cos 2 x + 3 x 2 sin 2 x ÷ dx
ò
ç 123 124 4 3 ÷
è I II ø
f( x) = x 3 sin 2 x + c
f ( p) = 0 + c = 0 Þ c = 0
f( x) = x 3 sin 2 x

Paragraph for Question Nos. 6 to 8


6. g( x) = x - A
1
A= ò f(t) dt
0
124 Solution of Advanced Problems in Mathematics for JEE

x
x3 x3 x3
f( x) = + 1 - x ( x - A) dx =
ò + 1- + Ax 2
2 2 2
0
2
f( x) = Ax + 1
1
3
A = Ax 2 + 1 Þ A =
ò 2
0

3x2
f( x) = + 1 ; min. f( x) = 1
2
3 2 3
7. x + 1= x -
2 2
3x2 - 2x + 5 = 0
D < 0, no solution
3
8. g( x) = x -
2
1 3 3 9
A= ´ ´ =
2 2 2 8

Paragraph for Question Nos. 9 to 11


a 1
9. ò f( x) dx - ò f( x) dx = 2 f( a) + 3 a + b …(1)
0 a

Diff. w.r.t. ‘ a’ on both sides,


( f( a) - 0) - (0 - f( a)) = 2 f ¢( a) + 3
2 f( a) = 2 f ¢( a) + 3
(2 f( a) - 3) = 2 f ¢( a)
2 f ¢( a)
=1
2 f( a) - 3
2 f ¢( a)
ò 2 f( a) - 3 da = ò da
ln|2 f( a) - 3| = a + c
2 f( a) - 3 = e a + c

2 f( a) - 3 = ke a

2 f( a) = ke a + 3
Indefinite and Definite Integration 125

3
Put a = 1 Þ 0 = ke + 3 Þ k = -
e
3 a
\ 2 f( a) = - e +3
e
3 3 a
f( a) = - e
2 2e
3 3 x
f( x) = - e
2 2e
Put f( x) in (1) (By taking limiting case)
a 1
æ3 3 x ö æ3 3 x ö 3
ò çè 2 - 2 e e ÷ dx - ç -
ò e ÷ dx = 3 - e a + 3 a + b
ø è 2 2e ø e
0 a

éæ 3 a 3 a ö æ 3 ö ù éæ 3 3 ö æç 3 a 3 e a ö÷ ù 3 a
ç
ê 2 2e- e ÷ - ç 0 - ÷ ú - êç - ÷ - ç - ú = 3 - e + 3a + b
ëè ø è 2 e ø û êëè 2 2 ø è 2 ÷
2 e ø úû e

3
-3 = b
2e
y
10. Length of subtangent =
( dy dx)
3 3 x
y = f( x) = - e
2 2e
dy 3 x
= f ¢( x) = 0 - e
dx 2e
dy 3
=-
dx x =1/ 2 2 e
1 æ 1 ö 3 3 1/ 2 3 æ 1 ö
when x = , y = fç ÷= - e = ç1 - ÷
2 2
è ø 2 2 e 2 è eø
3æ 1 ö
ç1 - ÷
2è eø
Length of subtangent = = e -1 = e -1
3
-
2 e
1 1 1
æ3 3 x ö 3x 3 x æ 3 3e ö æ 3 ö
11. ò f( x) dx = ç -
ò e ÷ dx = - e = ç - ÷ - ç0 - ÷
è 2 2e ø 2 2e 0 è 2 2e ø è 2e ø
0 0

æ3 3ö 3 3
=ç - ÷ + =
è 2 2 ø 2e 2e
126 Solution of Advanced Problems in Mathematics for JEE

Paragraph for Question Nos. 12 to 13


12. f3¢¢¢( x) = f0 ( x) see options or 3 times by parts.
xn æ 1 1 1ö
13. fn ( x) = ç ln x - 1 - - ¼ - ÷
|n è 2 3 nø
Paragraph for Question Nos. 14 to 15
2
æ 1ö 3
f( x) = a ç x - ÷ +
è 2ø 4
f(1) = 1 a =1
2
f( x) = x - x + 1
14. g( x) = 1 + x 2 now integrate
ex
15. ò e2x - e x + 1 ex =t

Paragraph for Question Nos. 16 to 17


1/ 2
æ 1 ö
( xe x ) 2 ç + 3÷
xe 2 x (1 + 3 x 2 ) 1/ 2 èx 2
ø
17. L = lim = lim
x ®¥ C × ( xe x ) C -1 × ( e x + xe x ) x ®¥ x C æ1 ö
C × ( xe ) × ç + 1÷
èx ø

Exercise-4 : Matching Type Problems

1
dx
dx 3
2. (A) x
ò ( x 2 + 1) x 2 + 2
=
æ 1 ö 2
ò
ç1 + 2 ÷ 1 + 2
è x ø x
2
Let 1 + =t2
2
x
æ 1 ö
4 8 ç x + 3 ÷ dx
x +x è x ø
(C) ò (1 - x 4 ) 7 / 2 = ò æ 7/2
1 2ö
ç 2 -x ÷
èx ø
1
Let - x2 =t2
x2
Indefinite and Definite Integration 127

(D) Let x = cos 2q Þ dx = -2 sin 4q dq

1- x
ò 1+ x
dx = - 2 tan q sin 4q dq
ò
3. (A) Let sin x = t Þ cos x dx = dt
1 1
dt æ 1 1 ö 1
ò (1 + t )(2 + t )
= ò çè t + 1 - t + 2 ÷ø dt = [ln(1 + t) - ln(t + 2)]0
0 0
41p / 4 p p/ 4
(B) ò|cos x|dx = 10 ò|cos x|dx + ò cos x dx
0 0 0

é p/ 2 p ù p/ 4
= 10 ê cos x dx - cos x dx ú + cos x dx
ò ò ò
êë 0 p/ 2 úû 0
0 1/ 2 1/ 2
æ1+ x ö
(C) ò [ x] dx + ò [ x] dx + ò ln ç ÷ dx
-1/ 2 0 -1/ 2
è 1- x ø
0 1/ 2
1
= ò -1 dx + ò 0 dx = - 2
-1/ 2 0
p/ 2 p/ 2
2 cos q 2 sin q
(D) I = ò dq = ò dq
0
3 ( cos q + sin q ) 0
3 ( sin q + cos q )
p/ 2
2 p
Þ 2I = ò dq =
3 3
0

p
Þ I=
6
4. (A) Common root a = b - a Þ 3 ( b - a) 2 + a( b - a) + 1 = 0 Þ 2 a 2 + 3 b 2 - 5ab + 1 = 0
1
4 x2 +
(B)
x +1
= sin 2
px
Þ x 2 = sin 2 px Þ x = ±1
2x2 2 2 2
1 1
(C) y = x ¹1 ; ¹ -2 , 1
1 1 x -1
+ -2
2 x -1
( x - 1)
7/6
æ x ö dx
(D) ò ç ÷ .
è1+ x ø x2
128 Solution of Advanced Problems in Mathematics for JEE

x +1 6 1
Let =t Þ - dx = 6t 5 dt
x 2
x
1/ 6
6 æ x ö
\ I = 6 t -7 ( -t 5 ) dt =
ò + C = 6ç ÷ +C
t è x + 1ø
5. (A) We have,
1. 5 1 2 1. 5

ò [ x 2 ] dx = [ x 2 ] dx +
ò ò [ x 2 ] dx + ò [x
2
] dx
0 0 1 2

1 2 1. 5
= 0 dx +
ò ò 1 × dx + ò 2 × dx = 0 + ( 2 - 1) + 2 (15
. - 2) = 2 - 2
0 1 2

(B) We have,
4 1 4
2 2 7
ò { x } dx = ò x dx + ( x - 1) dx =
ò + (8 - 1) - 3 =
3 3 3
0 0 1
4 4 4
Aliter : ò{ x } dx = ò x dx - [ x ] dx
ò
0 0 0

(C) We have,
æp ö
sin x + cos x = 2 sinç + x ÷
è4 ø
é æp öù
\ [sin x + cos x] = ê 2 sin ç + x ÷ ú
ë è4 øû
é æp öù
The graph of y = ê 2 sin ç + x ÷ ú is obtained from the graph of y = [ 2 sin x] by
ë è 4 øû
p
translating it by units in the direction of OX ¢. The graph so obtained is shown in figure.
4
It is evident from the graph of y = [ 2 sin( x + p 4)] that
ì1, 0£ x£p 2
ï0 , p 2 < x £ 3p 4
ï
ï-1, 3p 4 < x £ p
f( x) = [sin x + cos x] = í
ï-2 , p < x < 3p 2
ï-1, 3p 2 £ x < 7p 4
ï
î0 , 7p 4 £ x < 2p
Indefinite and Definite Integration 129

(0,–1)
p/4 p/2 3p/4 p 5p/4 3p/2 7p/4 2p
x¢ x

(0,–1)
(0,–2)


2p
\ ò [sin x + cos x] dx
0
p/ 2 3p/ 4 p 3p/ 2 7 p/ 4 2p
= ò 1 × dx + ò 0 dx + ò ( -1) dx + ò ( -2) dx + ò (-1) dx + ò 0 dx
0 p/ 2 3p/ 4 p 3p/ 2 7 p/ 4

p p p p
= + 0 - - 2 ´ + ( -1) + 0 = -p
2 4 2 4
(D) We have,
p

ò||sin x| -|cos x||dx


0
p/ 4 3p/ 4 p
= ò (|sin x| -|cos x|) dx + ò (|sin x| -|cos x|) dx + ò -(|sin x| -|cos x|) dx
0 p/ 4 3p/ 4
y

y = |sin x|

y = |cos x|

x¢ x
0 p/4 p/2 3p/2 p


p/ 4 p/ 2 3p/ 4
= - (sin x - cos x) dx +
ò ò (sin x - cos x) dx + ò (sin x + cos x) dx
0 p/ 4 p/ 2
p
+ ò -(sin x + cos x) dx
3p/ 4
130 Solution of Advanced Problems in Mathematics for JEE

= - [- cos x - sin x]p0/ 4 + [- cos x - sin x]pp// 24 + [- cos x - sin x]3p/p2/ 4 - [- cos x + sin x]p3p/ 4

= - [- 2 + 1] + [-1 + 2 ] + [ 2 - 1] - [1 - 2 ] = 4 2 - 4 = 4 ( 2 - 1)

Exercise-5 : Subjective Type Problems

x dx (cos -1 3 x) 2
1. ò + ò dx = I 1 + I 2
1 - 9x2 1 - 9x2
x dx (cos -1 3 x) 2
I1 = ò I2 = ò dx
1 - 9x2 1 - 9x2
Let 1 - 9 x 2 = t 2 Let cos -1 3 x = k
¥
x 3 dx
2. I = ò (a 2 + x 2 )5
0
p/ 2 p/ 2
1 1
I= ò sin 3 q cos 5 q dq = ò cos
3
q sin 5 q dq (Let x = a tan q)
6 6
a 0 a 0
p/ 2 p/ 2
1 3 1
2I = ò sin 2q dq = ò (3 sin 2q - sin 6q) dq
6
8a 0 32 a 6 0

1
Þ I=
24 a 6
2p
3. ò g( x) dx
0
2p

ò t f ¢(t) dt (Q x = f(t ))
3p/ 2
2p
2 3p
= ò (t cos t - t sin t ) dt = 4 p 2 - -1
2
3p/ 2

5
4. ò(x + x 3 + x) 2 x 6 + 3 x 4 + 6 x 2 dx

Let 2 x 6 + 3 x 4 + 6 x 2 = t 2 Þ 12( x 5 + x 3 + x) dx = 2t dt
1 1
= 2t 2 dt =
ò (2 x 6 + 3 x 4 + 6 x 2 ) 3/ 2 + C
12 18
Indefinite and Definite Integration 131

5. Put x = sin q
tan x
6. ò tan 2 x + tan x + 1 dx
dt
Let tan x = t , sec 2 x dx = dt, dx =
1+ t2
t dt æ 1 1 ö dt dt
ò (1 + t + t 2 ) × 1 + t 2 = ò çè 1 + t 2 - 1 + t + t 2 ÷ø dt = ò 1 + t 2 - ò 2
æ 3ö
2
æ 1ö
ç t + ÷ + çç ÷
÷
è 2 ø è 2 ø
1 æ 2t + 1 ö
= tan -1 (t ) - tan -1 ç ÷+C
æ 3ö è 3 ø
ç ÷
ç 2 ÷
è ø
2 æ 2 tan x + 1 ö
= tan -1 (tan x) - tan -1 ç ÷+C
3 è 3 ø
7. Let x 4 = t
4 x 3 dx = dt
1
-2011
æ 1ö
1 1 1 1 ç1 + ÷
1 + t 2010 1 1 (1 + t ) -2011 è tø
ò (1 + t) 2012 dt = ò (1 + t) 2012 dt + ò æ 1ö
2012
dt =
-2011
+
2011
0 0 0
t 2 ç1 + ÷ 0
è tø 0

-1 æ 1 ö 1 æ 1 ö -1 æ 1 1 ö 1 l
ç 2011 - 1÷ + ç 2011 - 0 ÷ = ç 2011 - 1 - 2011 ÷ = =
2011 è 2 ø 2011 è 2 ø 2011 è 2 2 ø 2011 m
3 3
2 2 2
8. ò ( x 2 x x + 2 x 2 x × x ln x) dx = ò x 2 x ( x + 2 x ln x) dx
1 1
3 2
x2 2 dt
ò æçè x ö ( x + 2 x ln x) dx
÷
ø
Let x x = t Þ x 2 ln x = ln t ; (2 x ln x + x) dx =
t
1

( 3 )3 ( 3 )3 33 / 2
2 dt t2 33 -1
ò t ×
t
= ò t dt =
2
=
2
= 13
1 1 1

dx (cos x - sin x) dx
9. ò (cos x - sin x)(1 + sin x cos x) = 2 ò (cos x - sin x) 2 (2 + (sin x + cos x) 2 - 1)
(cos x - sin x) dx
=2 ò ((sin 2 x + cos 2 x) - 2 sin x cos x)(1 + (sin x + cos x) 2 )
132 Solution of Advanced Problems in Mathematics for JEE

(cos x - sin x) dx
=2 ò (2 - (sin x + cos x) 2 )(1 + (sin x + cos x) 2 )
dt
=2 ò (2 - t 2 )(1 + t 2 ) where t = sin x + cos x
dt 2é 1 dt ù
=2 ò (2 - t 2 )(1 + t 2 ) = 3 êë ò 1 + t 2 dt + ò 2 - t 2 úû
2é -1 1 æ 2 + t öù
= êtan (t ) + lnçç ÷ú + C
3 êë 2 2 è 2 - t ÷ø úû
2 1
\ A= , B=
3 3 2
2 1
\ 12 A + 9 2 B - 3 = 12 × +9 2 -3 =8
3 3 2
la
10. x a × y = la ; y= (0, y1–mx1)
xa
dy xa × y (x1, y1)
= - ala x -a -1 = - a
dx x a +1
- ay 1
Þ m=
x1 y
x1 – m1 , 0
1 y 1
A = | y 1 - mx 1| x 1 - 1 = y 1 x 1 (1 + a) 2
2 m 2
1 a 1- a
= l × x 1 (1 + a) 2
2
For A to be constant 1 - a = 0
p p p
æ x 6 ( p - x) 9 ö 9
11. I ( 6, 8) 6
= x ( p - x) dx = ç -8 ÷ + 6 x 5 ( p - x) dx
ò ç
è 9 ÷
ø0 0
ò 9
0
p
6 6 5 4 3 2 1 6 ! ´ 8 ! 15
I ( 6, 8) = I ( 5, 9) = ´ ´ ´ ´ ´ ( p - x) 14 dx = ò
p
9 9 10 11 12 13 14 15 !
0
2 2 2
100 é1 2 3 102 ù
14. I = ò x dx - 0 × dx + dx +¼¼ + 2 dx +¼¼ + 9 dx ú
ê
ò ò ò ò
ê ú
0 ë0 12 22 92 û
155
I=
3
Indefinite and Definite Integration 133

1 1
sin q dx -1 æx - cos q ö
17. f(q) = ò = tan ç ÷
-1( x - cos q) 2 + sin 2 q è sin q ø -1

Clearly, f(q) is not defined when sin q = 0


q = 0, p,2p
x x x x
1 1é ù
20. f( x) = ( x - t ) 2 g(t ) dt = ê x 2 g(t ) dt - 2 x tg(t ) dt + t 2 g(t ) dt ú
ò ò ò ò
2 2ê úû
0 ë 0 0 0

é x x ù
f ¢( x) = ê x g(t ) dt - t g(t ) dt ú
ò ò
êë 0 0 úû
x
f ¢¢( x) = g(t ) dt
ò
0

f ¢¢¢( x) = g( x)
2 4
22. f(2 - x) = f(2 + x), it means it symmetric about x = 2 Þ ò f( x) dx = ò f( x) dx = 5
0 2

Let 2 - x = t ; f(t ) = f( 4 - t ) i . e. , f( x) = f( 4 - x) = f( 4 + x)
50 æ2 ö
ò f( x) dx = 25 ç f( x) dx ÷ = 25 ´ 5 = 125
ò
ç ÷
0 è0 ø
1 é1 æ
æ x2 x3 x 2n ö÷ x3 x5 x 2n +1 ö ù
23. I n = | x|ç 1 + x +
ò + +¼¼ + dx = 2 ê ç x + + +¼¼ + ò ÷ dx ú
ç 2 3 2n ÷ø ç 2 4 2n ÷
-1 è êë 0 è ø úû
1
é x2 x4 x6 x 2n +1 ù
=2ê + + +¼¼ + ú
ë1 × 2 2 × 4 4 × 6 2n × ( 2n + 2) û
0

é 1 1 1 1 ù
=2ê + + +¼¼ +
ë1 × 2 2 × 4 4 × 6 2n × (2n + 2) úû
1æ 1 1 1 ö 1æ1 1 ö
In =1+ ç + +¼¼ + ÷ =1+ ç - ÷
2 è 1×2 2 ×3 n(n + 1) ø 2 è 1 n + 1ø
1 3 p
lim I n = 1 + = =
n® ¥ 2 2 q
pq( p + q) = 3 ´ 2(5) = 30
b
25. ò|sin x|dx = 8 Þ b - a = 4p
a
134 Solution of Advanced Problems in Mathematics for JEE

a+ b
9p p 17 p
ò|cos x|dx = 9 Þ a+ b=
2
Þ a = ;b=
4 4
0
b 17 p / 4
1 1
2p
ò x sin x × dx =
2p
ò x sin x dx =2
a p/ 4
x
-y
28. f( x) = 0 Þ òe f ¢( y) dy = x 2 - x + 1
0

Þ e - x f ¢( x) = 2 x - 1 Þ f( x) = (2 x - 3)e x
p
æp ö æ 1 - cos np ö
29. I n = 2 ç -| x|÷ cos nx dx = 2 ç
ò ÷
0
è2 ø è n2 ø

æ 1 ö 40
I1 + I 2 + I 3 + I 4 = 4 ç 1 + ÷ =
è 9ø 9

❑❑❑
Area Under Curves 135

6
AREA UNDER CURVES

Exercise-1 : Single Choice Problems

1. O 1 2 3 4

–1

1£ x + 3 y < 2
1
Area of shaded region =
3
1
2. Area = 2 ( x - x 2 ) dx
ò
0
y

x
–1 O 1
1
8
3. y 2 = ( x 2 - 1) 2 = 4 (1 - x 2 ) dx =
ò 3
0
y

1
x
–1

ln 2
æ -x 1ö
4. Area = 4 ò çè e - ÷ dx = 2 - 2 ln 2

0
136 Solution of Advanced Problems in Mathematics for JEE

y
(0, 1/2)

x
(–ln2, 0) O (ln2, 0)
(0, –1/2)

5. As given relations are inverse of each other so A lies on y = x y

æ n n ö
i . e. , ç , ÷ A
ç 2 2 ÷
è n + 1 n + 1ø
O
So, required area = 8 area (OACBO) = 8 ( DOAB + area BACB ) x
BC
æ æ 2 1 ö
ç 1ç n ö ÷
=8ç ÷ + n × 1 - x 2 dx ÷
ò
ç
ç 2 è n2 + 1ø ÷ ÷
è n / n 2 +1 ø
3
æ x2 ö
ò ççè 12 x - 12 ÷÷ø dx 15
y
0
6. =
12 49
æ x2 ö
ò ççè 12 x - 12 ÷÷ø dx x
3
O 3 12

dy
8. =r
dx (1, 1)
1
Normal at (1, 1) is Þ y = - ( x - 1) + 1 P
r (1,1)
1 ((r+1),0)
1 1
Required area = x r dx + ar ( DPAB ) =
ò + r = f( r) O A r B nr
r+1 2
0
1 1
f ¢( r) = - + =0
2 2
( r + 1)
é1 1ù 2
9. Area = 4 ê 1 - x dx - ú =
ò
êë 0 2ú 3
û
y

x
(–1,0) O (1,0)
Area Under Curves 137

1 -x2
10. Area of DAOB = xe y
2
dA 2 (0,1)
= (1 - 2 x 2 ) e - x
dx 2
A(x,e–x )
1
Area is maximum at x =
2 (0,0) O B(x, 0)

2
12. ò g( x) dx
1

Let x = f(t ) Þ dx = f ¢(t ) dt


1
2 1
ò t(3t - 6 t + 3) dt =
4
0

13. x 2 y 2 + y 4 - x 2 - 5 y 2 + 4 = 0
( x 2 + y 2 - 4)( y 2 - 1) = 0
y
)
–1 (x (5, 5)
f
=
y
14. f(x
)
y =
(1, 1)
x
y=x
1 p3
15. Ar. of shaded region = Ar. of circle = .
2 2
y

x
O

16. We have,
y 2 = x (1 - x 3 ) …(1)
For x > 1, y 2 is negative. Since the square of a real number cannot be negative, y does not exist
1
at x = 0 or at x = 1 ; y = 0. Let x = . Therefore, from Eq. (1), we get
2
1 æ 1ö 7 y
y 2 = ç1 - ÷ =
2è 8 ø 16
7 x
y =± O 1
4
138 Solution of Advanced Problems in Mathematics for JEE

Also, for x < 0, y 2 is negative. Therefore, the required area is


1 1
2 ò y dx = 2 ( +) x 1 - x 3 dx
ò
0 0
1
=2 ò x - x 4 dx
0

17.
y

x
0 x1 p x2 2p

x2 y2
18. | x| + | y| ³ 2 and + =1
9 4
y

x
O

Ar. of ellipse - Ar. of square = p (2)(3) - 8 = 6 p - 8

Exercise-2 : One or More than One Answer is/are Correct

1. (a) f( x) = - ( x - a)( x - b)( x + c) = ( x - a)( x)( x + c) [Q b = 0]


Clearly option (a) is correct.
c b c
(b) ò f( x) dx = ò f( x) dx + ò f( x) dx > 0 (from graph)
a a b

which incorrect
b b
(c) ò f( x) dx < 0 & ò f( x) dx < 0 but second term is large negative value so option (c) is
a c
incorrect.
(d) Clearly, (d) is incorrect.
Area Under Curves 139

1 3n -1 ( r n) 1 3n ( r n)
2. T n = å
n r = 2 n 1 + ( r n) 2
, Sn = å
n r = 2 n + 1 1 + ( r n) 2
x
Let f( x) = + –
1+ x2 –
–1 1 2
(1 + x 2 ) - 2 x 2 1- x2
f ¢( x) = =
(1 + x 2 ) 2 (1 + x 2 ) 2
\ f( x) is decreasing in (2, 3).
3 3
2 3
Tn > ò f( x) dx, S n < ò f( x) dx
2 2

3. a + b=2 …(1)
4
2a
ò (a x + bx) dx = 8 Þ + b=1 …(2)
3
0
7
4. Normal y + x =
4
1/ 2
æ7 ö 2
ò ç - x ÷ - ( x + 1) dx
è4 ø –3/2 1/2
-3/ 2

Exercise-3 : Comprehension Type Problems

Paragraph for Question Nos. 1 to 3


x+a
Sol. f( x) =
2
bx + cx + 2
f( -1) = 0 Þ a = 1
x +1 1 - 2x
If y = 1 is asymptotes, then b = 0 and c = 1 Þ f( x) = and g( x) =
x +2 x -1
y = g(x)
y = f(x)

1 O
1 x
–2 –1 O x –1
–2
140 Solution of Advanced Problems in Mathematics for JEE

Paragraph for Question Nos. 4 to 6


Sol. y = e - x sin x
dy
= e - x [cos x - sin x] = 0
dx 0 p 2p 3p
p
Þ tan x = 1 Þ x = np +
4
So, (i . e. , 0 £ x £ p)
- e-x
I = e - x sin x dx =
ò [sin x + cos x] + c
2
( j+ 1)p ( j+ 1)p
e-x e - jp - p
Sj = ò e - x sin x dx = - [sin x + cos x] = ( e + 1)
2 2
jp jp
-p
1+ e
4. Put j = 0 , S 0 =
2
S 2009 e -2009p
5. = = ep
S 2010 -2010p
e
S j+1
6. = e -p
Sj

1 + e -p
¥
S 2 1 + ep
Q å S j = 1 - e0-p = =
1 - e -p 2( e p - 1)
j= 0

Exercise-5 : Subjective Type Problems

1. f( x) = x 2
1
p 1
A = 2 ( 2 - x 2 - x 2 ) dx =
ò +
2 3
0

2. f( x) = 2 ln x
1
17
A = ( - x 3 + 6 x 2 - 11x + 6 - 2 ln x) dx =
ò 4
0

4. At x = 0 , y = 0
x + 5y - y 5 = 0 Þ 1 + 5y¢ - 5y 4 y¢ = 0
at x = 0 , y = 0
1
y¢ = -
5
Area Under Curves 141

x
Equation of tangent : y = - , equation of normal : y = 5 x
5
1
Area = ´ 5 ´ 26 = 65
2
5. [ x]2 = [ y]2 y

\ [ y] = ± [ x] 5
[ y] = ± 1, 1£ x < 2 4
= ± 2, 2£ x<3 3
= ± 3, 3£ x<4 2
= ± 4, 4£ x<5 1
= ± 5, x =5 1 2 3 4 5 x
0
Now, when, x Î[1, 2) –1
then y Î [-1, 0) È [1, 2) –2
when x Î[2 , 3] –3
then y Î [-2 , - 1) È [2 , 3) –4
when x Î[3 , 4) –5

then y Î [-3 , - 2) È [3 , 4)
when x Î[4 , 5)
then y Î [-4 , - 3) È [4 , 5)
when x =5
then y Î [-5, - 4) È [5, 6)
Hence, required area = 2 ( 4) = 8 sq. unit
6. f( x) + f( z ) = f( x + z ) and f(0) = 0 and f ¢(0) = 4 Þ f( x) = 4 x
4
So, area bounded = D = ( 4 x - x 2 ) dx
ò
0
1 1
p
7. Required area = 4 ò x 2 - x 6 dx = 4 x 1 - x 4 dx =
ò 2
0 0
y

x
–1 O 1

Put x 2 = sin q
1 1
æ 5 ö 8
8. Ar = 4 (1 - x 2/ 5 ) dx = 4 ç x - x 7 / 5 ÷ =
ò è 7 ø0 7
0
142 Solution of Advanced Problems in Mathematics for JEE

7
DIFFERENTIAL EQUATIONS

Exercise-1 : Single Choice Problems

dy xy
4. =
dx x + y 2
2

y dy dt
Let =t Þ =t + x
x dx dx
dx 1+ t2 x2 1
ò =- ò dt ; ln y = - (Q y(1) = 1)
x t3 2y 2 2
y
5. ò dy = dx
ò
1- y2

- 1- y2 = x + c
Þ ( x + c) 2 + y 2 = 1
dy dx
6. ò =- ò
y ( x - 3) 2
1
Þ ln y = +c
x -3
7. Let f( x) = y
2
dy ex
- 2 xy =
dx ( x + 1) 2
8. Let x 2 y 2 = t
dy dt
2 xy 2 + 2 x 2 y =
dx dx
dt
= tan t
dx
Differential Equation 143

9. y = (C 1 cos C 2 ) cos x + (C 5 - C 1 sin C 2 ) sin x - C 3 eC 4 e - x


y = A cos x + B sin x + Ce - x
Q C 1 , C 2 , C 3 , C 4 are arbitrary constants.
10. y = e( a +1) x
y ¢ = e( a +1) x (a + 1)
y ¢¢ = e( a +1) x (a + 1) 2
dy æ f ¢( x) ö
12. - çç 1 + ÷ y = f( x)
dx è f( x) ÷ø
æ f ¢( x ) ö
- ò çç 1+ ÷ dx
è f ( x ) ÷ø e-x
I.F. = e =
f( x)
ye - x ye - x
= e - x dx + C
ò Þ = -e - x + C
f( x) f( x)
æ t2 ö
13. Equation of tangent at ç t , ÷ is
ç 2 ÷
è ø
t2 dy
y = tx - Þ t=
2 dx
Differential equation is
2
æ dy ö dy
ç ÷ - 2x + 2y =0
è dx ø dx
2
dy dt 1 dt x e-x 2
14. Let x + y = t Þ 1 + = ; + =x Þ = e-x + C
dx dx 3 2 2
t dx t ( x + y)
dy
15. - 2 y tan x = tan 2 x
dx
-2 tan x dx
I.F. = e ò = cos 2 x
1
y cos 2 x = sin 2 x dx =
ò (1 - cos 2 x) dxò
2
16. f( x) = 2 e x + 1
dy d 2 y dt
17. Let = t then =
dx dx 2 dx
dt 2tx
=
dx x 2 + 1
dy
Þ t= = 3( x 2 + 1) (Q y ¢(0) = 3)
dx
144 Solution of Advanced Problems in Mathematics for JEE

Þ y = x3 + 3x + 1 (Q y(0) = 1)
18. cy 2 = 2 x + c
1
2 cyy¢ = 2 Þ c =
yy ¢
y 2 = 2 x yy ¢ + 1
19. Let cosec y = t
Þ - cosec y cot y dy = dt
dt t 1
- =-
dx x x2
t 1 1 1
Þ = +c Þ = +c
x 2x2 x sin y 2 x 2

xdy - ydx x2 + y2
20. = dx
x2 x2
æ yö
dç ÷
è xø dx
ò 2
= ò x
æ yö
1+ ç ÷
è xø
t 3 f ( x) - x 3 f (t ) 1
21. lim = Þ 3 x f( x) - x 2 f ¢( x) = 1
t®x 2 2 2
t -x
dy 3 y -1 1 3
Þ - = Þ y= + x2 (Q f(1) = 1)
dx x x2 4x 4
2 dp(t )
22. = p(t ) - 900
dt
dp(t )
2 ò = dt ò
p(t ) - 900
2 ln|900 - p(t )| = t + c
p(t ) = 900 - 50 et / 2 (Q p(0) = 850)
p(t ) = 0 Þ t = 2 ln 18
sin y dy tan x
23. + = sec x
cos 2 y dx cos y
1 sin y
Let =t Þ dy = dt
cos y cos 2 y
dt
+ t × tan x = sec x
dx
Differential Equation 145

t × sec x = sec 2 x × dx + C
ò
sec y × sec x = tan x + C
dy
24. = ( 4 x + y + 1) 2
dx
Let 4 x + y + 1 = t
dy dt dt
4+ = Þ =t2 + 4
dx dx dx
1 ætö
tan -1 ç ÷ = x + C
2 è2ø
dy y
-
dx
25. tan q = dx x = -
y dy dy
1+ ×
x dx
y

q P

x
O
2
æ dy ö 2 y dy
Þ ç ÷ - =1
è dx ø x dx
1
ò x dx
26. I.F. = e = e ln x = x
x4
y × x = x 3 dx =
ò +c
4
27. x 3 dy + 3 x 2 y dx = y 2 dx + 2 xy dy
d ( x 3 y) = d ( xy 2 )
3
ò d( x y) = d ( xy 2 ) Þ x 3 y = xy 2
ò
Exercise-2 : One or More than One Answer is/are Correct

xdy - ydx x2 -2
1. = dx
x2 x2
æ yö æ 2 ö
ò dç ÷ = ç1 -
x
è ø è
ò ÷ dx
x2 ø
y = x2 - 2x + 2 (Q f(1) = 1)
146 Solution of Advanced Problems in Mathematics for JEE

2. I.F. = x sec x ; yx sec x = tan x + c


3. Put y = h
Þ x [ f( x + h) - f( x - h)] - h [ f( x + h) + f( x - h)] = 2( x 2 h - h 3 )
[ f( x + h) - f( x - h)]
or lim x - [ f( x + h) + f( x - h)] = lim 2( x 2 - h 2 )
h ®0 h h ®0

Þ xf ¢( x) - f( x) = x 2 Þ f( x) = x 2 + x
4. L.D.E., I.F. = 1 + sin 2 x ; (1 + sin 2 x) f = sin x + C , C = 0
5. 2 ydx + 2 xdy + (2 x 2 y 3/ 2 dx + x 3 y 1/ 2 dy) = 0
2
d( x 3 ´ y 3/ 2 ) = 0
2 d( xy) +
3
1 y sin 2 x y
6. I.F. = ; = ò dx ; = 2 cot x × cosec x dx = -2 cosec x + c
ò
3 3 3
sin x sin x sin x sin 3 x
æ æpö ö
y = -2 sin 2 x + 4 sin 3 x ççQ y ç ÷ = 2 ÷÷
è 2
è ø ø

Exercise-3 : Comprehension Type Problems

Paragraph for Question Nos. 1 to 2


x x
Sol. x g(t ) dt + (1 - t ) g(t ) dt = x 4 + x 2
ò ò
0 0

differentiate w.r.t. ‘ x’
x
3
x g( x) + ò g(t) dt + (1 - x) g( x) = 4 x + 2x …(1)
0

1. From (1)
x
3
ò g(t) dt + g( x) = 4 x + 2x
0
dy
Let g( x) + g¢( x) = 12 x 2 + 2 Þ + y = 12 x 2 + 2 (Q y = g( x))
dx
2. Put x = 0 in (1) we get g(0) = 0

Paragraph for Question Nos. 3 to 5


3. f( g( x)) = e -2 x
x × [ f( g( x))]¢ [ g( f( x))]¢
=
f( g( x)) g[ f( x)]
Differential Equation 147

2
Þ g( f( x)) = e - x
2
H( x) = e -( x -1) +1

4. f( g(0)) + g( f(0)) = 2
5. H( x) max = e

Paragraph for Question Nos. 6 to 8


g( x + h) - g( x) e x g( h) æ eh - 1ö
Sol. g¢( x) = lim = lim + lim g( x) ç ÷ (Q g¢(0) = 2)
h ®0 h h ®0 h h ®0 ç h ÷
è ø
= 2e x + g( x)
dy
- y = 2e x Þ y = 2 x e x + ce x
dx
Þ y = 2x ex (Q g(0) = 0)

Exercise-4 : Matching Type Problems

dx dx dx dx dy
1. (A) y - x = y2 + 1 Þ ( y 2 - y) = - (1 + x) Þ =-
dy dy dy 1+ x y( y - 1)
dx dx æ 2 ö
(B) y + 2 x = 10 y 3 Þ + çç ÷÷ x = 10 y 2
dy dy è y ø
2 2 ln y
ò y dy = e =y 2
I.E. = e
d( xy 2 ) = 10 y 4
dy
(C) = y¢
dx
y ¢ y ¢¢¢ = (3 y ¢¢) 2
y ¢¢¢ 3 y ¢¢
= then integrate it.
y ¢¢ y¢
(D) Put x 2 = t
dt t 1
+ =
dy y y 3
then solve it.
148 Solution of Advanced Problems in Mathematics for JEE

Exercise-5 : Subjective Type Problems

la
1. x a × y = la ; y= (0, y1–mx1)
xa
dy xa × y (x1, y1)
= -ala x -a -1 = -a
dx x a +1
-ay 1
Þ m=
x1 y
x1 – m1 , 0
1 y 1
A = | y 1 - mx 1| x 1 - 1 = y 1 x 1 (1 + a) 2
2 m 2
1 a 1- a
= l × x 1 (1 + a) 2
2
For A to be constant 1 - a = 0.
dy
2. = xy(1 + y)
dx
dy
ò
(1 + y) y
= x dx ò
x2
2y 2
=e (Q f(0) = 1)
1+ y
e2
Þ f ( 2) =
2 - e2
3. y 2 = cos 2 x + 2
dy
2y = -sin 2 x
dx
2
d2 y æ dy ö
y + ç ÷ = -cos 2 x
dx 2 è dx ø

d2 y é æ dy ö 2 ù
y4 + y3 = (cos 2 x + 2) 2 + (cos 2 x + 2) ê - ç ÷ - cos 2 x ú = 6
dx 2 êë è dx ø úû

t 2 f( x + 1) - ( x + 1) 2 f(t )
4. lim =1
t ® x +1 f(t ) - f( x + 1)
2tf( x + 1) - ( x + 1) 2 f ¢(t )
Þ lim =1
t ® x +1 f ¢(t )
Þ [ x + 1][2 f( x + 1) - ( x + 1) f ¢( x + 1)] = f ¢( x + 1)
Differential Equation 149

2 xf( x)
Þ f ¢( x) =
x2 + 1
Þ f( x) = x 2 + 1
ln( f( x)) - ln 2 f ¢( x)
Þ lim = lim =1
x ®1 x -1 x ®1 f ( x )

❑❑❑
150 Solution of Advanced Problems in Mathematics for JEE

8
QUADRATIC EQUATIONS

Exercise-1 : Single Choice Problems

1. Let 3 x / 2 = a , 2 y = b
a 2 - b 2 = 77 , a - b = 7 Þ a = 3 x / 2 = 9 Þ x = 4
b=2y =2 Þ y =1
3 3
2. f( x) = Õ( x - ai ) + å ai - 3 x = ( x - a1 )( x - a 2 )( x - a 3 ) + ( a1 + a 2 + a 3 ) - 3 x
i =1 i =1

f ( a1 ) = a 2 + a 3 - 2 a1 > 0 ( a1 < a 2 < a 3 )


f ( a 3 ) = a1 + a 2 - 2 a 3 < 0

x
a1 a3

3. x 4 - 2 ax 2 + x + a 2 - a = 0
a 2 - a(2 x 2 + 1) + x 4 + x = 0
2 x 2 + 1 ± (2 x - 1)
a=
2
a = x 2 + x, a = x2 - x + 1
1 3
a³ - , a³ (Q x Î R )
4 4
a
4. x 3 - 3 x 2 - 4 x + 12 = 0 b
g
Equation whose roots are a - 3 , b - 3 , g - 3 is
( x + 3) 3 - 3 ( x + 3) 2 - 4( x + 3) + 12 = 0
a–3
f( x) = x 3 + 6 x 2 + 5 x = 0 b–3
g–3
Quadratic Equations 151

5. | K - 1| < 3
3
-3 < K - 1< 3
-2 < K < 4
5 –1 1 3

–1
2
x 1 x - x -6 ( x - 3)( x + 2)
6. - £0 Þ £0 Þ £0
x +6 x x( x + 6) x( x + 6)
+ – + – +
–6 –2 0 3
x Î ( - 6 , - 2] È ( 0 , 3]
7. P( x) = x 4 - 8 x 2 + 15 + 2 x 3 - 6 x = ( x 2 - 3)( x 2 - 5) + 2 x( x 2 - 3)
= ( x 2 - 3)( x 2 + 2 x - 5)
Q ( x) = ( x + 2)( x 2 + 2 x - 5)
l -5 -7
8. a = 1, h = , b = 1, g = ,f= ,c =6
2 2 2
1 l 2 -5 2
5 10
l2 1 -7 2 = 0 Þ l = ,
2 3
-5 2 -7 2 6
10. \ Let f( x) =| x - a| + | x - b|
a+b–2x
Suppose a > b 2x–a–b
\ f(0) = f(1) = f( -1) b–a

f( x) = const. in [b, a] b a

So, b £ -1 < a £ 1
a - b£ 2
\ Minimum |a - b| = 2
2
x + 2x + c
12. y = Þ ( y - 1) x 2 + 2 (2 y - 1) x + (3 cy - c) = 0 ( D ³ 0)
x 2 + 4 x + 3c
D ³ 0 " y Î R and D £ 0
But at c = 0 and 1 there will be common factors among numerator and denominator.
Þ c( c - 1) < 0
2
13. f(t ) = t - mt + 2 = 0
f(2) < 0
Þ 4 - 2m + 2 < 0 Þ m > 3
152 Solution of Advanced Problems in Mathematics for JEE

3| x| 3 3
But = £ (by A.M. G.M. in equality)
2 9 6
9 + | x| + | x|
| x|
m
æ 3| x| ö 1
çç ÷÷ £ <1 [Qm > 3]
2
è9 + x ø 2m
éæ 3| x| mù
ö
So, êçç ÷÷ ú =0
êëè 9 + x 2 ø úû

14. x 2 ( x 6 - 24 x 5 - 18 x 3 + 39) = - 3 ´ 5 ´ 7 ´ 11
If ‘ x’ is integer, then there is no value of ‘ x’ .
1
15. m4 + = 119
m4
1
Þ m2 + = 11
m2
2
æ 1ö
Þ çm - ÷ = 9
è mø
1 æ 1 öæ 1 ö
m3 - = çm - ÷çm 2 + + 1÷ =|3 ´ 12| = 36
3 m øè 2
m è m ø
a
16. ax 2 + 2 bx + c = 0
b
a
ax 2 + 2 cx + b = 0
g
By condition of common root
1 a
Þ Common root a = and + b + c = 0
2 4
2c 2b
Þ b= and g =
a a
Equation whose roots are b and g is
æ 2c 2b ö 4 bc
x2 -ç + ÷ x + 2 =0
è a a ø a
æa ö
2 a 2 x 2 + a 2 x + 8 bc = 0 ç + b + c = 0÷
è4 ø
17. 9 x 2 ( x - 1) - 1( x - 1) = 0
1 1
x = 1, x =
,-
3 3
1 1
cos a = 1, cos b = , cos g = -
3 3
Quadratic Equations 153

a = 0, b + g = p
\ (Sa , S cos a ) = ( p , 1) = centre
é -1 æ pö ù é æpö ù
ê2 sin ç tan 4 ÷ , 4 ú = ê2 ç 2 ÷ , 4 ú = ( p , 4) ® point lies on the circle.
ë è ø û ë è ø û
\ Radius is 3.
11x 2 - 12 x - 6
18. y =
x2 + 4x + 2
( y - 11) x 2 + ( 4 y + 12) x + (2 y + 6) = 0 " x Î R
D³0
2
( 4 y + 12) - 4( y - 11)(2 y + 6) ³ 0
y 2 + 20 y + 51 ³ 0
( y + 17)( y + 3) ³ 0
y Î ( -¥, - 17] È [- 3 , ¥)
x +3 1
19. - ³0
x2 - x -2 x - 4
( x 2 - x - 12) - ( x 2 - x - 2)
³0
( x 2 - x - 2)( x - 4)
-10
³0
( x - 2)( x + 1)( x - 4)
Þ ( x + 1)( x - 2)( x - 4) < 0
x Î ( -¥, - 1) È (2 , 4)
20. x = 4 + 3i
( x - 4) 2 = - 9 Þ x 2 - 8 x + 25 = 0
x 3 - 4 x 2 - 7 x + 12 = ( x 2 - 8 x + 25)( x + 4) - 88 = -88
x2 + x -1
21. f( x) =
x2 - x + 1
y

5/3
1
–1
x
0 1 2 3
–1

By graph : Min. = f(0); Max. = f(2)


5
If x Î [-1, 3]; y max. =
3
154 Solution of Advanced Problems in Mathematics for JEE

22. By graph min. = f(0); max. = f(1)


if x Î [-1, 1]; y Î [-1, 1]
1 1 1
23. + =
x+p x+q r
x 2 + x( p + q - 2 r) + pq - r( p + q) = 0
If one root is a. Then other root must be -a.
p+q
p + q - 2r = 0 Þ r =
2
( p + q) 2 ( p 2 + q2 )
Product of the roots = pq - r( p + q) = pq - =-
2 2
24. If a1 x 2 + b1 x + c1 = 0 has one root a.
1
Þ a 2 x 2 + b2 x + c 2 = 0 has one root .
a
Þ c 2 x 2 + b2 x + a 2 = 0 has one root a
Condition of common root is
( a1 a 2 - c1 c 2 ) 2 = ( a1 b2 - b1 c 2 )( a 2 b1 - b2 c1 )
25. If a 2 - 5a + 3 = 0 and b 2 - 5 b + 3 = 0
Þ x 2 - 5 x + 3 = 0 has two roots a and b.
Þ a + b = 5, ab = 3
a b (a + b) 2 - 2ab 19
Sum of the roots = + = =
b a ab 3
a b
Product of roots = ´ =1
b a
a b
Equation whose roots are and is 3 x 2 - 19 x + 3 = 0
b a
26. |a - b| =|a 1 - b 1|
a 2 - 4b = b2 - 4a
a 2 - b 2 = 4 ( b - a)
( a - b)( a + b + 4) = 0
a ¹ b Þ a + b+ 4 =0
S1 - S 3 sin 2 b - cos b cos b (2 sin b - 1)
27. tan(q 1 + q 2 + q 3 + q 4 ) = = = = cot b
1 - S 2 + S 4 1 - cos 2 b - sin b sin b (2 sin b - 1)
28. ( a 2 + b 2 ) x 2 + 2 x( bd + ac) + ( c 2 + d 2 ) = 0
( a 2 x 2 + 2 acx + c 2 ) + ( b 2 x 2 + 2 bdx + d 2 ) = 0
Quadratic Equations 155

( ax + c) 2 + ( bx + d) 2 > 0
Þ This equation has imaginary roots.
29. If a , b are roots of ax 2 + bx + c = 0
Þ a + 2 , b + 2 are roots of a( x - 2) 2 + b( x - 2) + c = 0
Þ ax 2 + x( b - 4 a) + 4 a - 2 b + c = 0
30. a +b =1+ l
ab = l - 2
a + b - ab = 3
(a - 1)(b - 1) = - 2
Þ atleast one root is positive.
4
31. D ³ 0 Þ 3 k 2 + 8 k - 16 £ 0 Þ - 4 £ k £
3
a 2 + b 2 = (a + b) 2 - 2ab = k 2 - 2( k 2 + 2 k - 4) = -k 2 - 4 k + 8 = 12 - ( k + 2) 2
32. P ( x ) = ( x - 2) Q 1 ( x ) + R ( x )
Q ( x ) = ( x - 2) Q 2 ( x ) + R ( x )
Þ P ( 2) = Q ( 2)
33. a + b = - a and ab = b
if b ¹ 0 , a = 1 and b = -2
2
æ 1ö 9
x 2 + ax + b = x 2 + x - 2 = ç x + ÷ -
è 2ø 4
3
æ böæ c ö æ cö
34. x2 + ç ÷ç ÷ x + ç ÷ =0
è aøè aø è aø
a2b
2 3 3
x - (a + b) × ab x + a b = 0
ab2

35. x 2 + 2 ( a + b + c) x + 6 k( ab + bc + ca) = 0
D³0
Þ 4( a + b + c) 2 - 24 k( ab + bc + ca) ³ 0
1 æç a 2 + b 2 + c 2 ö
Þ k£ + 2÷
ç
6 è ab + bc + ca ÷
ø
also, |a - b| < c , |b - c| < a , |c - a| < b
Þ a 2 + b 2 + c 2 - 2( ab + bc + ca) < 0
a 2 + b2 + c 2 2
Þ <2 Þ k<
ab + bc + ca 3
156 Solution of Advanced Problems in Mathematics for JEE

36. 9| x|2 -18| x| + 5 = 0


Þ (3| x| - 1)(3| x| - 5) = 0
1 5
Þ x =± ,±
3 3
and x 2 - x - 2 > 0 Þ ( x - 2)( x + 1) > 0 Þ x < -1 or x > 2
37. Difference of roots is same in both equation
b2 - c = B 2 - C
38. | x - p| + | x - 15| + | x - p - 15| = ( x - p) - ( x - 15) - ( x - P - 15) = 30 - x
min. = 15
a = –1/2
39. 4 p( q - r) x 2 - 2 q( r - p) x + r( p - q) = 0
b = –1/2
1
If x = - is also the root of 4 x 2 - 2 x - m = 0
2
Þ m =2
40. Let cos x = t
Þ t Î [-1, 1]
Þ kt 2 - kt + 1 ³ 0 " t Î [-1, 1]
Case I : k ³ 0
1
x coordinate of vertex is .
2
–1 1/2 1
æ 1ö
Þ fç ÷³0
è2ø
k k
Þ - + 1³ 0
4 2
Þ k£ 4
Also, k³0
Þ k Î[0 , 4]
Case II : k < 0
Þ f(1) ³ 0 and f( -1) ³ 0
k - k + 1³ 0 and k + k + 1³ 0
–1 1/2 1
1
Þ 1³ 0 and k³ -
2
Also, k<0
é 1 ö é 1 ù
Þ k Î ê- , 0 ÷ Þ k Î ê- , 4ú
ë 2 ø ë 2 û
Quadratic Equations 157

1+ x y -1
41. =y Þ x=
1- x y +1
3
æ y - 1ö æ y - 1ö
H( y) = 3 çç ÷÷ - 2 çç ÷÷ + 5 = 0
è y + 1ø è y + 1ø
H( y) = 3 ( y - 1) 3 - 2 ( y - 1)( y + 1) 2 + 5 ( y + 1) 3 = 0
H( y) = 3 ( y 3 - 3 y 2 + 3 y - 1) - 2 ( y - 1)( y 2 + 2 y + 1) + 5 ( y 3 + 3 y 2 + 3 y + 1) = 0
H( y) = 3 ( y 3 - 3 y 2 + 3 y - 1) - 2 ( y 3 + y 2 - y - 1) + 5 ( y 3 + 3 y 2 + 3 y + 1) = 0
H( y) = 3 y 3 + 2 y 2 + 13 y + 2 = 0
H ¢( x) = 9 x 2 + 4 x + 13 Þ D < 0
H( x) > 0 " x > 0
Hence, it has one –ve real root.
42. (l2 + l - 2) x 2 + (l + 2) x - 1 < 0 " x Î R
l 2 + l - 2 < 0 Ç ( l + 2) 2 + 4 ( l 2 + l - 2) < 0
(l + 2)(l - 1) < 0 Ç 5l2 + 8l - 4 < 0
æ 2ö
l Î ( - 2 , 1) Ç l Î ç -2 , ÷
è 5ø
æ 2ö
Þ l Î ç - 2, ÷
è 5ø
l = -2 is also the solution of this equation.
43. a = 1, b = 1, g = 1, d = 1 (as) (a - 1) 2 + (b - 1) 2 + ( g - 1) 2 + (d - 1) 2 = 0
\ The roots of given equation is equal to 1.
a
\ S2 = 2 = 6
a0
44. | x - 1| + | x - 2| + | x - 3| ³ 6
Case I : x³3
3x - 6 ³ 6 Þ x ³ 4
Case II : 2< x<3
x³6 (Not possible)
Case III : 1£ x £ 2
4-x³6
Þ x £ -2 (Not possible)
Case IV : x<1
6 - 3x ³ 6
x£0
x Î ( -¥, 0] È [4 , ¥)
158 Solution of Advanced Problems in Mathematics for JEE

45. x x
0 3 0 3

Case-I : f(0) > 0 Ç f(3) £ 0 Case-II : f(3) > 0 Ç f(0) £ 0


a
46. x 3 + 3 px 2 + 3 qx + r = 0 b
g

2 1 1
= + ¼¼ (Q a , b , g are in H.P.)
b a g
3 1 1 1 ab + bg + ag
Þ = + + =
b a b g abg
r
Þ b=- which satisfy the given equation.
q
47. 4 y 2 + 4 xy + ( x + 6) = 0 " y Î R
D ³ 0 Þ x2 - x -6 ³ 0
48. log cos x 2 (3 - 2 x) < log cos x 2 (2 x - 1)

0 < cos x 2 < 1 Ç 3 - 2 x > 2 x - 1 Ç 3 - 2 x > 0 Ç 2 x - 1 > 0


x<1 x<3 2 x>12
a
49. px 2 + qx + r = 0
b
Þ ab < 0
a( x - b) + b ( x - a ) 2 = (a + b) x 2 - 4abx + ab (a + b) = 0
2

Product of roots = ab < 0


D = 16a 2b 2 - 4ab (a + b) 2 = - 4ab (a - b) 2 > 0
a
3 2
50. x + 2 x - 4 x - 4 = 0 b
c
1/a
4x3 + 4x2 - 2x - 1 = 0 1/b
1/c
1 1
q = 1, r = - ,s=-
2 4
51. log 2 ( x 2 + 3 x) £ 2
0 < x2 + 3x £ 4
Quadratic Equations 159

52. k - 2 > 0 Ç D < 0


k > 2 Ç ( k + 6)( k - 4) > 0
Þ k> 4
53. ab < 0
3m - 8 8
<0 Þ 2<m<
m -2 3
æ x2 + x ö
54. log 6 ç ÷>1
ç x+4 ÷
è ø
x2 + x x 2 - 5 x - 24 ( x - 8)( x + 3)
>6 Þ >0 Þ >0
x+4 x+4 x+4
a
55. ax 2 + c = 0
b
c
a + b = 0, ab =
a
a 3 + b 3 = (a + b)(a 2 + b 2 - ab) = 0
56. ( k - 1) x 2 - ( k + 1) x + ( k + 1) > 0 " x Î R
k - 1 > 0 Ç ( k + 1) 2 - 4( k - 1)( k + 1) < 0
k>1 Ç ( k + 1)(3 k - 5) > 0
5
Þ k>
3
b æ 4a ö
57. y = - 2 x 2 - 4 ax + k; abscissa corresponding to the vertex is - i . e. , ç ÷ = -2 Þ a = 2
2a è -4 ø
now, y( - 2) = 7
7 = - 8 + 16 + k Þ k = -1
58. If a + b + c = 0
Sum of coefficient ( b + c - a) + ( c + a - b) + ( a + b - c) = a + b + c = 0
Þ x = 1 is one root of the equation.
a+ b-c
Þ other root =
b+ c-a
a
59. x 3 - ax 2 + bx - c = 0 –a
b

Sum of roots a - a + b = a Þ b = a
If b is root of the equation, then ab = c.
60. a ¢b ¢ = 2 q 2 - r = 2a 2b 2 - (a 4 + b 4 ) = - (a 2 - b 2 ) 2 < 0
160 Solution of Advanced Problems in Mathematics for JEE

62. In DABC ,
A B C A B C
cot
+ cot + cot = cot × cot × cot
2 2 2 2 2 2
A B C A C B
If cot , cot , cot are in A.P. then cot + cot = 2 cot
2 2 2 2 2 2
A C
Þ cot cot = 3
2 2
63. f( x) = x " x Î [-9 , 9]
64. (3| x| - 3) 2 =| x| + 7
Þ (| x| - 2)(9| x| - 1) = 0
1 1
| x| = 2 , Þ x = ± 2, ±
9 9
y = x ( x - 4)
D f : ( -¥, 0] È [4 , ¥)
65. x 2 + 3| x| + 2 = 0 Þ (| x| + 2)(| x| + 1) = 0
a
66. x 2 - bx + c = 0
a+1
b-1
Sum of roots 2a = b - 1 Þ a =
2
2
æ b - 1ö æ b - 1ö 2
If a is the root of equation, then ç ÷ - bç ÷ + c = 0 Þ b - 4c = 1
è 2 ø è 2 ø
3 x 2 + 9 x + 17
67. y =
3x2 + 9x + 7
3 ( y - 1) x 2 + 9 ( y - 1) x + (7 y - 17) = 0
y - 1 ¹ 0 then D ³ 0
81( y - 1) 2 - 12 ( y - 1)(7 y - 17) ³ 0
( y - 1)( y - 41) £ 0
1 < y £ 41
x2 + 2x + 7
68. -6 < 0" xÎR
2x + 3
x 2 - 10 x - 11
<0
( 2 x + 3)
( x - 11)( x + 1)
<0
2x + 3
æ 3ö
x Î ç -¥, - ÷ È ( - 1, 11)
è 2ø
Quadratic Equations 161

3x - 2 5y + 2 ì3 ü
69. y = Þ x= Þ yÎR -í ý
7x + 5 3 -7y î7 þ
x +2
70. £ 0 Þ x Î [ - 2 , 4)
x -4 a b
2 –2
x - ax - 4 £ 0 4

f( -2) ³ 0 Ç f( 4) > 0
a³0 Ça<3
Þ a Î[0 , 3)
71. P( x) = ( P - 3) x 2 - 2 Px + (3 P - 6) " x Î R
P -3 > 0 Ç D =0
x
P>3 Ç P 2 - ( P - 3)(3 P - 6) = 0
Þ P =6
72. Graph is downward Þ a < 0 y

Graph cut y-axis Þ c > 0


-b
x-coordinate of vertex < 0 Þ b< 0 x
2a O

73. ax 2 + bx + c = 0 does not have real roots.


D<0

a>0 a<0
x

x Î [–4, 1]
y
y = x2–3x+5

5
74. 11
4
x
–4 0 1 3/2
y Î[3 , 33]
75. 3 x 2 - 17 x + 10 = 0 Þ ( x - 5)(3 x - 2) = 0
If x = 5 is common root, then m = 0
2 26
If x = is common root, then m =
3 9
162 Solution of Advanced Problems in Mathematics for JEE

76. x 2 + ( y + 2) x - ( y 2 + y - 1) = 0
æ 8ù
D ³ 0 Þ ( y + 2) 2 + 4 ( y 2 + y - 1) ³ 0 Þ y Î ç -¥, - ú È [0 , ¥)
è 5û
77. If x = 3 is root of this equation, then k = -5
Þ 3 x 4 - 6 x 3 - 5 x 2 - 8 x - 12 = ( x - 3)(3 x 2 + 4)( x + 1)
( 4 + sin 4 x)
78. a = - put sin 2 x = t Þ t Î[0 , 1]
2
sin x
æ4 ö
a = - ç + t ÷ = f (t )
èt ø O 1
4 –5
Here, f ¢(t ) = - 1> 0 y = f(t)
t2
\ For atleast one real root, a Î ( -¥, - 5]
–¥

79. ( rs) 2 + ( st ) 2 + (tr) 2 = ( rs + st + tr) 2 - 2 rst ( r + s + t ) = b 2 - 2 ( - c)( - a)


80. Let the roots be t , t + 1 and t + 2.
t + (t + 1) + (t + 2) = - a Þ 3 (t + 1) = - a
å t (t + 1) = b Þ b + 1 = 3 (t + 1) 2
a2 [3(t + 1)]2
= =3
b + 1 3(t + 1) 2
81. (3 x 2 + kx + 3)( x 2 + kx - 1) = 0
D1 = k 2 - 36 and D 2 = k 2 + 4 > 0
2 3
1 1 1 æ rö æ rö æ rö
82. = + Þ ç ÷ + ç ÷ + 1=0 Þ ç ÷ =1
r+s r s è sø è sø è sø
84. If x Î ( -¥, - 2] È [3 , ¥)
x 2 - 2 x - 8 = 0 Þ x = -2 , 4
if x Î ( -2 , 3)
x2 = 4 Þ x = ±2
85. 5 x 2 + 12 x + 3 = 0 has D < 0
Þ Both roots common.
86. a +b + g =6
ab + bg + ag = 5
abg = 1
Þ a 2 + b 2 + g 2 = 26
a 2b 2 + b 2 g 2 + a 2 g 2 = 13
Quadratic Equations 163

a + 5i
2 2
87. 2 x - 6 x + k = 0
a – 5i
2
Sum of roots = a = 3
a 2 + 25 k
Product of roots = = Þ k = 17
4 2
88. x 12 + x 22 = ( k - 2) 2 - 2 ( k 2 + 3 k + 5) = - ( k 2 + 10 k + 6) £ 18
89. a( x 2 - x + 1) - ( x 2 + x + 1) ³ 0
x2 + x + 1
Þ a³
x2 - x + 1
90. f(1) = l - 13 > 0 Þ l > 13
f(2) = l - 18 < 0 Þ l < 18
f(3) = l - 15 > 0 Þ l > 15
Þ l Î (15, 18)
94. D = ( b - c) 2 + 4 a(2 b + a + c) = ( b - c) 2 + ( 4 ac - 4 b 2 ) + (2 a + 2 b) 2 > 0
a
3
95. x - x + 1 = 0 b
c 1
a+1
(1 - x) 3 - x 2 (1 - x) + x 3 = 0 1
b+1
1
c+1
Þ x3 + 2x2 - 3x + 1 = 0
1 1 1
Sum of roots = + + = -2
1+ a 1+ b 1+ c
96. x 2 - 2 ( 4 k - 1) x + 15k 2 - 2 k - 7 ³ 0 " x Î R
D£0
Þ k2 - 6 k - 8 £ 0 Þ 2 £ k £ 4
x3 -1 x2 + x + 1
97. f( x) = = (Q x ¹ 1)
( x - 1)( x 2 - x + 1) x2 - x + 1
y
3

1
1/3
x
–1 0 1
164 Solution of Advanced Problems in Mathematics for JEE

2x2 + 2
98. > 0" xÎR Þ x 2 + mx + 4 > 0 " x Î R
x 2 + mx + 4
Þ D < 0 Þ m 2 - 16 < 0
99. x 2 - 2|a + 1| x + 1 = 0
D ³ 0 Þ 4 ( a + 1) 2 - 4 ³ 0 Þ a Î ( -¥, - 2] È [0 , ¥)
100. P( x) = a1 x 2 + 2 b1 x + c1 > 0 ; D1 = 4 ( b12 - a1 c1 ) < 0 , a1 > 0 , c1 > 0
Q ( x) = a 2 x 2 + 2 b2 x + c 2 > 0 ; D 2 = 4 ( b22 - a 2 c 2 ) < 0 , a 2 > 0 , c 2 > 0
f( x) = a1 a 2 x 2 + b1 b2 x + c1 c 2
D = b12 b22 - 4 a1 a 2 c1 c 2 < 0
101. x 2 - 2 x + 4 = -3 cos( ax + b)
( x - 1) 2 + 3 = -3 cos( ax + b)
Þ x = 1 and ax + b = p
102. a + b = a + a × r = 4
g + d = a × r 2 + ar 3 = 36 Þ r = 3 , a = 1
A = 3 , B = 243 = 3 5
103.

(0,2) y = |2 – |x||

y = 3–|x|
(0,1)
(–2,0) (2,0)

3 5
104. We have 4 x 2 - 16 x + 15 < 0 Þ < x < Þ cot a = 2 , the integral solution of the given
2 2
inequality and sin b = tan 45° = 1
1 1 4
\ sin(a + b) sin(a - b) = sin 2 a - sin 2 b = -1= -1= -
2 1+ 4 5
1 + cot a
105. f1 ( x) = f2 ( x)
Þ 2 + log e x = x
(1,0)

(2,0)
Þ log e x = ( x - 2)
Clearly graphs intersect once in (0,1). (0,–2)
Now check
Þ g( x) = 2 + ln x - x
g( e) > 0
g( e 2 ) < 0
Þ one root between ( e, e 2 )
Quadratic Equations 165

3 1
106. x 4 - 3 x 3 - 2 x 2 - 3 x + 1 = 0 Þ x2 - 3x - 2 - + =0
x x2
2
æ 2 1 ö æ 1ö æ 1ö æ 1ö
Þ ç x + 2 ÷ - 3 ç x + ÷ - 2 = 0Þ ç x + ÷ - 3 ç x + ÷ - 4 = 0
è x ø è xø è xø è xø
1
Þ t 2 - 3t - 4 = 0 (where x + = t )
x
Þ (t - 4)(t + 1) = 0 Þ t = 4 or t = -1
1 1
Þ x + = 4 or x + = -1
x x
1
Real solutions are from x + = 4 Þ x 2 + 1 = 4 x Þ x 2 - 4 x + 1 = 0
x
Hence, sum of roots = 4.
107. f( x) = x 2 - ( k + 4) + k 2 - 12
f( 4) = 16 - 4( k + 4) + k 2 - 12 < 0
Þ -2 < k < 6
108. a + b 2 = (a + b) 2 - 2ab = k 2 - 2( k 2 + 2 k - 4) = -k 2 - 4 k + 8
2

Maximum value = 12
109. f( x) = a x - x ln a
f ¢( x) = ( a x - 1) × ln a
110. As a , b and c are the roots of x 3 + 2 x 2 + 1 = 0 , we have
a + b + c = -2
ab + bc + ca = 0
a b c
Now, for finding the value of b c a , evaluating using first row, we get
c a b
a( bc - a 2 ) - b( b 2 - ac) + c( ab - c 2 ) = abc - a 3 - b 3 + abc + abc - c 3
= 3 abc - a 3 - b 3 - c 3
= - ( a 3 + b 3 + c 3 - 3 abc)
= - ( a + b + c)( a 2 + b 2 + c 2 - ab - bc - ca)
= - ( -2)[( -2) 2 - 3(0)] = 8
111. x 2 + px + q = 0 , p , q Î R , q ¹ 0 a , b real roots.
1 1
g( x) = 0 a+ ,b +
a b
166 Solution of Advanced Problems in Mathematics for JEE

1 1 æ 1 öæ 1ö
a +b + + = ç a + ÷ çç b + ÷÷
a b è a øè bø
-p a b 1
-p + = ab + + +
q b a ab
-p p 2 - 2q 1
-p + =q+ +
q q q
- pq - p = q 2 + p 2 - 2 q + 1
p 2 + p( p + 1) + q 2 - 2 q + 1 = 0
( q + 1) 2 - 4 ( q 2 - 2 q + 1) ³ 0
q2 + 2q + 1 - 4q2 + 8q - 4 ³ 0
-3 q 2 + 10 q - 3 ³ 0
3q2 - 2q - q + 3 £ 0
3 q ( q - 3) - ( q - 3) £ 0
é1 ù
êë 3 , 3 úû 1/3 3

112. ln( x 2 + 5 x) = ln( x + a + 3) Þ x 2 + 5 x = x + a + 3 > 0


a+3>0
–5 O
a > -3
y =x +a+3 Þ -5 + a + 3 £ 0
a£2
-3 < a £ 2
2
1 6 æ 1ö æ 1ö
113. f( x) = x 2 + - 6x - + 2 =ç x + ÷ -6ç x + ÷
x2 x è xø è xø
1
Let x + =t
x
f( x) = t 2 - 6t " t Î ( -¥, - 2] È [2 , ¥)
min. value = -9 at t = 3
æ cö
114. x 3 + 2 x 2 + 2 x + c = ( x 2 + bx + b) ç x + ÷
è b ø
c
Þ b + = 2 and b + c = 2 Þ b = c = 1
b
115. ab + bg + ag = 0 Þ (ab) 3 + (bg) 3 + (ag) 3 = 3 (ab)(ag)(bg)
Quadratic Equations 167

¥
118. å(a r + b r ) = (a + a 2 + a 3 +¼ ) + (b + b 2 + b 3 +¼ )
r =1
a b
= +
1 - a 1 -b
a
4x2 + 2x - 1 = 0
b
a
2
æ x ö æ x ö 2 1– a
4ç ÷ + 2ç ÷ - 1 = 0 Þ 5x - 1 = 0
è1+ x ø è1+ x ø b
1– b
x x x
æ 2011 ö æ 2012 ö æ 2013 ö
119. ç ÷ +ç ÷ +ç ÷ =1
è 2014 ø è 2014 ø è 2014 ø
x x x
æ 2011 ö æ 2012 ö æ 2013 ö
Let f( x) = ç ÷ +ç ÷ +ç ÷ Þ f( x) is a decreasing function for x Î R .
è 2014 ø è 2014 ø è 2014 ø
123. x 2 + ax + 12 = 0 …(1)
2
x + bx + 15 = 0 …(2) Common roots
2
x + ( a + b) x + 36 = 0 …(3)
(1) + (2) – (3)
a2 =9 Þ a = ±3
positive root a = 3
124. e sin x = t
t 2 - 4t - 1 = 0 Þ t = 2 ± 5 Þ e sin x = 2 ± 5 (Not possible)
125. Maximum value of f( x) = 3
Minimum value of f( x) = -1
126. f(1) = l - 2 < 0
a b c
127. 2 x 2 + 5 x + 7 = 0 has non-real roots Þ = =
2 5 7
Min. value of a + b + c = 2 + 5 + 7 = 14
Max. value of a + b + c = 28 + 70 + 98 = 196
128. Distance = ( x B - x A ) 2 + ( y B - y A ) 2 = (1 - 2t ) 2 + t 2 = 5t 2 - 4t + 1
1 2
Min. distance = at t =
5 5

129. a –3 2 b –3 2

af(1) < 0 and af( -1) < 0 ; f( -3) f(2) > 0


2
We have the equation ax + bx + c = 0 has two roots a and b such that a < -3 and b > 2.
168 Solution of Advanced Problems in Mathematics for JEE

If a > 0, then we have the following graphical representation :


y

–3 2
x
a b

Then, for all x Î [-3 , 2], f( x) < 0 , we have the following graphical representation :
This implies that
f( -1) < 0 and f(1) < 0 y

Þ a - b + c < 0 and a + b + c < 0


Þ a( a +|b| + c) < 0
If a < 0, then for all x Î [-3 , 2], f( x) > 0. This imply that a b
x
Þ f( -1) > 0 and f(1) > 0 –3 2
Þ a - b + c > 0 and a + b + c > 0
Þ a ( a +|b| + c) < 0
2
130. Let x + 5 x = t
t 2 - 2t - 24 = 0 = (t - 6)(t + 4)

x 2 + 5 x - 6 = 0 = ( x + 6)( x - 1)
x 2 + 5 x + 4 = 0 = ( x + 4)( x + 1)
131. Case-1: x ³ 2
4
3( x - 2) - (1 - 5 x) + 4(3 x + 1) = 13 Þ x = (Not possible)
5
1
Case-2: £ x<2
5
2
- 3( x - 2) - (1 - 5 x) + 4(3 x + 1) = 13 Þ x = (Possible)
7
1 1
Case-3: - £ x<
3 5
1
- 3( x - 2) + (1 - 5 x) + 4(3 x + 1) = 13 Þ x = (Not possible)
2
1
Case-4: x < -
3
1
- 3( x - 2) + (1 - 5 x) - 4 (3 x + 1) = 13 Þ x = - (Possible)
2
Quadratic Equations 169

132. log cos x sin x ³ 2 Þ sin x £ cos 2 x


sin 2 x + sin x - 1 £ 0
5 -1
0 < sin x £ (sin x > 0)
2
-D
133. Minimum value = -5 Þ D = 20
4
D
|a - b| = = 20
1
134. | x - 3| + | x + 5| = 7 x
2x + 2 = 7x x³3
- ( x - 3) + ( x + 5) = 7 x -5 < x < 3
- ( x - 3) - ( x + 5) = 7 x x £ -5
136. ( a + b + c) 2 = a 2 + b 2 + c 2 + 2 ( ab + bc + ac) Þ ab + bc + ac = - 4
a 3 + b 3 + c 3 - 3 abc = ( a + b + c)( a 2 + b 2 + c 2 - ab - bc - ac) Þ abc = - 4
140. x 2 - 3 x + 4 < x 2 + 3 x + 4
Þ x>0
a
2
142. x + 4 x + 3 = 0
b
a = -3 , b = -1
143. a 3 + b 3 + c 3 = 3 abc
Þ a + b+ c =0
Þ ax 2 + bx + c = 0 has one root x = 1
145. x 1 + x 2 + x 1 x 2 = a
x1 x 2 + x1 x 2( x1 + x 2 ) = b
x 12 x 22 = c Þ b + c = x 1 x 2 ( a + 1)
147. (| x| - 2)(| x| - 1) = 0 Þ x = ± 1, ± 2
149. a 2 + b 2 = (a + b) 2 - 2ab = 4 (1 - sin 2q) 2 + 4 cos 2 2q
= 4(2 - 2 sin 2q)
2
150. sin x + sin x = -b " x Î [0 , p]
0 £ -b £ 2
-2 £ b £ 0
152. x 2 + px - r = 0 = ( x - g)( x - d)
a 2 + pa - r = (a - g)(a - d) = -q - r
153. 2 x + 2 - 4 x £ 9 Ç 2 x + 2 - 4 x > 0; 2 x ( 4 - 2 x ) > 0
170 Solution of Advanced Problems in Mathematics for JEE

Exercise-2 : One or More than One Answer is/are Correct

2x - 1 – –
1. >0 + + +
x( x + 1)(2 x + 1) –1 –1/2 0 1/2
æ 1 ö æ1 ö
Þ x Î ( -¥, - 1) È ç - , 0 ÷ È ç , ¥ ÷
è 2 ø è2 ø
y

3. (0,2)
x
–2 0 3
x +2
y=
a
4. Apply D ³ 0 Ç f(2) > 0 Ç f( -2) > 0 Ç -2 < <2
2
y
5. f( x) = x - 3 x>4
=5 - x 2< x<4 3
=x +1 1< x < 2 2
1
=3 - x x<1
x
1 1 1234
6. a - b = - Þ ab = 1 (Q a ¹ b)
b a
a 1
a-b= Þ a - = a2 Þ a3 - a2 + 1 = 0
b a
7. If f(2 + x) = f(2 - x) and D > 0
æ D ö th
Vertex of parabola is ç 2 , - ÷ lies in IV quadrant.
è 4a ø
f(0) > f(1) > f(2)
8. If f(2 + x) = f(2 - x) and D < 0
f( -2) = 4 a - 2 b + c > 0
If log f ( 2) f(3) is not defined then f(2) = 1
Þ f( x) ³ 1
-b
If = 2 Þ a and b are opposite sign.
2a
9. Case-I : f( -1) ³ 0 Ç f(1) < 0 Ç f(2) ³ 0
3
a£0 Ça<0 Ça³ -
2 O1
é 3 ö –1 2
Þ a Î ê- , 0 ÷
ë 2 ø
Quadratic Equations 171

Case-II: f(1) ³ 0 Ç f( -1) < 0 Ç f( -2) ³ 0 y


3
a³0 Ça>0 Ça£
2 –1 O
x
æ 3 ù –2 1
Þ a Î ç 0, ú
è 2û
10. As expression taking minimum value
So, a > 0
-b -D
< 0; <0
2a 4a
Þ a > 0, b > 0, D > 0
2
11. ax + bx + c > 0 " x Î R
a > 0, D < 0
f ( 0) = c > 0
f( -3) + f( -2) = 13 a - 5b + 2 c > 0
f( -3) + f(2) = 13 a - b + 2 c > 0
12. D > 0 Þ k> 5 y

f(1) < 0 Þ k > 3


21 x
f(2) > 0 Þ k < 0 1 2
4
21
Þ 3< k<
4
13. x 2 + px + q = 0
Sum of the roots = -13
Product of the roots = 30

Þ x 2 + 13 x + 30 = 0 = ( x + 10)( x + 3)
Þ Correct roots are x = -10 , - 3
2
14. x - 3 x + 2 > 0
( x - 2)( x - 1) > 0 Þ x Î ( -¥, 1) È (2 , ¥)
x2 - 3x - 4 £ 0
( x - 4)( x + 1) £ 0 Þ x Î [-1, 4]
then x Î [-1, 1) È (2 , 4]
15. 5 x + (2 3 ) 2 x - 169 £ 0
5 x + 12 x - 169 £ 0
172 Solution of Advanced Problems in Mathematics for JEE

if x =2 5 2 + 12 2 = 169
x>2 5 x + 12 x > 169
x<2 5 x + 12 x < 169
Þ x Î ( -¥, 2]
16. f( x) = x 2 + ax + b
D1 : a 2 - 4 b
g( x) = x 2 + cx + d
D2 : c 2 - 4d
D1 + D 2 = a 2 + c 2 - 4 ( b + d) = ( a - c) 2 > 0 Þ atleast one of them is positive.
17. Let x - 1 = t 2
1 1 1 1
+ = +
2 2
x + 2 x -1 x -2 x -1 t + 2t + 1 t - 2t + 1
1 1 1 1
= + = +
|t + 1| |t - 1| |1 + x - 1| | x - 1 - 1|
If 1 < x < 2, then 0 < x -1< 1
1 1 1 1 2
+ = + =
|1 + x - 1| | x - 1 - 1| 1 + x - 1 1 - x - 1 2 - x
If x > 2, then x -1> 1
1 1 1 1 2 x -1
+ = + =
|1 + x - 1| | x - 1 - 1| x -1 + 1 x -1 -1 x -2
2 2
18. log 1/ 3 ( x + 2 px + p + 1) ³ 0
Þ ( x + p) 2 + 1 £ 1 Þ ( x + p) 2 £ 0 Þ x = - p
kp 2 - kp - k 2 £ 0 " k Î R
k2 + (p - p2) ³ 0 " k Î R
D£0
19. (a) a +b =a2 +b2
and ab = a 2b 2 Þ ab(ab - 1) = 0 Þ a = 0 or b = 0 or ab = 1
sin 5q np
(b) tan 2q + tan 3q = = 0 Þ sin 5q = 0 Þ q =
cos 2q cos 3q 5
æ 2 x 1 128 x 32 3
x 2 ö÷
ç + +
ç x2 x 22 4 x 1 x 32 ÷ø
è
(c) ³4 (Q AM ³ GM)
3
Quadratic Equations 173

(d) Equation of chord with mid-point ( h, k) is T = S 1


Þ ( h - 1) x + ( k - 3) y + ( h + 3 k - h 2 - k 2 ) = 0
If it is passes from (0 , 0).
Then, h 2 + k 2 - h - 3 k = 0
20. - 2 < a < 2 Þ a 2 Î[0 , 4)

x 2 - 4x - a2 = 0 Þ x = 2 ± 4 + a2
21. If a + 2b = 0
Þ ab < 0 Þ - 2b 2 < 0 Þ q < 0
a + b = -b = p
ab = - 2 b 2 = q Þ 2 p 2 + q = 0
22. f( x) = ( a + b - 2 c) x 2 + ( b + c - 2 a) x + ( c + a - 2 b) = 0 Þ f(1) = 0
(a) if a > b > c > 0 y = f(x)
Þ a + b > 2c
f ( 0) = a + c - 2 b < 0
x
–1 0 1
a
(c) g( x) = ax 2 + 2 bx + c = 0
b
g(0) = c > 0 y = g(x)
g( -1) = a - 2 b + c < 0
1/a
(d) cx 2 + 2 bx + a = 0
1/b –1
x
0

23. f( x) = 4 x 2 - 8 ax + a
D = 48 a 2 ³ 0
(a) If f( x) is non-negative " x Î R , then a = 0
(b) If a < 0, then f(0) < 0
(c) If f( x) = 0 has two distinct solutions in (0 , 1), then
f(0) > 0 Þ a > 0
4
f(1) > 0 Þ a <
7
-b
0< <1 Þ 0< a<1
2a
24. ax 2 + bx + c = 0 has no real roots, then D < 0
æ 1ö
f ç - ÷ = a - 2b + 4c > 0 Þ a > 0
è 2ø
174 Solution of Advanced Problems in Mathematics for JEE

4a + 2b + c f ( 2)
= >0
a + 3b + 9c æ 1ö
fç ÷
è3ø
25.

x = –1 x = 0 x = 1 x=3 x=4 x=5


x=2
26. ax 2 + bx + c = 0
-b ± b 2 - 4 ac -b ± i 4 ac - b 2
x= =
2a 2a
2
æ -b ö 4 ac - b 2 c
|a| =|b|= ç ÷ + =
2
è 2a ø 4a a
27. 3 x - 6 > 6 x>3
x>6 2£ x£3
4-x>6 1£ x < 2
6 - 3x > 6 x<1
28. f( x) = ax 2 + x + b - a
D < 0 and f(1) = b + 1 > 0
f ( 0) = b - a > 0
f ( 1 2) = 4 b + 2 - 3 a > 0
29. a 2 + b 2 = ( a + b) 2 - 2 ab = 7 …(1)
a 3 + b 3 = ( a + b)(7 - ab) = 10 …(2)
2
Þ ( a + b)[21 - ( a + b) ] = 20
Let ( a + b) = x
3
x - 21x + 20 = 0
( x - 1)( x + 5)( x - 4) = 0
31. a + b + g + d = 0
1 1 1 1
Root = - , - , - , -
d g b a
1
Put x ® -
x
Quadratic Equations 175

32. D ³ 0 Ç f( -1) > 0 Ç f(1) > 0


1
-2 < K £ –1 1
4
y y
a<0 a<0
33. (a) x c<0 (b) x c<0
b<0 b>0

y y
x a<0 a<0
(c) c>0 (d) x c<0
b>0 b<0

æ 1ö
34. (a) f(1) f( -1) > 0 (b) f(1) f ç - ÷ > 0
è 2ø
(c) f( -1) f( -2) > 0 (d) b 2 - 4 ac < 0
but a can be +ve or –ve.
b c
35. a + b = - , ab =
a a
b2 2 c b 2 - 2 ac
a 2 + b 2 = (a + b) 2 - 2ab = - =
a2 a a2
1 1 (a + b) 2 - 2ab b 2 - 2 ac
+ = =
a2 b2 a 2b 2 a2

1 1 (a + b)(a 2 + b 2 - ab) -b( b 2 - 3 ac)


+ = =
a3 b3 (ab) 3 c3
36. l = sin 2 x + sin x - 1
-1 < l < 1
37. x 2 + 5 x = x + a + 3 " x Î ( -5, 0)
x 2 + 4 x - 3 = a " x Î ( -5, 0)
Þ a Î ( -3 , 2]
2 2
39. x - 2 ax - a = 0 x>a
Þ x = a (1 ± 2 )
x 2 + 2 ax - 5a 2 = 0 x<a
43. (a + b) + ( g + d) = 12 Þ a +b = g + d =6 …(1)
ab ( g + d) + gd (a + b) = 54 Þ ab + gd = 9 …(2)
(ab)( gd) = 14 …(3)
176 Solution of Advanced Problems in Mathematics for JEE

Þ ab = 7 , gd = 2
æ K3 ö æ 2 ö K=a
44. l ç ÷ + mç K -3÷ + n =0 K=b
ç K - 1÷ ç K -1 ÷
è ø è ø K=c

lK 3 + mK 2 + nK - (3m + n) = 0

Exercise-3 : Comprehension Type Problems

Paragraph for Question Nos. 1 to 2


1. f( -1 - x) = f( -1 + x) " x Î R
Þ graph of f( x) is symmetric about x = -1.
b
- = -1 Þ b = 2 a
2a
a = f( -2) = 4 a - 2 b + c
b = f ( 3) = 9 a + 3 b + c
x = –3 x = –2 x=0 x=1 x=3 x axis
g = f( -3) = 9 a - 3 b + c
x = –1
Using graph f(3) > f( -3) > f( -2) Þ b > g > a
2. p = b - 4 a = - 2 a < 0
q = 2a + b = 4a > 0
Þ p ´ q< 0

Paragraph for Question Nos. 3 to 4


Sol. ( k + 1) x 2 - (20 k + 14) x + 91k + 40 = 0
f( 4) = 27 k > 0 ù
® One root is lie (4, 7)
f(7) = - 9 < 0 úû
f(10) = - 9 k < 0 ù
® Other root is lie (10, 13)
f(13) = 27 > 0 úû

Paragraph for Question Nos. 5 to 7


5. f( x) = x 2 + bx + c " x Î R
-b
Least value at = -1 Þ b = 2
2
Graph of f( x) cuts y-axis, when x = 0
Þ c =2
Quadratic Equations 177

Þ f( x) = x 2 + 2 x + 2
Least value of f( x) = 1
6. f( - 2) + f(0) + f(1) = 9
7. a Î (1, ¥)

Paragraph for Question Nos. 8 to 9


Sol. (log 2 x) - 4 (log 2 x) - m 2 - 2m - 13 = 0
2

8. D > 0 Þ m 2 + 2m + 17 > 0 " m Î R


9. m 2 + 2m - (log 2 x) 2 + 4 (log 2 x) + 13 = 0
D³0
æ 1ù
Þ (log 2 x - 6)(log 2 x + 2) ³ 0 Þ x Î ç 0 , ú È [64 , ¥)
è 4û

Paragraph for Question Nos. 10 to 11


1 -1
Sol. x 4 - 2 x 3 - 3 x 2 + 4 x - 1 = 0 has four roots a , , b,
a b
æ 1ö æ 1ö
ça + ÷ + çb- ÷ =2 …(1)
è aø è bø
æ 1ö æ 1ö
çb - ÷ -ça + ÷ = -4 …(2)
è b ø è a ø

Paragraph for Question Nos. 12 to 14


Sol. f( x) - (6 - x) = 0 = ( x - 1)( x - 2)( x - 3)( x - 4)( x - 5)
f( x) = ( x - 1)( x - 2)( x - 3)( x - 4)( x - 5) + (6 - x)

Paragraph for Question Nos. 15 to 16


Sol. x 3 - x 2 (1 + sin q + cos q) + x (sin q + cos q + sin q cos q) - sin q cos q = 0
Þ Roots are 1, sin q , cos q.

Paragraph for Question Nos. 17 to 18


Sol. 2 [1 + P( x)] = P( x - 1) + P( x + 1)
2 + 2 [ax 2 + bx + c] = a( x - 1) 2 + b( x - 1) + c + a( x + 1) 2 + b( x + 1) + c
Þ a =1
P ( 0) = c = 8
P(2) = 4 a + 2 b + c = 32 Þ b = 10
178 Solution of Advanced Problems in Mathematics for JEE

Paragraph for Question Nos. 19 to 21


3 2
2t – 9t + 30 = l
y
10

Sol. t
–2 0 2 3
–22

Paragraph for Question Nos. 22 to 23


22. D > 0
(2t - 1) 2 - 4t (5t - 1) > 0
-1 1
16t 2 - 1 < 0 Þ <t< ( t ¹ 0)
4 4
23. t > 0 y
Case-I: D < 0
1
(2t - 1) 2 - 4t (5t - 1) < 0 Þ t >
(Q t > 0)
4
-b 1 1 O
x
Case-II: D ³ 0 Ç f(0) ³ 0 Ç £0 Þ £t£
2a 5 4

Exercise-4 : Matching Type Problems

(2 x - 1) + – + – +
1. (A) >0
x(2 x + 1)( x + 1) –1 –1/2 0 1/2

(B) 3 x 2 + 2 ( a 2 + 1) x + ( a 2 - 3 a + 2) = 0
ab < 0
a 2 - 3 a + 2 < 0 Þ ( a - 1)( a - 2) < 0

(C) x + 3 - 4 x -1 + x + 8 -6 x -1 =1
2
Let x - 1 = t ; |t - 2| + |t - 3| = 1
a +b + g + d
(D) A.M. = =2
4
G.M. = (abgd) 1/ 4 = 2
A.M. = G.M. Þ a = b = g = d = 2
Quadratic Equations 179

3. (A) x 4 - 8 x 2 - 9 = 0
( x 2 - 9)( x 2 + 1) = 0 Þ x = 3, - 3

(B) x 2/ 3 + x 1/ 3 - 2 = 0
( x 1/ 3 + 2)( x 1/ 3 - 1) = 0 Þ x = -8 , 1

(C) ( 3 x + 1) 2 = ( x - 1) 2
Þ 3x + 1 = x + 1 - 2 x Þ 2 x = -2 x (Not possible)

(D) (3 x - 9)(3 x - 1) = 0 Þ x = 0, 2
4. (A) \ ( a + b) = -a & ab = b Þ (1, - 2) and (0, 0)
p 2p 4p
(B) P = O , Q = 8 cos cos cos =1
9 9 9
(C) ar 6 = 2
Product = ( 2 ) 11 = 2 11/ 2
\ m = 11
n=4
(D) x = y = 3
Q ( x - y ) 2 + ( y - 3) 2 = 0
\ 5x - 4 y = 3

Exercise-5 : Subjective Type Problems

æ pö p 3p 3p 5p p 5p
1. f ç cos ÷ = sin sin + sin sin + sin sin
è 7ø 7 7 7 7 7 7
p p
= 2 cos 2 + cos - 1
7 7
Þ f( x) = 2 x 2 + x - 1
2. ( r - a)( r - b)( r - c)( r - d) = ( - 1) ´ ( - 3) ´ (1) ´ (3)
Þ ( r - a) + ( r - b) + ( r - c) + ( r - d) = 0
é3 ö
3. Let x 2 + x + 1 = t " t Î ê , ¥÷
ë4 ø
t 2 - ( m - 3) t + m = 0
180 Solution of Advanced Problems in Mathematics for JEE

æ3ö
Case-I : fç ÷<0
è4ø
x
9 3 3/4
- ( m - 3) + m < 0
16 4
-45
Þ m< y
4
Case-II : D = 0 Þ m = 1, 9
-b 3 9
> Þm>
2a 4 2 0
x
3/4
There is one positive integral value of m = 9.
4. t 2 - (m - 3) t + m = 0 y

t Î [3 4 , ¥) has four distinct real roots, then


D>0
Þ m 2 - 10m + 9 > 0
x
Þ m Î ( -¥, 1) È (9 , ¥) 0 3/4
-b 3 9
> Þ m>
2a 4 2
æ3ö - 45
fç ÷>0 Þ m> Þ m Î (9 , ¥)
è4ø 4
5. f(t ) = (m 2 - 12) t 2 - 8 t - 4 = 0 ( t ³ 0)
f(0) = - 4 < 0
m 2 - 12 £ 0 Þ m Î [-2 3 , 2 3 ]
Case-I : D<0
Þ m 2 - 8 < 0 Þ m Î ( -2 2 , 2 2 )
Case-II : D ³ 0 Þ m Î ( -¥, - 2 2 ] È [2 2 , ¥)
-b 4
= < 0 Þ m Î ( -2 3 , 2 3 )
2
2 a m - 12
Þ m Î [-2 3 , 2 3 ]
é æ p öù
6. ( e x - 2) êsin ç x + ÷ ú ( x - ln 2)(sin x - cos x) < 0
ë è 4 øû
1 1
( x - ln 2) × (sin x + cos x)( x - ln 2)(sin x - cos x) < 0 = ( x - ln 2) 2 (sin 2 x - cos 2 x) < 0
2 2
Þ cos 2 x > 0 , x ¹ ln 2
Quadratic Equations 181

é p ö æ 3p ù
x Î ê0 , ÷ È ç , p ú - {ln 2}
ë 4ø è 4 û
Least positive integral value is 3.
7. x 2 + 17 x + 71 = l2 Þ lÎZ
2 2
x + 17 x + (71 - l ) = 0
D = perfect square = m 2 (say)
m 2 = 289 - 4 (71 - l2 )
(m - 2l)(m + 2l) = 1 ´ 5
Þ m - 2l = 1
m + 2l = 5
8. P( x) = ( x - x 3 - x 2 - 1)( x 2 + 1) + ( x 2 - x + 1)
4

P(a ) + P(b) + P( g) + P(d) = (a 2 - a + 1) + (b 2 - b + 1) + ( g 2 - g + 1) + (d 2 - d + 1) = 6


a
9. If - £1
2
f( x) max = f( 4) Þ 4 a + 18 = 6 Þ a = -3 (Not possible)
a
if - ³ 1
2
f( x) max = f( -2) Þ a = 0 (Not possible)
There is no real value of ‘ a’ .
10. x 2 - 8 x - (n 2 - 10n) = 0
D<0 Þ n 2 - 10n + 16 < 0
(n - 8)(n - 2) < 0
Þ 2 < n < 8 and n ¹ 10
2
11. x + 2 (m - 1) x + (m + 5) > 0 " ( x > 1)
Case-I : D<0
m 2 - 3m - 4 < 0 Þ -1 < m < 4
Case-II : D ³ 0 x
Þ m Î ( -¥, - 1] È [4 , ¥) 1

4
f(1) ³ 0 Þ m ³ -
3
-b
<1 Þ m>0
2a
Þ m Î ( -1, ¥)
182 Solution of Advanced Problems in Mathematics for JEE

12. ax 4 + bx 3 - x 2 + 2 x + 3 = ( x + 2)( x - 1) Q ( x) + ( 4 x + 3)
Put x = 1 a + b=3
x = -2 b = 2a
-b
13. D > 0 Ç >4 Ç f( 4) ³ 0
2a
k - 1> 0 Ç 4k > 4 Ç k 2 - 3k + 2 ³ 0
k>0 Ç k>1 Ç ( k - 2)( k - 1) ³ 0
Þ k³2
14. x 2 - 3 x + 2 = ( x - 1)( x - 2)
If ( x - 1) is a factor of x 4 - px 2 + q = 0. Then
p - q=1 …(1)
4 2
If ( x - 2) is a factor of x - px + q = 0. Then
4 p - q = 16 …(2)
Þ p = 5, q = 4
Þ p + q=9
15. x 2 + 2 xy + ky 2 + 2 x + k = 0
if it can be resolved into two linear factors, then
abc + 2 fgh - bg 2 - af 2 - ch 2 = 0
k2 - k - k = 0
k = 0, 2
2
16. ( a + 1) x + 2 = ax + 3 has exactly one solution.
Þ D =0
a 2 + 4 ( a + 1) = 0
( a + 2) 2 = 0 Þ a = -2 Þ a2 = 4
x 2 - ax + 1
17. y =
x2 - 3x + 2
x 2 ( y - 1) - x (3 y - a) + 2 y - 1 = 0 " x Î R
D³0
(3 y - a) 2 - 4 ( y - 1)(2 y - 1) ³ 0 " y Î R
y 2 - 6 y ( a - 2) + a 2 - 4 ³ 0 " y Î R
D£0
36 ( a - 2) 2 - 4 ( a 2 - 4) £ 0
( a - 2)(2 a - 5) £ 0
Quadratic Equations 183

5
2£ a£
2 (x ¹ 1)
Þ Integral value of a = 2 y

At a = 2
x2 - 2x + 1 ( x - 1) 2 1/2
f( x) = = 0
x
x2 - 3x + 2 ( x - 2)( x - 1) 1

x -1
f( x) = ( x ¹ 1)
x -2
Range R - {0 , 1} x=2

Þ No integral values of ‘ a’ for which range is R.


100
18. x = ( x 2 - 3 x + 2) × Q ( x) + ( ax + b)
at x =1 Þ a + b=1 …(1)
100
at x =2 Þ 2a + b = 2 …(2)
Þ a = 2 100 - 1, b = 2 - 2 100
Remainder = (2 100 - 1) x + 2 (1 - 2 99 )
Þ k = 99
1/ 3
19. x = 7 + 7 2/ 3
x 3 = 7 + 49 + 3 ´ 7 ( x) Þ x 3 - 21x - 56 = 0
Product of all roots = 56
21. Clearly P( x) is a second degree polynomial.
\ P( x) = ax 2 + bx + c
P ¢( x) = 2 ax + b
P( x) - P ¢( x) = ax 2 + ( b - 2 a) x + c - b = x 2 + 2 x + 1
a = 1, b - 2 a = 2 , c - b = 1
a = 1, b = 4 , c = 5
P( x ) = x 2 + 4 x + 5
P( -1) = 1 - 4 + 5 = 6 - 4 = 2
2
23. Let x = t
t 2 + kt + k = 0
D>0 Þ k Î ( -¥, 0) È ( 4 , ¥)
f(0) < 0 Þ k < 0
184 Solution of Advanced Problems in Mathematics for JEE

4
25. f(1) < 0 Þ a < -
3
1 3
8 x
f(3) < 0 Þ a < -
7
Integral values of ‘ a’ are - 5, - 4 , - 3 , - 2.
æpö
26. f(0) f ç ÷ £ 0
è2ø
- (n + 1)(2n + 1)(n - 3) £ 0 Þ n Î [3 , ¥)
27. f( x) = ax 2 + bx + c a , b, c Î I
2
ax + bx + c = a( x - a )( x - b) + p a,b Î I
ax 2 + bx + c - 2 p = a( x - a )( x - b) - p = 0
Not possible for integral values of x.
28. 9 x 2 + 2 x ( y - 46) + y 2 - 20 y + 244 = 0
D³0 Þ y 2 - 11 y + 10 £ 0
( y - 1)( y - 10) £ 0 Þ 1 £ y £ 10
y + 2 y ( x - 10) + 9 x 2 - 92 x + 244 = 0
2

D³0 Þ x 2 - 9 x + 18 £ 0
( x - 3)( x - 6) £ 0 Þ 3 £ x £ 6
29. a + b = 3 and a 3 + b 3 = 7 Þ a 3 + (3 - a) 3 = 7 Þ 9 a 2 - 27 a + 20 = 0
Sum of distinct values of ‘ a’ is 3.
30. ( y 2 - 3) 2 + ( x - 4) 2 = 1
Þ x = 4 + cos q , y 2 = 3 + sin q
M = 36 , m = 1
31. x 1 + x 2 + x 1 x 2 = a
x1 x 2 + x1 x 2( x1 + x 2 ) = b
x 12 x 22 = c
If b + c = 2( a + 1) Þ x 1 x 2 = 2
32. x 3 + 3 x 2 + 4 x + 5 = 0 Þ x = a is root
x 3 - 3 x 2 + 4 x - 5 = 0 Þ x = b is root
Þ a +b =0
33. 5
(1) - (2) ( 2) - ( 3)
( x - z )( x + y + z ) = 1 and ( y - x)( x + y + z ) = -2
Quadratic Equations 185

x+ y
Divide z =
2
2
æ x + yö æ x + yö
y 2 + yç ÷+ç
2 2
÷ = 1 and x - 2 xy - 5 y = 0
è 2 ø è 2 ø
x = (1 + 6 ) y
2
y2 = put values
9+3 6
4 (1 - a - b) - ( a - b) 2 4 (1 + a + b) - ( a + b) 2
34. >
4 -4
Þ 8 > ( a + b) 2 + ( a - b) 2 Þ a 2 + b 2 < 4
3
35. 20 x + 3 20 x + 13 = 13

3
20 x + 3 20 x + 3 20 x + 3 20 x + 3 20 x +¼¥ = 13
3
Þ 20 x + 13 = 13 Þ 20 x = 2197 - 13
2184 546
Þ x= =
20 5
36. Let f( x) = x 2 - 2 ( a + 1) x + a ( a - 1)
f(1 - a) < 0 Ç f(1 + a) < 0
2
4a - 3a - 1 < 0 Ç 3a + 1 > 0
1 1
- < a<1 Ç a> -
4 3
æ 1 ö
Þ a Î ç - , 1÷
è 4 ø
37. ( x - 8)( x - 2) < 0
Þ 2< x<8
-b
38. sin q + cos q =
a
c
sin q × cos q =
a
b2 2c
(sin q + cos q) 2 = 1 + 2 sin q cos q = =1+
2 a
a
2 2
b -a 2c
Þ =
2 a
a
186 Solution of Advanced Problems in Mathematics for JEE

39. cos 2 x + (1 - a) cos x - a 2 £ 0 " x Î R


Let cos = t " t Î [-1, 1]
–1 1
t 2 + (1 - a) t - a 2 £ 0 " t Î [-1, 1] x

f( -1) £ 0
Þ a2 - a ³ 0
Þ a Î ( -¥, 0] È [1, ¥)
f(1) £ 0
Þ a2 + a - 2 ³ 0
( a + 2)( a - 1) ³ 0 Þ a Î ( -¥, - 2] È [1, ¥)
a
40. 2 x 2 - 35 x + 2 = 0
b
2 2
2a - 35 = - and 2b - 35 = -
a b
42. xF ( x) - 1 = k( x - 1)( x - 2)( x - 3) ¼( x - 9)
k( x - 1)( x - 2)( x - 3) ¼( x - 9) + 1
Þ F ( x) =
x
Constant term = k( -9 !) + 1 = 0
1
Þ k=
9!
44. cos A + cos B + cos C = -a
cos A cos B + cos B cos C + cos A cos C = b
cos A cos B cos C = -c
a 2 - 2 b - 2 c = cos 2 A + cos 2 B + cos 2 C + 2 cos A cos B cos C
=1
45. k > 0 y
-b
D>0Ç >0
2a
k -3
k 2 - 10 k + 9 > 0 Ç <0 O
x
k
k Î ( -¥, 1) È (9 , ¥) Ç k Î (0 , 3) Þ k Î(0 , 1)

❑❑❑
Sequence and Series 187

9
SEQUENCE AND SERIES

Exercise-1 : Single Choice Problems

1. AM ³ GM
3. 2 sec a = sec (a - 2 b) + sec (a + 2 b)
2 cos(a + 2 b) + cos(a - 2 b)
=
cos a cos(a - 2 b) cos(a + 2 b)
cos 2a + cos 4 b = cos a (2 cos a cos 2 b)
2 cos a - 1 + 2 cos 2 2b - 1 = 2 cos 2 a cos 2 b
2

Þ cos 2 a(1 - cos 2 b) + (cos 2 b + 1)(cos 2 b - 1) = 0


Þ cos 2 a = 1 + cos 2 b
a+c
4. If a , b, c A.P. Þ b=
2
2 ec
if c , d , e H.P. Þ d=
e+c
2
if b, c , d G.P. Þ c = bd
æ a + c ö æ 2 ec ö
c2 = ç ÷ç ÷
è 2 øè e + cø
Þ c 2 = ae
5. ( a + nd) 2 = ( a + md)( a + rd)
a mr - n 2
Þ =
d 2n - m - r
2mr a -n
if m , n , r in H.P., then n = Þ =
m+r d 2
4
6. A.M. (a , b , g , d) = =1
4
G.M. (a , b , g , d) = 1 Þ a = b = g = d = 1
So, equation is ( x - 1) 4 = 0
188 Solution of Advanced Problems in Mathematics for JEE

S 14 S 22 - S 22 S 32 S 22 ( S 14 - S 32 )
7. S 3 = S 12 Þ = =0
S 12 + S 32 S 12 + S 32
r ×2r
8. T r =
( r + 2)!
( r + 2 - 2) 2 r 1 1
Tr = = 2r - 2 r +1
( r + 2)! ( r + 1)! ( r + 2)!
2! 2 n +1
Sn = -
2 ! (n + 2)!
é 2 n +1 ù
lim S n = S ¥ = 1 êas lim = 0ú
n® ¥
ë n ® ¥ (n + 2)! û
p æ p ö æ p ö
9. tan 2 = tan ç - x ÷ tan ç + x÷
12 è 12 ø è 12 ø
p
tan 2 - tan 2 x
2 p 12 æ p ö
tan = Þ tan 2 x ç tan 4 - 1÷ = 0 Þ tan x = 0
12 p è 12 ø
1 - tan 2 tan 2 x
12
x = 0 , p , 2 p , 3 p , ¼¼ 99 p
Sn n n+1
10. =
Sn - 1 n - 1 n + 2
æ2 3 4 5 n ö æ3 4 5 n n + 1ö
Q n = ç ´ ´ ´ ´ ¼¼ ÷ ´ ç ´ ´ ´ ¼¼ ´ ´ ÷
è1 2 3 4 n - 1ø è 4 5 6 n +1 n +2ø
ænö æ 3 ö 3n
Q n = ç ÷ ×ç ÷=
è 1ø è n + 2 ø n + 2
lim Q n = 3
n® ¥
log l p 1 log A + ( p - 1) log R p 1
11. log m q 1 = log A + ( q - 1) log R q 1
log n r 1 log A + ( r - 1) log R r 1
log A p 1 ( p - 1) log R p 1
log A q 1 + ( q - 1) log R q 1 =0
log A r 1 ( r - 1) log R r 1
12. Numbers divisible by 6 ® 49
Numbers divisible by 18 ® 16
y+z
13. = yz Þ 1 - x ³ 2 yz
2
Sequence and Series 189

Thus, (1 - x)(1 - y)(1 - z ) ³ 2 yz × 2 zx × 2 xy = 8 xyz


xyz 1
Þ £
(1 - x)(1 - y)(1 - z ) 8
17. Clearly, both roots are lies in between -1 and 1.
n æ n ö æ n ö
\ lim (a r + b r ) = ç lim
å a r ÷ + ç lim
å å br ÷
n® ¥ ç ÷ ç ÷
r =1 è n ® ¥ r =1 ø è n ® ¥ r =1 ø
a b 1
= + =
1 - a 1 - b 12
a a a a a a a a a a a a a
18. å aij = a12 + a13 + a14 + a12 + a 23 + a 24 + a13 + a 32 + a 34 + a14 + a 42 + a 43
æ 1 ö
³ 12 çQ x + ³ 2 ÷
è x ø
x 2 + 2 xy + 2 xy + 4 y 2 + z 2 + z 2 6 2
19. ³ 2 ×4 × x4 × y4 ×z4
6
20. Let first term be ‘ a’ and difference be d.
Þ 5 ( a + 4 d) = 8 ( a + 7 d)
Þ a + 12 d = 0
25
S 25 = [2 a + 24 d]
2
S 25 = 25 ( a + 12 d) = 0
21. 10 sin x = 5 ( 4 sin 2 x + 1) sin x ¹ 0
5 ±1
Þ sin x =
4
22. Let first term of G.P. be a and ratio be r.
Þ a + ar + ar 2 = 70 and 10 ar = 4 a + 4 ar 2
1
Þ a = 40 r=
2
a 40
S= = = 80
1- r 1
1-
2
¥
k k ¥ 1 k
23. å n+k
= å =
2 k n =1 2 n 2 k
n =1 2
¥
k 1 2 3 4
å 2 k = 2 + 2 2 + 2 3 + 2 4 +¼¼ = 2
k =1
p+ q+ r
24. ( pqr) 1/ 3 ³ Þ p=q=r
3
if 3 p + 4 q + 5r = 12 Þ p = q = r = 1
190 Solution of Advanced Problems in Mathematics for JEE

1æ 1 1 1 1 1 ö 1 é1æ 1ö 1æ 1 1ö 1æ 1 1ö ù
25. ç + + + + +¼¼ ÷ = ê ç 1 + ÷ + ç + ÷ + ç + ÷ + ¼¼ú
3 è 3 6 10 15 21 ø 3 ë3 è 2ø 5è2 3ø 7 è3 4ø û
1é 1 1 1 ù 1
= + + + ¼¼ú =
3 êë 1 × 2 2 × 3 3 × 4 û 3
a b
[2 A + ( a - 1) D] [2 A + ( b - 1) D]
26. 2 =2 = c Þ D = 2 c, A = c
a2 b2
x r x
27. =4 Þ = r - r2
1- r 4
1
if -1 < r < 1 then -2 < r - r 2 <
4
x 1
-2 < < Þ - 8 < x < 1
4 4
n+1
28. t 1 + t 3 + t 5 +¼¼ + t 2n +1 = [2 a + n(2 d)] = 248
2
n
t 2 + t 4 + t 6 +¼¼ + t 2n = [2 ( a + d) + (n - 1) 2 d] = 217
2
t 2n +1 - t 1 = 2n × d = 56
n+1 n
Þ [2 a + 56] = 248 and [2 a + 56] = 217
2 2
Þ n = 7, a = 3
29. length of side A1 = 20
20
length of side A 2 =
2
20
length of side A 3 =
( 2)2
20
length of side A n =
( 2 ) n -1
400
Area of A n = <1
2 n -1
¥
1 1 1 k+1
30. S k = å ( k + 1)i =1+ +
k + 1 ( k + 1) 2
+¼¼ =
k
i =0
n n n n
n (n + 1) n ( n + 3)
å k S k = å( k + 1) = å k + å 1 = 2
+n=
2
k =1 k =1 k =1 k =1
2
( r + 1) r -1 æ 1 2 ö r -1
31. T r = ×2 = ç1 + - ÷2
r ( r + 1) è r r + 1ø
Sequence and Series 191

n n n æ 2 r -1 2 r ö÷ é 2n ù æ n ö n
Sn = å Tr = å 2 r -1 + å ç - = (2 n -1 ) + ê1 - ú =ç ÷2
ç r + 1 ÷ø
r =1 è r
r =1 r =1 ë n + 1û è n + 1 ø
29
32. å(1.5) n = (1.5) 2 + (1.5) 3 +¼¼ + (1.5) 29
n =2
é(1.5) 28 - 1ù
= (1.5) 2 ê ú = 2 k - 2 (1.5)
2

ë 0.5 û
33. 7 , A1 , A 2 , A 3 ,¼¼ , A n , 49 are in A.P.
æn +2ö
A1 + A 2 + A 3 +¼¼ + A n = ç ÷ (7 + 49) - (7 + 49)
è 2 ø
n
Þ ´ 56 = 364 Þ n = 13
2
2 2
34. , , 2 , 2 r, 2 r 2
2 r
r
35. S n = 5n 2 + 4n
t n = S n - S n -1 = 10n - 1
é æ a 2 - b öù æ 2 2 2 ö
36. x 3 + y 3 = ( x + y)( x 2 + y 2 - xy) = a ê b - ç ÷ú çQ xy = ( x + y) - ( x + y ) ÷
ç ÷ ç ÷
êë è 2 ø úû è 2 ø
3 ab a 3
= -
2 2
1
37. S 1 = =3
2
1-
3
3
S2 = =5
2
1-
5
M
M
2n - 1
Sn = = 2n + 1
2
1-
2n + 1
1 1 1 1 1 1
+ + +¼¼ = + + +¼¼
S1 S 2 S 3 S 2 S 3 S 4 S 3 S 4 S 5 3 × 5 × 7 5 × 7 × 9 7 × 9 × 11
¥ ¥ ¥
1 é 1 1 ù
S¥ = å tr = å ( 2 r + 1)( 2 r + 3 )( 2 r + 5)
= å ê(2 r + 1)(2 r + 3) - (2 r + 3)(2 r + 5) ú
r =1 r =1 r =1 ë û
38. ar 5 , 2 , 5, ar 13 are in G.P.
Þ ( ar 9 ) 2 = 10
t 1 t 2 t 3 ¼¼ t 19 = a 19 r 9´19 = ( ar 9 ) 19 = 10 19/ 2
192 Solution of Advanced Problems in Mathematics for JEE

39. A.M. ³ G.M.


1 1 1 1
A + + 1³ 3 ; B + + 1³ 3 ; C + + 1³ 3 ; D + + 1³ 3
A B C D
æ 1 öæ 1 öæ 1 öæ 1 ö 4
ç A + + 1÷ ç B + + 1 ÷ ç C + + 1÷ ç D + + 1÷ ³ 3
è A øè B øè C øè D ø
40. (Sr) 2 = Sr 2 + 2 Sr1 r2
a-b
S r1 r2 =
2
2n n
41. [2 a + (2n - 1)d] = x and [2( a + 2nd) + (n - 1) d] = y
2 2
2y x 2y - x
Þ - = 3nd Þ d =
n n 3n 2
44. 2 , 6 , 2 ( k - 1) are in G.P.
Þ 6 2 = 2 ´ 2 ( k - 1)
Þ k = 10
Þ x 2 - x - 6 > 0 and | x| < 100
Þ x Î ( -100 , - 2) È (3 , 100)
Number of integers = 193
n n
æ 3 öæ 1öæ 1öæ 3ö
45. å 1 + T r T r +1 T r + 2 T r + 3 = å 1+ çr - ÷çr - ÷çr + ÷çr + ÷
è 2 øè 2 øè 2 øè 2ø
r =1 r =1
n 2 n
æ 2 5ö 5
= å çr - ÷ = å r2 -
r =1 è 4ø r =1 4
2 n n n
5 5 1 5
= å r2 - + år2 - = + r2 - å å
r =1 4 r =2 4 4 r =2 r =2 4

46. T r = å T r - å T r -1 = r 2 + r
n n n
2008 1 æ1 1 ö n
å Tr
= 2008 å
r( r + 1)
= (2008) ç - å
÷ = (2008)
r r + 1ø n+1
r =1 r =1 r =1 è
(2008) n
lim = 2008
h®¥ n + 1
n
1 1 1
48. P( x) = å æçè x - r ö÷ø æçè x - r + 1 ö÷ø æçè x - r + 2 ö÷ø
r =1
n n
1 1é 1 1 ù
Absolute term = - å r( r + 1)( r + 2) = - 2 êê å r( r + 1) - ( r + 1)( r + 2) úú
r =1 ë r =1 û
Sequence and Series 193

1 é1 1 ù
=- -
2 ë 2 (n + 1)(n + 2) úû
ê

1 é1 1 ù 1
lim - - =-
n ®¥ 2 êë 2 (n + 1)(n + 2) úû 4
1 1 1 1
50. , , ,¼¼ , are in A.P.
T1 T 2 T 3 Tk
1
+ 5d
T2 a 2
= =9 Þ d = -
T6 1 a
+d
a
1
+ 3d
T10 a 5
= =
T4 1 17
+ 9d
a
æ 1 1 ö 2æ 1 1 ö 15
52. ç 1 + + + ¼¼ ÷ + ç 1 + + +¼¼ ÷ =
2 4 3 2 4
è 3 3 ø è 3 3 ø 8
53. ( x - 1)( x - 2)( x - 3)( x - 4) ¼¼( x - 10)
Coefficient of x 8 = sum of terms taken two at a time
1
= [(1 + 2 + 3 +¼¼ + 10) 2 - (12 + 2 2 +¼¼ + 10 2 )]
2
55. AM = GM
a +b + g + d 1
= (abgd) 1/ 4 =
4 2
1
Þ a =b = g = d =
2
56. Use AM ³ GM
¥
57. å(a r + b r ) = (a + a 2 + a 3 +¼ ) + (b + b 2 + b 3 +¼ )
r =1
a b
= +
1 - a 1 -b
a
4x2 + 2x - 1 = 0
b
a
2
æ x ö æ x ö 2 1– a
4ç ÷ + 2ç ÷ - 1 = 0 Þ 5x - 1 = 0
è1+ x ø è1+ x ø b
1– b
194 Solution of Advanced Problems in Mathematics for JEE

2
é 10 ´ 11ù
58. 2 2 [1 + 2 3 + 3 3 + 4 3 +¼ + 10 3 ] = 4 ê = 12100
ë 2 úû
59. AM ³ HM
a a
b+ +
2 2³ 3
3 4 1
+
a b
60. 4 x - 15 = 4 2- x Þ 4 x = 16 Þ x = 2
æ 2011p ö æ pö 1
Common ratio = cos ç ÷ = cos ç 670 p + ÷ =
è 3 ø è 3ø 2
61. AM ³ GM
c2 c2
a 4 + b4 + + 4 4 4
1/ 4
2 2 ³ æç a b c ö÷
4 ç 4 ÷
è ø
1
62. x 2 + y 2 = x 2 + ³2
x2
2 6 12 20
63. + + + +¼¥
3 3 3 3 3 3
1 1 +2 1 +2 +3 1 + 2 + 33 + 43
3 3

1´ 2 2 ´3 3´4
= + + +¼¥
3 3 3
1 1 +2 1 + 23 + 33
3

n n
n(n + 1) n(n + 1)
= lim
n ®¥
å 13 + 2 3 +¼ + n 3 = nlim
®¥
å 2
1 1 æ n(n + 1) ö
ç ÷
è 2 ø
n n
1 1 1
= lim 4
n ®¥
å n(n + 1)
= 4 lim
n ®¥
å æçè n - n + 1 ö÷ø
1 1
æ1 1 1 1 1 1 ö
= 4 lim ç - + - +¼ + - ÷
n ®¥ è 1 2 2 3 n n + 1ø
n
= 4 lim =4
n ®¥ n + 1
¥
1 æ 1 1 ö 1 æ 1 ö
64. å çç - ÷÷ = çç ÷
( k - 1) n =1 è (n + 1)(n + 2) ¼(n + k - 1) (n + 2)(n + 3) ¼(n + k) ø ( k - 1) è 2 × 3 × 4 ¼ k ÷ø
3 6
65. A - G = and G - H =
2 5
As we know,
æ3 öæ 6ö 15
G 2 = AH Þ G2 ç + G÷çG - ÷ Þ G = 6 and A =
è 2 øè 5ø 2
Sequence and Series 195

ab = 36 and a + b = 15 Þ a = 12 and b = 3
Þ
2+5 5+8 8 + 11 1 æ 5 2 - 2 2 8 2 - 5 2 112 - 8 2 ö
66. S = + + +¼ = ç + + +¼ ÷
2
2 ×5 2 2
5 ×8 2 2
8 × 11 2 ç 2
3 è 2 ×5 2 2
5 ×8 2 2
8 × 11 2 ÷
ø
1æ 1 1 1 1 1 1 1 1 ö
= ç - + - + - +¼ + - ÷÷
3 çè 2 2 5 2 5 2 8 2 8 2 112 29 2
32 2 ø
1æ 1 1 ö 85
= ç - ÷=
3 è 4 32 2 ø 1024
10 10
r r
67. å ( r 2 - 1) 2 - r 2 = å ( r 2 - r - 1)( r 2 + r - 1)
r =1 r =1

1 10 æ 1 1 ö
= å ç
2
-
2
÷
2 r =1 è r - r - 1 r + r - 1 ø
¥ ¥
r
68. åt r = å r4 + r2 + 1
r =1 r =1
¥ ¥
r r
= å ( r 2 + 1) 2 - r 2 å ( r 2 - r + 1)( r 2 + r + 1)
=
r =1 r =1
¥
1 æ 1 1 ö
= å ç - ÷
2 r =1 è r 2 - r + 1 r 2 + r + 1 ø
4 7 10
69. S¥ = 1 + + + +¼
5 52 53
1 1 4 7
× S¥ = + + +¼
2
5 5 5 53
4 3 3 3 7
Þ S¥ =1+ + + +¼ =
5 5 5 2 3 4
5
35
Þ S¥ =
16
71. x 1 , x 2 , x 3 ¼ x 2n
2n
å( -1) r +1 x r2
r =1

x 12 - x 22 + x 32 +¼ - x 22n
( x 1 - x 2 )( x 1 + x 2 + x 3 +¼ x 2n )
- ( x 2 - x 1 )( x 1 + x 2 + x 3 +¼ x 2n )
( x - x1 ) 2 x
- 2n [ x 1 + x 2n ]
2n - 1 2
x
( x 12 - x 22n )
2x - 1
196 Solution of Advanced Problems in Mathematics for JEE

a +b
72. =9; ab = 4
2
73. rms ³ AM
p 2 + q2 p + q
³
2 2
n
74. 150 ´ 9 + [300 + (n - 1)( -2)] = 4500 Þ n = 25
2
Total term = n + 9 = 34
20
75. S 20 = [2 (1 - ad) + 19 d] = 20
2
Þ 19 d - 2 ad = 0
¥ ¥
1 1 æ 1 1 ö
76. å (n - 2)(n - 1) n(n + 1)(n + 2) = 4 å ççè (n - 2)(n - 1) n(n + 1) - (n - 1) n(n + 1)(n + 2) ÷÷ø
n =3 n =3
5 1 1 1 1 1
78. 2 x + 2 2 x +1 + = 2 x + 2 2x + 2 2x + + + + +
x x x x x
2 2 2 2 2 2x
1/ 8
2 x + 2 2 x +1 + (5 2 x ) æ x 1 ö
Þ ³ ç 2 ´ (2 2 x ) 2 ´ ÷ =1
8 ç (2 x ) 5 ÷
è ø
5
Þ 2 x + 2 2 x +1 + ³8
2x
¥ ¥ ¥ æ ö 1
æ ( 4 r + 5) ö 1 1 1 1 1 1
79. å çè r(5r + 5) ÷ø × 5 r = å æçè r - 5r + 5 ö÷ø × 5 r = å çç r × 5 r -
( r + 1) × 5 r +1
÷=
÷ 5
r =1 r =1 r =1 è ø

Exercise-2 : One or More than One Answer is/are Correct

a1 + a n 2 a1 a n
1. a = , b = a1 a n , c =
2 a1 + a n
Þ a ³ b ³ c and b 2 = ac
2. D1 : b 2 - 4 ac < 0
D 2 : c 2 - 4 ab < 0
D 3 : a 2 - 4 bc < 0
D1 + D 2 + D 3 : a 2 + b 2 + c 2 < 4 ( ab + bc + ac)
a 2 + b2 + c 2
1< <4
ab + bc + ac
Sequence and Series 197

3. If a , b, c are in H.P.
A.M. > H.M.
a+c
Þ >b Þ a + c > 2b
2
Þ a - b> b - c
1 1
or - <0
a-b b-c
G.M. > H.M.
also ac > b or ac > b 2
1
4. T p = a + ( p - 1) d =
q( p + q)
1 1
T q = a + ( q - 1) d = Þ a=d=
p ( p + q) pq( p + q)
5. (a) a , H 1 , H 2 , H 3 , ¼¼ , H n , b are in H.P.
1 1 1 1 1 1
, , , , ¼¼ , , are in A.P.
a H1 H 2 H 3 Hn b
1 1 1 1 n æ 1 1ö
Þ + + +¼¼ + = ç + ÷
H1 H 2 H 3 Hn 2 è a bø
(c) a , A1 , A 2 , A 3 , ¼¼ , A 2n , b are in A.P.
A1 + A 2n = A 2 + A 2n -1 = A 3 + A 2n -2 = ¼¼ = a + b
(d) 4 g 2 + 5 g 3 = 4 r + 5r 2
2
This is minimum at r = -
5
6. a , b, c are in H.P.
1 1 1
, , are in A.P.
a b c
a+ b+ c a+ b+ c a+ b+ c
(a) - 2, - 2, - 2 are in A.P.
a b c
a+ b+ c a+ b+ c a+ b+ c
(b) - 1, - 1, - 1 are in A.P.
c b c
2 1 1 2
(c) = + ³ Þ ac ³ b
b a c ac
a 5 + c 5 ³ 2 ( ac) 5/ 2 ³ b 5
(d) 2ac = ab + bc
7. Let the roots be a , ar , ar 2 , ar 3 and ar 4 .
a( r 5 - 1)
= 40 …(1)
r -1
198 Solution of Advanced Problems in Mathematics for JEE

æ 1 ö
ç - 1÷
1èr 5
ø
and = 10 …(2)
a 1
-1
r
r 5 - 1 40
put = in (2) we get ar 2 = ± 2
r -1 a
Now, d = ( ar 2 ) 5 = ( ± 2) 5
8. (a) Q 2 a k +1 = a k + a k + 2
\ fk ( -1) = 0
-1 is a root.
\ Other is also real root.
(b) From (a) ( -1) is root for any ‘ k’ so any pair of equation has a common root.
(c) If one root is -1, other roots are - c a (form)
a k+ 2 a3 a4 a5
i . e. , , , ¼¼ are not in A.P.
ak a1 a 2 a 3
a+c 2 ce
9. b = ,d=
2 c+e
if c 2 = bd , then c 2 = 36 (Q a = 2 , e = 18)
10. If a , b, c are in A.P. then
a = b - d and c = b + d
a + b + c = 60 Þ b = 20
If ( a - 2), b,( c + 3) are in G.P., then
400 = (18 - d)(23 + d) Þ d = 2 , - 7
81 + 144 a + 16 b + 9 c 4
4 4
12. ³ 36 abc
4
Þ A.M. = G.M.
Þ 81 = 144 a 4 = 16 b 4 = 9 c 4
13. x , y , z A.P.
Let x = y - q and z = y + q
3q
sin
cos( y - q) + cos y + cos( y + q) = 1 = 2 × cos( y)
q
sin
2
Sequence and Series 199

3q
sin
1
2 × sin( y) Þ cot y = 2
sin( y - q) + sin y + sin( y + q) = =
2 sin q
2
3q
sin
2 = 3 = 3 - 4 sin 2 q Þ cos q = 3 - 2
q 2 2 2 2
sin
2
10 n +1 + 1 10 m +1 + 1
15. <
10 n + 2 + 1 10 m + 2 + 1
Þ 10 n +1 × 10 m + 2 + 10 n +1 + 10 m + 2 + 1 < 10 n + 2 × 10 m +1 + 10 n + 2 + 10 m +1 + 1
Þ 10 m +1 < 10 n +1
16. S r = r + S r Þ S r2 - S r = r
17. 50, 48, 46, 44, ……… A.P.
T n = 50 + (n - 1)( -2) = 0
Þ n = 26
n n n
2r + 1 1 1 1
18. S n = S n å t r = å 12 + 2 2 + 3 2 +¼ + r 2 = å 6 æçè r - r + 1 ö÷ø = 6 æçè 1 - n + 1 ö÷ø
r =1 r =1 r =1

Exercise-3 : Comprehension Type Problems

Paragraph for Question Nos. 1 to 2


Sol. T1 = A + B = 0 Þ A = -B
T 2 = Aa + Bb = 1 Þ A(a - b) = 1
T 3 = Aa 2 + Bb 2 = 1 Þ A(a 2 - b 2 ) = 1
T 4 = Aa 3 + Bb 3 = 2 Þ A(a 3 - b 3 ) = 2
Þ a + b = 1 and ab = -1

Paragraph for Question Nos. 3 to 4


Sol. Set A : 5 - D , 5, 5 + D and
Set B : 5 - d , 5, 5 + d
p 25 - D 2 7
= =
q 25 - d 2 8
Þ 25 = 8 D 2 - 7 d 2 = d 2 + 16 d + 8 (Q D = 1 + d)
Þ d = 1 and D = 2
Set A {3 , 5, 7} and set B {4 , 5, 6}
200 Solution of Advanced Problems in Mathematics for JEE

Paragraph for Question Nos. 5 to 7


( x - 3) + ( y + 1) + ( z + 5)
5. ³ [( x - 3)( y + 1)( z + 5)]1/ 3
3
Þ ( x - 3)( y + 1)( z + 5) £ (21) 3
1 5
( x - 3) + y + +z+ 1/ 3
æ 1ö æ 5ö 2 3 ³ é( x - 3) æ y + 1 ö æ z + 5 ö ù
6. Term is 6 ( x - 3) ç y + ÷ ç z + ÷ Þ ê ç ÷ç ÷
è 2 øè 3 ø 3 ë è 2 øè 3 ø úû
æ 1öæ 5 ö (355) 3
( x - 3) ç y + ÷ ç z + ÷ £
è 2 øè 3 ø 63 ´33
(355) 3
Maximum value =
62 ´33
x+ y+z
7. ³ ( xyz ) 1/ 3 ; xyz £ (20) 3
3

Paragraph for Question Nos. 8 to 10


Sol. Let removed number are A and A + 1.
n(n + 1) 105
- 2 A - 1 = ( n - 2)
2 4
2n 2 - 103n + 206 = 8 A
Þ n = 50 , A = 7

Paragraph for Question Nos. 11 to 13


2
Sol. a n +1 - 1 = ( a n - 1)
a n - 1 = ( a n -1 - 1) 2
a n -1 - 1 = ( a n -2 - 1) 2
( a 2 - 1) = ( a1 - 1) 2
a1 - 1 = ( a 0 - 1) 2
2 n-1 n
Þ ( a n - 1)( a n -1 - 1) 2 ( a n -2 - 1) 2 ¼¼( a1 - 1) 2 = ( a n -1 - 1) 2 ( a n -2 - 1) 2 ¼¼( a 0 - 1) 2
n
Þ ( a n - 1) = 3 2
0 n-1
2 × (3 2 + 1)(3 2 + 1) ¼¼(3 2 + 1)
bn =
2n
(3 + 1)
n
32 -1
bn = n
32 + 1
Sequence and Series 201

Paragraph for Question Nos. 14 to 15


n n
4 æ 1 1 ö æ 1 1 ö
f ( n) = å ( r - 1) r( r + 1) = 2 å ççè ( r - 1) r - r( r + 1) ÷÷ø = 2 ççè 1 × 2 - n (n + 1) ÷÷ø ; a = nlim
®¥
f ( n) = 1
r =2 r =2
122
14. f(7) + f(8) =
63
3 a
15. x 2 + x + t =0
2 b

Paragraph for Question Nos. 16 to 17


a1 a2 a3 a1005 1
Sol. = = = ¼¼ =
a1 + 1 a 2 + 3 a 3 + 5 a1005 + 2009 k
1 3 5 2009
a1 = , a2 = , a3 = , ¼¼ a1005 =
k -1 k -1 k -1 k -1
(1005) 2 1005
a1 + a 2 + a 3 +¼¼ + a1005 = = 2010 Þ k - 1 =
k -1 2

Exercise-4 : Matching Type Problems

1. (A) a , b, c are in A.P.


b-a=c-b
b - a , c - b, a are in G.P.
c-b a
= Þ c-b=a (Q b - a = c - b)
b-a c-b
(B) a , x , b are in A.P.
a+b
x=
2
a , y , z , b are in G.P.
y = a 2 / 3 b1 / 3 , z = a 1 / 3 b 2 / 3
(C) a , b = ar , c = ar 2
If c > 4 b - 3 a
r 2 - 4r + 3 > 0 (Q a > 0)
( r - 3)( r - 1) > 0
(D) 7 x 2 - 8 x + 9 < 0
a = 7 > 0, D = 64 - 252 < 0
No solution
202 Solution of Advanced Problems in Mathematics for JEE

2. (A) a + d = b + c = 20
(B) 2 , G1 , G 2 , G 3 , G 4 , G 5 , G 6 , 5 are in G.P.
G1 G 6 = G 2 G 5 = G 3 G 4 = 10
(C) a 4 h7 = a1 h10 = a10 h1 = 6
æ 7ö
(D) (2 x - 5) 2 = 2 ç 2 x - ÷ Þ (2 x - 8)(2 x - 4) = 0 Þ x = 3
è 2ø
2 3
3. (A) 2 × 2 x = 2 x + 2 x
Exponential series can’t be in A.P.
(B) If a1 , a 2 , a 3 ,¼¼ , a n are in A.P.
a 2 - a1 = a 3 - a 2 = a 4 - a 3 = ¼¼ = a n - a n -1 = d
æ 1 1 1 ö
S = -dç + + ¼¼ + ÷
ç a + a a + a a + a ÷
è 1 2 2 3 n -1 n ø

æ a n - a1 ö
= -d ç ÷= a - a
1 n
ç d ÷
è ø
2n
[2 a + (2n - 1) d]
S 2n
(C) = 2 =3
Sn n
[2 a + (n - 1) d]
2
Þ 2 a = (n + 1) d
3n
[2 a + (3n - 1) d]
S 3n
= 2 =3
2S n 2n
[2 a + (n - 1) d]
2
(D) t 1 + t 5 = t 2 + t 4 = 2t 3
4(t 1 - t 2 - t 4 ) + 6t 3 + t 5 3t 1 + (t 1 + t 5 ) - 4 (t 2 + t 4 ) + 3 (2t 3 )
= =1
3t 1 3t 1
4. A ® Q; B ® P; C ® T; D ® S
1 1
5. (A) log 2 x + log 2 y = 5 and log 2 y + log 2 x = 7
3 3
Þ log 2 x = 6 and log 2 y = 3
Þ x = 2 6 and y = 2 3
(B) ÐB = 60° and b 2 = ac
a 2 + c 2 - b2 1
cos B = = Þ a 2 + c 2 = 2 ac
2 ac 2
Þ a=c
Sequence and Series 203

(C) AM ³ GM
b c a c a b
+ + + + +
a a b b c c ³1
6
(D) ( b + c) 2 - a 2 = lbc
Þ b 2 + c 2 - a 2 = (l - 2) bc
b2 + c 2 - a 2 l - 2
Þ =
2 bc 2
l -2
-1 < <1
2
0<l<4
6. P(n) × ( f(n + 2) - f(n)) = q(n)
æ 1 1 ö
P ( n) × ç + ÷ = q ( n)
èn +1 n +2ø
P ( n) × ( 2n + 3) = ( n 2 + 3n + 2) × q ( n)
Þ P(n) = n 2 + 3n + 2 and q(n) = (2n + 3)

Exercise-5 : Subjective Type Problems

1. If a , b, c , d are in A.P. with common difference ‘ k’ , then


9 k 3 + ( x - 4) k 2 + 4 k = 0
k {9 k 2 + ( x - 4) k + 4} = 0
D ³ 0 Þ ( x - 4) 2 - 144 ³ 0
( x + 8)( x - 16) ³ 0
Þ x Î ( -¥, - 8] È [16 , ¥)
2. S = 1 × 2 + 2 × 2 2 + 3 × 2 3 + 4 × 2 4 +¼¼ + n × 2 n
2 × S = 1 × 2 2 + 2 × 2 3 + 3 × 2 4 +¼¼ + (n - 1) × 2 n + n × 2 n +1
Þ S = (n - 1) × 2 n +1 + 2 = 2 + 2 n +10
Þ 2(n - 1) = 2 10
Þ n = 513
n ¥ é 1 ù 1
r+2 1
3. lim å 2 r +1 r ( r + 1) = å ê r × 2 r -
r +1 ú=
n ®¥
r =1 r =1 ë ( r + 1) × 2 û 2
¥ ¥
8r æ 1 1 ö
4. å 4 r 4 + 1 = 2 å çè 2 r 2 - 2 r + 1 - 2 r 2 + 2 r + 1 ÷ø = 2
r =1 r =1
204 Solution of Advanced Problems in Mathematics for JEE

5. Let three terms in A.P. a - d , a , a + d


If ( a - d) 2 , a 2 ,( a + d) 2 are in G.P. Þ d = ± 2 a
a2 1
r= =
2
( a - d) (1 ± 2 ) 2

10 2n - 1 æ 10 n - 1 ö æ n ö
÷ = P ç 10 - 1 ÷ Þ P = 3
6. -2ç
9 ç 9 ÷ ç 9 ÷
è ø è ø
7. a - d , a , a + d , a - d + 30
If last three terms are in G.P.
( a + d) 2 = a ( a - d + 30)
d2
Þ a=
30 - 3 d
n
1
8.
8n 4
å[k( k + 2)( k + 4)( k + 6) - ( k - 2) k ( k + 2)( k + 4)]
k =1
1 é(n - 1)(n + 1)(n + 3)(n + 5) + n (n + 2)(n + 4)(n + 6) + 15 ù 1
8 êë ú = 4 (n ® ¥)
n4 û
2
é n(n + 1) ù
9. Unit digit of ê úû = 1
ë 2
n(n + 1)
Then unit digit of is 1 because unit digit of n(n + 1) can not be 8.
2
10. 2 log b c = log c a + log a b
æ log a + 2 log r ö æ log a ö æ log a + log r ö
2 çç ÷÷ = çç ÷÷ + çç ÷÷
è log a + log r ø è log a + 2 log r ø è log a ø
A - 3 + 33
Let A = log a and R = log r Þ 3 A 2 + 3 Ar - 2 R 2 = 0 Þ =
R 6
A + 2R A 3
d = log b c - log c a =- =
A+R A + 2R 2
3 3r 2 r 6r 3
11. 3 , , , 7s ; = 1 + and = + 7s
r s r s s r
Þ 7 r 3 - 6 r 2 + 21r - 18 = 0 Þ ( r 2 + 3)(7 r - 6) = 0
6 9
Þ r = and s =
7 14
12 22 32 42
12. S= + + + +¼¼
31 32 33 34
S 12 2 2 3 2
= + + +¼¼
3 32 33 34
Sequence and Series 205

2S S 1 3 5 7
=S - = + + + +¼¼
3 3 3 32 33 34
2S 1 3 5
= + + +¼¼
2 3
9 3 3 34
2S 2S 1 2 2 2
- = + + +
+¼¼
3 9 3 32 3 343

4S 1 2 æ 1 1 ö
= + ç1 + + +¼¼ ÷
9 3 32 è 3 3 2
ø
4S 1 2 æ 1 ö 2 3 p
= + çç ÷= Þ S= =
9 3 9 è 2 3 ÷ø 3 2 q
13. S ¥ = f( x) max x Î [-4 , 3]
a - ar = f ¢(0) = 3
f ¢( x) = 3 x 2 + 3 > 0 \ f( x) max = f(3) = 27 + 9 - 9 = 27
a
S ¥ = 27 =
1- r
1 a
a(1 - r) = 3 Þ =
1- r 3
æ aö
\ 27 = a ç ÷
è3ø
a 2 = 81 Þ a = ±9
3 1
If a =9 1- r = If a = -9 1- r = -
9 3
2 4
r= r= > 1 (rejected)
3 3
p 2
\ = \ p + q=5
q 3
14. Total runs from 1 to 9 = 1350
Let, number of terms in A.P. be n.
n
Þ [300 + (n - 1) ´ ( -1)] = 4500 - 1350 = 3150
2
Þ n = 25 or 126, n = 126 (not possible)
Þ n = 25, total matches = 34
10 100 æ 1 1 ö 10 æ 1 1 1 1 1 1 1 ö
15. x = å ç - ÷= ç1 + + + -
4 n = 3è n - 2 n + 2 ø 4 è
- - - ÷
2 3 4 102 101 100 99 ø
206 Solution of Advanced Problems in Mathematics for JEE

(2n + 1) + (2n - 1) + (2n + 1)(2n - 1)


16. f ( n) =
2n + 1 + 2n - 1
Let 2n + 1 = a and 2n - 1 = b
( a + b + ab) ( a - b) a 3 - b 3
2 2
f ( n) = =
( a + b) ( a - b) a 2 - b 2
(2n + 1) 3/ 2 - (2n - 1) 3/ 2
Þ f ( n) =
2
60 60
(2n + 1) 3/ 2 - (2n - 1) 3/ 2 (121) 3/ 2 - 1
å f ( n) = å 2
=
2
= 665
n =1 n =1

17. 3 0 {2 0 + 2 -1 + 2 -2 ¼¥} = 1{2}


1
3 -1 {2 0 + 2 -1 + 2 -2 ¼¥} = {2}
3
1
3 -2 {2 0 + 2 -1 + 2 -2 ¼¥} = {2}
3
M M
¥
2 ´1
Hence, =3
1
1-
3
18. 15 2 + (15 + d) 2 + (15 + 2 d) 2 +¼ + (15 + 9 d) 2 = 1185
Þ 19 d 2 + 90 d + 71 = 0
Þ d = -1
S n ³ S n -1
n æ n - 1ö
(31 - n) ³ ç ÷ (32 - n) Þ n £ 16
2 è 2 ø
a/r
3 2 a
19. 24 x - 14 x + kx + 3 = 0
ar
1 1
Product of roots a 3 = - Þ a=-
8 2
Þ k = -7
If x = 7 lies between the roots, then
f(7) = 49 + 7a 2 - 112 < 0
a2 -9 < 0
20. 9 x 3 + 3 y 3 + 1 = 9 xy
(9 1/ 3 x) 3 + (3 1/ 3 y) 3 + 13 = 3 (9 1/ 3 x)(3 1/ 3 y) Þ 9 1/ 3 x = 3 1/ 3 y = 1
Sequence and Series 207

21. If a , x , y , z , b A.P.
3a + b a+b a + 3b
x= ,y= and z =
4 2 4
If a , x , y , z , b H.P.
4 ab 2 ab 4 ab
x= ,y= and z =
3b + a a+b 3a + b
æ 3a + b ö æ a + b ö æ a + 3b ö æ 4 ab ö æ 2 ab ö æ 4 ab ö 343
If ç ÷ç ÷ç ÷ = 55 and ç ÷ç ÷ç ÷= Þ ab = 7
è 4 øè 2 øè 4 ø è 3 b + a ø è a + b ø è 3 a + b ø 55

❑❑❑
208 Solution of Advanced Problems in Mathematics for JEE

10
DETERMINANTS

Exercise-1 : Single Choice Problems

1. Direct expansion.
k 1 1
2. D = 1 k 1 =0 Þ ( k - 1) 2 ( k + 2) = 0
1 1 k
1 1 1
also D1 = k k 1 ¹0 Þ k ¹ 1 Þ k = -2
k2 1 k

a a2 1 + a3 1 a a2
3. b b2 1 + b = (1 + abc) 1 b b 2 = 0
3

c c2 1 + c3 1 c c2

1 2a a
2 1 1
4. D = 1 3 b b = 0 Þ = +
b a c
1 4c c
6. 2 x + ay + 6 z = 8
and 4 x + 2 ay + 6 z = 8 Þ 2 x + ay = 0
and 6 x + 12 y + 6 z = 30 Þ 4 x + (12 - a) y = 22
22
Þ y= a¹4
12 - 3 a
7. R 1 ® R 1 + R 2 + R 3
x2 - 4 x2 - 4 x2 - 4 1 1 1
2 x 2 - 13 2 2
= ( x - 4) 2 2
x - 13 2 = 0
x 2 - 13 3 7 x 2 - 13 3 7

Þ ( x 2 - 4)( x 2 - 15)(20 - x 2 ) = 0
Determinants 209

k k + 1 k -1
8. D = k + 1 k k + 2 =0
k -1 k + 2 k
R1 ® R1 - R 2 R2 ® R2 - R3
-1 1 -3
D= 2 -2 2 =0
k -1 k + 2 k
log a + (n - 1) log r log a + (n + 1) log r log a + (n + 3) log r
9. D = log a + (n + 5) log r log a + (n + 7) log r log a + (n + 9) log r
log a + (n + 11) log r log a + (n + 13) log r log a + (n + 15) log r
C3 ® C3 -C2 C2 ® C 2 - C1
log a + (n - 1) log r 2 log r 2 log r
Þ log a + (n + 5) log r 2 log r 2 log r = 0
log a + (n + 11) log r 2 log r 2 log r
a1 2a3 5a 2 a1 a3 a2 a1 a2 a3
10. D 2 = b1 2 b3 5b2 = 10 b1 b3 b2 = -10 b1 b2 b3
c1 2c3 5c 2 c1 c3 c2 c1 c2 c3
1 bc a
11. D 2 = 1 ac b
1 ab c
R 1 ® aR 1 R 2 ® bR 2 R 3 ® cR 3
2
a abc a a 1 a2 1 a a2
1
D2 = b abc b 2 = b 1 b 2 = - 1 b b 2 = -D 1
abc
c abc c 2 c 1 c2 1 c c2

0 0 -1
12. C 1 ® C 1 - C 2 + C 3 0 1 1- a =1
1 a 1+ a - b
1 2 x
13. 2 3 x 2 = 10 Þ x 2 + x - 12 = 0
3 5 2
Sum = -1
14. R 1 ® R 1 - R 2 , R 2 ® R 2 - R 3
210 Solution of Advanced Problems in Mathematics for JEE

-1 -1 -1
D = d-a+1 e-b+1 f - c + 1 , C 1 ® C 1 - C 2 and C 2 ® C 2 - C 3
x+a x+b x+c
On solving D does not depend on x.
15. R 1 ® R 1 + R 2 + R 3
1 1 1
D = ( x + y + z) 2 y y - z - x 2y C 1 ® C 1 - C 2 and C 2 ® C 2 - C 3
2z 2z z-x-y
0 0 1
3
D = ( x + y + z) 1 -1 2y Þ D = ( x + y + z) 3
0 1 z-x-y
16. ÐBOC = 60°
Þ BC = OB = OC = r
D C
AB = 2 r cos 30° = 3 r 30° 60° Q
P
2 O
Area or rectangle 3r 3 A B
= =
Area of circle 2 p
pr
17. C 1 ® C 1 - bC 3 , C 2 ® C 2 + aC 3
1 0 -2 b
2 2 2
(1 + a + b ) 0 1 2a = (1 + a 2 + b 2 ) 3
2 2
b -a 1 - a - b
2 a+ b+ c+ d ab + cd 1 1 0 1 a + b ab
18. a+ b+ c+ d 2( a + b)( c + d) ab( c + d) + cd( a + b) = c + d a + b 0 1 c + d cd
ab + cd ab( c + d) + cd( a + b) 2 abcd cd ab 0 0 0 0
=0
l m n
19. |B| = p q r = [2 Ar( DABC )]2
1 1 1
C
(l, p)

5
3

A B
(m, q) 4 (n, r)
1 2 1
20. D = 1 3 4 = 0
1 5 10
Determinants 211

1 2 1
D1 = K 3 4 = 5 ( K 2 - 3 K + 2) = 5 ( K - 1)( K - 2)
K2 5 10
1 1 1
D2 = 1 K 4 = -3 ( K 2 - 3 K + 2) = -3 ( K - 2)( K - 1)
1 K2 10
1 2 1
D3 = 1 3 K = K 2 - 3 K + 2 = ( K - 2)( K - 1)
1 5 K2

1 x + 1 ( x + 1) 2
21. ( x + 1)( x + 2)( x + 3) 1 x + 2 ( x + 2) 2 = 2( x + 1)( x + 2)( x + 3)
1 x + 3 ( x + 3) 2

-2 cos C cos B
22. cos C -1 cos A
cos B cos A -1
-2 (1 - cos 2 A) - cos C( - cos C - cos A cos B ) + cos B (cos C cos A + cos B)
1 + cos 2C 1 + cos 2 B
-2 + cos 2 A + + + 2 cos A cos B cos C
2 2
cos 2 A + cos 2C + cos 2 B + 2 cos A cos B cos C
2 cos( A + B ) cos( A + B ) + 2 cos 2 C - 1 + 2 cos A cos B cos C
2 cos C [cos C - cos( A - B )]
-2 cos C cos A cos B - 1 + 2 cos A cos B cos C = -1
24. As a , b and c are the roots of x 3 + 2 x 2 + 1 = 0 , we have
a + b + c = -2
ab + bc + ca = 0
abc = -1
a b c
Now, for finding the value of b c a , evaluating using first row, we get
c a b
a( bc - a 2 ) - b( b 2 - ac) + c( ab - c 2 ) = abc - a 3 - b 3 + abc + abc - c 3
= 3 abc - a 3 - b 3 - c 3 = - ( a 3 + b 3 + c 3 - 3 abc)
= - ( a + b + c)( a 2 + b 2 + c 2 - ab - bc - ca)
= - ( -2)[( -2) 2 - 3 (0)] = 8
212 Solution of Advanced Problems in Mathematics for JEE

l l + 1 l -1
25. For non-trivial solution,| A| or D = 0, That is, l + 1 l l + 2 =0
l -1 l + 2 l
l l + 1 l -1
Now, R 2 ® R 2 - R 1 ; R 3 ® R 3 - R 1 gives 1 -1 3 =0
-1 1 1
l l + 1 l -1
Also, R 3 ® R 3 + R 2 gives 1 -1 3 =0
0 0 4
Evaluation using third row, we get
1
4( -l - l - 1) = 0 Þ l = -
2
which is exactly the real value of l.

Exercise-2 : One or More than One Answer is/are Correct

1. f( a , b) = a( a + b)( a + 2 b)
2. R 1 ® R 1 - R 2 and R 2 ® R 2 - R 3
1 -1 0
1
0 1 -1 = 0 Þ tan q = -
3
cos 2 q sin 2 q 1 + 2 3 tan q
3. R 1 ® R 1 - R 2 , R 2 ® R 2 - R 3
-1 -1 3
2
D =d -1 2 -1 = -d 2 (13 d + 12 a)
a + 2d a a + d
1-l 3 -4
4. 1 - ( 3 + l) 5 = 0
3 1 -l

x 2 + 4 x - 3 2 x + 4 13
5. D( x) = 2 x 2 + 5 x - 9 4 x + 5 26
8 x 2 - 6 x + 1 16 x - 6 104

C 3 ® C 3 - 4C 2 , C 2 ® C 2 - 2C 1
3x + 3 3 0
D( x) = 26 x - 37 26 0
8 x 2 - 6 x + 1 16 x - 6 104
Determinants 213

a 1 2
7. D = 1 2 1 = 0 Þ a 2 - a - 2 = 0 Þ ( a - 2)( a + 1) = 0
2 1 a
0 1 2 a 0 2 a 1 0
D1 = b 2 1 D2 = 1 b 1 D3 = 1 2 b
0 1 a 2 0 a 2 1 0
a = 2 infinite solution
a = -1, b ¹ 0 has no solution.
a 1 2
8. D = 1 2 1 = 0 Þ a 2 - a - 2 = 0
2 1 a
Þ ( a - 2)( a + 1) = 0
0 1 2 a 0 2 a 1 0
D= b 2 1 D= 1 b 1 D= 1 2 b
0 1 a 2 0 a 2 1 0
a = 2 infinite solution
a = -1, b ¹ 0 has no solution.

Exercise-3 : Comprehension Type Problems

Paragraph for Question Nos. 1 to 3


2 l 6 8 l 6
D = 1 2 m = (l - 2)(m - 3) ; D1 = 5 2 m = (l - 2)( 4m - 15)
1 1 3 4 1 3
2 8 6 2 l 8
D2 = 1 5 m = 0 ; D 3 = 1 2 5 = ( l - 2)
1 4 3 1 1 4

Exercise-4 : Subjective Type Problems

2. R 1 ® R 1 + R 2 + R 3
a1 + b1 + c1 a 2 + b2 + c 2 a 3 + b3 + c 3
3 2 b1 + c1 2 b2 + c 2 2 b3 + c 3
2 c 1 + a1 2c2 + a2 2c3 + a3
214 Solution of Advanced Problems in Mathematics for JEE

a1 + b1 + c1 a 2 + b2 + c 2 a 3 + b3 + c 3
R 2 ® R 2 + R1 - R 3 = 9 b1 b2 b3
2 c 1 + a1 2c2 + a2 2c3 + a3
Now, operate as R 3 ® R 3 - R 1 + R 2
then R 1 ® R 1 - R 2 - R 3
(1 + x) 2 (1 + x) 4 (1 + x) 6
3. Let f( x) = (1 + x) 3 (1 + x) 6 (1 + x) 9
(1 + x) 4 (1 + x) 8 (1 + x) 12

Coefficient of ‘ x’ is f ¢(0).
2(1 + x) 2 4(1 + x) 3 6(1 + x) 5 (1 + x) 2 (1 + x) 4 (1 + x) 6
f ¢( x) = (1 + x) 3 (1 + x) 6 (1 + x) 9 + 3(1 + x) 2 6(1 + x) 5 9(1 + x) 8
(1 + x) 4 (1 + x) 8 (1 + x) 12 (1 + x) 4 (1 + x) 8 (1 + x) 12

(1 + x) 2 (1 + x) 2 (1 + x) 6
+ (1 + x) 3 (1 + x) 6 (1 + x) 9
4(1 + x) 3 8(1 + x) 7 12(1 + x) 11

Put x = 0, f ¢(0) = 0
5. For non-zero solution, D =0
2 3 -1
3 2 k =0 Þ k =0
4 1 1

Now, let x = l
3l 5l
So, y =- , z =-
2 2
Þ Minimum positive integer value of z is at l = -2 ; z = 5
2 a -2 3
6. 1 a 2 =0 Þ a =2
2 0 a

7. Let three terms be A - d , A , A + d.


Þ A 4 = ( A - d) 2 ( A + d) 2 = A 4 + d 4 - 2 A 2 d 2
Þ d = ± 2 A , r = 3 + 2 2 or r = 3 - 2 2
3 a1 + b1 3 a 2 + b2 3 a 3 + b3 3 a1 3a2 3a3 a1 a2 a3
8. D 3 = 3 b1 3 b2 3 b3 = 3 b1 3 b2 3 b3 = 27 b1 b2 b3
3 c1 3c2 3c3 3 c1 3c2 3c3 c1 c2 c3
Determinants 215

6 a1 2a2 2a3 a1 a2 a3
D2 = 3 b1 b2 b3 = 24 b1 b2 b3
12 c1 4c2 4c3 c1 c2 c3
1 cos q 1
9. D = - cos q 1 cos q = 3 (1 + cos 2 q)
-1 - cos q 2

Its minimum value = 3


1 1 1
10. D = 1 2 3 = l - 8 = 0 Þ l = 8
2 5 l
1 1 6
D 3 = 1 2 14 = m - 36 = 0 Þ m = 36
2 5 m
æ 1 1 1 ö÷
11. n sin 2 p ç 1 + 1 + + ¼ |n
ç |2 |3 | N ÷ø
è
æ 1 1 1 ö
n sin 2 p ç 1 + + +¼ ÷
è n + 1 ( n + 1)( n + 2 ) ( n + 1)( n + 2 )¼ ( N ) ø
Using sin(2np + q) = sin q
æ 1 1 1 ö
= n(2 p) ç + +¼ ÷
è n + 1 (n + 1)(n + 2) (n + 1)(n + 2)¼ N ø
sin q
Using lim =1
q®0 q
= 2p
cos q sin q cos q
12. sin q cos q sin q = 0 Þ -2 cos q cos 2q = 0
cos q sin q - cos q

❑❑❑
216 Solution of Advanced Problems in Mathematics for JEE

11
COMPLEX NUMBERS

Exercise-1 : Single Choice Problems

y -7 1
2. arg ( z - 2 - 7i ) = cot -1 (2) Þ =
x -2 2
æ z - 5i ö p
arg ç ÷=± Þ x( x + 2) + ( y - 5)( y - 1) = 0
è z + 2 -i ø 2
4. z 12 + z 22 = z 1 z 2
1
5. Let w = reiq then z = ei ( p/ 2+ q )
r
1
z w = e -i ( p/ 2+ q ) r × eiq = e -i p/ 2
r
n n
6. a å rw r -1 + bå w r -1 = a(1 + 2w + 3w 2 +¼¼ + nw n -1 ) + b(1 + w + w 2 +¼¼ + w n -1 )
r =1 r =1
ìï1 + w + w 2 +¼¼ + w n -1 nw n üï
=aí - ý + b ( 0)
ïî 1-w 1 - w ïþ
æ n ö
= aç0 - ÷+0
è 1 - wø
8. z 4 + z 3 + 2 = 0 has roots z 1 , z 2 , z 3 and z 4 .
Þ ( z - 1) 4 + 2 ( z - 1) 3 + 32 = 0 has roots (2 z 1 + 1),(2 z 2 + 1),(2 z 3 + 1) and (2 z 4 + 1)
æ z - 6 - 3i ö p
9. arg ç ÷=
è z - 3 - 6i ø 4
Þ ( x - 6) 2 + ( y - 6) 2 = 9
11. |iz + z 1| =|i||z - iz 1| =|z - iz 1|
Maximum distance of iz 1 ( -3 + 5 i ) from z is 2 + 3 2 + (5 - 1) 2 = 7
Complex Numbers 217

z 1 - z 2 |z 1 - z 2| iq z1
12. = e
z 3 - z 2 |z 3 - z 2|
z 1 - z 3 |z 1 - z 3| -iq
= e
z 2 - z 3 |z 2 - z 3|
æ z - z 2 z1 - z 3 ö æ z - z 2 iq z 1 - z 3 -iq ö z2 q q z3
arg çç 1 + ÷÷ = arg ç 1
ç z -z e - z -z e ÷
÷
è z3 - z2 z3 - z2 ø è 3 2 2 3 ø
p

2
z 2 3 i p/ 3
13. = e
z1 2
3 ip/ 3 æ pö p
1+ e ç 2 + 3 cos ÷ + 3i sin
z1 + z 2 2 è 3ø 3
= =
z1 - z 2 3 ip/ 3 p p
1- e 2 - 3 cos - 3i sin
2 3 3
2 2
æ7ö æ3 3 ö
ç ÷ + çç ÷
÷
è2ø è 2 ø 49 + 27 133
= = =
2 2 1 + 27 7
æ 1ö æ3 3 ö
ç ÷ + çç ÷
÷
è2ø è 2 ø
14. z 1 z 2 z 3 = -c
Þ 1 =|c| Þ |c| = 1
|z 1 + z 2 + z 3| £ |z 1| + |z 2| + |z 3|
|a| £ 3
|b| =|z 1 z 2 + z 2 z 3 + z 3 z 1| £ |z 1 z 2| + |z 2 z 3| + |z 3 z 1|
Þ |b| £ 3
1
15. £ |z| £ 4
2
2 2
1 ææ 1ö ö ææ 1ö ö 1
z+ = çç ç r + ÷ cos q ÷÷ + çç ç r - ÷ sin q ÷÷ = r2 + + 2 (cos q - sin q)
z èè rø ø èè rø ø r2
16. |3 + i ( z - 1)| =|z - 1 - 3i|
Maximum distance of A from ( z ) = OA + r 2 O
A(1,3)
(0,2)
= 1+ 1 + 2 =2 2
17. x 2 - ( 2 i ) x - 1 = 0
2 i ± -2 + 4 1
x= = ( ± 1 + i)
2 2
218 Solution of Advanced Problems in Mathematics for JEE

p 3p
x = cis , cis
4 4
3p p
x 2187 = cis , cis
4 4
1 æ - 3p ö æ pö 1 3p p
= cis ç ÷ , cis ç - ÷ Þ x 2187 - = 2i sin , 2i sin = 2 i
2187 2187
x è 4 ø è 4ø x 4 4
(1 + z 9 )
18. 1 × = 0 , z ¹ -1
1+ z
Þ z 9 = -1
i ( 2n +1) p
iq 9
Þ re = e , n = 1, 2 , ¼¼ 8
ia ib
19. Let P( re ) & Q ( re )
Point of intersection of tangents at ‘ a ’ , ‘ b’ to circle x 2 + y 2 = r 2 is
æ a +b a +b ö æ a +b ö
iç ÷
ç cos r sin ÷ è 2 ø
çr× 2 -i 2 ÷ = re 2w 1 w 2
=
ç a -b a -b ÷ a - b w1 + w 2
ç cos cos ÷ cos
è 2 2 ø 2
® ® ® ® ® ®
20. |z 1 - z 2|2 + |z 2 - z 3|2 + |z 3 - z 1|2 = 2 ( 4 + 9 + 16) - 2 ( a × b + b × c + c × a)
® ® ®
where a , b , c are position vectors of points z 1 , z 2 , z 3
æ 1ö
Þ Maximum value = 58 - 2 (6 + 12 + 8) ç - ÷ = 84
è 2ø
7+i
21. We have Z=
3 + 4i
Simplifying (i . e. , rationalizing the denominator), we get
7 + i 3 - 4i 21 + 4 - 28i + 3i
´ =
3 + 4i 3 - 4i 9 + 16
25 - 25i
= =1-i
25
14
æ 7+i ö
Therefore, ç ÷ = (1 - i ) 14
è 3 + 4i ø
= [(1 - i ) 2 ]7 = (1 + i 2 - 2i ) 7
= ( +2 7 ) i
22. |Z - 4| + |Z + 4| = 10
PS + PS ¢ = 2 a
which implies that foci at 4 and -4 and a = 5 as shown in the following figure.
Complex Numbers 219

bi

–5 –4 0 4 5
–bi

Now, b 2 = 25 (1 - e 2 ) = 25 - (5e) 2
= 25 - 16 = 9
Þ b=3
Z lies on the ellipse circumference |Z| denotes the distance from the origin. Therefore,
|Z|max = 5
|Z|min = 3
Thus, the difference between the maximum and the minimum values of |Z| is
|Z|max -|Z|min = 5 - 3 = 2

Exercise-2 : One or More than One Answer is/are Correct

1. Let z 1 = rei q and z 2 = rei f


|z 1 + z 2| =|z 1|
Þ |ei q + ei f| =|ei q| = 1
Þ (cos q + cos f) 2 + (sin q + sin f) 2 = 1
1 2p 2p
Þ cos(q - f) = - Þ q -f = or -
2 3 3
z1
= ei ( q -f) = ei 2p/ 3 or e -i 2p/ 3
z2
æz ö p
2. (a) If arg çç 1 ÷÷ = then z 1 and z 2 subtend right-angle at circumcentre origin.
è z2 ø 2
\ the chord joining z 1 and z 2 will subtend an angle q at ‘ z ’ such that
ìq = p 4 if |z| = 1
ï
íq < p 4 if |z| > 1 z

îïq > p 4 if |z| < 1 q


1 1 1 q
(b) |z 1 z 2 + z 2 z 3 + z 3 z 1| =|z 1| ×|z 2| ×|z 3| + +
z1 z 2 z 3 q

=|z 1 + z 2 + z 3|
æ ( z + z 2 )( z 2 + z 3 )( z 3 + z 1 ) ö æ ( z 1 + z 2 )( z 2 + z 3 )( z 3 + z 1 ) ö
(c) çç 1 ÷÷ = ç ÷
z1z 2z 3 ç z1z 2z 3 ÷
è ø è ø
220 Solution of Advanced Problems in Mathematics for JEE

(d) The triangle formed by joining z 1 , z 3 and z 2 is isosceles and right angled at z 3 .

B(iz)
A(z)

3. O

C(i2z)

Method I : Multiplying a complex number by i rotates a vector for z in the anticlockwise


direction by an angle of 90°.
\ Ð AOB = ÐBOC = 90°
As shown in figure, the DABC is a right angled isosceles triangle.
Method II : Let z , iz , i 2 z are vertices A , B and C of the triangle ABC.
\ | AB| =|BC | also | AB|2 + |BC |2 =| AC|2
Since, | AB| =|BC| also | AB|2 + |BC|2 =| AC|2
\ the DABC is a right angled isosceles triangle.
7. ( z + i ) 4 = 1 + i
æ p 2m p ö
z = -i + 2 1/ 8 cos ç + ÷
è8 4 ø
æ 2 1/ 8 × 2 ö
Square side length = ç ÷
ç 2 ÷
è ø
æ1 3 ö÷
8. z = 4 çç - i
è2 2 ÷ø
æ 2mp 60° ö
Roots = 4 1/ 4 cos ç - ÷
è 4 4 ø
m = 0 , 1, 2 , 3 m =1
2
9. a + bw + cw = a
a + bw 2 + cw = a
æ 1 3 i ö÷ æ 1 3 i ö÷
|a| = 1 Þ a + b ç - + + cç - - =1
ç 2 2 ÷ø ç 2 2 ÷ø
è è
10. Check option for z = w
w 62 + w + 1 = 0 w2 + w + 1 = 0
w155 + w + 1 = 0 w2 + w + 1 = 0
Complex Numbers 221

Exercise-3 : Comprehension Type Problems

Paragraph for Question Nos. 1 to 2


Sol. f( z ) + f( z ) = f( z ) + f( z )
(az + b) + ( a z + b ) = a z + b + a z + b
Þ (a - a )( z - z ) = 0
Þ Im (a ) = 0 (Im( z ) ¹ 0)
f( z ) + f( z ) = 0
Þ a( z + z ) + (b + b ) = 0 (Q a = a )
Þ Re(b) = 0 (Re( z ) = 0)
2 2
| f( z )| > ( z + 1)
2 2
Þ a z + b 2 > z 2 + 2z + 1
Þ (a 2 - 1) z 2 - 2 z + (b 2 - 1) > 0 " z Î R
Paragraph for Question Nos. 3 to 5
Sol. |a - b| = 2 7
2
Þ |(a + b) - 4ab| = 28
Þ |z 12 - 4( z 2 + m )| = 28
Þ |m - ( 4 + 5 i )| = 7
greatest (|m|) = 16 + 25 + 7
least |m| = 7 - 16 + 25
Paragraph for Question Nos. 6 to 7
Sol. C 1 :|z - z 1|2 + |z - z 2|2 = 10 Þ C 1 : ( x - 5) 2 + y 2 = 1
C 2 :|z - z 1|2 + |z - z 2|2 = 16 Þ C 2 : ( x - 5) 2 + y 2 = 4
Paragraph for Question Nos. 8 to 9
Z 2 - Z1 Z - Z 1 i q/ 2 Z 3 - Z1 Z - Z 1 i q/ 2
Sol. = 4 ×e , = 4 e B(Z3)
|Z 2 - Z 1| |Z 4 - Z 1| |Z 3 - Z 1| |Z 4 - Z 1|
( Z 2 - Z 1 )( Z 3 - Z 1 ) ( Z 4 - Z 1 ) 2 q i
Þ = e I(Z4)
|Z 2 - Z 1||Z 3 - Z 1| |Z 4 - Z 1|2
( Z 2 - Z 1 )( Z 3 - Z 1 ) AB × AC
q/2

Þ =
2
(Z 4 - Z1 ) ( IA) 2
q/2
Similarly, others. B(Z1) (Z1+Z2) B(Z2)
222 Solution of Advanced Problems in Mathematics for JEE

Exercise-4 : Matching Type Problems

1. Let BC = n , CA = n + 1, AB = n + 2
æ z - z3 ö æ z - z1 ö
(A) arg çç 1 ÷÷ = 2 arg çç 3 ÷÷ = ÐC = 2 ÐA
z
è 2 - z 3 ø è z 2 - z1 ø
sin C sin A sin 2 A sin A
\ = Þ =
c a n+2 n
n+2 (n + 2) 2 + (n + 1) 2 - n 2 n + 2
Þ cos A = Þ =
2n 2(n + 2)(n + 1) 2n
Þ n(n 2 + n + 5) = (n 2 + 3n + 2)(n + 2) Þ n 2 - 3n - 4 = 0 Þ n = 4
\ biggest side = n + 2 = 6
® ® ® ®
(B) ( c - a ) × ( b - c ) = 0 Þ ÐC = 90° Þ a 2 + b 2 = c 2
Þ n 2 + (n + 1) 2 = (n + 2) 2 Þ n = 3
1
\ Area = × 3 × 4 = 6 = D
2
® ® ® ® ® ®
\ | a ´ b + b ´ c + c ´ a| = 2 D = 12
a1 b2 - a 2 b1 4 3
(C) = tan A = \ cos A =
a1 a 2 + b1 b2 3 5
(n + 2) 2 + (n + 1) 2 - n 2 3
\ =
2(n + 2)(n + 1) 5
Þ 5(n 2 + 6n + 5) = 6(n 2 + 3n + 2)
Þ n 2 - 12n - 13 = 0 Þ n = 13
1 1
\ S - c = ( a + b - c) = (13 + 14 - 15) = 6
2 2
(D) Altitudes are in H.P. Û sides are in A.P.
Also, b > a + c , a > b + c , c > a + b Þ least value of a = 2
\ least value of b = 3
3. (A) {0 , 1, w + 1} m = {0 , 1, - w 2 } m
0 , 1, - 1, - w 2 , - w, w
(B) 2w,( x 2 - x + 10) = 0 roots are 2 + 3w, 2 + 3w 2
Last number is 3.
(C) Central angle = 60° Equilateral D
(D) Put z = 1 z 1 = 1, z 2 = w, z 3 = w 2
Complex Numbers 223

Exercise-5 : Subjective Type Problems


Im(z)

p/4
1. p/4
O
O Re(z)

2 £ |z| £ 4
1
Probability =
4
2.
Im(z)
5
4
3
2
1
Re(z)
O 1 2 3 4 5

3. z + z = 2 |z - 1| Þ y2 = 2x - 1
p
arg ( z 1 - z 2 ) = Þ y1 - y 2 = x1 - x 2
4
y 12 - y 22 = 2 ( x 1 - x 2 ) = 2 ( y 1 - y 2 ) Þ y 1 + y 2 = 2 ( y 1 ¹ y 2 )

❑❑❑
224 Solution of Advanced Problems in Mathematics for JEE

12
MATRICES

Exercise-1 : Single Choice Problems

écos q ù é sin q ù
1. A = ê ú [cos q sin q] + ê - cos q ú [sin q - cos q]
ë sin q û ë û
é cos 2 q cos q sin q ù é sin 2 q - sin q cos q ù
=ê 2 ú+ê ú =I
ësin q cos q sin q û ë - sin q cos q cos 2 q û
0 0 -1
2. | A| = 0 -1 0 =1
-1 0 0
æ0 0 -1ö æ 0 0 -1ö æ 1 0 0 ö
2ç ÷ç ÷ ç ÷
A =ç 0 -1 0 ÷ ç 0 -1 0 ÷ = ç 0 1 0 ÷
ç -1 0 0 ÷ø çè -1 0 0 ÷ø çè 0 0 1 ÷ø
è
é3 0 0ù
3. A = ê0 3 0ú
ê ú
êë0 0 3 úû
det (adj (adj ( A))) =| A|4 = 27 4
ì27 4 ü 1
í ý=
î 5 þ 5
4. A -1 B -1 = B -1 A -1 Þ C = ( A -1 + B -1 ) 5 = ( I ) 5
5. A 4 = I Þ A( A 3 ) = I
7. (adj A) A =| A|I
| A| = xyz - 8 x - 4 y - 3 z + 28 = 2l - l = l
8. ( x - 2) + ( x 2 - x + 3) + ( x - 7) = 0
x 2 + x - 6 = 0 Þ ( x + 3)( x - 2) = 0
Matrices 225

é -1 3 ù 1 é0 3 ù
9. A = ê ú Þ A -1 = ê
ë 3 0û 9 ë3 1úû
é 1 - tan q ù é cos 2 q - sin q cos q ù é a -bù
10. ê ê ú =ê
ëtan q 1 úû ësin q cos q cos 2 q û ë b a û
ú

Þ a = cos 2q , b = sin 2q
11. P2 =I -P
or P 3 = P - P 2 = 2P - I
or P 4 = 2I - 3P
or P 5 = -3 I + 5P
or P 6 = 5I - 8 P
2
12. |adj (adj ( A))| =| A|( n -1)
Þ | A| = x + y + z = 12
x ³ 1, y ³ 1, z ³ 1
11
Þ C 2 = 55
T
é a bù é d -c ù é d -bù é a bù
13. Let A = ê ú ; adj( A) = ê ú =ê ú ; adj (adj ( A)) = ê ú
ë c dû ë -b a û ë -c a û ë c dû
14. M = A 2m × A -1
A 2m +1
M=
a 2 + b2
If A 2 = ( a 2 + b 2 ) × I Þ A 2m = ( a 2 + b 2 ) m × I
A 2m +1 = ( a 2 + b 2 ) m × A
15. A 2 + 5 A + 6 I = I
( A + 2 I )( A + 3 I ) = I
Þ A + 2 I and A + 3 I are inverse of each other.
é3 -5 ù é12 -5 ù é 1 0 ù
16. AB = ê úê ú =ê ú =I
ë7 -12 û ë 7 -3 û ë0 1û
é 3 -2 ù
17. adj( A) = ê ú
ë -2 2 û
18. AA 1 = I
écos q 2 sin q ù é cos q sin q ù é 1 0 ù
ê sin q cos q ú ê2 sin q cos q ú = ê0 1ú
ë û ë û ë û
écos 2 q + 4 sin 2 q 3 sin q cos q ù é 1 0 ù
Þ ê ú =ê ú Þ sin q = 0
ë 3 sin q cos q sin 2 q + cos 2 q û ë0 1û
226 Solution of Advanced Problems in Mathematics for JEE

æ 1 2ö æ cos q sin q ö
20. If A = ç ÷, P = ç ÷ , Q = P T AP , we have
è 0 1ø è - sin q cos q ø
P( P T AP )( P T AP ) ¼( P T AP )P T
PQ 2014 P T =
2014 times
= ( PP T ) A( PP T ) A( PP T ) ¼( PP T ) A( PP T )
Matrix multiplication is associative.
æ cos q sin q ö æ cos q - sin q ö
PP T = ç ÷ç ÷
è - sin q cos q ø è sin q cos q ø
æ 1 0ö
=ç ÷ = I2
è 0 1ø
Hence, PQ 2014 P T = A 2014
æ1 2ö æ1 2ö æ 1 2ö æ 1 4ö
A =ç ÷ Þ A2 =ç ÷ç ÷ =ç ÷
è0 1ø è0 1ø è 0 1ø è 0 1 ø
æ1 4öæ 1 2ö æ 1 6ö
A3 =ç ÷ç ÷ =ç ÷
è0 1 ø è 0 1ø è 0 1ø
æ1 6ö æ 1 2ö æ 1 8ö
A4 =ç ÷ç ÷ =ç ÷
è0 1ø è 0 1ø è 0 1ø
æ1 2n ö æ 1 4028 ö
Þ An =ç ÷ and A 2014 = ç ÷
è0 1ø è0 1 ø
2 2
æMö M æ1 ö
21. adj ç ÷ = = ç |M|÷
2
è ø 2 è 8 ø
1 1
22. | A -1| = =
| A| 5
|( AB ) T | =| AB| =| A × ( adj A)| =| A||
× adj ( A)|= 5 ´ 5 2 = 5 3
1 1
\|| A -1|( AB ) T |=| ( AB ) T |= | AB|= 1
5 53

Exercise-2 : One or More than One Answer is/are Correct

3. A a Ab = A a +b
é1 0 0ù
Also, A 0 = ê0 1 0 ú = I
ê ú
êë0 0 1úû
and A a A -a = A a -a = A 0 = I
Matrices 227

we get A a-1 = A -a
However, A a-1 = - A a and A a2 = -I do not hold.
4. A( A 2 - I ) - 2( A 2 - I ) = 0
( A 2 - I )( A - 2 I ) = 0

Exercise-3 : Matching Type Problems

1. (A) Possible non-negative value of | A| = 2 , 4 , 8


(B) Sum is 0.
(C) |adj(adj(adj A)))| =| A|
least absolute value of | A| = 2
Þ | A| = ± 2
(D) least | A| = -8
16
|4 A -1| = = -2
| A|
2. (A) Since A is idempotent, A 2 = A 3 = A 4 =¼¼ = A. Now,
( A + I ) n = I + n C 1 A + n C 2 A 2 +¼¼ + n C n A n
= I + n C 1 A + n C 2 A +¼¼ + n C n A
= I + ( n C 1 + n C 2 +¼¼ + n C n ) A
= I + (2 n - 1) A
Þ 2 n - 1 = 127 Þ n = 7
(B) We have,
( I - A)( I + A + A 2 +¼¼ + A 7 )
= I + A + A 2 +¼¼ + A 7 + ( - A - A 2 - A 3 - A 4 ¼¼ - A 8 )
= I - A8
=I (if A 8 = 0)
(C) Here matrix A is skew-symmetric and since | A| =| A T | = ( -1) n| A|, so | A|(1 - ( -1) n ) = 0.
As n is odd, hence| A| = 0. Hence A is singular.
(D) If A is symmetric, A -1 is also symmetric for matrix of any order.
æ ö
n ç ÷ 1
1 ç 1 ÷ = 1 dx
5. (A) å
n r =1 ç r ÷ x
ò
ç ÷ 0
è nø
228 Solution of Advanced Problems in Mathematics for JEE

(B) D = 4 cos t cos 2t


(C) 3 x 2 + 2 px + g < 0
æ 5ö
f ç- ÷ =0 f( -1) = 0
è 3ø
(D) (2 x - 2) 2 + 1 + ||b - 1|-3| =|sin y|
b -1= ±3
|sin y| = 1

Exercise-4 : Subjective Type Problems

1. ( AB ) 2 = AB × AB = A 3 B 2
( AB ) 3 = ( AB ) 2 × AB = A 3 B 2 × AB = A 7 B 3
( AB ) 4 = ( AB ) 3 × AB = A 7 B 3 × AB = A 15 B 4 Þ ( AB ) 10 = A 1023 B 10
æ 3 32 33 ö 1
2. l = lim 18 ç + + +¼¼ ÷ = 18 ´ =9
n® ¥ ç 2 4 6 ÷ æ 1ö
è3 3 3 ø 3ç1 - ÷
è 3ø
æ 2 22 ö 1
m = lim 12 ç + +¼¼ ÷ = 12 ´ = 12
n® ¥ ç 2 4 ÷ æ 1ö
è2 2 ø 2ç1 - ÷
è 2ø
a1 a 2 a 3
4. b1 b2 b3 = a1 b2 c 3 - a1 c 2 b3 …six elements
c1 c2 c3
All cannot be simultaneously 1.
5. First element of matrix A10 = 286 (10 th of sequence 1, 2, 6, 15, …)
Trace of A10 = 286 + 297 + 308 + 319 +¼ + 385 = 3055
❑❑❑
Permutation and Combinations 229

13
PERMUTATION AND COMBINATIONS

Exercise-1 : Single Choice Problems


7 7 7
1. = 81; = 72; = 72
9 9 8 9 8 9

æ 8! ö 2 æ 8! ö
2. ç ÷ ´ C1 ´ 3! + ç ÷ ´ 3 ! = 8400
è 3 !3 !2 !2 ! ø è 3 !2 !2 !2 ! ø
æ 3! ö
3. Number of ways = 6 ´ ç ´ 3 !÷ = 108
è 2! ø
4 5!
4. C1 ´ = 240
2!
6
5. C 2 ´ 1 ´ 4 ! = 360
a
6. x 2 - 5x + 3 = 0
b
Þ a + b = 5, ab = 3
a b a 2 + b 2 19
Sum of roots = + = =
b a ab 3
5
7. C 4 ´ 8 C 6 + 5 C 5 ´ 8 C 5 = 196
21
8. (1 + 2 + 3 +¼¼ + 22) C 10
2009 ´ 2008 ´ 2007 + 1 1
9. x = = 2008 +
2008 ´ 2007 ´ 2007 2009 ´ 2007
Þ [ x] = 2008
10. N = p1n p 2 p 3 ¼¼ p m +1
No. of factors = (n + 1) 2 m
11. Number of ways = (11)! ´ 2 12
230 Solution of Advanced Problems in Mathematics for JEE

12.
5 5

m = 5 ´ 5 ´ 8C 2 ´ 2 !

4 5

n = 4 ´ 5 ´ 8C 2 ´ 2 !
13. Three different digits (not including zero)
9
C 3 ´ 2!
Two digits (not including zero)
9
C2 ´2
Three digits (including zero)
9
C2 ´1
14. Let no. of elements in A = n
No. of elements in B = m
2 n - 2 m = 1920 = 2 7 ´ 15
Þ n = 11, m = 7
n( A È B ) = n( A) + n( B ) - n( A Ç B ) = 15
15. C ¼¼¼ = 4 ! = 24
D¼¼¼ = 4 ! = 24
M ¼¼¼ = 4 ! = 24
S C …… = 3! = 6
S D …… = 3! = 6
S M C DW=1
S M C WD=1
5! 6!
16. P = All A’s together = ; Q = All B’s together =
3! 4!
5! 6 !
n( P Ç Q ) = 3 ! ; n( P È Q ) = + - 3 ! = 50 - 6 = 44
3! 4!
17. 5 6 ´ 6 7 ´ 7 8 ´ 8 9 ´ 9 10 ´ 10 11 ´¼¼´ 30 31
No. of zero’s = no. of 5’s
= 6 + 11 + 16 + 21 + (2 ´ 26) + 31 = 137
18. ( x - y)( x + y) = 10 ´ 337
Þ x - y = 10 and x + y = 337
347
x= (not possible)
2
19. Total number of different things = n + 2
Permutation and Combinations 231

20. Let the numbers are 10 - d , 10 , 10 + d.


d Î {-9 , - 8 , - 7 , ¼¼ , 7 , 8 , 9}
22. m = 2 ´ 5 ! ´ 5 !
n = 4 ! ´ 5!
23. Total ways = 4 ´ 4 ! = 96
4
25. C 2 ´ 5 2 ´ (21) 2 = 66150
26. Total all letters are different.
Þ 10 5 - 10 C 5 ´ 5 ! = 69760
29. M = 1440
M = 25 ×32 ×5
No. of divisions = 6 ´ 3 ´ 2 = 36
P = Product of divisors = (1440) 18
P = 2 90 × 3 36 × 518
Hence, x = 30
30. Case-1 : All digits same = 9
Case-2 : Excluding zero :
4!
(i) No’s having 3 digits same : 9 C 2 ´ 2 C 1 ´ = 288
3!
4!
(ii) No’s having 2 digits same, 2 other same : 9 C 2 ´ = 216
2 !2 !
Case-3 : Including zero :
(i) No’s having 3 zero’s : 9
3!
(ii) No’s having 2 zero’s : 9 C 1 ´ = 27
2!
3!
(iii) No’s having 1 zero = 9 C 1 ´ = 27
2!
Hence, total no’s = 576
31. Case-I : When two T’s contain exactly one vowel between them,
5 ! ´ ( 5 C 1 ´ 5 C 4 ´ 4 !) = 15 ´ 5 ! ´ 5 !
Case-II : When two T’s also contain consonant between them,
4 ! ´ ( 5 C 2 ) ´ ( 7 C 5 ´ 5 !) = 42 ´ 5 ! ´ 5 !
32. 6 6 6 6 6 0 ® 6
6!
666633®
4 !2 !
6!
666642®
4!
232 Solution of Advanced Problems in Mathematics for JEE

6!
666444®
3 !3 !
6!
33. Five 4 runs + one 0 run =
5!
6!
Four 4 runs + two 2 runs =
4 !2 !
6!
Three 4 runs + two 3 runs + one 2 runs =
3 !2 !
6!
Two 4 runs + four 3 runs =
2 !4 !
Þ N = 96
7
34. C 2 = 21
35. x 1 + x 2 + x 3 + x 4 + x 5 = 101
Let x 1 = 2 k1 + 1, x 2 = 2 k 2 + 1, x 3 = 2 k 3 + 1, x 4 = 2 k 4 + 1, x 5 = 2 k 5 + 1
48+ 5-1
Þ k1 + k 2 + k 3 + k 4 + k 5 = 48 ; C 5-1
36. Total ways = (largest number is 4)
6 4 - ( 4 4 - 3 4 ) = 1121
6
37. C 3 ´ 4!
38. If two points are selected from one side of main diagonal = 6 C 2 .
Then other two points are selected on other side of main diagonal = 1.
Total ways = 6 C 2 ´ 1 = 15

39. (9 - x 1 ) + (9 - x 2 ) + (9 - x 3 ) + (9 - x 4 ) + (9 - x 5 ) = 43
Þ x1 + x 2 + x 3 + x 4 + x 5 = 2
2+ 5-1
Number of ways = C 5-1 = 6 C 4 = 15

Exercise-2 : One or More than One Answer is/are Correct

1. Case-I : All five letters are different.


= 5!
Case-II : Two letters are same and remaining are different.
3 5!
C 1 ´ 4 C 3 ´ = 720
2!
Case-III : Two alike, two other alike and remaining different.
3 5!
C 2 ´ 3C 1 ´ = 270
2 !2 !
Total number of words = 1110
Permutation and Combinations 233

100
2. å 100 C k ( x - 2)100-k × 3 k = ( x + 1)100
k =0

Coeff. of x 50 = 100 C 50
Total - Row 1 - Row 2
3. {|2 for N }
|2
8
C 5|6 -|6 -|6
|2
4. = (four odd) + (4 even) + (3 even + 1 odd) + (2 even + 2 odd)
= 5C 4 ´ 4 ! + 4C 4 ´ 4 ! + 4C 3 ´ 5C 1 ´ 4 ! + 4C 2 ´ 5C 2 ´ 4 ´ 4
= 1584

Exercise-3 : Comprehension Type Problems


Paragraph for Question Nos. 1 to 2
1. 0
2. Digit 6 always come at last three place digit 5 always come at last four place and digit 4 always
come at last five place.
3
C 1 ´ 3 C 1 ´ 3 C 1 ´ 3 ! = 162

Exercise-4 : Matching Type Problems

6! 7
1. (A) ´ C 2 = 7560 (B) 5 !´ 6 C 2 = 1800
2!
4!
(C) 7560 - 1800 = 5760 (D) 4 ! ´ 5 C 4 ×
= 720
2 !2 !
2. (A) Total ways - (No repeating letter is at odd position)
11! 11!
-0 =
2 !2 !2 ! (2 !) 3
7! 4!
(B) ´ 8C 4 ´ = 210 ´ 7 !
2 !2 ! 2!
(C) MM TT HEICS

7 ! ´ 8 C 2 ´ 1 = 28 ´ 7 !
æ 4 !ö æ 7 ! ö 4 ! 7 !
(D) ç ÷ ´ ç ÷=
è 2 ! ø è 2 !2 ! ø (2 !) 3
234 Solution of Advanced Problems in Mathematics for JEE

Exercise-5 : Subjective Type Problems


9
1. C 4 ´ 5 C 4 = 630
9 7! 9!
2. C2 ´ =
2 !2 ! 8
10
4. C 3 - 8 C 3 = 64
5. Case-I: If Ravi is include.
7
C 5 ´ 9 C 8 = 189
Case-II: If Ravi is not include.
7
C 6 ´ [ 8 C 7 + 9 C 8 ] = 119
Total number of ways = 308
6
6. C 4 - 4C 2 = 9
7. 5 ! - (1 + 5 C 2 ´ 1) = 109
8. Let other two sides are a and b.
\ a + b > 11 0 < a £ 11, 0 < b £ 11
9.
a3 b2 c3
a2 c2
b1
a1 c1

( a1 , a 2 , a 3 ),( b1 , b2 ) and ( c1 , c 2 , c 3 ) are alike things so these can be arranged is


8! 4 ×5 ×6 ×7 ×8
= = 560
2 !3 !3 ! 2 ´6
n
10. C 2 - n = 14 Þ n = 7
11. x 1 + x 2 + x 3 +¼¼ + x 7 + x 8 = 93
x 1 ³ 0 , x 2 ³ 6 , x 3 ³ 6 , ¼¼ x 7 ³ 6 , x 8 ³ 0
x 1 + x ¢2 + x ¢3 +¼¼ + x 7¢ + x 8 = 57
64
No. of ways = C7
4
12. C 4 ( 2 C 1 ) 4 = 16
13. Let x 1 objects of one type
x 2 objects of second type
x 3 objects of third type
x 1 + x 2 + x 3 = 3n
Permutation and Combinations 235

0 £ x 1 £ 2n, 0 £ x 2 £ 2n , 0 £ x 3 £ 2n
3n + 2 n +1
Number of ways = C2 -3 ´ C 2 = 3n 2 + 3n + 1
14. x + y + z + w = 15
x ³ 0, y ³ 6, z ³ 2, w ³ 1
x + y¢ + z ¢ + w¢ = 6
Number of ways = 9 C 3 = 84

❑❑❑
236 Solution of Advanced Problems in Mathematics for JEE

14
BINOMIAL THEOREM

Exercise-1 : Single Choice Problems


153
1. Let x = 2 2 N = x 16 - 1
a = x2 + 2 x + 1 N = ( x 4 - 1)( x 4 + 1)( x 8 + 1)
N = ( x 4 - 1)( x 2 + 2 x + 1)( x 2 - 2 x + 1)( x 8 + 1)
Let y = 2 204 N = y 6 - 1 = ( y 3 - 1)( y 3 + 1)
b = y2 - y + 1 = ( y 3 - 1)( y + 1)( y 2 - y + 1)
4
3. C 2a 2 = - 6C 3a 3
3
Þ a =-
10
5. a n = (2 + 3 ) n
Let a ¢n = (2 - 3 ) n Þ a n + a ¢n = integer
Þ [a n ] + {a n } + a ¢n = integer Þ {a n } = 1 - a ¢n
So, lim (a n - [a n ]) = lim [1 - (2 - 3 ) n ] = 1 - 0 = 0 (Q0 > {a n }, a ¢n < 1)
n ®¥ n ®¥
20
6. N = C 7 - 20 C 8 + 20
C9 - 20
C 10 +¼¼ - 20
C 20
= ( 20 C 7 + 20
C9 + 20
C 11 +¼¼ + 20
C 19 ) - ( 20 C 8 + 20
C 10 +¼¼ + 20
C 20 )
= ( 20 C 0 + 20
C2 + 20
C4 + 20
C 6 ) - ( 20 C 1 + 20
C3 + 20
C 5)
= (1 + 190 + 4845 + 38760) - (20 + 1140 + 15504)
= 43796 - 16664 = 27132 = 3 ´ 4 ´ 7 ´ 19 ´ 17
é 1 ù
7. log 2 ê1 + (2 12 - 2)ú = log 2 2 11 = 11 [QS n C r = 2 n ]
ë 2 û
r
æ 1 ö
8. T r +1 = n C r × x n -r × y r = 12 C r × x 12-r × ç ÷
è x3 ø
Binomial Theorem 237

12 - 4 r = 0 Þ r = 3
12 ´ 11 ´ 10
T 4 = 12 C 3 = = 220
3 ×2 ×1
3 4 5 1 1
9. + + +¼¼ = -
4 ! 5! 6 ! 3 ! ( k + 3)!
52 52
r é1 1 ù 1 1
å ( r + 1)! = å êë r ! - ( r + 1)!úû = 3 ! - 53 !
r =3 r =3

Þ k = 50
n
10. f( x) = å[( r + 1) 2 n C r - r 2 n C r -1 ]
r =1

f(n) = (n + 1) 2 - 1
f(30) = 960
n
12. C 1 × a + C 2 × a + C 3 × a 3 +¼¼ n C n × a n = (1 + a ) n - 1
n 2 n

æ 2p i ö
ç where a = e n = a 2 ÷
ç a1 ÷
è ø
13. 2 30 × 3 20 = 2 10 × (6) 20 = 1024 (7 - 1) 20 = 1024 (7 K + 1) = 7 k ¢ + 1024 = 7 k ¢ + 1022 + 2
26 26 26 26
14. C0 + C1 + C 2 +¼¼ + C 26 = 2 26
Þ 2( 26 C 0 + 26
C 1 +¼¼ + 26
C 13 ) = 2 26 + 26
C 13
15. (1 + x + x ) = a 0 + a1 x + a 2 x + a 3 x +¼+ a 2n x 2n
2 n 2 3

differentiate w.r.t. x
n(1 + x + x 2 ) n -1 (1 + 2 x) = a1 + 2 a 2 x + 3 a 3 x 2 +¼ + 2n × a 2n x 2n -1
Put x = 1 n × 3 n = a1 + 2 a 2 + 3 a 3 +¼ + 2n a 2n …(1)
Put x = w 0 = a1 + 2 a 2 w + 3 a 3 w 2 +¼ + 2n a 2n w 2n -1 …(2)
2 2 4 4n -2
Put x = w 0 = a1 + 2 a 2 w + 3 a 3 w +¼ + 2n a 2n w …(3)
(1) + (2) + (3)
n × 3 n -1 = a1 + 4 a 4 + 7 a7 + 10 a10 +¼
n
16. C r + n C r -1 = n +1
Cr
3
C 0 + 3 C 1 + 4 C 2 + 5 C 3 +¼ + 99
C 97 = 100 C 97
17. Last digit of 9 ! = 0
Last digit of 3 9966 = 9
Hence last digit 9.
18. x = T7 = n C 6 (3 1/ 3 ) n -6 × ( 4 -1/ 3 ) 6
238 Solution of Advanced Problems in Mathematics for JEE

y = T n -5 = n C n -6 (3 1/ 3 ) 6 × ( 4 -1/ 3 ) n -6
y = 12 x
n
C n -6 (3 1/ 3 ) 6 ( 4 -1/ 3 ) n -6 = 12 × n C 6 (3 1/ 3 ) n -6 ( 4 -1/ 3 ) 6
Þ 12 = (12 1/ 3 ) 12-n Þ n = 9
r
æ2ö
20. t r +1 = 15 C r ( x 2 ) 15-r × ç ÷
è xø
Coeff. of x 15 = 15 C 5 × 2 5
Coeff. of x 0 = 15 C 10 × 2 10
21. (1 + x) 2 (1 + y) 3 (1 + z ) 4 (1 + w) 5
General term = 2 C a 3 C b 4 C d 5 C e x a + b + d + e
2 14 ´ 13
å C a ´ 3 C b ´ 4 C d ´ 5 C e = 14 C 12 or 14
C 12 = = 91
a + b + d + e =12 2
n n n n
1 2
22. å r × nC r + 2 å r + 1 × n C r ; n å n -1 C r -1 + n + 1 å n +1 C r +1
r =0 r =0 r =0 r =0
n -1 2
Þ n ×2 + × (2 n +1 - 1)
n+1

Exercise-2 : One or More than One Answer is/are Correct


20 20 20 20 20
1. N = C7 - C8 + C9 - C 10 +¼¼ - C 20
= ( 20 C 7 + 20
C9 + 20
C 11 +¼¼ + 20
C 19 ) - ( 20 C 8 + 20
C 10 +¼¼ + 20
C 20 )
20 20 20 20 20 20 20
=( C 20 + C2 + C4 + C 6) -( C1 + C3 + C 5)
2. For B and D put x = 1, - 1
For A differentiate with respect to x then put x = 0
1
For C replace x with
x
4
3. å( -1) r 16C r = 16C 0 - 16C 1 + 16C 2 - 16C 3 + 16C 4 = 1365
r =0
1 n 1 n
4. 2 ´ ´ C1 = 1 + ´ C 2 Þ n = 8, 1
2 22
r 16- 3r
æ 1ö
8 4
T r +1 = Crç ÷ x Þ r = 0, 4, 8
è2ø
5. LHS = (1 + 2 x 2 + x 4 )(1 + C 1 x + C 2 x 2 + C 3 x 3 +¼¼ )
Binomial Theorem 239

RHS = a 0 + a1 x + a 2 x 2 + a 3 x 3 +¼¼
Comparing the coefficients of x , x 2 , x 3 ,¼¼
Now, 2 a 2 = a1 + a 3
2( n C 2 + 2) = n C 1 + ( n C 3 + 2 n C 1 )
n(n - 1) n(n - 1)(n - 2)
2 + 4 = 3n +
2 6
or n 3 - 9n 2 + 26n - 24 = 0
\ (n - 2)(n 2 - 7n + 12) = 0 (Q8 + 52 = 36 + 24)
or (n - 2)(n - 3)(n - 4) = 0
\ n = 2, 3, 4
n n n æ n öæ n öæ n ö
6. å å å n C i × n C j × n C k = çç å n C i ÷÷ çç å n C j ÷÷ çç å n C k ÷÷ = 2 3n
i = 0 j= 0 k = 0 è i =0 øè j= 0 ø è k =0 ø
100 100 100 100 100 100
7. ( C6 + C7 ) + 3( C7 + C 8) + 3( C8 + C 9 ) + ( 100 C 9 + 100
C 10 )
= 101 C 7 + 3 101
C8 + 3 101
C9 + 101
C 10
( 101 C 7 + 101
C 8 ) + 2 ( 101 C 8 + 101
C 9 ) + ( 101 C 9 + 101
C 10 ) = 102 C 8 + 2 × 102 C 9 + 102
C 10
102 102 102 102 103 103
=( C8 + C 9) + ( C9 + C 10 ) = C9 + C 10 = 104 C 10
15
C 2r 1 2r + 1 1 6 r - 13 13 15
8. > Þ > Þ <0 Þ < r<
15 2 15 - 2 r 2 2 r - 15 6 2
C 2r +1
9. f( x) = 1 + x 111 + x 222 +¼ + x 999
if f( x) is divided by x + 1, then remainder f( -1) = 0
if f( x) is divided by x - 1, then remainder f(1) = 10
f( x) = (1 + x 222 + x 444 + x 666 + x 888 ) + x 111 (1 + x 222 + x 444 + x 666 + x 888 )
= (1 + x 111 )(1 + x 222 + x 444 + x 666 + x 888 )

Exercise-3 : Matching Type Problems


n
2. (B) P = å nCr =2n
r =0
m
Q= å m C r (15) r = (1 + 15) m = 16 m
r =0

Þ n = 4m
(C) 1 + 6 + 120 + 56 K
Reminder = 15
240 Solution of Advanced Problems in Mathematics for JEE

a 2 + b 2 + ab ( a - b)( a 2 + b 2 + ab) a 3 - b 3
3. (A) = =
a+b ( a - b)( a + b) a 2 - b2
4+ 3 8 + 15 12 + 35
1
+ + +¼ = (( 169 ) 3 - 13 ) = 1098
3 +1 5+ 3 7+ 5 2
8 8
(B) (2 cos 2 q - 3 sin q) = ( -2 sin 2 q - 3 sin q + 2)
5 5
3
Greatest value = 5 at sin q = - (Q 4 £ q £ 6)
4
(C) Let ( 3 + 1) 6 = I + f
and ( 3 - 1) 6 = f ¢ Þ ( 3 + 1) 6 + ( 3 - 1) 6 = 416 = I + 1
Þ I = 415 = 1 ´ 5 ´ 83
1 - x 256 1 - x n +1
(D) (1 + x)(1 + x 2 ) ¼(1 + x 128 ) = =
1- x 1- x
Þ n + 1 = 256

Exercise-4 : Subjective Type Problems

2. Coefficient of x 60 = -6 + 5 + 8 - 6 = 1
7. (1 + x) 3n = 3n
C0 + 3n
C1 x + 3n
C 2 x 2 +¼¼ + 3n
C 3n x 3n
Put x = 1 2 3n = 3n
C0 + 3n
C1 + 3n
C 2 +¼¼ + 3n
C 3n
2 3n 3n 3n 3n 2 3n
Put x = w (-w ) = C0 + C 1w + C 2 w +¼¼ + C 3n
Put x = w 2 ( - w) 3n = 3n
C0 + 3n
C 1w 2 + 3n
C 2 w 4 +¼¼ + 3n
C 3n
3n 2 3n 3n 3n 3n 3n
2 + (-w ) + ( - w) = 3[ C0 + C 3 +¼¼ + C 3n ]
5
10. å 20 C 2K -1 = 2 18 Þ 2 108 = 2 3 (2 5 ) 21 = 8 (33 - 1) 21
K =1
Remainder = -8 or 3
11. f(n) = n C 0 a n -1 - n C 1 a n -2 +¼¼
( a - 1) n
Þ f ( n) =
a
f(2007) + f(2008) = 3 7 K
3 9 + ( a - 1) 3 9
Þ = 37 K Þ K = 9
a
13. (360 + 1) 44 - 1 = 44
C 0 × (360) 44 + 44
C 1 × (360) 43 +¼ + 44
C 43 × (360) 1
= 360 [ 44 C 0 × (360) 43 + 44
C 1 × (360) 42 +¼ + 44
C 43 ]
Binomial Theorem 241

14. (3|x -2| + (3|x -2|-9 ) 1/ 5 ) 7


T 6 = 7 C 5 × (3|x -2| ) 2 × 3|x -2|-9 = 567
Þ 3 3|x -2|- 9 = 27 Þ | x - 2| = 4 Þ x = 6 , - 2
10 10
15. 1 + å 3 r × 10C r + å r × 10C r
r =1 r =1
10 10
1 + ((1 + 3) - C 0 ) + 10 × 2 9 = 4 10 + 5 × 2 10 = 2 10 ( 4 5 + 5)
a = 1, b = 5
if a , b lies between the roots of f( x) = 0
f(1) < 0 Ç f(5) < 0
-k 2 < 0 Ç 16 - k 2 < 0
16. S n = n C 0 n C 1 + n C 1 n C 2 +¼ + n C n -1 × n C n = 2n
C n -1
2n + 2
S n +1 = Cn
2n + 2
S n +1 Cn 15
= =
Sn 2n 4
C n -1
(2n + 2)(2n + 1) 15
Þ =
n(n + 2) 4
Þ n 2 - 6n + 8 = 0

❑❑❑
242 Solution of Advanced Problems in Mathematics for JEE

15
PROBABILITY

Exercise-1 : Single Choice Problems

2. f( x) = 3 x + 4 1 - x [where x = P( A)]
9
f( x) max. = 5 at x =
25
5
3. P( A È B ) = 1 - P ( A È B ) =
6
1 3
P( A Ç B ) = , P ( A) =
4 4
1
Þ P ( B ) = P ( A È B ) + P ( A Ç B ) - P ( A) =
3
n
æ 1ö 31
4. 1 - ç ÷ = Þ n =5
è2ø 32
3!´ 2 1
5. Required probability = =
9! 140
3 !3 !3 !
3 !(3n - 3)!
[(n - 1)!]3
6. Required probability =
(3n)!
(n !) 3
7. If product of two numbers equal to third number, then possibilities are (2 , 3 , 6), (2 , 4 , 8),
(2 , 5, 10).
3 1
Probability = =
10
C 3 40
3 2 2 1 4
8. P = ´ ´ ´ =
3 3 3 3 27
Probability 243

7!
9. Total word = n =
2 !2 !
T ITANIC I TTANIC A TTINIC
6! 6! 6! m 5
Favourable word = m = + + Þ P= =
2 ! 2 ! 2 !2 ! n 7
n! 3 6
10. Probability = = =
n 32 64
n
(n - 1)! 1 × 2 × 3
Þ = Þ n=4
n n -1 43
11. Total case = n = 9 ´ 10 3
Favourable case = m = (9 ´ 10 3 ) - 6 4
64 107
P =1- =
3 125
9 ´ 10
12. Total case = n = 6!
Favourable case = m = (3 ! ´ 2 !) + (2 ! ´ 2 !) = 16
16 1
Probability = =
6 ! 45
13. E 1 “No card is king from removed cards”
E 2 “1 card is king from removed cards”
E 3 “2 card is king from removed cards”
E 4 “3 card is king from removed cards”
E 5 “4 card is king from removed cards”
F = 3 cards are drawn from pack those are kings.
48 4 48
S
æ F ö C C C 25 × 4 C 1 3
C3
P( F ) = å P( E i ) × P ççè E i ÷÷ø = 52 C 26 × 26 C3 +
52
C 26
×
26
C3
+0+0+0
i =1 26 3
49
4 4´ C 26
= ( 48 C 26 + 48
C 25 ) =
52 26 52
C 26 × C3 C 26 × 26 C 3
1
=
(13)(17)(25)
3
C 2 × 10 C 4 1 15
14. ´ =
13 7 286
C6
15. Let f be function from {1, 2 , ¼ , 10} to itself total functions possible is 10 10 . The number of
one-one onto functions possible is 10!.
10 ! 9!
Hence, the probability of selected function to be one-one onto is = .
10
10 10 9
244 Solution of Advanced Problems in Mathematics for JEE

16. Let the friends come to the restaurant at 5 h x min and 5h y min, respectively, where
x , y Î [0 , 60].
Hence, the sample space consists of all points ( x , y) lying in 60 ´ 60 square as shown above
and for favourable cases,| x - y| £ 15, that is -15 £ x - y £ 15 which is shown by shaded region
in the graph shown below :
y
45
60

45

15

x
0 15 60

Hence, the probability that they will meet is given by :


1
2 ´ ´ 45 ´ 45 2
2 æ3ö 7
1- =1-ç ÷ =
60 ´ 60 4
è ø 16
91
17. Total ways = C3
Favourable ways = (Common ratio is 2) + (Common ratio is 3) = 16 + 2 = 18

Exercise-2 : One or More than One Answer is/are Correct


4
C 3 ´ 11 C 5 1 8
1. Probability = ´ =
15 7 195
C8
æ 1öæ 1öæ 1öæ 1ö æ 1 ö
2. Probability = ç 1 - ÷ ç 1 - ÷ ç 1 - ÷ ç 1 - ÷ ¼¼ ç 1 - ÷
è 2 øè 4 øè 6 øè 8ø è 2012 ø
1 3 5 7 2011 2012 !
= ´ ´ ´ ´¼¼´ =
2 4 6 8 2012 2 2012 (1006 !) 2
2 1
3. We have P( E i ) = = or i = 1, 2 , 3.
4 2
1
Also for i ¹ j , P( E i Ç E j ) = = P( E j ) P( E i ). Therefore, E i and E j are independent for i ¹ j .
4
1
Also, P( E 1 Ç E 2 Ç E 3 ) = ¹ P( E 1 ) P( E 2 ) P( E 3 )
4
\ E 1 , E 2 , E 3 are not independent.
3
4. Max. ( P( A Ç B )) = P( A) =
5
Probability 245

4
Min. ( P( A Ç B )) = P( A) + P( B ) - 1 =
15
19
P ( A È B ) = P ( A) + P ( B ) - P ( A Ç B ) = - P( A Ç B )
15
3
P ( A Ç B ) = P ( A) - P ( A Ç B ) =
- P( A Ç B )
5
æ A ö P( A Ç B ) P( B ) - P( A Ç B )
Pç ÷= =
èBø P( B ) P( B )

Exercise-3 : Comprehension Type Problems

Paragraph for Question Nos. 1 to 2


1 2 1 3 1 4 1 4 2
1. P( E 1 ) = ´1+ ´ + ´ + ´ = =
10 10 2 10 3 10 4 10 5
3 1
´
æ B 3 ö P( B 3 Ç E 2 ) 10 3 1
2. P çç ÷÷ = = =
E
è 2ø P ( E 2 ) 2 1 3 1 4 1 3
´ + ´ + ´
10 2 10 3 10 4

Paragraph for Question Nos. 3 to 5


3. Mr. A’ s 3 digit number is always greater than Mr. B’s 3 digit numbers then A should always pick
digit 9.
8
C 3 ´ 8C 2 1
Probability = =
8 9 3
C3 ´ C3
8
C3 ´1 1 1
4. Probability = = =
9 8 9 84
C3 ´ C3 C3
5. P( E ) = A picks 9 or A does not pick 9 and his number is greater than B
8
1 2 1æ C3 1 ö÷ 37
= + × ç1 - × =
3 3 2è ç 8
C3 8
C 3 ÷ø 56
Paragraph for Question Nos. 6 to 7
6. Let a n = number of ways of outcomes of n tosses when no 2 consecutive heads occur
a n = a n -2 + a n -1
Also, a1 = 2 (H or T)
a2 = 3 (TT or HT or TH)
\ a 3 = 5, a 4 = 8 ……
a10 = 144
246 Solution of Advanced Problems in Mathematics for JEE

144
\ Probability =
2 10
7. [HT HT HTH] T, T, T
4!
Number of ways of arranging = =4
3!
4
Probability =
2 10

Paragraph for Question Nos. 8 to 10


8. 6n > 2 n , n Î N
\ n = 1, 2 , 3 , 4
4 æ Number of solutions of x + y > 4 , 1 £ x , y £ 6 ö
9. ´ç ÷
6 è 36 ø
æ Number of solutions of x + y + z > 8 , 1 £ x , y , z £ 6 ö
´ç ÷
è 63 ø
4 30 160 100
= ´ ´ =
6 36 216 243
4 30 æ 160 ö 4 30 56 35
10. Probability = ´ ´ ç1 - ÷= ´ ´ =
6 36 è 216 ø 6 36 216 243

Paragraph for Question Nos. 11 to 12


11. Let p1 be the probability of being an answer correct from section 1. Then p1 = 1 5 . Let p 2 be
the probability of being an answer correct from section 2. Then p 2 = 1 15 .
1 1 1
Hence, the required probability is ´ =
5 15 75
12. Scoring 10 marks from four questions can be done in 3 + 3 + 3 + 1 = 10 ways so as to answer 3
questions from section 2 and 1 question from section 1 correctly.
10 10 3
C3 C1 1 æ 1 ö
Hence, the required probability is ç ÷ .
20 5 è 15 ø
C4

Exercise-5 : Subjective Type Problems

æ 2 1 2 1 1 2 ö æ 2 1 2 ö 2 416
1. ç + × + × × ÷ ç + × ÷ =
è 3 3 3 3 3 3 ø è 3 3 3 ø 3 729
6
C 5 + 7 C 4 + 8C 3 + 9C 2 + 10
C1 + 1 9
5. Probability = =
10 64
2
Probability 247

3
C1 3
6. p = =
7 7
C1
6!
7. Total ways = ´ 3 ! = 90
2 !2 !2 !3 !
Favourable cases = 90 - [3 ! + 3 C 1 ´ 3 C 1 ´ 2 ´ 2] = 48
48 8
Þ p= =
90 15
9. E 1 ® be the event of both getting the correct answer
E 2 ® both getting wrong answers.
E ® both obtaining same answer.
1 1 1 æ 1öæ 1 ö 77
P( E 1 ) = = , P( E 2 ) = ç 1 - ÷ ç 1 - ÷=
8 12 96 è 8 øè 12 ø 96
æ E
ö æ E ö 1
P çç ÷÷ = 1; P çç ÷÷ =
è E1
ø E
è 2ø 1001
1

æ E ö 96 13
Pç 1 ÷= =
è E ø 1× 1 1 77 14
+ ×
96 1001 96
10. Total ways = 9 C 7 ´ 7 !
Favourable ways Þ 9 C 7 ´ 7 ! - ( 7 C 3 ´ 3 !) ´ ( 6 C 4 ´ 4 !)
( 7 C 3 ´ 3 !) ´ ( 6 C 4 ´ 4 !) 15 7
P( E ) = 1 - =1- =
9 36 12
C7 ´ 7!
3
1 ì1 1 1 1 ü 1 ìïæ 1 1 1 ö üï 27
11. í ´ + 2 ´ ´ ý + ´ íç ´ ´ ÷ ý =
2 î2 2 4 4 þ 4 ïè 4 4 2 ø ï 128
î þ
1 1
12. 1st 2 nd ´ a
4 6
1 1
2 nd 1st ´ b
4 6
1 1
1st 1st ´ c
4 36
1 1
2 nd 2 nd ´ d
4 36
c+d 2
=
a+ b+ c+ d 5
❑❑❑
248 Solution of Advanced Problems in Mathematics for JEE

16
LOGARITHMS

Exercise-1 : Single Choice Problems

1. log 10 x = A x>0
log 10 ( x - 2) = B, x - 2 > 0 Þ x > 2
Þ A 2 - 3 AB + 2 B 2 < 0
Þ ( A - 2 B )( A - B ) < 0
Þ (log x - 2 log( x - 2))(log x - log( x - 2)) < 0
Case-I : log x - 2 log ( x - 2) < 0
and log x - log( x - 2) > 0 …(1)
Case-II : log x - 2 log ( x - 2) > 0
and log x - log( x - 2) < 0 …(2)
From (1) & (2), x Î ( 4 , ¥)
2. (log e x) 2 = (log e x) - (log e x) 2 + 1 (log e x > 0)
2
2(log e x) - log e x - 1 = 0
(2 log e x + 1)(log e x - 1) = 0
1
log e x = - (not possible)
2
log e x =1
log 3 7 log 3 7
3. S = (a ) + ( b log 7 11 ) log 7 11 + ( c log11 25 ) log11 25
log11 25
= 27 log 3 7 + 49 log 7 11 + 11 = 469
0
æ 1ö
4. a 2 - 3 a + 3 > ç x + ÷ and a 2 - 3 a + 3 > 0
è xø
a 2 - 3a + 2 > 0
( a - 1)( a - 2) > 0 Þ a Î ( -¥, 1) È (2 , ¥)
5
5. P = = log 120 x 5 ; (120) P = x 5 = 32 Þ x = 2
log x 120
Logarithms 249

z 1/ 3 z 1/ 6
6. x= , y=
2 5
z 1/ 3 z 1/ 6 1
If xy = z 3/ 2 ; × = z 3/ 2 Þ z =
2 5 10
7. log x (log 3 (log x y)) = 0 Þ y = x 3 , log y 27 = 1 Þ y = 27
-3 5
8. log 10-2 10 3 + log 10-1 10 -4 = +4=
2 2
3 3-a 4 log 3 2
9. a = Þ log 3 2 = ; log 6 16 =
1 + 2 log 3 2 2a 1 + log 3 2
10. log 2 (log 2 (log 3 x)) = 0 Þ x = 9
log 2 (log 3 (log 2 y)) = 0 Þ y = 8
x y 7 x y 2
11. Let log 3 a = x , log 3 b = y; + = and + =
3 2 2 2 3 3
12. a = log 2 5; b = log 5 8; c = log 8 11; d = log 11 14
2 abcd = 2 log 2 14 = 14
log 8 17 log 2 2 17
14. =
log 9 23 log 3 23
15. Case-I : 2 x - 3 > 1 Case-II : 0 < 2 x - 3 < 1
3x - 4 > 1 0 < 3x - 4 < 1
5 5 3 5
x> Þ x>2 x< Þ < x<
3 3 2 3
16. p £ log 10 N < p + 1 Þ P = 10 p + 1 - 10 p
- q £ log 10 1 N < - q + 1 Þ Q = 10 q - 10 q - 1
17. n + 1 = number of digits = 1 + characteristic
18. log 10 (0.15) 20 = 20 (log 10 15 - 2) = -16.478
19. log 2 (log 4 (log 10 10 16 )) = log 2 (log 4 16) = 1
20. 2 log x - log(2 x - 75) = 2
x2
= 100 Þ x 2 - 200 x + 7500 = 0
2 x - 75
log x a×log a y ×log y z
21. x = x log x z = z
22. x x x
= x 3x/2
3 9
x ¹ 0, 1 x x= x Þ x=
2 4
If x = 1, then it also satisfy.
250 Solution of Advanced Problems in Mathematics for JEE

23. (log 3 x) 2 = 2 log 3 x


Þ log 3 x = 0 or log 3 x = 2
x =1 or x =9
24. log 10 x + log 10 y = 2 Þ xy = 100 …(1)
x - y = 15 …(2)
Þ x = 20, y =5
1/ 2
æ x + 1 æç 2 x - 3 ö÷ ö 7
ç 3è xø÷ 3
25. ç 2 ÷ = 2
ç ÷
è ø
2 1 14
Þ x+ x- = Þ 5 x 2 - 14 x - 3 = 0
3 x 3
2 2
( 2 x - x 2 + 1) ( 2 x - x 2 + 1) 3 2x - x +1
52x - x +1
26. 25 +9 = 34 ×
3 5
2 2
Let 3 2 x - x +1
= a and 5 2 x - x =b +1

34
a 2 + b2 = ab
15
15a 2 - 34 ab + 15b 2 = 0 Þ (3 a - 5b)(5a - 3 b) = 0
a 5
Case-1 : if =
b 3
2 x - x 2+ 1
æ3ö 5
Þ ç ÷ =
è 5ø 3
Þ 2x - x 2 + 1 = -1 Þ x 2 - 2x - 2 = 0
Sum of two values of x = 2
a 3
Case-2 : if =
b 5
2 x - x 2+ 1
æ3ö 3
ç ÷ =
è 5ø 5
Þ 2 x - x 2 + 1 = 1 Þ x = 0 and 2
Sum of all values of x is 4.
27. a x = by = cz = dw
Þ b = a x/ y , c = a x/z , d = a x/w
æx x xö
çç + + ÷÷
y z wø x x x æ1 1 1ö
log a ( bcd) = log a aè = + + = x çç + + ÷÷
y z w è y z wø
Logarithms 251

4
28. x =
( 5 + 1)( 5 + 1)( 8 5 + 1)( 16 5 + 1)
4

Multiply and divide by (1 - 16 5) then


x = -1 + 16 5
( x + 1) 48 = 5 3 = 125
1
29. log x log 18 ( 2 + 8 ) =
3
1
log x log ( 3 2 )2 3 2 =
3
æ 1ö 1 1
log x ç ÷ = Þ x=
è2ø 3 8
1
30. f ( n) = log 2 n if log 8 n is integer
3
=0 otherwise
2011
å f(n) = log 8 2 3 + log 8 2 6 + log 8 2 9 = 1 + 2 + 3 = 6
n =1

32. log 0. 3 ( x - 1) < log ( 0. 3)2 ( x - 1) Þ ( x - 1) 2 > x - 1

Þ ( x - 1)( x - 2) > 0
Also, for log to be defined ( x - 1) > 0
x Î (2 , ¥)
2
33. 7 2x -5x -6
= ( 49) 3 log 2 2
=73
Þ 2 x 2 - 5x - 6 = 6
2 x 2 - 5 x - 12 = 0
Þ (2 x + 3)( x - 4) = 0
16
34. (log 2 x) 4 + 16(log 2 x) 2 log 2
x
Þ t 4 + 16t 2 ( 4 - t ) (where log 2 x = t )
Þ t 2 (t 2 + 64 - 16t )
Þ t 2 ( t - 8) 2
Since 1 £ x £ 256 Þ 0 £ t £ 8
Þ Maximum of (t - 8) 2 t 2 lies at t = 4.
Hence, maximum ( 4 - 8) 2 × 4 2 = 256
252 Solution of Advanced Problems in Mathematics for JEE

37. Q l>0
1 ± (1 - 4 log 16 l)
\ log 16 x =
2
Q The given equation will have exactly one solution, if
1
1 - 4 log 16 l = 0 or log 16 l = = 4 -1
4
-1
\ l = (16) 4 = (2 4 ) 1/ 4 = 2 , - 2 , 2i , - 2i , where i = -1
But l is real and positive.
\ l =2
Number of real values = 1
38. Let x be the rational number, then according to question,
x = 50 ´ log 10 x
By trial x = 100
39. Q x = log 5 (1000) = log 5 (5 3 ´ 8) = 3 + log 5 8
and y = log 7 (2058) = log 7 (7 3 ´ 6) = 3 + log 7 6

Þ x - y = log 5 8 - log 7 6 > 0 æQ log 5 8 > 1, log 7 6 < 1ö


ç \ log 8 - log 6 > 0 ÷
è 5 7 ø
\ x> y
æ 2 ö 4æ 2 ö æ 4 ö
40. 7 log ç ÷ + 5 log ç 5 ÷ + 3 log ç 3 ÷
ç5´3÷ ç 23 ´3 ÷ ç 24 ´5÷
è ø è ø è ø
= 7 {4 log 2 - log 5 - log 3} + 5 {2 log 5 - 3 log 2 - log 4} + 3 {4 log 3 - 4 log 2 - log 5}
= log 2
41. log 10 {tan 1° tan 2° tan 3°¼ tan 45°¼ tan 87° tan 88° tan 89° }
= log 10 {tan 1° tan 2° tan 3°¼ tan 45°¼ cot 3° cot 2° cot 1° }
= log 10 1 = 0
1 1 1
+ + æ 1 1 1ö æ7ö
42. log 7 log 7 7 2 4 8 = log 7 ç + + ÷ = log 7 ç ÷
è2 4 8ø è8ø
= 1 - log 7 8 = 1 - 3 log 7 2
log 23 2
43. ( 4) 3 + (9) log 2 2 = (10) log x 83
Þ ( 4) 1/ 2 + 9 2 = (10) log x 83
Þ (83) 1 = (83) log x 10
\ 1 = log x 10 Þ x = 10
æyö æzö æxö
log10 ç ÷ log10 ç ÷ log10 çç ÷÷
44. (10 log10 x
) è zø
(10 log10 y
) è xø
(10 log10 z
) èyø
Logarithms 253

45. Q log x 2 log 2 x 2 = log 4 x 2


\ x > 0 , 2 x > 0 and 4 x > 0 and x ¹ 1, 2 x ¹ 1, 4 x ¹ 1
1 1
Þ x > 0 and x ¹ 1, ,
2 4
1 1 1
Then, × =
log 2 x log 2 2 x log 2 4 x
Þ log 2 x × log 2 2 x = log 2 4 x
Þ log 2 x × (1 + log 2 x) = (2 + log 2 x)
Þ (log 2 x) 2 = 2
Þ log 2 x = ± 2
2
\ x =2±
2 2
\ x = {2 - ,2 }
46. Q 2 log 10 x - log x (0.01) = 2 log 10 x - log x (10 -2 )
= 2(log 10 x + log x 10) (Q x > 0 and x ¹ 1)
æ log e x log e 10 ö
= 2 çç + ÷÷ ³ 2 × 2 (Q AM ³ GM)
è log e 10 log e x ø
=4
47. Let log 2 x = a
a 2 - 2a + 1 Þ a = 1
if log 2 x = 1 Þ x = 2
48. log e ( e 2 x ln x ) = log e x 3
2 + (ln x) 2 = 3 ln x
Let ln x = a
a 2 - 3a + 2 = 0 Þ ( a - 2)( a - 1) = 0
Þ x1 = e 2 , x 2 = e
49. M = antilog 32 0.6 = (32) 0.6 = 2 3 = 8
N = 49 1 × 49 - log 7 2 + 5 - log 5 4
49 1 25
= + =
4 4 2
50. log 2 (log 2 (log 3 x)) = 0 Þ x = 9
log 3 (log 3 (log 2 y)) = 0 Þ y = 8
51. |log 1/ 2 10 + |log 4 625 - log 2 5| =|log 1/ 2 10 + log 2 5| = 1
254 Solution of Advanced Problems in Mathematics for JEE

æ 1 ö
ç ÷
log 5 2 è 2 a ø 1
52. log 3 2 = = =
log 5 3 1 2 ab - 1
b-
2a
55. ( x - 3) 2 = 9 Þ x = 6
éæ 16 ö 7 æ 25 ö 5 æ 81 ö 3 ù
57. log a êç ÷ × ç ÷ × ç ÷ ú = 8
êëè 15 ø è 24 ø è 80 ø úû
Þ log a 2 = 8 Þ a = 2 1/ 8
7 1 31
58. log 23 (2 7 ) - log 32 (3 -1/ 2 ) = + =
3 4 12
æ log 5 16 ö
2 - çç ÷÷ - log 3 2
æ 1 ö æ 1 ö è 2 log 5 9 ø æ 1 öæ 1 ö
59. çç ÷÷ × çç ÷÷ = ç ÷ çç ÷÷
è 27 ø è 27 ø è 27 ø è 27 ø
1
æ 1 ö - log 3 27 2 2
= ç ÷ ×2 =
è 27 ø 27
( x - 1)( x + 2) ( x - 1)( x + 2)
60. log 2 = log 2 4 Þ =4
3x - 1 3x - 1
Þ x 2 - 11x + 2 = 0
1
61. log 100 10 =
2
log 2 (log 4 2) = log 2 1 2 = -1
log 4 [log 2 (256) 2 ]2 = log 4 16 2 = 4
3
log 4 8 = log 22 2 3 =
2
62. l = log 5 (log 5 3) Þ 5 l = log 5 3
-l
3 k+ 5 = 3 k × 3 5-l = 3 k × 3 log 3 5 = 5 × 3 k
63. log 10 b 4 = 2 p × log 10 a 2
log 10 b
= log a b = p
log 10 a
64. 2 x = 3 y = 6 -z = k (let)
x = log 2 k , y = log 3 k , z = -log 6 k
1 1 1
+ + = log k 2 + log k 3 - log k 6 = 0
x y z
65. ( 2 - 1) 3 = 5 2 - 7
Logarithms 255

1 1
- log a b
1 3 3 2 17
66. 1 + log a b = Þ log a b = - Þ =
4 4 1 + log a b 6
68. Let log y x = t
5t 2 - 26t + 5 = 0 Þ (5t - 1)(t - 5) = 0
Either x = y 5 or y = x 5
1 1
69. 1 - = Þ (log 3 x - 1) 2 = log 3 x Þ (log 3 x) 2 - 3 log 3 x + 1 = 0
log 3 x log 3 x - 1
1 1
70. log 2 x + log 2 y + log 2 z = 2 Þ x y z =4
2 2
1 1
log 3 y + log 3 x + log 3 z = 2 Þ x × y × z = 9
2 2
1 1
log 4 z + log 4 x + log 4 y = 2 Þ x × y × z = 16 Þ xyz = 24
2 2
æ 1 ö log 71 / 49 - log1 / 55
71. ç ÷ ×2 +7
è 49 ø
1 1
´ +7
49 4
1 1
72. log 2 (3 - x) - log 2 + log 2 (5 - x) = + log 2 ( x + 7)
2 2
Þ log 2 (3 - x)(5 - x) = log 2 ( x + 7)
Þ x2 - 9x + 8 = 0 Þ x = 8
1
73. log 5 x = log x 5 Þ x = 5,
5
2
74. | x - 1|log 3 x - 2 log x 9
= ( x - 7) 7
either x = 2 or log 3 x 2 - 2 log x 9 = 7
(log 3 x - 4)(2 log 3 x + 1) = 0
x -1 x -1
75. 9 + 7 = 4 (3 + 1)
x
Let 3 =t
t2 æt ö
+ 7 = 4 ç + 1÷ Þ t 2 - 12t + 27 = 0
9 è 3 ø
(t - 3)(t - 9) = 0
76. If a > 1
log a 10 > log a 3 > log a e > log a 2
Þ log 10 a < log 3 a < log e a < log 2 a
256 Solution of Advanced Problems in Mathematics for JEE

4 4
r
78. å log 4 2 r = å2 =5
r =1 r =1

79. log 3 2 + log 3 5 = log 3 10


log 3 9 < log 3 10 < log 3 27
80. sin 3q = 3 sin q - 4 sin 3 q
3
kæ 3 1 ö 3æ 1ö 4 æ 1ö
ça + 3 ÷ = ça + ÷ - ça + ÷
2è a ø 2 è a ø 8 è a ø
-1 æ 3 1 ö
= ça + 3 ÷
2 è a ø
Þ k = -1
81. 2 x - 3 > 0 Ç x 2 - 5 x - 6 > 0 Ç 2 x - 3 ¹ 1
3
x > Ç ( x - 6)( x + 1) > 0 Ç x ¹ 2
2
Þ (6 , ¥)

Exercise-2 : One or More than One Answer is/are Correct

1. 6 (log x) 2 + log x - 1 = 0
(3 log x - 1)(2 log x + 1) = 0
x = 10 1/ 3 or x = 10 -1/ 2
3. 3(log 10 2) x 2 - (1 - log 10 2) x = 2 log 10 2 - x
log 10 2(3 x 2 + x - 2) = 0
log 10 2( x + 1)(3 x - 2) = 0
2
Roots of this eq. are x = -1,
3
Sum of coeff. = 2 log 10 2 (irrational)
Discriminant = b 2 - 4 ac = 25 (log 10 2) 2 (irrational)
4. A = min.( x 2 - 2 x + 7) " x Î R Þ A =6
2
B = min.( x - 2 x + 7) " x Î [2 , ¥) Þ B =7
Logarithms 257

Exercise-3 : Comprehension Type Problems

Paragraph for Question Nos. 1 to 3


1. If a 1 = 4 , then 3 4 £ N < 3 5
If a 2 = 2 , then 5 2 £ N < 5 3
Þ 81 £ N < 125
3. If a 1 = 5, then 3 5 £ N < 3 6
If a 2 = 3 , then 5 3 £ N < 5 4
If a 3 = 2 , then 7 2 £ N < 7 3
Þ 243 £ N < 343
Paragraph for Question Nos. 4 to 5
2 2
Sol. | x - y | = 221

Paragraph for Question Nos. 6 to 7


Sol. (1 + 4 log p 2 (2 p)) 2 + (1 + log 2 p) 2 = (1 + log 2 4 p) 2

Let log 2 p = t
2
æ æ 1 + t öö 2 2
ç1 + 2ç ÷ ÷ + (1 + t ) = (3 + t ) Þ t = log 2 p = 2
è è t øø

Exercise-4 : Matching Type Problems

1. (A) a = 3 (( 7 + 1) - ( 7 - 1)) = 6
b = 1296 = 36
(B) a = ( 3 + 1) - ( 3 - 1) = 2
b = (3 + 2 ) - (3 - 2 ) = 2 2
(C) a = ( 2 + 1), b = ( 2 - 1)
(D) a =2 + 3, b=2 - 3

Exercise-5 : Subjective Type Problems

1. N = 6 log10 40 × 6 2 log10 5 = 6 log10 1000 = 6 3 = 216


2. log b ( a log 2 b ) = log a ( b log 2 b )
258 Solution of Advanced Problems in Mathematics for JEE

1
Þ log b a = log a b Þ a = b or a = (not possible)
b
log a ( c - ( b - a) 2 ) = 3 Þ c = a3
Þ Minimum value of c = 8 at a = 2
3. log b 729 = 6 log b 3
if this is an integer, then b = 3 , 3 2 , 3 3 , 3 6
5 3
4. Case-1 : If x+ >1 Þ x>-
2 2
æ8 ö
then ( x - 5) 2 < (2 x - 3) 2 Þ 3 x 2 - 2 x - 16 > 0 Þ x Î ç , ¥ ÷
è3 ø
5 5 3
Case-2 : If 0< x + <1 Þ - < x<-
2 2 2
æ 3ö
then ( x - 5) 2 > (2 x - 3) 2 Þ x Î ç - 2 , - ÷
è 2ø
there is no negative integral value of x.
æ 1ö
ç log100 x + ÷
6 (log a x )(log10 a )(log a 5)
5. a - 3 (log10 x -1) = 9 è 2ø
5
6 × 5(log10 x -1) - 3 (log10 x -1) = 3 (log10 x +1)
3 log10 x
6 × 5(log10 x -1) = + 3 × 3 log10 x
3
10 log10 x
6 × 5(log10 x -1) = ×3
3
log10 x - 2
æ 5ö
ç ÷ =1
è3ø
Þ log 10 x - 2 = 0
Þ x = 100
Integer part of log 3 100 is 4.
æ a + b ö log 5 a + log 5 b
6. log 5 ç ÷=
è 3 ø 2
2
æ a + bö
Þ log 5 ç ÷ = log 5 ( ab)
è 3 ø
Þ ( a + b) 2 = 9 ab Þ a 2 - 7 ab + b 2 = 0
a 4 + b 4 + 2 a 2 b 2 = 49 a 2 b 2
a 4 + b4
Þ = 47
a 2 b2
Logarithms 259

8. log 10 1 + x + 3 log 10 1 - x = 2 + log 10 1 - x + log 10 1 + x


Þ log 10 1 - x = 1
1 - x = 10 Þ x = -99 (not possible)
9. x 2 = 1 + 6 log 4 y
y 2 - 2 x y - 2 2x +1 = 0
Þ y = 2 x +1 and y = -2 x
if y = -2 x (not possible, because y > 0)
if y = 2 x +1
Þ log 2 y = x + 1
x 2 = 1 + 3 log 2 y
Þ x 2 = 1 + 3 ( x + 1)
x2 - 3x - 4 = 0
Þ ( x - 4)( x + 1) = 0
x1 = 4
Þ y 1 = 2 5 = 32
x2 = -1
Þ y 2 = 2° = 1
log 2| x 1 x 2 y 1 y 2| = log 2 128 = 7

10. log 7 log 7 7 7 7 = log 7 log 7 (7 7 / 8 ) = log 7 (7 8) = 1 - 3 log 7 2


Þ a =3

log 15 log 15 15 15 15 15 = log 15 log 15 (1515/16 )


æ 15 ö
= log 15 ç ÷ = 1 - 4 log 15 2
è 16 ø
Þ b=4
Then a + b=7
æ 1 ö
11. log 1+ x (1 - y) + log 1- y (1 + x) = 2 ç t + = 2 Þ t = 1÷
è t ø
1+ x =1- y
x = -y
\ log 1- y (1 + 2 y) + log 1- y (1 - 2 y) = 2
260 Solution of Advanced Problems in Mathematics for JEE

log 1- y (1 - 4 y 2 ) = 2

1- 4y2 =1+ y2 -2y


5y2 - 2 y = 0
2
y = 0, y =
5
But y = 0 rejected.
12. log b n = 2
log n (2 b) = log n 2 + log n b = 2
1
log n 2 + = 2
2
3
log n 2 = Þ n = 2 2/ 3
2
if log b n = 2 Þ b = n 1/ 2 = 2 1/ 3
n × b = 2 2/ 3 × 2 1/ 3 = 2
1
13. log y x + =2
log y x

Þ log y x = 1 Þ x=y
2
x + y = 12
Þ x 2 + x - 12 = 0
Þ ( x + 4)( x - 3) = 0
Þ x = - 4 or x = 3
but x > 0, then x = 3
xy = 9
14. yx = xy
if x =2y then y 2 y = (2 y) y
Þ 2 y log y = y log(2 y)
if y ¹0 then log y 2 = log(2 y)
Þ y2 =2y Þ y =2
x 2 + y 2 = 5 y 2 = 20
15. (log 2 4 + log 2 ( 4 x + 1)) log 2 ( 4 x + 1) = 3
Let log 2 ( 4 x + 1) = t
Logarithms 261

t 2 + 2t - 3 = 0 Þ t = -3 or 1
log 2 ( 4 x + 1) = 1 Þ 4 x = 1 Þ x = 0
17. x 2 + 4 x + 3 = 0 ( x > 0)
(log 3 5
x)
18. log 31 / 4 (log 3 5
x) = 4 Þ log 3 =1

Þ log 5 x = 1 Þ x =5

❑❑❑
262 Solution of Advanced Problems in Mathematics for JEE

17
STRAIGHT LINES

Exercise-1 : Single Choice Problems

6l - 3 1
1. Let ratio be l : 1 Þ =0, l =
l+1 2
3.
y
C(2 +2p, p+2)
)
(0,1
–2,2
3 3
x
O (p, 0)
p p
B 2 ,– 3

if ( a , sin a) lie inside the triangle, then a Î(0 , p)


711 9 ´ 79
4. x= =
13 + 11m 13 + 11m
if x is an integer, then m = 6
2
æ yö æ yö
6. 7 ç ÷ + 2c ç ÷ - 1 = 0
x
è ø è xø
m 1 + m 2 = 4m 1 m 2 Þ c =2
10.
Y

(0,8)

(0,3)
(8,0)
X
(–4,0) O
Straight Lines 263

1 2
13. a = 72 (a, 2a)
2
a = ± 12
(a, a) (2a, a)
Centroid º (16 , 16) or ( -16 , - 16)
14. g( x) = ax + b
g(1) = 2
Þ a + b=2
g(3) = 0
Þ 2a = - 2
a = -1
b=3
g( x) = - x + 3
cot [cos (|sin x| + |cos x|) - sin -1 (|sin x|) + |cos x|]
-1

|sin x| + |cos x|Î [1, 2 ]


Þ cot [cos -1 1 - sin -1 1] = 0 = g(3)
15. Points A and B are mirror images about y = x.
Point P will lie on the ^ bisector of line joining A and B Þ P lie on y = x.
m1
16. 4m 3 - 3 am 2 - 8 a 2 m + 8 = 0 m2
m3

m 1 m 2 m 3 = -2
Þ m3 =2 (Qm 1 m 2 = - 1)

2x
–5
7x+y=8 y=
17. 6

(1,1)
x–7y+6=0

æ y - mx ö
18. 2 x 2 + 3 y 2 - 5 x ç ÷ =0
è C ø
Coefficient of x 2 + coefficient of y 2 = 0
5m y=mx+C
5+ =0 Þ m + C =0
C
Then the equation of family of line is y = m( x - 1)
264 Solution of Advanced Problems in Mathematics for JEE

20. Equation of line BC is y = 7 x - 10 D


C
Equation of line CD is y = x + 2 y = 7x
(2, 4)
(1,2)
(2 - 0)(10 - 0) 10
Area of rhombus = = A
(7 - 1) 3 B
(0, 0) y=x

A(3,3)

C(–6,0) y=x
21.
3 y = –x
B
3 (–3,–3)
3
22. y = ( x - 9) + 6
4
23. Acute angle bisector is
7x - y æ x - yö 7x – y = 0
= - çç ÷÷
50 è 2 ø
x–y=0
Þ y = 2x
3 x + 4 y - 12 3 x + 4 y - 12
24. Either x = y or x = or y = Þ (1, 1)
5 5
æ 1 10 ö y
25. Co-ordinate of point A ç - , ÷
è 7 7 ø B 2)
(0,
1 1 1 1 A (0,3/2)
Ar ( DABC ) = ´ ´ = C
2 2 7 28
x
O
26.
A(1,2)

B G(4,1) C
(x1,y1) (4,b)
x+y=5 x=4
x +4+1
Co-ordinate of centroid G( 4 , 1) Þ 1 =4
3
Þ x 1 = 7 and y 1 = - 2
Straight Lines 265

y= 1|
27. |x+ |x–
1 | y=

–1 O 1 2

The image of y =| x - 1| w.r.t. y-axis is y =| x + 1| Þ y = ± ( x + 1)


Required solution = ( y - ( x + 1))( y + ( x + 1)) = 0
Q(4,5)
P(1,4)
R(m,m)
28.
P¢(4,1)
X
O

Image of (1, 4) about the line y = x is ( 4 , 1) Þ P ¢( 4 , 1) Q ( 4 , 5) and R(m , m ) are collinear.


Þ m =4
AD AB 10 2
29. = = =
CD BC 5 1
B(5, 1)

A(–1, –7) D C(1, 4)


2
æ yö æ yö 3
30. 4 c ç ÷ - ç ÷ + 6 = 0 has one root is - Þ c = -3
x
è ø x
è ø 4
x y( a + c) 1
33. + + =0
a 2 ac c
Þ a( y + 2) + c(2 x + y) = 0
Passes through a fixed point (1, - 2)
2 m
1æ y ö 2æ yö 1
34. ç ÷ + ç ÷ + =0
bè x ø hè x ø a 2m
2b b ab 9
Þ 3m = - and 2m 2 = Þ =
h a h2 8
266 Solution of Advanced Problems in Mathematics for JEE

x y
35. Equation of line is + =1 y
2h 2k
(0,2k)
if it passes through fixed point ( x 1 , y 1 )
P(h,k)
x1 y
+ 1 =1
2h 2k x
0 (2h,0)
4
36. OA : OB = l : 1 Þ l =
3
B(9/5, 9/10) 4x + 2y – 9 = 0
1
O (0, 0)
l 2x + y + 6 = 0
A(–12/5, –6/5)
x = 2y

æ 7ö
37. G ç 1, ÷
è 3ø
A(1,1)

(–1,2) (3,2)

(–3,3)B C(5,3)

38. Diagonals are perpendicular.


39. Let point on the line x + y = 4 is ( a , 4 - a).
4( a) + 3 ( 4 - a) - 10 a1
= 1 Þ a 2 + 4 a - 21 = 0
5 a2
Þ a1 + a 2 = - 4 Þ b1 + b2 = 12
40. Equation of altitude on BC A(–3,4)

x + 4 y = 13
x+y=1 2x+3y=6
Equation of altitude on AB
7 x - 7 y + 19 = 0 B C(–3/7, 16/7)
4x–y+4= 0
æ 3 22 ö
Þ Hç , ÷
è7 7 ø
41. Equation of line is (3 x + 4 y + 5) + l ( 4 x + 6 y - 6) = 0
- ( 3 + 4l ) 7 1
Þ ´ = -1 Þ l=
4 + 6l 5 2
5 -1 7 -5
42. = Þ x = 11
8 -2 x -8
Straight Lines 267

S
R(7,5)

43. (3,2)
P Q(8,0)
(–1,–1)

Þ S( -2 , 4)
1 a a 1
44. Area = a + 1 a + 1 1 = 1
2 a+2 a 1
45. ( x - y) 2 = 1
Þ x - y = 1 and x - y + 1 = 0
46. AB subtend an acute angle at point C , then
a 2 + ( a + 1) 2 > 4
æ - 7 - 1 ö÷ æç 7 - 1 ö÷
a Î çç -¥, ÷ È ç 2 , ¥÷
Þ
è 2 ø è ø
48. h = cos q y

k = 2 sin q A 1
2 q P(h, k)
k
h2 + =1 2
4
Þ 4x2 + y2 = 4 q
x
O(0,0) B
50. Let the point of reflection is ( h, k).
h - a k - 0 -2( a + at 2 )
= = Þ x = -a
1 -t 1+ t2
51.
y R(6, 7)
(1, 3)
P

x
O

R¢(6, –7)

52. Let ( x , y) and ( X , Y ) be the old and the new coordinates, respectively. Since the axes are
rotated in the anticlockwise direction, q = + 60° . Therefore,
é x ù = écos 60° - sin 60° ù é X ù
ëê y ûú ëê sin 60° cos 60° ûú ëê Y ûú

Þ é x ù = é 1 2 - 3 2ù é X ù
êë y úû ê 3 2 ú
1 2 û êë Y úû
ë
268 Solution of Advanced Problems in Mathematics for JEE

éX 3 ù
ê - Yú
Þ é xù = ê 2 2 ú
êë y úû ê 3 Y
X+ ú
êë 2 2 úû
X 3 3 Y
Þ x= - Y and y = X+
2 2 2 2
2 2
æX ö æ ö
Þ ç - 3 Y ÷ - ç 3 X + Y ÷ = a2
ç2 2 ø ÷ ç 2 ÷ø
è è 2
Þ ( X 2 + 3Y 2 - 2 3 XY ) - (3 X 2 + Y 2 + 2 3 XY ) = 4 a 2
Þ -2 X 2 + 2Y 2 - 4 3 XY = 4 a 2
Þ Y 2 - X 2 - 2 3 XY = 2 a 2
which is the required equation.
53. The following figure depicts the condition. By observation from the figure, DABC is clearly an
obtuse angled and isosceles triangle.
y

4/3
C B

x
O 4/3 4

Alternate solution : The following figure depicts the condition.


y

C B(1, 1)
(–2, 2)
x
O

A x+3y–4=0
(2, –2)

y = –x
3x+y–4=0
Straight Lines 269

From the figure, we get


A : 3 x + y = 4 and y = - x Þ x = 2 ; y = -2
B : (1, 1) by solving the equations.
C : x + 3 y - 4 = 0 and y = - x Þ x = -2 ; y = 2
Thus, AB = BC = 1 + 9 = 10
AC = 4 2 + 4 2 = 4 2
10 + 10 - 16(2)
cos B = <0
2( 10 )( 10 )
Therefore, the given triangle is isosceles and obtuse angled triangle.
y 2 - y1 y 3 - y 2
56. = Þ Points are collinear.
x 2 - x1 x3 - x2
57. 3 h = a cos t + b sin t + 1
3k = a sin t - b cos t
Þ (3 h - 1) 2 + (3 k) 2 = ( a cos t + b sin t ) 2 + ( a sin t - b cos t ) 2 = a 2 + b 2
x y
58. Equation of line + = 1.
a -1 - a
Lines passes from (4, 3).
62. The given triangle is equilateral. Therefore, the orthocentre of the triangle is same as centroid
of the triangle. Thus, the orthocentre, that is, the centroid is given by
æ 5 + 0 + ( 5 2) 0 + 0 + ( 5 3 2) ö æ 5 5 ö
ç , ÷ ºç , ÷
ç 3 3 ÷ çè 2 2 3 ÷ø
è ø
63. Using homogenization,
æ y - mx ö æ y - mx ö P
3x2 - y2 - 2x ç ÷ + 4yç ÷ =0
è C ø è C ø
(0, 0) O 90º
Coefficient of x 2 + Coefficient of y 2 = 0 y = mx + C

æ 2m ö æ 4ö
ç3 + ÷ + ç -1 + ÷ = 0 Q
è C ø è Cø
C = -m - 2
64.

R(3, 3 3)

60º 60º
P(–1, 0) O(0, 0)

3x + y = 0
270 Solution of Advanced Problems in Mathematics for JEE

Exercise-2 : One or More than One Answer is/are Correct

x y
1. Let line be + =1
a b
a + b = 9 and ab = 20
Þ a = 5, b = 4 or a = 4, b = 5
æ 4ö
2. Centroid is ç 4 , ÷.
è 3ø
2 3 -5
3. t2 t -6 = 0 Þ t 2 + t - 6 = 0
3 -2 -1
2
æ yö æ yö
4. bç ÷ + 6 ç ÷ + a = 0
x
è ø è xø
bm 2 + 6m + a = 0
if m = 1 is root of the equation
Þ a + b = -6
if m = -1 is root of the equation
Þ a + b=6
6. Co-ordinate of other two points (0, 2 3)
(1 ± 2 cos q , 3 ± 2 sin q) 2

æ æ ö ö (1, 3)
ç 1 ± 2 ç 3 ÷ , 3 ± 2 æç 1 ö÷ ÷
ç ç 2 ÷ è 2 ø ÷ø 2
è è ø
x
(1 + 3 , 3 + 1) and (1 - 3 , 3 - 1) O (2,0)

8. Image of A(3 , - 1) about angle bisector x - 4 y + 10 = 0 is A ¢( a , b).


a - 3 b + 1 - 2(3 + 4 + 10)
= = A(3,–1)
1 -4 17 6x+10y=59
Þ A ¢(1, 7) x–4y+10=0

æ x + 10 ö
Let point B ç x 1 , 1 ÷ on the line x - 4 y + 10 = 0
è 4 ø B C

If mid-point of AB lie on the line 6 x + 10 y = 59
æ x +3ö æ x + 10 - 4 ö
6ç 1 ÷ + 10 ç 1 ÷ = 59
è 2 ø è 8 ø
Þ B(10 , 5)
Straight Lines 271

Exercise-3 : Comprehension Type Problems

Paragraph for Question Nos. 3 to 4


3. x + y = 2 and x - 3 y = 6 A
Meet at (3 , - 1)
4. Image of A(2 , - 4) about x + y = 2 lie on BC.
x2 - 2 y2 + 4 æ -4 ö
= = -2 ç ÷ I
1 1 è 2 ø
B C
Þ x 2 = 6, y 2 = 0
Image of A(2 , - 4) about x - 3 y = 6 lie on BC.
x3 - 2 y3 + 4 8
= = -2
1 -3 10
2 4
Þ x3 = , y3 =
5 5
Equ. of line BC , x + 7 y = 6
æ4 2ö
Þ B ç , ÷ and C(6 , 0)
è3 3ø

Exercise-4 : Matching Type Problems


n +1
2. (A) å( 1 C r -1 + 2C r -1 + 3C r -1 +¼ + n C r -1 )
r =1
n +1 n +1 n +1 n +1
1 2 3
= å C r -1 + å C r -1 + å C r -1 +¼ + å n C r -1
r =1 r =1 r =1 r =1
1 2 3 n n
= 2 + 2 + 2 +¼ + 2 = 2(2 - 1)
(B) Family of line ( x + y + 2) + l(2 x - y + 4) = 0 always passes from ( -2 , 0).
If almost one tangent can be drawn from ( -2 , 0) then
S 1 = 4 - 8 g - 36 + 4 g 2 £ 0
g2 - 2 g - 8 £ 0
(C) 2 sin 7 x × cos 2 x = cos 2 x
1
Þ cos 2 x = 0 or sin 7 x =
2
p 3p p 5p 13 p 17 p 25p 29 p 37 p 41p
x= , 7x = , , , , , , ,
4 4 6 6 6 6 6 6 6 6
(D) a + b = tan 65° tan 70° - tan 65° - tan 70°
272 Solution of Advanced Problems in Mathematics for JEE

tan 65° + tan 70°


tan 135° = = -1
1 - tan 65° + tan 70°
Þ tan 65° tan 70° - tan 65° - tan 70° = 1
3. (A) cos 40° - 2 cos 40° sin 10° = cos 40° - (sin 50° - sin 30° )
1 1 1
(B) 3 2l 4 = 0 Þ (3 - 2l)(1 + 3l) = 0
1 1 -3l
k 2 - 2k 1
(C) -k + 1 2k 1 =0 Þ 2k 2 + k - 1 = 0
-4 - k 6 - 2 k 1
1
Þ k = -1,
2
¥ k 3 4 5
æ 1ö æ 1ö æ 1ö æ 1ö 1
(D) å ç ÷ = ç ÷ + ç ÷ + ç ÷ +¼¼ =
k =3 è 2 ø è2ø è2ø è2ø 4

Exercise-5 : Subjective Type Problems

1. D = 132
2. ax + by + c = 3 x - 4 y + c
Þ a = 3 , b = -4
Distance of 3 x - 4 y + c from A(3 , 1) is 1.
|9 - 4 + c|
Þ =1
5
|c + 5| = 5
Also, 3 x - 4 y + c = 0 and 3 x - 4 y + 5 = 0 lie on same side of A
Þ c + 5> 0
Þ c + 5 =5 Þ c =0
3. xy( x + y - 2) = 0 y

a + a4 -2 £ 0 ( a > 0) (0, 2)

Þ a =1 x
O (2, 0)
x+y=2

R
5. PQ = 3 2
)
2,3
QR £ PQ P( (–1,6)
Q
6. x 2 ( y 2 - x 2 ) = 0
has 3 different lines x = 0, y = x and y = - x.
Straight Lines 273

10
8. 2 < a <
3
y

(–6,6) (–2,10/3)
y=x
(0,2)
(–2,2)
x
(–2,0) (3,0)
O

y = –x

9. Describe a circle whose diameter is AB.


\ centre = (1, 0)
Radius = 2
Let ‘ m ’ the slope of the line passing through ( 4 , 1). (4,1)
( y - 1) = m ( x - 4) intersect the circle A
(–1,0) B(3,0)
^ distance from centre < radius of circle.
-3m + 1
<2
m2 + 1
9m 2 - 6m + 1 < 4m 2 + 4
æ 6 - 96 6 + 96 ö ì 1 ü
Þ m Îç , ÷ - í , 1ý
ç 10 10 ÷ î5 þ
è ø
12 6
l1 + l 2 = =
10 5
5 (l 1 + l 2 ) = 6
2 2 3
10. = Þ b=
b 3 2

– 2, 0 2, 0
b b
60° 60°

(0,–2)

❑❑❑
274 Solution of Advanced Problems in Mathematics for JEE

18
CIRCLE

Exercise-1 : Single Choice Problems

3
1. CP = = ( h - 1) 2 + ( k - 1) 2 y
2
Locus of point P( h, k) is P(h,
1 k)
3
( x - 1) 2 + ( y - 1) 2 = C
4 (1,1)

x
0
2. d 2 - ( r1 - r2 ) 2 = 15 ; d 2 - ( r1 + r2 ) 2 = 5 Þ r1 r2 = 50
T
4. PT = 16 + 16 - 8 - 8 - 7 = 3 3 3
Þ TT ¢ = 2 BT = 2 × 3 cos 45° = 3 2 P O
(4,4) B(1,1)


5. It will be circle with diametric ends as (1, 1) and ( 4 , 2) i . e. , point of
intersection.

1
6. (–4,4) (0,6)

19

8. Let centroid be ( h, k).


cos a + sin a + 1 sin a - cos a + 2
Þ h= , k=
3 3
Þ 3 h - 1 = cos a + sin a , 3 k - 2 = sin a - cos a
Þ (3 h - 1) 2 + (3 k - 2) 2 = 2
2 2
æ 1ö æ 2ö 2
Þ çx - ÷ +çy - ÷ =
è 3ø è 3ø 9
Circle 275

9. (1,1) Q(–3,0)

Length of tangent = PQ = 4 2 + 12 - 5 = 12
10.
D(12,17)

a
a cosq
q
A
90

C
°–

a
q

a sinq a
a cosq
q 90°– q
O a cosq B L a sinq M

OA = a sin q = 12 ,
DL = a sin q + a cos q = 17
a cos q = 5
C = ( a cos q + a sin q , a cos q) = (17 , 5)
12. Centroid divide the line joining orthocentre and circumcentre in 2 : 1.
y
B(0,1)

2 G 1 A(1,0)
O(0,0) x
H 1+cosq , 1+sinq O
(h,k) 3 3 (cosq, sinq)C

Þ h = 1 + cos q , k = 1 + sin q
2 2
( x - 1) + ( y - 1) = 1
13. Co-ordinate of centre is C(1, 1).
( x - 1) 2 + ( y - 1) 2 = 1
(1,1)
x2 + y2 - 2x - 2 y + 1 = 0
x+y=2

(1,2) (3/2,3/2) (2,1)

y=x
276 Solution of Advanced Problems in Mathematics for JEE

p
14. a = 2 R cos A
6
Þ a = 4 3 cm
3 2
Area of DABC = a = 12 3 cm 2
4 p/6 p/6
B C

15. Image of centre C 1 (5, 0) about the line y = x + 3 is


C1 x2 + y2 – 10x = 0
x 2 - 5 y 2 - 0 - 2 ( 5 + 3)
= = (5,0)
1 -1 12 + 12
Þ C 2 ( -3 , 8)
y = x+3
Equation of reflected circle is (x2, y2)
( x + 3) 2 + ( y - 8) 2 = 25
x=4

19. C
x=0

(0,0)

20. Let the equation of line is 3 x + 4 y = C


C
= 3 Þ C = 15 (in first quadrant)
5
21. C 1 (5, 0), C 2 (3 , - 1), C 3 (3 2 , 2) do not lie on straight line.
22. Let equation of diameter is 3 x + 5 y = C
Þ C =7
23. Equation of circle is
( x + 1)( x - 2) + ( y - 2)( y - 3) + l ( x - 3 y + 7) = 0
If its radius is 5.
Þ l = ±1
25. Equation of circle is ( x - 1)( x - 3) + ( y - 4)( y - 2) = 0 y
(1,4) B(3,4)
A
(2,3)
(3,2)
C
x
O
26. Equation of tangent at O(0 , 0).
x(0) + y(0) + g( x + 0) + f( y + 0) = 0
Þ gx + fy = 0
Circle 277

27. Equation of normal at O(0 , 0) (0,0) y=x

y = -x O

æ æ 1 ö æ 1 öö
Centre çç 0 ± çç - ÷÷ , 0 ± çç ÷÷ ÷÷
è è 2ø è 2 øø
æ 1 1 ö æ 1 1 ö
Þ Either çç - , ÷÷ or çç ,- ÷÷
è 2 2ø è 2 2ø
28. Here, C 1 C 2 = r1 + r2 (Condition for external touch)
30. The triangle is right angled and the radical centre will be the orthocentre of the triangle.
32. Equation of common chord is 6 x + 14 y + (l + m ) = 0
If it passes through (1, - 4). Then, l + m = 50
33. x2 + y2 - 6x + 8 y = 0
Distance of line from centre
9 - 16 - 25 32
=
5 5 (3,–4)
32 7
Shortest distance = -5 =
5 5
p p
34. Ð AOB = Þ Ð ACB =
2 4 y
B

O C(1,0)
x
A

y=7x+5

37. Equation of required circle :


S : ( x - 1) 2 + ( y - 1) 2 + l( x - y) = 0
S¢: x 2 + y 2 + 2 y - 3 = 0
Common chord of S = 0 and S ¢ = 0 is S - S ¢ = 0
( l - 2) x - ( l + 4) y + 5 = 0
Centre of S ¢ : (0 , - 1) lies on common chord Þ l = -9
S : ( x - 1) 2 + ( y - 1) 2 - 9 ( x - y) = 0
9
Þ r=
2
40. Point lie inside the circle k + ( k + 2) 2 < 4 Þ 2 k 2 + 4 k < 0; -2 < k < 0
2

41. The length of the normal is


278 Solution of Advanced Problems in Mathematics for JEE

2
æ dy ö
y 1+ ç ÷
è dx ø

The length of radius vector of a point ( x , y) on the curve is | xi + yi|, that is x 2 + y 2 , it is


given that
x 2 + y 2 =| y| 1 + ( y ¢) 2
Squaring on both sides of this equation, we get
x 2 + y 2 = y 2 [1 + ( y ¢) 2 ]
2
æ dy ö
Þ x 2 + y 2 = y 2 + y 2ç ÷
è dx ø
2
2 æ dy ö
Þ x =ç y ÷
è dx ø
dy dy
Þ y = x or y = = -x
dx dx
dy
Now, y =x
dx
Þ y dy = x dx
Integrating on both sides, we get
y2 x2
= +c
2 2
Þ x 2 - y 2 = 2 c or x 2 - y 2 = constant
This answer does not exist in the given options. So, consider the other alternative.
y dy = - x dx
Integrating on both sides, we get
y2 x2
=- +c
2 2
Þ x 2 + y 2 = constant
and this constant is > 0 in practical sense.
44. Length of chord = 2 3 2 - 5 = 4

(–1,–1)
3
5
|| ||
(1,0)
Circle 279

47. Family of circles touching the line y = x at the point ( 4 , 4) is


( x - 4) 2 + ( y - 4) 2 + l( y - x) = 0
We need to find the member of this family which has length of chord = 6 2 on x + y = 0. For
different l’ s , we get different circles.
x 2 + y 2 - 8 x - 8 y + 32 + ly - lx = 0
Þ x 2 + y 2 + x( -8 - l) + y( -8 + l) + 32 = 0 …(1)

C
R
3
E 2
2
4

D A(4,4)

x
O
x

y
=

=
y

–x

Now,
OA = DC = 4 2
6 2
DE = 3 2 = (given)
2
Therefore, R 2 = (3 2 ) 2 + ( 4 2 ) 2
l2
Þ = 50 Þ l2 = 100 Þ l = ± 10
2
Substituting l = -10 in eq. (1), we get
x 2 + y 2 + 2 x - 18 y + 32 = 0
[Substituting l = 10; in eq. (1); we get x 2 + y 2 - 18 x + 2 y + 32 = 0 , which does not exist in
the given options]
Note : From eq. (1), we get
( l + 8) 2 ( l - 8) 2 l2
R 2 = (Radius) 2 = g 2 + f 2 - c = + - 32 =
4 4 2
280 Solution of Advanced Problems in Mathematics for JEE

48. Slope of line normal to circle and perpendicular to line


1
m = - = tan q
2 C

Co-ordinate of point lie on normal at a dist. of 3 from centre (1, –2)

æ y = 2x + 11
æ -2 ö æ 1 öö
ç 1 ± 3 çç ÷÷ , - 2 ± 3 çç ÷÷ ÷÷
ç
è è 5ø è 5 øø

Exercise-2 : One or More than One Answer is/are Correct

(3,3)
2.
(–3,3)

æpö æp ö
3. x 2 + y 2 - x ç ÷ + y ç - 2 sin -1 a ÷ = 0
2
è ø è 2 ø
2 2
æpö æp ö
Þ Length of chord = 2 ç ÷ + ç - sin -1 a ÷
è4ø è4 ø
7. ( x + 2) 2 + ( y - 3) 2 is nothing but square of distance between ( x , y) and ( -2 , 3) where ( x , y) is
point lies on the circle.
Centre = ( - 4 , 5), r = 16 + 25 + 40 = 9 A

Clearly, ( -2 , 3) is lies inside the circle.


C
\ PC = 2 2 a = PA 2 = (9 + 2 2 ) 2
P
b = PB 2 = (9 - 2 2 ) 2 B

\ a + b = 178, a - b = 72 2
8. Let point of intersection P( h, k)
Equation of chord of contact is
hx + ky = a 2
If it is tangent to x 2 + y 2 - 2 ax = 0

ha - a 2
Þ =a
h2 + k 2
Circle 281

9. Equation of tangent to circle


(5,5)
3 9
y - 3 = ( x - 2) ± 13 1 +
2 4 (2,3)

Þ 2 y = 3 x + 13 , 2 y = 3 x - 13
(0,6) (–1,1)
x2 - 2 y2 - 3 æ 13 ö
= = -ç ÷ Þ ( -1, 5)
3 2 è 13 ø
x3 - 2 y3 - 3 æ -13 ö
= = -ç ÷ Þ (5, 1)
3 -2 è 13 ø
10. 2 x 2 = 98 Þ x 2 = 49 Þ x = ± 7
(–6, 5)

y=5 (8, 5)

7
7
(1, –2)
98 x
(–6, –9)
y = –9 (8, –9)
x

x = –6 x=8

Exercise-3 : Comprehension Type Problems

Paragraph for Question Nos. 1 to 3


Sol. P1 ( -2 , - 1)
P1
æ 16 1ö C1(0,–2) C2 –3,– 1
P2 ç - ,- ÷ 2
è 7 7ø 5
æ1 ö 5 /2
P 3 ç , - 1÷
è2 ø P3
P2
3 5

C3(2,2)

Paragraph for Question Nos. 4 to 6


2 2
4. S : x + y + x (2l - 9) + y (3l - 12) + 53 - 27l = 0
C : x2 + y2 - 4x - 6 y - 3 = 0
282 Solution of Advanced Problems in Mathematics for JEE

Equation of line : S - C = 0
or x(2l - 5) + y(3l - 6) + 56 - 27l = 0
or 5 x + 6 y - 56 = 0 or 2 x + 3 y - 27 = 0
23
Þ x = 2, y =
3
5. Centre of C lies on common chord of S and C.
Þ (2 , 3) lies on x(2l - 5) + y(3l - 6) + 56 - 27l = 0
Þ S : x 2 + y 2 - 5x - 6 y - 1 = 0
6. Difference of squares of lengths of tangents from A and B is 3, which is equal to| AP 2 - BP 2|.
Paragraph for Question Nos. 7 to 8
7. Max. dist. between any two arbitrary points = 2
y
y=x
(1,1)

(1,–1)

8. x
O

Paragraph for Question Nos. 9 to 10


Sol. Let P( h, k)
L1 = h 2 + k 2 - 4

L2 = h2 + k 2 - 4h

L3 = h2 + k 2 - 4k
If L41 = L22 L23 + 16
Þ ( h 2 + k 2 - 4) 2 = ( h 2 + k 2 - 4 h)( h 2 + k 2 - 4 k) + 16
Þ ( h + k)( h 2 + k 2 - 2 h - 2 k) = 0
C1 : x + y = 0
C2 : x2 + y2 - 2x - 2 y = 0
Circle 283

Exercise-5 : Subjective Type Problems

1. Equation of chord of contact w.r.t. P A


hx + ky = 1
Equation of common chord is P(h,k)
( l - 3) x + ( 2l + 2) y + 3 = 0 B
l - 3 2l + 2
Þ = = -3
h k
Þ Equation of locus is 6 x - 3 y - 8 = 0
2. By using system of circles any circle passing through (1, 1) and ( -2 , 1) is
( x - 1)( x + 2) + ( y - 1) 2 + l( y - 1) = 0 …(1)
Given circles x2 + y2 -1=0 …(2)
Now radical axis of (1) and (2) is
( x - 2 y) + l( y - 1) = 0 …(3)
Q Radical centre of given circles is (0 , 0).
So, eq. (3) is passing through (0 , 0).
\ l =0
Put l = 0 in eq. (1) we get required circle.
y 3
3. = tan q =
x 4
y)
(x,
4 3
q
0 5

4. x 2 + y 2 - 26 x + 12 y + 210
( x - 13) 2 + ( y + 6) 2 + 5
(13,–6)

(5,0)

5. S º x 2 + y 2 + 2 gx + 2 fy + c = 0
Þ 2 g + 2 f = -c - 2 …(1)
(1, 1) satisfy circle.
Þ 2g + 2 f + c = -2
Þ c =0
284 Solution of Advanced Problems in Mathematics for JEE

and g + f = -1
\ Length of tangent = 8 + 4 g + 4 f + c = 2
6. Length of common external tangent
A
d 2 - ( r1 - r2 ) 2 = 5 …(1) r1
C1 r2
–B

30 °
cos(90° - B + 90° - C + 30° ) = cos 60°

90°
9
C2

–C
r1

90°–C
B
r 2 + r22 - d 2

°–
r2

90
= 1 …(2)
2 r1 r2 B C
5

(2, 2)
9.

y = –1 x=2 x=5

From diagram common points are 3.


10. (C 1 C 2 ) 2 = r12 + r22
18 = 2 r 2 Þ r 2 = 9
y
(2,4)

(2,3/2)

x
11. O
(2,–1)

12. PQ = PA = PB A P(h,k)
B
2 2 2 2 2
( h - 1) + ( k - 2) = 6 - ( h - 3) - ( k - 4)
Q C(3,4)
Þ h2 + k 2 - 4h - 6k - 3 = 0 (1,2)

13. c = 3 , a 2 + b 2 = 36

Length of chord AB = 2 r 2 - p 2
2
æ 2c ö
=2 c -ç ÷ =2 2
ç 2 2 ÷
è a +b ø
❑❑❑
Parabola 285

19
PARABOLA

Exercise-1 : Single Choice Problems

1. P ¢Q ¢ = PQ cos(90° - q)
4 y P¢
= (t 2 < 1)
2 P(1,2)
t +1
( P ¢Q ¢) min = 2 2 q x
O

Q(1,–2)
2. Equation of circle with SP as diameter
æ 9ö
( x - 4) ç x - ÷ + y( y - 6) = 0
è 4ø
æ 25 ö 25
Centre ç , 3 ÷ and radius =
è 8 ø 8
Equation of normal at P( 4 , 6) is
4 x + 3 y = 34
2
æ 25 ö
ç2 + 9 - 34 ÷
æ 25 ö 15
Length of chord = 2 ç ÷ - ç 8 ÷ =
è 8 ø ç 5 ÷ 4
ç ÷
è ø
3. The diagonals are the focal chord. B
2
A
AS = 1 + t = c (say)
1 1 æ 1 1 1ö S
+ =1 çQ + = ÷
c æ 25 ö è AS CS a ø
ç - c÷
è 4 ø D
C
5
Þ c = ,5
4
286 Solution of Advanced Problems in Mathematics for JEE

æ1 ö æ1 ö
A ç , 1÷ , B( 4 , 4), C( 4 , - 4) and D ç , - 1÷
è 4 ø è 4 ø
1 15
Area of trapzium = (2 + 8) ´
2 4
2
4. For normal chord t 2 = - t 1 -
t1
Also chord substends an angle of 90° at the vertex
\ t 1t 2 = - 4 Þ t 22 = 8
9. ( y - x + 2) + l( y + x - 2) = 0
The family of lines passes through (2 , 0).
The chord is x = 2 and end points are (2 , ± 4).
2
10. t 2 = - t1 - t 1)
t1 2 2
t 1,
P(
t 12 + t 22 2t + 2t 2
h= and k = 1
2 2
Put the value of t 2 and eliminate t 1 we get
4 k2
h -2 = + Þ a = 2, b = 4, c = 2 Q(t22, 2t 2)
k2 2
11. The parabola is ( y - 1) 2 = 4( x - 1). The coordinates of P(1 + t 12 , 1 + 2t 1 ) and
Q (1 + t 22 , 1 + 2t 2 ).
Here S(2 , 1) is the focus. The coordinates of T are G.M. of abscissa and A.M. of ordinates of P
and Q.
Þ ST 2 = 16 \ SP × SQ = ST 2
12. Let P(t 1 ) and Q (t 2 ) are point of y 2 = 8 x
2t 12 + 2t 22 = 17 and (2t 12 )(2t 22 ) = 11
ST 2 = SP × SQ = 2(1 + t 12 ) 2(1 + t 22 ) = 34 + 4 + 11
ST = 49
-1 -a A
13. ay = x 2 Þ = =- (slope of normal)
æ dy ö 2 x 1 B
ç ÷
dx
è ø
aB 1 a
Þ x1 = and y1 = - put ( x 1 , y 1 ) in ay = x 2
2A B 2
Parabola 287

dy y
14. =
dx 2 x
P
2 dy 1 (x,y)
= dx
y x
(0,y/2)
Þ 2 log y = log x + log c
Þ y 2 = cx put (3 , 1)
(–x,0)

15. ( x - a ) 2 = - ( y - (a + 4)) 4
x+
y=
The curve passes through (2 , 0)
(2 - a ) 2 = - (0 - (a + 4)) (0,4)
2
a - 5a = 0 Þ a = 0 or a = 5
( x - 5) 2 = - ( y - 9) put y = 0 Þ x = 2 , 8 A (–2,0) B(2,0) C

16. y = (tan 60° ) x is the focal chord. P


Coordinates of P and Q are intersection of y = 3 x with parabola
æ 4 -4ö 60°
P( 4 , 4 3 ), Q ç - , ÷
(–2,0) (0,0)
è 3 3ø
Find ^ bisector of PQ. Q

17. The director circle of the parabola is its directrix ( x + 11 = 0). Now apply condition of tangency.
18. The following figure depicts the condition. Chord of contact of a point A( x A , y A ) with respect
to y 2 = 4 x is y A y = 2( x + x A ). Since this chord passes through the point (3, 1), we have
y A = 2 ( x A + 3)
y
B

(xA, yA) (3,1)


A
x
O

AB and AC are tangents to the parabola.


BC is chord of contact of point A with respect to the parabola y 2 = 4 ax.
288 Solution of Advanced Problems in Mathematics for JEE

Given that point A lies on x 2 + y 2 = 25, we have


x 2A + y 2A = 25
Þ x 2A + 4 ( x A + 3) 2 = 25
Þ x 2A + 4 ( x 2A + 9 + 6 x A ) = 25
Þ 5 x 2A + 24 x A + 36 - 25 = 0
Þ 5 x 2A + 24 x A + 11 = 0

Exercise-2 : One or More than One Answer is/are Correct

1. Equation of normal at P( at 2 , 2 at ) is P(at2, 2at)

y = - tx + 2 at + at 3
G( 4 a + at 2 , - 2 at )
Þ Locus of point G( h, K ) is
G(h,K)
y 2 = 4 a( x - 4 a) (at2, –2at) Q

Exercise-3 : Comprehension Type Problems

Paragraph for Question Nos. 1 to 3


Sol. Tangent and normal are angle bisectors of focal radius and perpendicular to directrix.
Circle ‘ C ’ circumscribing DABP is
( x - 5)( x + 3) + ( y - 4)( y + 4) = 0
Length of latus rectum = 4(2 2 ) = 8 2
=5
+y
y =9 x P
x+ x–y–1=0
B(–3,–4)

2)
A (3 ,
)
(5,4
Parabola 289

Exercise-5 : Subjective Type Problems

1. b = 2a 2 + 4a - 2 …(1)
2 A
- b = 2a - 4a - 2 …(2) –1 (a,b)
(1) & (2) Þ 4a 2 - 4 = 0 Þ a = ± 1
B
Put a = 1, b = 2 + 4 - 2 = 4 (–a,–b)

\ A (1, 4), B ( -1, - 4)


AB 2 = l 2 = ( 4 + 64 ) 2 = 68
æ a + b æ a + bö2 ö æ a + b - a 2 - b2 ö
2. R = ç , -ç ÷ ÷ , M = çç , ÷
ç 2 è 2 ø ÷ø ÷
è è 2 2 ø
2 2
-b + a a b
PQ = y + b 2 = ( x - b) R
b-a Q
P
M
y + b 2 = - ( b + a)( x - b)
y = - ( b + a)( x - b) - b 2
b
D 1 = [[- ( a + b)( x - b) - b 2 ] + x 2 ] dx
ò
a
b
( x - b) 2 x3 ( a - b) 3
= - ( a + b) - b2 x + =
2 3 6
a

a -a 2 1
1
Area of DPQR = D 2 = b -b 2 1
2 2
a+b æ a + bö
-ç ÷ 1
2 è 2 ø
( a - b) 3
R 1 ® R 1 - R 2 , R 2 ® R 2 - R 3 , we get D 2 =
8
3. m AB ´ m BC = -1
-2 -2
Þ ´ = -1
(t 1 + t 2 ) (t 2 + t 3 )
Þ (t 1 + t 2 )(t 2 + t 3 ) = - 4
Similarly,
m AD ´ m CD = -1
Þ (t 1 + t 4 )(t 3 + t 4 ) = - 4
Þ (t 1 + t 2 )(t 2 + t 3 ) = (t 1 + t 4 )(t 3 + t 4 )
Solving this
t2 + t4
= -1
t1 + t 3
❑❑❑
290 Solution of Advanced Problems in Mathematics for JEE

20
ELLIPSE

Exercise-1 : Single Choice Problems

1. Length of perpendicular from C(0 , 0) to the tangent at P(2 3 cos q , 2 2 cos q) is

-1
CF =
cos 2 q sin 2 q
+
12 8
2 3x 2 2 y
Equation of normal at P is - = 12 - 8 which meets the major axis at
cos q sin q
æ 2 ö
Gç cos q , 0 ÷
è 3 ø
CF ´ PG = 8

(0,3)
(–4,3) (4,3)
y=3
2. (2,0)

The minimum length of intercept will be possible when


y = 3 or y = -3 Þ AB = 8

x+y=7

3.
Ellipse 291

dy x
=- = -1
dx 2y
Put x = 2 y in the equation of ellipse
The point lies in I quad Þ (2 , 1)
4. Equation of tangent at P is y
b2
ex + y = a P ae, a

2 10 x
Þ e= , a=
3 3 2
b
Q ae, – a
10 5
and b=
9
2 b 2 100
Length of latus rectum = =
a 27
5. Area bounded by circle & ellipse = pa 2 - pab = pa( a - b)
S F + S 2 F2
6. 1 1 ³ ( S 1 F1 )( S 2 F 2 ) = 16
2
Q Product of perpendiculars from two foci of an ellipse upon any tangent is equal to the
square of semi-minor axis.
7. f( k 2 + 2 k + 5) > f( k + 11)
Þ k 2 + 2 k + 5 < k + 11 Þ k Î ( -3 , 2)
8. Since sides of the square are tangent and perpendicular to each other,
so the vertices lie on director circle
2
æ 10 ö
x 2 + y 2 = 16 + b 2 = ç ÷ 10
è 2 ø
5 2 5 2
Þ b=3
A p( 4)(3)
= = 12
p p
æ xq + py ö 2 2
9. T = S 1 Þ px + qy + ç ÷ - 1 = p + q + pq - 1
è 2 ø
Þ p 2 + q 2 = - pq Þ p = 0 , q = 0
10. The combined equation of pair of tangents drawn from a point ( x 1 , y 1 ) to the ellipse
x2 y2
Sº + - 1 = 0 is T 2 = SS 1 . Therefore,
2 2
a b
2
æ xx 1 yy ö æ x2 y2 ö æ x2 y2 ö
ç 2 + 21 - 1÷ = ç 2 + 2 - 1÷ ç 12 + 12 - 1÷
è a b ç ÷ ç ÷
ø èa b øè a b ø
292 Solution of Advanced Problems in Mathematics for JEE

2
æ 4x ö æ x2 ö æ 42 ö
ç + 2 y - 1÷ = ç + y 2 - 1÷ ç + 2 2 - 1÷
è 9 ø ç 9 ÷ç 9 ÷
è øè ø
Þ 3 x 2 + 7 y 2 - 16 xy + 8 x + 36 y - 52 = 0

2 h 2 - ab
Þ tan a =
a+b
where, a = 3 , b = 7 and h = -8. Therefore,
2 64 - 21 43
tan a = =
10 5
Note : a is acute angle between the pair of tangents. Therefore,
( a + b - c) 2 = a 2 + b 2 + c 2 + 2 ab - 2 ac - 2 bc
Alternate solution : Any line passing through the point (4, 2) is given by
y - 2 = m ( x - 4)
y = mx - 4m + 2
For this line to be tangent to the given ellipse, put this y into the equation of the ellipse and
make
D =0
That is,
x2
+ (mx - 4m + 2) 2 = 1
9
(1 + 9m 2 ) x 2 + x(36m - 72m 2 ) + 16 (9)m 2 - 16 (9)m + 27 = 0
Now,
D = 0 Þ B 2 - 4 AC = 0
Þ (36m - 72m 2 ) 2 - 4 (1 + 9m 2 )(16 × 9m 2 - 16 × 9m + 27) = 0
Þ (36m ) 2 (1 - 2m ) 2 - 36 (1 + 9m 2 )(16m 2 - 16m + 3) = 0
Þ m 2 (1 + 4m 2 - 4m ) - 36 (16m 2 - 16m + 3 + 9 × 16m 4 - 9 × 16m + 27m 2 ) = 0
Þ 7m 2 - 16m + 3 = 0
Now,
m1 - m 2 ( m 1 + m 2 ) 2 - 4m 1 m 2
tan a = =
1 + m 1m 2 1 + m 1m 2

2
æ 16 ö 3
ç ÷ -4×
è 7 ø 2 7 æç 16 2 - 4 × 3 × 7 ö÷
Þ tan a = =
3 10 ç 7 ÷
1+ è ø
7
Ellipse 293

æ 1 ö 43
= ç ÷ 4 ( 43) =
è 10 ø 5
where a is the acute angle between the tangents.

Exercise-2 : Comprehension Type Problems

Paragraph for Question Nos. 1 to 2


1. SS ¢ = 2 ae
2 x2
2 2 3ö
÷2æ
ç S¢
( x 1 - x 2 ) + ( y 1 - y 2 ) = 4(2) ç ÷
è 2 ø C
(h,k) y
( x 1 + x 2 ) 2 + ( y 1 + y 2 ) 2 - 4 ( x 1 x 2 + y 1 y 2 ) = 12 x1
2
S
y1
(2 h) 2 + (2 k) 2 - 4 (1 + 1) = 12
(Q x 1 x 2 and y 1 y 2 are ^ distance of the foci from their tangents
= b 2 = 12 )
Þ h2 + k 2 = 5
1 + h2
2. (2 x - h)( h) = 1 Þ x=
2h
1 + k2 S¢(2x–h, 2y–k)
(2 y - k)( k) = 1 Þ y= C
2k
(x,y)
S(h,k)

❑❑❑
294 Solution of Advanced Problems in Mathematics for JEE

21
HYPERBOLA

Exercise-1 : Single Choice Problems

1
1. The normal is y - 4 = ( x - 1). Put the value of y in xy = 4 we get co-ordinates.
4
3. c 2 = a 2 m 2 - b 2 Þ c 2 = l2 m 2 - (l3 + l2 + l) 2
c2 ³ 0 Þ m 2 ³ (l2 + l + 1) 2
3 9
l2 + l + 1 has minimum value Þ m 2 ³
4 16
3
4. The asymptotes are y = ± x and the double ordinate be
2
æ 3 ö æ 3 ö
P çç h, h 2 - 4 ÷÷ and P ¢ çç h, - h 2 - 4 ÷÷
è 2 ø è 2 ø
Þ ( PQ )( PQ ¢) = 3
5. 2 ae = 5 and 2 a = 3
5
Þ e=
3 (1,2) (5,5)
1 1 5
Þ + = 1 Þ e¢ =
2 2 4
e (e )
¢
6. The equation of normal at (2 sec q , tan q) is 2 x cos q + y cot q = 5
1
Equal intercepts Þ sin q =
2
2 2 25
Also touches ellipse Þ a +b = Q c 2 = a 2m 2 + b 2
3
7. Let locus of point be ( h, k).
Equation of chord of contact is hx + ky = 4
æ 4 - hx ö
For tangent, x ç ÷ = 1 has two equal roots.
è k ø
Hyperbola 295

Þ hk = 4 Þ xy = 4
2 2
x2 y æ x cos a + y sin a ö
8. - -ç ÷÷ = 0
16 18 çè p ø
Þ Coeff. of x 2 + coeff. of y 2 = 0 Þ P = ± 12
2
æ dö d
The chord x cos a + y sin a ± 12 = 0 is tangent to the circle x 2 + y 2 = ç ÷ Þ =6
è2ø 4
9. Let the rectangular hyperbola be x 2 - y 2 = a 2 and the point be ( a sec q , a tan q).
æ 2 a ö æ a cos q ö æ 2 a sin q ö
a1 a 2 + b1 b2 = ( a cos q) ç ÷ + ç- ÷ç ÷
è cos q ø è sin q ø è cos q ø

Exercise-2 : One or More than One Answer is/are Correct

æ 1ö
3. Let ç t , ÷ be any point on xy = 1
è tø
xy = 1
Þ xy ¢ + y = 0
-y
Þ y¢ =
x
1
Þ y¢ = -
t2
-b
Þ =t2
a
Þ a and b are of opp. sign.
❑❑❑
296 Solution of Advanced Problems in Mathematics for JEE

22
COMPOUND ANGLES

Exercise-1 : Single Choice Problems

2. a sin x + b(2 cos c cos x) = a


a - a sin x
cos c =
2 b cos x
1
= (a sec x - a tan x) differentiate w.r.t. x
2b
a sec x tan x - a sec 2 x = 0
a
Þ sin x =
a
3 tan x - tan 3 x
3. tan x × < -1
1 - 3 tan 2 x
æ 3t - t 3 ö
tç ÷ + 1< 0 (Let tan x = t )
ç 1 - 3t 2 ÷
è ø
1-t4 (t - 1)(t + 1)
<0 Þ <0
2
1 - 3t (3t 2 - 1)

æ 1 ö
Þ t Î çç , 1÷÷
è 3 ø
8 8
é tan( r + 1) A - tan( rA) - tan A ù tan 9 A - 9 tan A
4. å tan( r A) tan {( r + 1) A} = å êë tan A úû = tan A
= -10
r =1 r =1

5. f( x) = 2 cosec 2 x + sec x + cosec x


1 + sin x + cos x
=
sin x cos x
sin 3 x + sin 2 x - cos 3 x - cos 2 x p
f ¢( x) = =0 Þ x=
2 2 4
sin x cos x
Compound Angles 297

2 p
f( x) min = at x =
2 -1 4
6. cosec q + cosec (60° – q) - cosec (60°+q)
where q = 10°
1 1
10. (2 sin x cos x + 2 cos 2 x) = (sin 2 x + cos 2 x + 1)
2 2
tan A tan B
11. = =k ( k > 0), if 2 sin A = 3 sin B
3 5
2 tan A 3 tan B 2 3k 3 ´ 5k 1
Þ = Þ = Þ k=
2 2 2 2 5
1 + tan A 1 + tan B 1 + 3k 1 + 5k
12. Gives equations can be written as
2 cos a + 9 cos d = -6 cos b - 7 cos g …(1)
2 sin a - 9 sin d = 6 sin b - 7 sin g …(2)
Square and add equation (1) and (2),
Þ 4 + 36 + 36 [cos a cos d - sin d sin a ] = 36 + 49 + 84 [cos b cos g - sin b sin g]
Þ 36 [cos(a + d)] = 84 [cos(b + g)]
cos(a + d) 84 7 m
= = = ; m + n = 10
cos(b + g) 36 3 n

1 + sin q + 1 - sin q 2
13. = = -2 sec q
1 - sin q 2 cos q
3
2 rp 2p 4p 6p 2p 4p 8p
14. A = å cos 7
= cos
7
+ cos
7
+ cos
7
= cos
7
+ cos
7
+ cos
7
=B
r =1
x 1
15. tanb = =
z 3
y 1
tan a = =
z 2
tan a + tan b p
tan(a + b) = =1 Þ a +b =
1 - tan a tan b 4
é p 2p ù
17. f( x) = -2 sin 2 x + sin x + 2 " x Î ê ,
ë 6 3 úû
Let sin x = t
é1 ù
f(t ) = -2 t 2 + t + 2 " t Î ê , 1ú
ë2 û
298 Solution of Advanced Problems in Mathematics for JEE

1 3
18. 1 + (cos 2 A - sin 2 B ) - cos A cos B = 1 + cos( A + B ) cos( A - B ) - [cos( A + B ) + cos( A - B )] =
2 4
19. (2 sin x - cosec x) 2 + (tan x - cot x) 2 = 0
1
\ sin 2 x = Ç tan 2 x = 1
2
20. cos 2 A = sin A × tan A Þ cos 3 A = sin 2 A
æ 3 + 1ö æ 3 + 1ö æ 3 + 1ö
21. f( x) = çç ÷ sin x + ç
÷ ç 2 ÷
÷ cos x = ç ÷
ç 2 ÷ (sin x + cos x)
è 2 ø è ø è ø
22. A = B + C
Þ tan A tan B tan C = tan A - tan B - tan C
23. E = sin A + sin 2 B + sin 3C
3 4 3
E = + 2 × × -1
5 5 5
15 24 39 - 25 14
= + -1= =
25 25 25 25
cos A cos C + cos A cos C
24. = cot C (Q A + B + C = p)
cos A sin C + cos A sin C
æa + gö æa -gö
2 cos ç ÷ sin ç ÷
sin a - sin g è 2 ø è 2 ø æa + gö
25. = = cot ç ÷ = cot b
cos g - cos a æa + gö æa -gö è 2 ø
2 sin ç ÷ sin ç ÷
è 2 ø è 2 ø
x x x x x sin x
26. cos × cos cos LL cos × cos =
256 128 64 4 2 æ x ö
256 sin ç ÷
è 256 ø
(sin 7a + sin 5a ) + 5(sin 5a + sin 3a ) + 12(sin 3a + sin a )
27.
sin 6a + 5 sin 4a + 12 sin 2a
2 sin 6a cos a + 5(2 sin 4a cos a ) + 12(2 sin 2a cos a )
= = 2 cos a
sin 6a + 5 sin 4a + 12 sin 2a
28. tan 2 A + tan 2 B + tan 2 C = tan A tan B + tan B tan C + tan A tan C
Þ tan A = tan B = tan C
p
Þ A = B =C =
3
29. log|sin x| |cos x |+ log|cos x| |sin x|= 2log|sin x| |cos x |= 1 Þ |cos x |=|sin x |
Þ
3
30. f( x) = sin x + cos x = 1 - 3 sin x cos x = 1 - sin 2 2 x
6 6 2 2
4
Compound Angles 299

2 sin a é(1 + sin a ) - cos a ù 2 sin a[(1 + sin a ) - cos a ]


31. y = ´ê =
1 + cos a + sin a ë(1 + sin a ) - cos a úû (1 + sin a ) 2 - cos 2 a
1 + sin a - cos a
=
1 + sin a
tan 3 A cot 3 A sin 3 A cos 3 A
32. + = +
1 + tan 2 A 1 + cot 2 A cos A sin A
sin 4 A + cos 4 A 1 - 2 sin 2 A cos 2 A
= =
sin A cos A sin A cos A
= sec A cosec A - 2 sin A cos A
1 - sin q 1 + sin q 2 2
33. + = =
1 + sin q 1 - sin q 1 - sin 2 q |cos q|
34. y = (sin 2 q + cosec 2 q + 2) + (cos 2 q + sec 2 q + 2) = 7 + (tan 2 q + cot 2 q) ³ 9
35. log 3 sin x - log 3 cos x - log 3 (1 - tan x) - log 3 (1 + tan x) = -1
æ tan x ö tan x 1 2
log 3 ç ÷ = -1 Þ = Þ tan 2 x =
è 1 - tan 2 x ø 1 - tan x 2 3 3
æ 1 ö
36. sin q + cosec q = 2 Þ sin q = cosec q = 1 ; ç x + ³ 2 ÷
è x ø
37. (tan q + cot q) (tan 2 q + cot 2 q - 1) = 52
(tan q + cot q) {(tan q + cot q) 2 - 3} = 52
Let tan q + cot q = t
t 3 - 3t - 52 = 0 Þ t =4
tan q + cot q = (tan q + cot q) 2 - 2 = 14
2 2

38. -5 £ 3 sin x - 4 cos x £ 5


10 £ 3 sin x - 4 cos x + 15 £ 20
log 20 10 £ log 20 (3 sin x - 4 cos x + 15) £ log 20 20
39. x2 + y2 =9
Let x = 3 cos q , y = 3 sin q
4 a 2 + 9 b 2 = 16
4
Let a = 2 cos f , b = sin f
3
4 a 2 x 2 + 9 b 2 y 2 - 12 abxy = (2 ax - 3 by) 2
= (12 cos q cos f - 12 sin q sin f) 2 = 144 cos 2 (q + f)
300 Solution of Advanced Problems in Mathematics for JEE

40. A 2 = sin 2 - sin 3 y=sinx

sin 3 > sin 2


A2 < 0
B 2 = cos 2 - cos 3
cos 3 > cos 2 x
O 1.57 3 2 3.14
B2 < 0
y=cosx

3 2 3.14
x
O 1.57

Both A and B are not real numbers.


41. (2 x + 2 - x - 2 cos x) (3 x + p + 3 - x -p + 2 cos x)(5 p- x + 5 x -p - 2 cos x) = 0
2 x + 2 -x
If = cos x Þ x =0
2
3 x + p + 3 - x -p
If = - cos x Þ x = -p
2
5 p- x + 5 x -p
If = cos x (Not possible)
2
There are two real values of x.
42. e sin x - e - sin x - 4 = 0
e 2 sin x - 4 e sin x - 1 = 0
Þ e sin x = 2 ± 5
If e sin x = 2 + 5
Þ sin x = ln (2 + 5) [ln(2 + 5) > 1, Not possible]
If e sin x = 2 - 5 (2 - 5 < 0) Not possible
There is no solution.
43. 4 sin 4 a + 4 sin 2 a × cos 2 a + 4 cos 2 ( p / 4 - a / 2)
= 4 sin 2 a + 2 [1 + cos( p / 2 - a )]
= 2 |sin a | + 2 + 2 sin a
3p
= -2 sin a + 2 + 2 sin a = 2 (If p < a < then sin a < 0)
2
Compound Angles 301

æ p p ö
ç sin cos ÷
æ p p öç 12 12 ÷
44. ç cos - sin ÷ +
è 12 12 ø ç p p ÷
ç cos sin ÷
è 12 12 ø
p p
cos - sin
= 12 12 = 2 1 - sin p / 6 = 2 2
p p sin p / 6
sin × cos
12 12
tan 100°+ tan 125°
45. tan(100°+125° ) = =1
1 - tan 100° tan 125°
Þ tan 100°+ tan 125°+ tan 100° tan 125° = 1
46. If sin x + sin 2 x = 1 Þ sin x = 1 - sin 2 x Þ sin x = cos 2 x
cos 8 x + 2 cos 6 x + cos 4 x = sin 4 x + 2 sin 3 x + sin 2 x
= sin 2 x(sin 2 x + 2 sin x + 1)
= (1 - sin x) (2 + sin x)
= 2 - sin x - sin 2 x = 1
47. Let x = 5cos q, y = 5sin q
0 < 3 x + 4 y £ 25 (Q 3 x + 4 y > 0)
æqö
48. 5 cos 2q + 2 cos 2 ç ÷ + 1 = 0
è2ø
1 3
10 cos 2 q + cos q - 3 = 0 Þ cos q = , -
2 5
4
49. sinb = where 0 < b < p and tanb > 0
5
3
then cosb =
5
é3 4 ù
5 ê sin(a + b) - cos(a + b)ú coseca = 5
ë5 5 û
æ pö æ pö é p æ pö p æ p öù æ 5p ö
50. sin ç x + ÷ + cos ç x + ÷ = 2 êcos sin ç x - ÷ + sin × cos ç x + ÷ ú = 2 sin ç x + ÷
è 6 ø è 6 ø ë 4 è 6 ø 4 è 6 øû è 12 ø
5p p p
This attained maximum value when x + = Þ x=
12 2 12
51. sin 2 x - cos 2 x = 2 a - 1
- 2 £ 2a - 1 £ 2
1- 2 1+ 2
£ a£
2 2
52. (cos 12°× cos 24°× cos 48°× cos 84° ) (cos 36° cos 72° ) × cos 60°
( - cos 12°× cos 24°× cos 48°× cos 96° ) (cos 36° cos 72° ) × cos 60°
302 Solution of Advanced Problems in Mathematics for JEE

é sin(2 4 ´ 12° ) ù æ 5 + 1 5 - 1ö 1 1
ê- 4 ú ´ çç ´ ÷´ =
÷
ë 2 sin 12° û è 4 4 ø 2 128

53. 2 cos 2 q + cos q + 1


7 1
y min = at cos q = -
8 4
y max = 4 at cos q = 1
y max 32
=
y min 7

–1 –1/4 –1
3
æ 3 tan x - tan x ö 4
54. tan x ç ÷ + 1 < 0 ; tan x - 1 < 0
ç 1 - 3 tan 2 x ÷ 3 tan 2 x - 1
è ø
(tan 2 x + 1)(tan x + 1)(tan x - 1)
Þ <0
( 3 tan x + 1)( 3 tan x - 1)
p p
Þ < x<
6 4
p p
55. a = 2 R sin = 2 r tan
n n
p
n
R r

a/2
56. (cos12°+ cos 132° ) + (cos 84°+ cos 156° )
12°+132° 12°-132° 84°+156° 84°-156°
= 2 cos cos + 2 cos cos
2 2 2 2
= 2 cos 72° cos 60°+2 cos 120° cos 36°
5 -1 1 æ 1ö 5 +1 1
=2 ´ ´ + 2 ´ç- ÷´ =-
4 2 è 2 ø 4 2
1 é 2 sin q cos q 2 sin 3q cos 3q 2 sin 9q cos 9q 2 sin 27q cos 27q ù
57. + + +
2 êë cos q cos 3q cos 9q cos 3q cos 9q cos 27q cos 27q cos 81q úû
1 é sin(3q - q) sin(9q - 3q) sin(27q - 9q) sin(81q - 27q) ù
= + + +
2 êë cos q cos 3q cos 3q cos 9q cos 9q cos 27q cos 27q cos 81q úû
1 1 é sin 80q ù
= [tan 81q - tan q] = ê
2 2 ë cos q cos 81q úû
Compound Angles 303

æ 4 cos 20° + 1 ö 2 sin 40° + sin 20° 2 sin(60° - 20° ) + sin 20°
58. sin 20° ç ÷= = = 3
è cos 20° ø cos 20° cos 20°
59. Let us draw the graph of
æ xp ö
f( x) = sin ç ÷
è 2 ø
and g( x) = cos( xp)
On the same xy-plane as shown in the following figure.
y

g(x)

1 2 3 4
x
0 1 3 5 7
2 2 2 2
f(x)

æ5 7ö
From this graphical representation, it is clear that y is strictly increasing in ç , ÷
è2 2ø
Because for all values of x,
5 7
< x<
2 2
æ xp ö
That is, sin ç ÷<0
è 2 ø
and cos( xp) < 0
dy
which imply that >0
dx
which means that y is strictly increasing.
æ sin 8q ö
60. 8 sin q sin 3q ç ÷ = cos 6q
è 4 sin 2q ø
sin 3q sin 8q = cos 6q cos q
cos 5q - cos 11q = cos 7q + cos 5q
cos 7q + cos 11q = 0
2 cos 9q × cos 2q = 0
1 1 3
61. tan A = - Þ sin A = ; cos A = -
3 10 10
-3
63. (2 cos q) 2 = (1 - sin q) 2 Þ sin q = 1 or sin q =
5
304 Solution of Advanced Problems in Mathematics for JEE

1
64. sin q + = 2 Þ sin q = 1
sin q
65. tan 2 q + cot 2 q = a Þ tan 3 q + cot 3 q = a + 2 ( a - 1) = 52
5
66. tan A = - tan C =
12
3 4
cos B = - cos D = - Þ tan D =
5 3
67. tan 2 q - sin 2 q = tan 2 q sin 2 q =|tan q sin q|
sin 10° + sin 20°
68. = tan 15° = 2 - 3
cos 10° + cos 20°
69. (sin 2 q) 3 + (cos 2 q) 3 = (sin 2 q + cos 2 q)(sin 4 q + cos 4 q - sin 2 q cos 2 q)
= 1 - 3 sin 2 q cos 2 q
tan x + 1 sec 2 x + 2 (tan x + 1) 2 - (sec 2 x + 2)
70. - Þ
tan x - 1 tan 2 x - 1 tan 2 x - 1
2 tan x - 2 2
Þ Þ
2 tan x + 1
tan x - 1
cot a - tan a
71. - [cos 450° + cos(2a - 180° )]
cot a + tan a
Þ (cos 2 a - sin 2 a ) + cos 2a = 2 cos 2a
æ 1 + tan a ö æ 1 + tan a ö
72. ç ÷ ×ç ÷+1
è 1 - tan a ø è 1 - tan a ø
æp ö æp ö æp ö
1 + tan 2 ç + a ÷ = sec 2 ç + a ÷ = cosec 2 ç - a ÷
è4 ø è4 ø è4 ø
tan a + sin a
73. = tan a
1 + cos a
74. (cos 2a + cos 5a ) - (cos 3a + cos 4a )
7a 3a 7a a
2 cos × cos - 2 cos × cos
2 2 2 2
7a é 3a aù a 7a
2 cos cos - cos ú = -4 sin sin a cos
2 êë 2 2û 2 2
2
æ 1 + sin a sin b ö
1 - çç ÷÷
1 - tan 2 g è cos a cos b ø
75. cos 2 g = =
1 + tan 2 g æ 1 + sin a sin b ö
2
1 + çç ÷÷
è cos a cos b ø
(cos a cos b) 2 - (1 + sin a cos b) 2 [1 + cos(a - b)][cos(a + b) - 1]
Þ = £0
2 2
(cos a cos b) + (1 + sin a sin b) (cos a cos b) 2 + (1 + sin a sin b) 2
Compound Angles 305

2p
76. x = (II nd quadrant)
3
sin 50 x æ 101x ö 1
cos x + cos 2 x + cos 3 x +¼ + cos 100 x = × cos ç ÷=-
x è 2 ø 2
sin
2
p 2p 10 p 1
77. cos 3 0° + cos 3 + cos 3 + cos 3 p +¼ + cos 3 =-
3 3 3 8
1 - 2(cos 60° - cos 80° ) 2 cos 80°
78. = =1
2 sin 10° 2 sin 10°
79. ( x + 5) 2 + ( y - 12) 2 = 14 2
Let x = -5 + 14 cos q , y = 12 + 14 sin q
Þ x 2 + y 2 = 365 + 336 sin q - 140 cos q
80. tan q = l has three distinct solution in [0 , 2p] Þ l = 0 and q = 0 , p , 2 p.
1 + tan a 1 - tan a 2
81. + =
1 - tan a 1 + tan a 1 - tan 2 a
æ3 4 ö
82. 3 sin q + 4 cos q = 5 ç sin q + cos q ÷ = 5 sin(q + 53° )
è5 5 ø
n
83. f(n) = Õ cos r
r =1

f( 4) = cos 1 × cos 2 × cos 3 × cos 4 < 0


f(5) = cos 1 × cos 2 × cos 3 × cos 4 × cos 5 < 0
( p 2 - q 2 ) 2 ( 4 tan A sin A) 2
84. = = 16
pq tan 2 A - sin 2 A
æ pö
85. 0 < sin a < cos a < 1 a Î ç 0, ÷
è 4ø
(sin a ) cos a < (sin a ) sin a
(cos a ) cos a < (cos a ) sin a
A 5A
86. 32 sin sin = 16 (cos 2 A - cos 3 A)
2 2
= 16 [(2 cos 2 A - 1) - ( 4 cos 3 A - 3 cos A)]
87. cos a cos b - sin a sin b + sin a cos b - cos a sin b = 0
cos a(cos b - sin b) + sin a(cos b - sin b) = 0
(cos b - sin b)(cos a + sin a ) = 0
cos a = - sin a (Q cos b ¹ sin b)
tan a = -1
306 Solution of Advanced Problems in Mathematics for JEE

88. 2 x = 3 y = 6 -z = k
x = log 2 k , y = log 3 k , z = - log 6 k
1 1 1
+ + =0
x y z
2 2
æ -21 ö æ -27 ö
89. (sin a + sin b) 2 + (cos a + cos b) 2 = ç ÷ +ç ÷
è 65 ø è 65 ø
1170 æ a -b ö
2 + 2 cos(a - b) = = 4 cos 2 ç ÷
2
(65) è 2 ø
90. m 2 = a 2 + b 2 + 2 ( a 2 cos 2 q + b 2 sin 2 q)( a 2 sin 2 q + b 2 cos 2 q)

= a 2 + b 2 + 2 a 2 b 2 + ( a 4 + b 4 - 2 a 2 b 2 ) sin 2 q cos 2 q
n
sin(3 r q) cos(3 r q) 1 n 1
91. Q = å = å tan(3 r +1 q) - tan(3 r q) = P
r
r = 0 cos(3 q) cos(3 r +1 q) 2 r =0 2
92. When 270° < q < 360° , we have
æ qö
2 (1 + cos q) = ç 2 cos 2 ÷
è 2 ø
which is non-negative. Now, the above equation can be written as
q
2 (1 + cos q) = 2 cos
2
q æ q q ö
= -2 cos çQcos < 0 when 135° < < 180° ÷
2 è 2 2 ø
Now, let us consider that 2 + 2 (1 + cos q)
which is not-negative. That is,
q
2 + 2 (1 + cos q) = 2 - 2 cos
2
q q
= 2 1 - cos = 2 2 sin 2
2 4
q
= 2 sin
4
q æ q 135° q ö
= 2 sin çQsin > 0 when < < 90° ÷
4 è 4 2 4 ø
93. We know that - 2 £ sin x + cos x £ 2
3p
When x = - , we have sin x + cos x = - 2
4
Compound Angles 307

3p
when x = - , we have y = - 2 + 1 < 0
4
which implies that options (1) and (2) are incorrect.
p
Now, at x = , we have sin x + cos x = 2
4
That is, (sin 4 x + cos 4 x) 2 ¹ 2. Therefore, y ¹ 2 + 2 for any x Î R .
which implies that option (4) is incorrect.
p
Note : The maximum value of sin x + cos x is 2 , for x = and the maximum value of
4
p
(sin 4 x + cos 4 x) 2 is 2, for x = .
16
94. (cos x + cos y) 2 + (sin x + sin y) 2 = ( - cos z ) 2 + ( - sin z ) 2
2 + 2 cos( x - y) = 1
1 1 1 sin 50° sin 70° + sin 10° sin 70° - sin 10° sin 50°
95. + - =
sin 10° sin 50° sin 70° sin 10° sin 50° sin 70°
1
(cos 20° - cos 120° + cos 60° - cos 80° - cos 40° + cos 60° )
= 2
1
sin 30°
4
1æ3 ö
ç + cos 20° - 2 cos 60° cos 20° ÷
2è2 ø
= =6
1
sin 30°
4

Exercise-2 : One or More than One Answer is/are Correct

1. cot 12° cot 24° cot 48° [cot 28° cot(60°-28° ) cot(60°+28° )] = (cot 12° cot 48° )(cot 24° cot 84° )

cot 36° cot 72°


= ´ =1
cot 72° cot 36°
2. cot 4 x - 2(1 + cot 2 x) + a 2 = 0
Þ cot 4 x - 2 cot 2 x + a 2 - 2 = 0
Þ (cot 2 x - 1) 2 = 3 - a 2
to have atleast one solution
3 - a2 ³ 0
Þ a2 - 3 £ 0
a Î [- 3 , 3 ]
308 Solution of Advanced Problems in Mathematics for JEE

Integral values –1, 0, 1


\ Sum = 0
p
3. (A) tan 1 > tan -1 1 Þ tan 1 >
4
(B) sin 1 > cos 1
sin 57.3° > cos 57.3°
(C) tan 1 < sin 1 (not possible)
Because tan 57.3 > 1 > sin 57.3°
p
(D) cos 1 <
4
æpö
Þ cos (cos 1) > cos ç ÷
è4ø
4. (A) tan 1 > 1 and sin 1 < 1, then log sin 1 tan 1 < 0
(B) 1 + tan 3 < 1 and cos 1 < 1, then log cos1 (1 + tan 3) > 0
(C) cos q + sec q > 2 and log 10 5 < 1, then log log 10 5(cos q + sec q) < 0
(D) 2 sin 18° < 1 and tan 15° < 1, then log tan 15° 2 sin 18° > 0
æaö æaö
2 tanç ÷ 1 - tan 2 ç ÷
è2ø , è2ø
5. Put sin a = cos a =
æ a ö æaö
1 + tan 2 ç ÷ 1 + tan 2 ç ÷
è2ø è2ø
sin(2a + b) 3
6. Given =
sin b 1
sin(2a + b) + sin b 3 + 1
Option (C) = (Use C and D method)
sin(2a + b) - sin b 3 - 1
tan(a + b) = 2 tan a
Option (B) 3 sin b = sin(2a + b)
2 sin b = sin(2a + b) - sin b
2 sin b = 2 cos(a + b) sin a
Option (D) 3 sin b = sin{a + (a + b)}
3 sin b = sin a cos(a + b) + cos a sin(a + b)
Subtract from (B) option
2 sin b = cos a sin(a + b)
cos b cos(2a + b)
Option (A) cot b - 3 cot(2a + b) = -3
sin b sin(2a + b)
cos b cos(2a + b) 2 sin(a + b) sin a
= -3 = = 4 tan a (from D)
sin b 3 sin b sin b
3
Also cot a + cot(a + b) = cot a (from C)
2
Compound Angles 309

Now multiply the two relations.


7. sin( x + 20° ) = sin( x + 40° ) + sin( x - 40° )
sin( x + 20° ) - sin( x - 40° ) = sin( x + 40° )
cos( x - 10° ) = sin( x + 40° ) = cos[90°-( x + 40° )]
Þ x = 30° now check the option, only (a) and (b) satisfy
8. 2S(cos x cos y) + 2S(sin x sin y) + 3 = 0
(S cos x) 2 + (S sin x) 2 = 0
Þ S cos x = 0 and S sin x = 0
cos 3 x + cos 3 y + cos 3 z = 4(cos 3 x + cos 3 y + cos 3 z ) - 3(cos x + cos y + cos z )
= 12 cos x cos y cos z
9. 0 < sin x < 1, 0 < cos x < 1
If sin n x + cos n x = 1 n =2
n n
sin x + cos x > 1 n<2
n n
sin x + cos x < 1 n>2
10. If x = sin(a - b) sin( g - d)
2 x = cos(a - b - g + d) - cos(a - b + g - d)
y = sin(b - g) sin(a - d)
Þ 2 y = cos(b - g - a + d) - cos(b - g + a - d)
z = sin( g - a ) sin(b - d)
Þ 2z = cos( g - a - b + d) - cos( g - a + b + d)
2 x + 2 y + 2z = 0 Þ x + y + z =0
If x + y + z = 0 then x 3 + y 3 + z 3 = 3 xyz
11. X 2 + 4 XY + Y 2 = ( x cos q - y sin q) 2 + ( x sin q + y cos q) 2
+ 4( x cos q - y sin q)( x sin q + y cos q)
= x + y + 4 { x sin q cos q - y sin q cos q + xy(cos 2 q - sin 2 q)}
2 2 2 2

= x 2 (1 + 4 sin q cos q) + y 2 (1 - 4 sin q cos q) + 4 xy(cos 2 q - sin 2 q)


cos 2 q - sin 2 q = 0
p
Þ q= ( 0 £ q £ p / 2)
4
x 2 + 4 XY + Y 2 = 3 x 2 - y 2
Þ A = 3 and B = -1
12. (A) 2( a + d) = 2( b + c)
1 1 - tan 2 20°
(B) tan 50° = =
tan 40° 2 tan 20°
Þ tan 20° + 2 tan 50° = tan 70°
310 Solution of Advanced Problems in Mathematics for JEE

Þ 2a + 2b = 2c
(D) tan 20°-2 tan 10° = tan 20° tan 2 10° > 0
Þ tan 20° > 2 tan 10°
Þ b > a and d > c
1 1 1
13. (A) (2 sin 75° cos 75° ) = sin 150° =
2 2 4
(B) log 28 7
2 = 2 + log 2 (irrational)

(C) log 53 × log 65 = log 63 = 1 + log 23 (irrational)


3 1
(D) 8 - log 27 = 8 -1/ 3 =
2
1 1
14. a - b = sin x cos x(cos 2 x - sin 2 x) = sin 2 x cos 2 x = sin 4 x
2 4
1
a + b = sin x cos x = sin 2 x
2
15. 2 + 2 + 2 cos 4q = 2 + 2(1 + cos 4q)

= 2 + 4 cos 2 2q
= 2 + 2 |cos 2q|
p 3p
If p < 2q < 3 p / 2 then <q<
2 4
2 + 2 |cos 2q| = 2 - 2 cos 2q = 2 |sin q|= 2 sin q
3p 3p
If < 2q < 2 p then <q< p
2 4
2 + 2 |cos 2q| = 2 + 2 cos 2q = 2 |cos q|= -2 cos q
16. 1 + tan a + tan 2 a = tan 3 a
Þ 1 + tan 2 a = tan a(tan 2 a - 1)
1 1 1
18. a > Þa> ; 1£ £4
6 6 2 2
sin x + cos x 1 - 3 sin x cos x 1 - 3 sin x cos 2 x
2

19. log 10 sin x + log 10 cos x + 2 log 10 cot x + log 10 tan x = -1


log 10 (sin x × cos x × cot x) = k = log 10 cos 2 x = -1
20. tan A + tan B + tan C = tan A tan B tan C
3 6 3 6
+ + tan C = × × tan C
tan C tan C tan C tan C
Þ tan 2 C = 9 Þ tan C = 3
Compound Angles 311

(1 - cot x)
21. = (1 - cot x) × cosec 2 x
2
sin x
= (1 - cot x)(1 + cot 2 x)
1é æ 2p ö 2æ 4p öù
22. f( x) = ê2 sin 2 x + 2 sin 2 ç x + ÷ + 2 sin ç x + ÷
2ë è 3 ø è 3 ø úû

tan x 1 - 3 tan 2 x
23. y = =
tan 3 x 3 - tan 2 x
1-3y
tan 2 x = >0
3-y

24. 2 sin( A - B ) = cos B (sin B - sin 3 B ) - sin B (cos B + cos 3 B )


= -sin B cos B
1 sin 2 B
= - sin 2 B Þ sin( A - B ) = -
2 2 2
1 1 1
25. a > Þa> ; 1£ £4
6 6 2 2
sin x + cos x 1 - 3 sin x cos x 1 - 3 sin x cos 2 x
2

26. 1 + tan a + tan 2 a = tan 3 a


Þ 1 + tan 2 a = tan a(tan 2 a - 1)

Exercise-3 : Comprehension Type Problems

Paragraph for Question Nos. 1 to 3


1. q = 286.5° (IV quadrant) l < 0, m > 0
2. tan( -1042° ) = - tan(1080° - 38° ) = tan 38° < tan 45°
3. q = 4011
. ° (I quadrant) l > 0, m > 0

Paragraph for Question Nos. 4 to 6


æ 2p ö æ 4p ö
a = sin a b = sin ç a + ÷ c = sin ç a + ÷
è 3 ø è 3 ø
æ 2p ö æ 4p ö
p = cos a q = cos ç a + ÷ r = cos ç a + ÷
è 3 ø è 3 ø
312 Solution of Advanced Problems in Mathematics for JEE

æ 2p ö æ 4p ö
4. a + b + c = sin a + sin ç a + ÷ + sin ç a + ÷
è 3 ø è 3 ø
æpö
= sin a + 2 sin(a + p) cos ç ÷ = 0
è3ø
æ 2p ö æ 2p ö æ 4p ö æ 4p ö
5. ab + bc + ac = sin a sin ç a + ÷ + sin ç a + ÷ sin ç a + ÷ + sin a sin ç a + ÷
è 3 ø è 3 ø è 3 ø è 3 ø
1é 2p æ 2p ö 2p 4p æ 4 p ö ù -3
= cos - cos ç 2a + ÷ + cos - cos(2a + 2 p) + cos - cosç 2a + ÷ =
2 êë 3 è 3 ø 3 3 è 3 ø úû 4
æ 2p ö æ 4p ö æ 4p ö æ 2p ö 2p 3
6. qc - rb = cos ç a + ÷ sin ç a + ÷ - cos ç a + ÷ sin ç a + ÷ = sin =
è 3 ø è 3 ø è 3 ø è 3 ø 3 2
Paragraph for Question Nos. 7 to 8
7. tan A = 7 + 4 3 = cot C A

tan A + cot C = 2 7 + 4 3 F 6 + 2
1 E
= 2(2 + 3 ) = 4 + 2 3 2

= 3 +1
B C
æ 2 + 6ö 1 D
æ AC ö 2
8. log AE ç ÷ = log 2 çç ÷ = log
÷ 2
=1
è CD ø è 1+ 3 ø
Paragraph for Question Nos. 9 to 10
9. In a DABC , cot A + cot B + cot C ³ 3 Þ cot q ³ 3
sin 2 A sin q
10. cot q - cot A = cot B + cot C Þ sin( A - q) =
sin B sin C
sin 2 B × sin q sin 2 C sin q
sin( B - q) = and sin(C - q) =
sin A sin C sin A sin B

Paragraph for Question Nos. 11 to 12


x x
cos+ sin
2 2
11. f( x) =
x x
cos - sin
2 2
x x x
- cos + sin 1 - tan
p x 2 2 = 2
12. If < < p Þ f( x) =
2 2 x x x
- cos - sin 1 + tan
2 2 2
Compound Angles 313

Exercise-4 : Matching Type Problems

1. (A) If A + B = 45° then (1 + tan A) (1 + tan B ) = 2


(B) a 2 - 5a £ 6 sin x " x Î R
a 2 - 5a £ -6
a 2 - 5a + 6 £ 0 Þ ( a - 3) ( a - 2) £ 0
4 4 2
æ 1ö æ 4 1 ö æ 1ö æ 2 1 ö
ç a + ÷ - ç a + 4 + 2÷ ç a + ÷ - ç a + 2 ÷ 2
è a ø è a ø è a ø è a ø æ 1ö æ 1 ö
(C) = = ç a + ÷ - ç a2 + ÷ =2
2 2
æ 1ö 2 1 æ 1ö 2 1 è aø è a2 ø
ça + ÷ + a + 2 ça + ÷ + a + 2
è a ø a è a ø a
3
(D) å( x - k) 2 = ( x - 1) 2 + ( x - 2) 2 + ( x - 3) 2 = 0 No real root
k =1

1 - tan 2 ( p / 4 - x)
2. (A) y = = cos( p / 2 - 2 x) = sin 2 x
1 + tan 2 ( p / 4 - x)
æ 5 sin x - 12 cos x + 26 ö
(B) 0 £ log 3 ç ÷£1
è 13 ø
2
æ 1ö 7
(C) y = -2 sin 2 x + cos x + 3 = 2 cos 2 x + cos x + 1 = 2 ç cos x + ÷ +
è 4 ø 8
(D) y = 4 sin 2 q + 4 sin q cos q + cos 2 q = (2 sin q + cos q) 2
2
æ1 ö
4. (A) cos 2 x = ç - sin x ÷
è5 ø
Þ (5 sin x - 4) (5 sin x + 3) = 0
4 3
Þ sin x = or -
5 5
q
(B) cot = 1 + cot q
2
q
Þ 2 cos 2 = cos q + sin q
2
3p p
Þ sin q = 1 Þ q=- ,
2 2
(C) f( x) = - sin 4 x + 8 sin 2 x + 2
Þ f( x) Î [2 , 9]
(2 x 2 + 5 x + 27) æ 1ö
(D) log 2 ³0 çx> ÷
2 2
(2 x - 1) è ø
314 Solution of Advanced Problems in Mathematics for JEE

Þ 2 x 2 - 9 x - 26 £ 0
13
Þ -2 £ x £
2
5. (A) f( x) = -2 sin 2 x + sin x - 6
y min = -9 at sin x = -1
47 1
y max = - at sin x =
8 4
(B) f( x) = 2 cos 2 x + 6
y min = 6; y max = 8
1
(C) f( x) = [4 sin 2 x - 1 + cos 2 x + 3 (1 + cos 2 x)]
2
1
= [2 + 4 sin 2 x + 4 cos 2 x]
2
= 1 + 2 (sin 2 x + cos 2 x)
y max = 1 + 2 2 ; y min = 1 - 2 2
æp ö
(D) f( x) = 2 sin ç + sin x ÷
è4 ø

Exercise-5 : Subjective Type Problems

sin 80° sin 65° sin 35° sin 80° sin 65° sin 80° sin 65° 1
1. = = =
2 sin 35° cos 15°+2 sin 35° cos 35° 2(cos 15°+ cos 35° ) 4 cos 25° cos 10° 4
2. If A + B = 45°
(1 - cot A) (1 - cot B ) = 2
Þ (1 - cot 23° ) (1 - cot 22° ) = 2
3. 4x2 - 7x + 1 = 0
7
tan A + tan B =
4
1
tan A × tan B =
4
tan A + tan B 7
tan( A + B ) = =
1 - tan A + tan B 3
4 sin 2 ( A + B ) - 7 sin( A + B ) cos( A + B ) + cos 2 ( A + B )
4 tan 2 ( A + B ) - 7 tan( A + B ) + 1
= =1
1 + tan 2 ( A + B )
Compound Angles 315

(18 - 2) ´ 180° (n - 2) ´ 180°


4. + + 60° = 360° Þ n = 9
18 n
5. 10 (1 - cos 2a ) 2 + 15 (1 + cos 2a ) 2 = 24
1 3
Þ (5 cos 2a + 1) 2 = 0 Þ cos 2a = - Þ tan 2 a =
5 2
æ 3p p ö æ 3p pö æ 5p 3 p ö æ 5p 3p ö
6. tan ç - ÷ ç tan - tan ÷ + tan ç - ÷ ç tan - tan ÷
è 8 8øè 8 8ø è 8 8 øè 8 8 ø
æ 7 p 5p ö æ 7p 3p ö æ 9p 7p ö æ 9p 7p ö 9p p
+ tan ç - ÷ ç tan - tan ÷ + tan ç - ÷ ç tan - tan ÷ = tan - tan = 0
è 8 8 øè 8 8 ø è 8 8 øè 8 8 ø 8 8
2p p 4 æ cos 2 p + 2 cos 2 p ö sin p 4 æ 1 + 2 cos 2 p ö sin p
cos + 2 cos 2 ç ÷ ç ÷
7 7 = è 7 7ø 7 è 7 ø 7
7. =
p 2p 4p 3p
cos cos sin sin
7 7 7 7
æ 2p ö p
4 ç 1 + 2 cos ÷ sin
è 7 ø 7
= =4
pæ pö
sin ç 3 - 4 sin 2 ÷
7è 7ø
8. a 2 sec 2 200° = c 2 tan 2 200° + d 2 + 2 cd tan 200°
b 2 sec 2 200° = c 2 + d 2 tan 2 200° - 2 cd tan 200°
Þ a 2 + b2 = c 2 + d 2
( a sec 200° - c tan 200° ) 2 = d 2
( b sec 200° + d tan 200° ) 2 = c 2
Þ ( c 2 + d 2 ) (sec 2 200° + tan 2 200° ) + (2 bd - 2 ac) sec 200° tan 200° = c 2 + d 2
Þ ( c 2 + d 2 ) (2 tan 2 200° ) = (2 ac - 2 bd) sec 200° tan 200°
2( c 2 + d 2 ) 2 sec 200° 2 -2
Þ = = =
ac - bd tan 200° sin 200° sin 20°
p 9p 7p 9p 10 p 8p 7p 9p
9. 2 cos cos + cos + cos = cos + cos + cos + cos =0
17 17 17 17 17 17 17 17
cos q - cos 3q + cos 3q - cos 9q + cos 9q - cos 17q cos q - cos 17q
10. = = tan 9q
sin 3q - sin q + sin 9q - sin 3q + sin 17q - sin 9q sin 17q - sin q
11. 8 abc = 8 sin 10° sin 50° sin 70° = 1
a + b sin 10°+ sin 50° 2 sin 30° cos 20°
= = =1
c sin 70° sin 70°
1 1 1 1 1 1 sin 50° sin 70°+ sin 10° sin 70°- sin 10° sin 50°
+ - = + - = =6
a b c sin 10° sin 50° sin 70° sin 10° sin 50° sin 70°
316 Solution of Advanced Problems in Mathematics for JEE

1é æ 2p ö 3æ 4p öù
12. ê 4 sin 3 q + 4 sin 3 ç q + ÷ + 4 sin ç q + ÷
4ë è 3 ø è 3 ø úû
1é ìæ 2p ö æ 4p ö üù
= ê3 sin q - sin 3q + 3 sin íç q + ÷ - sin(3q + 2 p) + 3 sin ç q + ÷ - sin(3q + 4 p)ýú
4ë îè 3 ø è 3 ø þû
1é ì æ 2p ö æ 4p öü ù 3
= ê3 ísin q + sin ç q + ÷ + sin ç q + ÷ ý - 3 sin 3q ú = - sin 3q
4ë î è 3 ø è 3 øþ û 4
n n
sin(2 r - 2 r -1 )
13. å cos 2 r cos 2 r -1 = å(tan 2 r - tan 2 r -1 ) = tan 2 n - tan 1
r =1 r =1

14. x = sec q - tan q, y = cosec q + cot q


1 + cos q 1 - sin q (1 - sin q)(1 + cos q)
y - x - xy = - - =1
sin q cos q sin q × cos q
15. cos 18° - cos 72° = 2 sin 45° sin 27°
= 2 sin 27°
16. 3(sin 1 - cos 1) + 6(sin 1 + cos 1) 2 + 4(sin 6 1 + cos 6 1)
4

= 3(1 - 2 sin 1cos 1) 2 + 6(1 + 2 sin 1cos 1) + 4(1 - 3 sin 2 1cos 2 1)


= 3(1 + 4 sin 2 1cos 2 1 - 4 sin 1cos 1) + 10 + 12 sin 1cos 1 - 12 sin 2 1cos 2 1
= 13
2 33
17. 3 sin 2 x + 2 cos x
+ 2
= 28
3 sin 2 x + 2 cos x
2
Let 3 sin 2 x + 2 cos x
= t, t 2 - 28t + 27 = 0 Þ t = 1, 27
If t = 1 Þ sin 2 x + 2 cos 2 x = 0
p 3p 7p
2 cos x(sin x + cos x) = 0 Þ x= , ,
2 4 4
If t = 27
Þ sin 2 x + 2 cos 2 x = 3 (Not possible)
p 3p 7p
(sin 2a - cos 2a ) 2 + 8 sin 4a = 1 + 7 sin 4a = 1 (at a = , , )
2 4 4
18. (sin q + cosec q) 2 + (cos q + sec q) 2
= 5 + cosec 2 q + sec 2 q
= 7 + tan 2 q + cot 2 q
³9
19. tan 20° + tan 40° + tan 80° - tan 60°
sin 20° cos 80° + sin 80° cos 20° sin 40° cos 60° – sin 60° cos 40°
= +
cos 20° cos 80° cos 40° cos 60°
Compound Angles 317

sin 100° sin 20°


= -
cos 20° cos 80° cos 40° cos 60°
sin 80° 2 sin 20°
= -
cos 20° cos 80° cos 40°
sin 80° cos 40° - sin 40° cos 80° sin 40°
= =
cos 20° cos 40° cos 80° cos 20° cos 40° cos 80°
8 sin 40° sin 20°
= = 8 sin 40°
sin(8 ´ 20° )
20. 1 + cos 10 x cos 6 x = 2 cos 2 8 x + sin 2 8 x
2 + cos 16 x + cos 4 x = 2(1 + cos 16 x) + 1 - cos 16 x
Þ cos 4 x = 1
np
x= (n = 0 , ± 1, ± 2 , ± 3 .......)
2
If 360° < k < 540°
Þ k = 450° (n = 5)
21. cos 20°+2 sin 2 55° = 1 + 2 sin k°
= cos 20° + 1 - cos 110°
= 1 + cos 20° + sin 20°
= 1 + 2 sin( 45°+20° )
Þ k = 65
cos 96°+ cos 6° 2 cos 51° cos 45°
23. tan 19 x = =- = - cot 51° = tan 141°
cos 96°- cos 6° 2 sin 51° sin 45°
Þ 19 x = 180° n + 141
2 sin 40° + sin 20° 2 sin(60° - 20° ) + sin 20°
24. =
cos 20° cos 30° cos 20° cos 30°
3p
sin
2p 4p 6p 7 × cos 4 p - 1 = -3
25. cos + cos + cos -1=
7 7 7 p 7 2
sin
7
k 11
26. (cos 2 A - cos 3 A) =
2 8
k 11
[2 cos 2 A - 1 - 4 cos 3 A + 3 cos A] =
2 8
Þ k=4
27. 3 sin 2 x + 4 cos 2 x = 3 + cos 2 x
28. tan a + tan b = 12
tan a × tan b = -3
318 Solution of Advanced Problems in Mathematics for JEE

tan a + tan b
tan(a + b) = =3
1 - tan a tan b
cos 24° cos 33° æ sin 18° cos 9° ö
29. +ç - cos 18° ÷
2 sin 9°
2 sin 33° sin 57° è ø
cos 24° cos 33° sin 9°
+ =2
sin 57° cos 24° sin 9°
æ 1 + cos 2q ö æ 1 + cos 4q ö æ 1 + cos 8q ö
30. tan q ç ÷ç ÷ç ÷
è cos 2q ø è cos 4q ø è cos 8q ø
sin q æç 2 cos 2 q ö÷ æç 2 cos 2 2q ö÷ æç 2 cos 2 4q ö÷ 8 sin q cos q cos 2q cos 4q sin 8q
= = = tan 8q
cos q çè cos 2q ÷ø çè cos 4q ÷ø çè cos 8q ÷ø cos 8q cos 8q

31. y = sin 2 x + cos 2 x + tan 2 x + cot 2 x + cosec 2 x + sec 2 x + 6


y = 9 + 2(tan 2 x + cot 2 x) ³ 13

❑❑❑
Trigonometric Equations 319

23
TRIGONOMETRIC EQUATIONS

Exercise-1 : Single Choice Problems

5a
1. tan 2 x - sec 2 y = - 3 = -2 - a 2 Þ 6 a 2 + 5a - 6 = 0
6
2. [tan( x + y) - cot( x + y)]2 + ( x + 1) 2 = 0
Þ x = -1 and tan 2 ( x + y) = 1
p
x + y = np ±
4
3. sin x + cos x = 1
æ pö 1
sin ç x + ÷ =
è 4ø 2
p p
x = np + ( -1) n -
4 4
4. sin 2 (sin x) - 3 sin(sin x) + 2 = 0
{sin(sin x) - 2}{sin(sin x) - 1} = 0
Equation has no solution.
5. tan 2 x = tan 6 x Þ sin 4 x = 0
4 x = p , 2 p , 3 p , ¼¼ , 11p
p 2p 3p 11p
x= , , , ¼¼ ,
4 4 4 4
p 3 p 5p 7 p 9 p 11p
But , , , , , are rejected. So number of solutions = 5.
4 4 4 4 4 4
6. 3 sin 2 x - 6 sin x - sin x + 2 = 0
(3 sin x - 1)(sin x - 2) = 0
sin x ¹ 2 , then
1
sin x =
3
320 Solution of Advanced Problems in Mathematics for JEE

1
sin x = has 6 solutions for x Î[0 , 5p]
3
7. cos q + cos 2q = -1
Þ 2 cos 2 q + cos q = 0
1
Þ cos q = 0 or cos q = -
2
p 2p 4p 3p
Þ q= , , ,
2 3 3 2

1
1/2
8. p 2p
x
0 p/2 3p/2

1
in [0 , 2p) max. (sin x , cos x) = has two solutions.
2
9. (cot 2 x + 2 3 cot x + 3) + (cot 2 x + 1) + ( 4cosec x + 4) = 0
(cot x + 3 ) 2 + (cosec x + 2) 2 = 0
Þ cot x = - 3 and cosec x = -2
2 2
10. sin x = sin 3 x
Þ 3 x = np ± x
np np np
x= , x= , hence general solution is .
4 2 4
11. sin x > 0
Þ 8 sin 2 x cos 2 x = 1
Þ 2 sin 2 2 x = 1
Þ cos 4 x = 0
p
x = (2n + 1) (n Î I )
8
12. cos x + cos 2 x + cos 3 x + cos 4 x + cos 5 x = 5
Þ cos x = 1 Ç cos 2 x = 1 Ç cos 3 x = 1 Ç cos 4 x = 1 Ç cos 5 x = 1
2np 2np 2np
x = 2np Ç x = np Ç x = Çx= Çx=
3 4 5
Þ x = 2np
13. (2 sin x - cosec x) 2 + (tan x - cot x) 2 = 0
1 p
Þ sin 2 x = Ç tan 2 x = 1 Þ x = np ±
2 4
Trigonometric Equations 321

14. cos 3 3 x + cos 3 5 x = (2 cos 4 x cos x) 3


cos 3 3 x + cos 3 5 x = (cos 5 x + cos 3 x) 3
Þ 3 cos 5 x cos 3 x(cos 5 x + cos 3 x) = 0
Þ cos 5 x cos 3 x × cos 4 x cos x = 0
15. sin 100 x = 1 + cos 100 x Þ sin 100 x = 1 and cos 300 x = 0
p
16. sin q £ 1 and sec 2 4q ³ 1 Þ sin q = sec 4q = 1; q =
2
17. ( 4 sin 2 x + cosec 2 x) + (tan 2 x + cot 2 x) = 6
(2 sin x - cosec x) 2 + (tan x - cot x) 2 = 0
Þ 2 sin x = cosec x and tan x = cot x
4 2
18. sin q - 2 sin q + 1 = 2
(sin 2 q - 1) 2 = 2 = cos 4 q (not possible)
19. cos( P sin x) = sin( P cos x)
æp ö
cos( P sin x) = cos ç - P cos x ÷
è 2 ø
p
P sin x + P cos x = 2np +
2
p
P sin x - P cos x = 2np -
2
20. | x| + | y| = 2 –2 £ x £ 2
æ px 2 ö –2 £ y £ 2
sin ç ÷ =1
ç 3 ÷
è ø
px 2 p
=
3 2
3
x =±
2
æ p ö
21. x Î ç - , p ÷
è 2 ø
cos 2 x > |sin x|
sin x ³ 0 sin x < 0
2
1 - 2 sin x - sin x > 0
2
2 sin x + 2 sin x - sin x - 1 < 0 2 sin 2 x - 2 sin x + sin x - 1 < 0
(2 sin x - 1)(sin x + 1) < 0 (2 sin x + 1)(sin x - 1) < 0

–1 0 1/2 –1/2 0 1
322 Solution of Advanced Problems in Mathematics for JEE

1 1
- < sin x £
2 2
æ p p ö æ 5p ö
ç- , ÷ Èç , p÷
è 6 6ø è 6 ø
22. sin 4 x + cos 4 x = sin x cos x
1 - 2 sin 2 x cos 1 x = sin x cos x
2y2 + y -1=0
(2 y - 1)( y + 1) = 0
1
y= y = -1
2
2 sin x cos x = 1 sin x cos x ¹ -1
sin 2 x = 1
p 5p
2x = ,
2 2
p 5p
x= ,
4 4
5x x
23. sin = 1 Ç sin = -1
2 2
1 1
24. cos 2q = sin 2 q Þ sin 2 q = Þ sin q = ±
3 3
25. b sin q = -c - a cos q
b 2 (1 - cos 2q) = c 2 + a 2 cos 2q - 2 ac cos q
Þ ( a 2 + b 2 ) cos 2q - 2 ac cos q + ( c 2 - b 2 ) = 0
c 2 - b2
cos a × cos b = …(1)
a 2 + b2
a 2 (1 - sin 2q) = c 2 + b 2 sin 2q - 2 bc sin q
( a 2 + b 2 ) sin 2q - 2 bc sin q + ( c 2 - a 2 ) = 0
c2 - a2
sin a × sin b = …(2)
a 2 + b2
a 2 - b2
cos(a + b) = cos a cos b - sin a sin b =
a 2 + b2
Trigonometric Equations 323

Exercise-2 : One or More than One Answer is/are Correct

1. 2 cos 2 q + 2 2 cos q - 3 = 0
1 -3
( 2 cos q + 1) 2 = 4 Þ cos q = or (Not possible)
2 2
3. 4 sin 3 x + 5 ³ 4 cos 2 x + 5 sin x
Þ (sin x - 1)( 4 sin x + 1) 2 £ 0 " x Î R
4. 4 cos x (2 - 3 sin 2 x) + cos 2 x + 1 = 0
cos x (3 cos x + 2)(2 cos x - 1) = 0
p
Least difference =
6
5. cos x cos 6 x = - 1
Case-1 : cos x = 1 and cos 6 x = - 1
Not possible
Case-2 : cos 6 x = 1 and cos x = -1
Þ x = (2n - 1) p , (n Î I )
7. 2 k = sin 2 2 x - 2 sin 2 x - 2
Let sin 2 x = t t Î [-1, 1]
é 3 1ù
2 k = t 2 - 2t - 2 Þ k Î ê- , ú
ë 2 2û
æ pöæ 3p ö æ 5p ö æ 7p ö 4 2 1
8. f(q) = ç cos q - cos ÷ ç cos q - cos ÷ ç cos q - cos ÷ ç cos q - cos ÷ = cos q - cos q +
è 8 øè 8 øè 8 øè 8 ø 8
4 sin 2 x cos 2 x + 4 sin 4 x - 4 sin 2 x cos 2 x 1
9. = tan 4 x =
2 2 2 9
4 cos x - 4 sin x cos x
1
Þ tan x = ±
3
1
10. tan q(1 - sin 2 q) + cot q(1 - cos 2 q) + 1 + sin 2q = 0 Þ sin 2q = -
2
æ1 3 ö
11. 2 çç sin q + cos q ÷÷ = - ( x - 3) 2 - 2
è2 2 ø
324 Solution of Advanced Problems in Mathematics for JEE

Exercise-3 : Comprehension Type Problems

Paragraph for Question Nos. 1 to 3


2 2
1. h( x) = f ( x) + g ( x) = 2 + 2 sin 4 x
Þ 8 cos 4 x ³ 0
Þ cos 4 x ³ 0
p
Longest interval =
4
2. 2 + 2 sin 4 x = 4
Þ sin 4 x = 1
p
Þ x = ( 4n + 1)
8
3. sin 3 x + cos x = cos 3 x + sin x
Þ sin 3 x - sin x = cos 3 x - cos x
Þ 2 sin x cos 2 x = -2 sin 2 x sin x
Þ sin x = 0 or tan 2 x = -1
3p 7p
Þ x = 0, p or x= ,
8 8

Exercise-4 : Matching Type Problems


2
2 æ1 ö
1. (A) cos x = ç - sin x ÷
è5 ø
Þ (5 sin x - 4)(5 sin x + 3) = 0
4 3
Þ sin x = or -
5 5
q
(B) cot = 1 + cot q
2
q
Þ 2 cos 2 = cos q + sin q
2
3p p
Þ sin q = 1 Þ q = - ,
2 2
(C) f( x) = - sin 4 x + 8 sin 2 x + 2
Þ f( x) Î [2 , 9]
(2 x 2 + 5 x + 27) æ 1ö
(D) log 2 ³0 çx> ÷
2 2ø
(2 x - 1) è
Þ 2 x 2 - 9 x - 26 £ 0
Trigonometric Equations 325

13
Þ -2 £ x £
2
2. (A) sin x = 1, cos y = 1 or sin x = -1, cos y = -1
(B) f ¢( x) = cos x + sin x - K
Þ k³ 2
(C) | x - 1| £ 1 and |2 x 2 - 5| £ 1
2

Þ x2 =2
(D) sin x + sin y = sin( x + y)
æ x + yö æ x - yö æ x + yö æ x + yö
Þ 2 sin ç ÷ cos ç ÷ = 2 sin ç ÷ cos ç ÷
è 2 ø è 2 ø è 2 ø è 2 ø
æ x + yö æ x - yö æ x + yö
Þ sin ç ÷ = 0 or cos ç ÷ = cos ç ÷
è 2 ø è 2 ø è 2 ø
np np
Þ x + y = 2np or x= , y=
2 2
if x = 0, y = ± 1
1 1
if x = , y =-
2 2
1 1
if x =- , y =
2 2
if y = 0, x = ± 1

Exercise-5 : Subjective Type Problems

1. Let sin x - 1 = a , cos x - 1 = b, sin x = c


Þ a + b 3 + c 3 = ( a + b + c) 3
3

Þ a + b = 0 or b + c = 0 or c + a = 0
1
sin x + cos x = 2 or sin x + cos x = 1 or sin x =
2
Þ Total solution = 5
2. sin y - 2014 cos y = 1
p
Þ y=
2
2 sin 6 x
3. <0
sin x - 1
Þ sin 6 x > 0
æ pö æp pö
Þ x Î ç 0, ÷ È ç , ÷
è 6ø è3 2ø
326 Solution of Advanced Problems in Mathematics for JEE

1 + tan 2 x - 2 2 tan x £ 0
é p 3p ù é p p ö æ p 3p ù
Þ xÎê , ú Þ xÎê , ÷ Èç ,
ë8 8 û ë 8 6 ø è 8 8 úû
4. sin 4 x - 4 sin 2 x + (2 + k) = 0
Let sin 2 x = t t Î[0 , 1]
t 2 - 4t + (2 + k) = 0
f(0) f(1) £ 0
( k + 2)( k - 1) £ 0 Þ - 2 £ k £ 1
5.
1

–p/2 –p/6 p/6


–p/3 O p/3 p/2

–1

6. 2 sin 2 x + sin 2 2 x = 2
2 sin 4 x - 3 sin 2 x + 1 = 0 Þ (2 sin 2 x - 1)(sin 2 x - 1) = 0
sin 2 x + cos 2 x = tan x
2 tan x + 1 - tan 2 x = tan x(1 + tan 2 x)
Þ tan 3 x + tan 2 x - tan x - 1 = 0 Þ (1 + tan x)(tan 2 x - 1) = 0
2 cos 2 x + sin x £ 2
2 sin 2 x - sin x ³ 0
sin x (2 sin x - 1) ³ 0
7. (3 cot q + 1)(cot q + 3) = 0
1
cot q = - and cot q = -3
3
p p
q = a, p + a q = - a, p + - a
2 2
8. (8 cos 4q - 3)(cot q - tan q) 2 = 12
æ 4 cos 2 2q ö
8 (2 cos 2 2q - 1) - 3 ç ÷ = 12
ç sin 2 2q ÷
è ø
16 cos 4 2q - 8 cos 2 2q - 3 = 0
Þ ( 4 cos 2 2q - 3)( 4 cos 2 2q + 1) = 0
3
Þ cos 2q = ±
2
Trigonometric Equations 327

9. 2 sin 2 x + 4 sin 2 x cos 2 x = 2


2 sin 4 x - 3 sin 2 x + 1 = 0 Þ (sin 2 x - 1)(2 sin 2 x - 1) = 0
1
sin x = ± ,±1
2
sin 2 x + cos 2 x = tan x
2 tan x 1 - tan 2 x
+ = tan x
1 + tan 2 x 1 + tan 2 x
Þ tan 3 x + tan 2 x - tan x - 1 = 0
(tan x + 1) 2 (tan x - 1) = 0
2 cos 2 x + sin x £ 2
2 sin 2 x - sin x ³ 0
sin x(2 sin x - 1) ³ 0
❑❑❑
328 Solution of Advanced Problems in Mathematics for JEE

24

Exercise-1 : Single Choice Problems

cot A + cot B cos A sin B + cos B sin A sin 2 C c2


1. = = =
cot C
(sin A sin B )
cos C (sin A sin B ) cos C a 2 + b2 - c 2
ab ×
sin C 2 ab
2c 2 18
= =
æ 17 ö 2 8
ç - 1÷ c
è 9 ø
p æp - Aö
2. ÐBIC = +ç ÷ A
2 è 2 ø
p æB +Cö
= +ç ÷
2 è 2 ø H
I
B C
1
3. [(2 R cos A) 2 + a 2 ][(2 R cos B ) 2 + b 2 ][(2 R cos C ) 2 + c 2 ]
64
1
[(2 R cos A) 2 + (2 R sin A) 2 ][(2 R cos B ) 2 + (2 R sin B ) 2 ][(2 R cos C ) 2 + (2 R sin C ) 2 ]
64
=R6
4. B = 60°
1
2 sin 2 B = 3 sin 2 C Þ sin C = Þ C = 45°
2
A C 1
5. tan tan =
2 2 3
s-b 1 2 a+c
= Þ b= sÞ = bÞ b³ 2 (A.M. ³ G.M.)
s 3 3 2
a
6. cos A cos B cos C å
cos A
= 2 R cos A cos B cos C å
tan A = 2 R cos A × cos B × cos C × Õ(tan A)
Solution of Triangles 329

sin A sin B sin C


= 2 R cos A × cos B × cos C × = 2 R sin A sin B sin C
cos A cos B cos C
BD AD
8. In DBAD, =
sin q sin 60°
CD AD
In DCAD, = A
sin(75°-q) sin 45°
BD CD sin 45° q
Þ sin 60° = 75°–q
sin q sin(75°- q)
sin q BD sin 60° 1
Þ = = 60° 60°+q 45°
sin(75°-q) CD sin 45° 6 B C
1 D 3
2 bc A
9. Length of angle bisector AD = cos
b+ c 2
2 ac B
Length of angle bisector BE = cos
a+c 2
2 ab C
Length of angle bisector CF = cos
a+b 2
3 3
H. M. = =
b+ c a+ c a+ b 1 1 1
+ + + +
2 bc 2 ac 2 ab a b c
10. 2b = a + c
2 sin B = sin A + sin C
æ B Bö æ A+Cö æ A -C ö B 1
2 ç 2 sin cos ÷ = 2 sin ç ÷ cos ç ÷ Þ sin =
è 2 2 ø è 2 ø è 2 ø 2 2 2
æ B - C ö b + c sin B + sin C
11. 2 cos ç ÷= =
è 2 ø a sin A
A 1
Þ sin = Þ Ð A = 60°
2 2
4 + c2 - 1 1 æ 3ö 3
12. cos A = = çc + ÷³
4c 4è cø 2
A
I3 I2
60° 60°
60°

F F
60° 60°

60°
60° 60°
B D C
13.

60°
I1
330 Solution of Advanced Problems in Mathematics for JEE

If DABC is an equilateral triangle then DDEF and D I 1 I 2 I 3 are also equilateral triangle
3
Side of DDEF = 1 unit Þ Ar( DDEF ) =
4
1
2x ×
14. AD = x cos p = 1 A
1 3 1
x+ x+ 120°
x x X 1/X

1
AD max =
2 B C
3a 3a 3a
15. r = ,R = , r1 = Þ r , R , r1 are in A.P.
6 3 2
16. sin( B + C ) sin( B - C ) = sin( A + B ) sin( A - B )
sin 2 B - sin 2 C = sin 2 A - sin 2 B
Þ 2 sin 2 B = sin 2 A + sin 2 C
Þ 2b2 = a 2 + c 2 (Using sine rule)
tan A + tan B
17. tan( A + B ) = = tan( p - C )
1 - tan A tan B
7 7
Þ tan C = Þ sin C =
4 65
Using sine rule
c 65
R= =
2 sin C 14
cos A cos B cos C
18. + +
a b c
b 2 + c 2 - a 2 a 2 + c 2 - b 2 a 2 + b 2 - c 2 a 2 + b 2 + c 2 ( a + b + c) 2 - 2( ab + bc + ac)
+ + = =
2 abc 2 abc 2 abc 2 abc 2 abc
a + c b + c a 2 + b 2 + ac + bc c 2 ( a + b + c) c 2 (2 s) 2 R c
19. + = = = = =
b a ab abc 4 RD r r
20. a 2 (sin B - 1) = b 2 + c 2 - a 2 = 2 bc cos A Þ cos A < 0
a a
21. 2 R ¢ = = = 2R C
sin( p - A) sin A
Þ R¢ = R
O

A B
Solution of Triangles 331

22. a = d1 d 2
d1 d
= a cos q , 2 = a sin q
2 2
1 = 4 sin q cos q
1
Þ sin 2q = Þ 2q = 30° q
2 a

23. Q (m + n) cot q = m cot a - n cot b


\ (2 + 1) cot q = 2 cot a - cot b A
1 æp ö a b
Put cot q = ,cot b = cot ç - a ÷ = tan a
3 è 2 ø
2
We have 1 = - tan a Þ tan 2 a + tan a - 2 = 0
tan a q
C
B
\ tan a = 1 \ a = 45° D
2:1
l l
24. Circumradius of equilateral D , R = =
2 sin 60° 3
l 2 2l 2
Diagonal of square = 2 R Þ a 2 = 2 R \ a = R 2 = \ Area of square =
3 3
2 2 2
2 + ( 6 ) - ( 3 + 1) 3 -1
25. cos q = =
4 6 2 2
26. If a , b, c are in A.P.
B 1
Þ 2 sin B = sin A + sin C Þ sin =
2 4
B
6 cos
s 2
= = 3 15
r B
1 - 2 sin
2
æ A-Bö
1 - tan 2 ç ÷
27. cos( A - B ) = è 2 ø = 31 Þ tan æ A - B ö = 1
ç ÷
æ A - B ö 32 è 2 ø 3 7
1 + tan 2 ç ÷
è 2 ø
æ A-Bö a-b C 1
tan ç ÷= cot Þ cos C =
è 2 ø a+b 2 8
a 2 + b2 - c 2 1
cos C = = Þ c =6
2 ab 8
28. ( b + c) cos( B + C ) + ( c + a) cos(C + A) + ( a + b) cos( A + B )
= -[( b cos A + a cos B ) + ( c cos A + a cos C ) + ( c cos B + b cos C )] = -[a + b + c] = -30
332 Solution of Advanced Problems in Mathematics for JEE

p 2p 4p
30. Ð A = ,Ð B = ,ÐC =
7 7 7
æ b 2 ö÷ æç c 2 ö÷ æç a 2 ö÷
( a 2 - b 2 )( b 2 - c 2 )( c 2 - a 2 ) = a 2 b 2 c 2 ç 1 - 1- 1-
ç a 2 ÷ø çè b 2 ÷ø çè c 2 ÷ø
è
æ 2p ö æ 4p ö æ p ö
ç sin 2 ÷ç sin 2 ÷ç sin 2 ÷
2 2 2ç
= a b c 1- 7 ÷ç1 - 7 ÷ç1 - 7 ÷ = a 2 b2 c 2
ç p ÷ç 2 p ÷ç 4 p ÷
ç sin 2 ÷ç sin 2 ÷ç sin 2 ÷
è 7 øè 7 øè 7 ø
a
36. h =
2 3
Ar( DABC ) = Ar ( DAPB ) + Ar( DBPC ) + Ar( DAPC )
3 2 1 a
a = a( h + h1 + h2 ) Þ h1 + h2 =
4 2 3
2 2 2
6 +7 -x
37. cos 60° = Þ x = 43
2 ´6 ´7
2 ab p
39. CD = cos B
a+b 3
ab D
= a
a+b
p/3
42. a + b + c = 48 p/3
A
C b
a = 20
b + c = 28
Þ a + b > c, a + c > b
Þ 20 + b > 28 - b, 20 + c > 28 - c
Þ b> 4 ,c > 4
43. In an equilateral triangle
a=b=c
( a + b + c)( b + c - a)( c + a - b)( a + b - c)
44.
4b2 c 2
2 s(2 s - 2 a)(2 s - 2 b)(2 s - 2 c) æ 2( s - a) ö æ ( s - b)( s - c) ö 2 A A
= = 4ç ÷ç ÷ = 4 sin cos 2 = sin 2 A
2 2 bc bc 2 2
4b c è øè ø
45. R = 4 r A
æ A B Cö q
R = 4 ç 4 R sin sin sin ÷
è 2 2 2ø
q
1 = 16 sin 2 × cos q = 8 (1 - cos q) cos q B
q p-2q
C
2
Solution of Triangles 333

46. DDMN ~
= DAMN Þ DM = AM
C

N
A B
a b c
47. b c a = - ( a 3 + b 2 + c 3 - 3 abc)
c a b
= - ( a + b + c)( a 2 + b 2 + c 2 - ab - bc - ac)
p 2p 4p
48. A = ,B = ,C =
7 7 7
æ 2 öæ 2 öæ 2 ö
ç1 - b ÷ç1 - c ÷ç1 - a ÷ = l
ç a 2 ÷ø çè b 2 ÷ø çè c 2 ÷ø
è
æ 2p ö æ 4p ö æ p ö
ç sin 2 ÷ç sin 2 ÷ç sin 2 ÷
ç1 - 7 ÷ ç1 - 7 ÷ ç1 - 7 ÷ =l
ç p ÷ç 2p ÷ ç 4p ÷
ç sin 2 ÷ç sin 2 ÷ç sin 2 ÷
è 7 øè 7 øè 7 ø
æ 2 p 2p ö æ 2 2p 4p ö æ 2 4p pö
ç sin - sin 2 ÷ ç sin - sin 2 ÷ ç sin - sin 2 ÷
ç 7 7 ÷ç 7 7 ÷ç 7 7 ÷ =l
ç 2 p ÷ ç 2 2p ÷ ç 2 4p ÷
ç sin ÷ç sin ÷ç sin ÷
è 7 øè 7 øè 7 ø
Þ l = 1(sin 2 A - sin 2 B = sin( A - B ) × sin( A + B ))
D D D
49. r1 = , r2 = , r3 =
s-a s-b s-c
r1 r2 r3 s3
=
r3 ( s - a)( s - b)( s - c)
s-a s-b s-c
+ +
s s s ³ æ s - a öæ s - böæ s - c ö
ç ÷ç ÷ç ÷
3 è s øè s øè s ø
3
50. sin A = B
5
4
sin B = 3
5
5
sin C = 1 90°
C A
4
334 Solution of Advanced Problems in Mathematics for JEE

A B C A B C
4 R sin cos cos + 4 R cos sin cos
r + r2 2 2 2 2 2 2
51. 1 =
1 + cos C C
2 cos 2
2
æ A B A Bö
2 R ç sin cos + cos sin ÷
è 2 2 2 2ø
= = 2R
C
cos
2
sin 2 a + cos 2 a - (1 + sin a cos a ) 1
53. cos q = =
2 sin a cos a 2
55. Since we need to compute the radius of an escribed circle, we would be needing the length of
all the sides of the given triangle ABC.
From the question, we already know AB = AC = 5.
For finding the length of side BC , let us draw a line AD which is the bisector of angle BAC , as
shown in the figure below.
ÐBAD = ÐDAC = 15° y

BD BD 3 -1 C
Therefore, sin 15° = = and sin 15° =
AB 5 2 2
B
5 ( 3 - 1)
Therefore, BD = 5 sin 15° =
2 2
x
We also know that BC = 2 BD A N
5 ( 3 - 1)
Therefore, BC =
2
Now, we know that the required radius
æ A ö æ AB + BC + CA ö æ Aö
r1 = s tan ç ÷ = ç ÷ tan ç ÷
2
è ø è 2 ø è2ø
æ 5 ( 3 - 1) ö
ç5+ + 5÷
ç 2 ÷ æ 10 2 + 5 3 - 5 ö
=ç ÷ (tan 15° ) = çç ÷ (2 - 3 )
2 2 2 ÷
çç ÷÷ è ø
è ø
a
56. ED = BE - BD = - C cos B A
2
a æ a 2 + c 2 - b2 ö
= -Cç ÷
2 ç 2 ac ÷
è ø
b2 - c 2 C B
= E D
2a
Solution of Triangles 335

57. 2R(sin A cos B cos C + cos A sin B cos C + cos A cos B sin C )
= 2R(sin( A + B ) cos C + cos A cos B sin C )
abc D rs
= R(2 sin A sin B sin C ) = = =
4R 2 R R
b cos A
58. In DAFE , = 2R 1 A
sin B
Þ R 1 = R cos A
F E
Similarly, R 2 = R cos B
and R 3 = R cos C
3 B C
R 1 + R 2 + R 3 = R (cos A + cos B + cos C ) £ R D
2
59. Ar ( DABC ) = Ar ( DOAB ) + Ar ( DOBC ) + Ar ( DOAC )
1
8 = R 2 (sin a + sin b + sin g)
2
4p æ 2 20 ö
Þ sin a + sin b + sin g = çQ R = ÷
5 è p ø

Exercise-2 : One or More than One Answer is/are Correct

1. x 2 - r( r1 r2 + r2 r3 + r1 r3 ) x + ( r1 r2 r3 - 1) = 0
x 2 - ( r1 r2 r3 ) x + ( r1 r2 r3 - 1) = 0
Þ Roots are 1 and r1 r2 r3 - 1
A

60°

2. c b

90° 30°
a C
B=H
s
3. R = 2 r , r = ( s - a) tan 30° = tan 30° Þ s is irrational Þ D is irrational
3
r1 = s tan 30° = 3 r (rational)
p
4. D + E + F =
2
5. a = 4 , b = 8 , Ð C = 60°
1 a 2 + b2 - c 2
cos C = = Þ c=4 3
2 2 ab
336 Solution of Advanced Problems in Mathematics for JEE

r r2 s-a s-c
6. If = Þ =
r1 r3 s s-b
Þ a 2 + b2 = c 2
Þ Ð C = 90°
7. Ð BOC = 2 Ð A
Ð BIC = p 2 + A 2
Ð BHC = p - A
8. 3x2 - 4x + 3 < 0
1
Þ ( 3 x - 1)( x - 3 ) < 0 Þ < x< 3
3
30° < A , B < 60° Þ 60° < C < 120°
2
9. cos 2q = 2 cos q - 1
1 p
= 2 cos 2 -1
2 8
p 1
2 cos 2 = 1 +
8 2
p 2 +1
cos 2 =
8 2 2
a 2 + b2 - c 2
cos C = then solve it
2 ab
11. (3 sin A + 4 cos B ) 2 + ( 4 sin B + 3 cos A) 2 = 37 ; 9 + 16 + 24 sin( A + B ) = 37
b+ c 2
12. =
a 1
A

2 b+c
1 a
B C
13. Ð A , ÐB , ÐC A.P. Þ ÐB = 60° B
2 2 2
a + 10 - 9 60°
cos 60° = 10
20 a

60°+q 60°–q A
C
9
1
14. D = ab sin C
2
a+b sin A + sin B
³ ab Þ ³ sin A ´ sin B
2 2
Solution of Triangles 337

15. 3 cos A = cos( B - C ) - cos( B + C ) Þ 2 cos A = cos B - C ) = - cos( A + 2C )


2 = (tan A sin 2C - cos 2C )

Exercise-3 : Comprehension Type Problems

Paragraph for Question Nos. 2


æp Aö æp Bö æp Cö
2. r ¢ = 4 r sin ç - ÷ sin ç - ÷ sin ç - ÷ A
è 4 4 ø è 4 4 ø è4 4ø
r¢ æp Aö æp Bö æp Cö
= 4 sin ç - ÷ sin ç - ÷ sin ç - ÷
r è4 4ø è4 4ø è4 4ø
F E
A B C p–C p–B
= sin + sin + sin - 1 2 2 2 2
2 2 2
p–A
æp Aö æp Bö æp Cö 2 2
r1¢ = 4 r sin ç - ÷ cos ç - ÷ cos ç - ÷
è 4 4 ø è 4 4 ø è4 4ø
B D C
r1¢ æp Aö æp Bö æp Cö
= 4 sin ç - ÷ cos ç - ÷ cos ç - ÷
r è4 4ø è4 4ø è4 4ø
A B C
= 1 - sin + sin + sin
2 2 2

Paragraph for Question Nos. 3 to 4


3. Ar ( DDEF ) A
æp Aö æp Bö æp Cö
= 2 R 2 sin ç - ÷ sin ç - ÷ sin ç - ÷ F E
è 2 2 ø è 2 2 ø è2 2ø
I
A B C
= 2 R 2 cos cos cos
2 2 2 B/2 C/2

Ar( DABC ) 2 R 2 sin A sin B sin C A B C B C


B/2
C/2

4. = = 8 sin sin sin £ 1


Ar( DDEF ) A B C 2 2 2
2 R 2 cos cos cos
2 2 2 D

Paragraph for Question Nos. 5 to 6


Sol. c 2 = R Þ c = 82
2r = a + b - c
Þ a + b = 98 ...(1)
2 2 2 2
a + b = c = (82) ...(2)
Þ a = 18 , b = 80
338 Solution of Advanced Problems in Mathematics for JEE

Paragraph for Question Nos. 7 to 8


p A
Sol. Ð A1 = -
2 2
p 1 p 1æ p Aö
Ð A2 = - ( Ð A1 ) = - ç - ÷ A
2 2 2 2è2 2 ø
p A
= + C1 B1
4 4
p 1
Ð A3 = - (Ð A2 )
2 2 B
A1
C
3p A
= -
8 8
n
pæ 1 1 1 ö ( -1) A
Ð An = ç 1 - + - + ........ ÷ +
2è 2 4 8 ø 2n

Paragraph for Question Nos. 9 to 10


1
´ BD ´ h1
D1 2 h h
Sol. = = (1 - x) 1 ´ 3 = (1 - x) yz A
D 1 h2 h3
´ AB ´ h2
2 h3 E
h1
1 F
´ EC ´ h4 D
D2 2
= = x(1 - y)(1 - z )
D 1
´ AC ´ h5 h2
2
B C

Paragraph for Question Nos. 11 to 13


æ cö
Sol. log ç 1 + ÷ + log a - log b = log 2
è a ø
Þ a + c = 2b
( c - a) x 2 + 2 bx + ( a + c) = 0 has equal roots, then
a 2 + b2 = c 2

Paragraph for Question Nos. 14 to 16


BE ED EC ED
Sol. = , =
sin C A sin B A
sin sin
2 2
Solution of Triangles 339

ED
Þ BE + EC = a = (sin B + sin C )
A
sin
2
A
a sin ´ 2R
Þ ED = 2
b+ c
2 bc A
la = cos A
b+ c 2
æ A ö A/
ç 2 bc a sin ´ 2 R ÷ A/2 2
A 2
ç cos + ÷
çb+ c 2 b+ c ÷
ç ÷
è ø
2 sin B sin C sin B sin C E
B C
Þ la = = A/2 A/2
A æ Aö
2 sin B sin C + 2 sin 2 sin 2 ç B + ÷
2 è 2 ø C B
D

Exercise-4 : Matching Type Problems

2. (A) 3 0 {2 0 + 2 -1 + 2 -2 ¼¼¼¥} = 1{2}


1
3 -1 {2 0 + 2 -1 + 2 -2 ¼¼¼¥} = {2}
3
1
3 -2 {2 0 + 2 -1 + 2 -2 ¼¼¼¥} = {2}
3
M M
M M
¥
2 ´1
Hence, =3
1
1-
3
(B) b 2 + c 2 - a 2 = 2 bc cos A = 54
bc cos A = 27 = a 3 Þ a = 3
b 2 + c 2 63
= =7
9 9
(C) Circumcentre of DABC is ( -1, 0).
Point A lie on the circle ( x + 1) 2 + y 2 = 4 Þ x 2 + y 2 + 2 x - 3 = 0
(D) (cos q sin q + 6) = 6 (sin q - cos q) Þ 36 + sin 2 q cos 2 q + 12 sin q cos q = 36(1 - 2 sin q cos q)
Let sin q cos q = t
340 Solution of Advanced Problems in Mathematics for JEE

t 2 + 84t = 0 Þ t = 0
If sin q = 0 Þ cos q = -1 Þ q = p
p
If cos q = 0 Þ sin q = 1 Þ q =
2
3. r1 r2 + r3 r2 + r1 r3 = S 2 Þ S = 42
1 1 1 1
+ + = Þ r =8
r1 r2 r3 r
D
r=
Þ D = 336
S
D D D D
4. Use r = , r1 = , r2 = , r3 =
s s-a s-b s-a
A B C
(C) r = 4 R sin sin sin
2 2 2
and similarly r1 , r2 , r3

Exercise-5 : Subjective Type Problems

2. Ð O 1 EO 2 = 90° , E is the orthocentre of D O 1 EO 2 A


x 3
= ;x =1
sin 30° sin 120° D E
x

30° 30°
B C
1 Ö3
3. r ( AD + AE ) = 5
2
1 A
r( BF + BD) = 10
2
B B D E
r cot
+ r cot
BF + BD 2 2 =2
Þ =2 Þ
AD + AE A A I
r cot + r cot
2 2
C B C
cos F
Applying C and D, 2 =3
A-B
sin
2
2
D 1D 2D 3 ( r1 r2 r3 ) 2 æ s s s ö
4. = =ç ´ ´ ÷
3 6 s - a s - b s - cø
D r è
( s - a) + ( s - b) + ( s - c)
³ [( s - a)( s - b)( s - c)]1 3
3
Solution of Triangles 341

s3
Þ ³ 27
( s - a)( s - b)( s - c)
Minimum value = 1
AB AM A
5. In DABM , =
sin 150° sin 7° 83°

°
23
AC AM
In DACM , =
sin(97°-q) sin q M
Þ sin q = 2 sin 7° sin(97°- q)
Þ sin q = sin q - cos(104°- q) 7° q

30°
Þ cos(104°- q) = 0
B C
Þ q = 14°
AH BH CH R R2
6. + + = ( a cos A + b cos B + c cos C ) = (sin 2 A + sin 2 B + sin 2C )
AD BE CF D D
4R 2 bc sin A
= sin A sin B sin C = =2
D D
c AA1
7. = A
sin C æ Aö
sin ç B + ÷
è 2ø
A
AA1 cos = sin B + sin C (Q R = 1) B A/2 C
2
A B C c
AA1 cos + BB 1 cos + CC 1 cos
Þ 2 2 2 =2 A1
sin A + sin B + sin C
8. ax 2 + bx + c = 0 has equal roots, then
b 2 = 4 ac …(1)
2 2 2
sin A sin C a c a + c b + 2 ac cos B
+ = + = =
sin C sin A c a ac ac
= 4 + 2 cos B
A B C
9. cot , cot , cot is AP
2 2 2
In DABC ,
A B C A B C
cot + cot + cot = cot × cot × cot
2 2 2 2 2 2
A C
Þ cot × cot = 3
2 2
AM ³ GM
342 Solution of Advanced Problems in Mathematics for JEE

A C
cot + cot
2 2 ³ cot A × cot C Þ cot
B
³ 3
2 2 2 2
10. ( R 2 - 4 Rr + 4 r 2 ) + ( 4 r 2 - 12 r + 9) = 0
( R - 2 r) 2 + (2 r - 3) 2 = 0
3
Þ r = ; R = 2r
2
DABC is an equilateral triangle.
11. In DBCP , A
3 PC 3 3
=
sin 60° sin q B
60° 3
C
q a
PC = 2 3 sin q
60°

2 ab cos C + 2 3 ab sin C ( a 2 + b 2 - c 2 ) + 12
12. b + c = =
2b 2b
13. R = 3, D = 6
PDDEF = DE + EF + DF = R(sin 2 A + sin 2 B + sin 2C )
= 4R sin A sin B sin C
æ b c ö 1
= 4Rç sin A ÷ = (2 D ) = 4
è 2R 2R ø R

❑❑❑
Inverse Trigonometric Functions 343

25
INVERSE TRIGONOMETRIC
FUNCTIONS

Exercise-1 : Single Choice Problems

æp ö p æp ö
2. (cot -1 x) ç - cot -1 x ÷ + 2 cot -1 x - cot -1 x + 3 ç - tan -1 x ÷ - 6 > 0
è 2 ø 2 è 2 ø
- (cot -1 x) 2 + 5 cot -1 x - 6 > 0
(cot -1 x) 2 - 5 (cot -1 x) + 6 < 0
(cot -1 x - 3)(cot -1 x - 2) < 0
2 < cot -1 x < 3
cot 3 < x < cot 2 (Q cot -1 x is decreasing)
3. 1 + tan 2 (tan -1 2) + 1 + cot 2 (cot -1 3) = 1 + 2 2 + 1 + 3 2 = 15
a æ (n + 1) 2 + (n + 1) - ((n + 1) 2 - (n + 1)) ö
4. å tan -1 çç 1 + ( n + 1) 4
- ( n + 1) 2
÷
÷
n =1 è ø
5. cot -1 ( cos a ) - tan -1 ( cos a ) = x
p
- 2 tan -1 cos a = x
2
p x
- = tan -1 cos a
4 2
x
1 - tan
cos a = 2
x
1 + tan
2
x 1 - cos a a
Þ tan = Þ sin x = tan 2
2 1 + cos a 2
344 Solution of Advanced Problems in Mathematics for JEE

æ æ 1ö æ 1ö ö
ç çn + ÷ - çn - ÷ ÷
æ 4 ö æ 1 ö 2ø è 2ø ÷
6. T n = tan -1 çç ÷÷ = tan -1 ç ÷ = tan -1 ç è
2
è 4n + 3 ø ç 2 ÷ ç æ 1 ö æ 1ö ÷
è n + ( 3 4) ø ç 1 + çn + ÷çn - ÷ ÷
è è 2 øè 2øø
æ 1ö æ 1ö
T n = tan -1 ç n + ÷ - tan -1 ç n - ÷
è 2ø è 2ø
æ 1ö æ 1ö p æ 1ö
S n = tan -1 ç n + ÷ - tan -1 ç ÷ Þ S ¥ = - tan -1 ç ÷
è 2 ø 2
è ø 2 è2ø
np
7. cos -1 (1 - x) + m cos -1 x =
2
Domain x Î[0 , 1]
cos -1 (1 - x) + m cos -1 x > 0 (Qm > 0)
There is no solution. p
8. 2 tan -1 (2 x - 1) = cos -1 x
2x - 1³ 0 1³ x > 0
O
1 –1 +1

2
–p
Only one solution
9. Put x = 2 sin q , y = 3 cos q
x y sin q cos q
+ -2 = + - 2 Î [-3 , - 1]
2 2 3 2 2 2
sin q cos q
\ + - 2 = -1 only
2 2
10. (cos -1 x) 2 - (sin -1 x) 2 > 0 Þ (cos -1 x + sin -1 x)(cos -1 x - sin -1 x) > 0
Þ cos -1 x - sin -1 x > 0
p p p 1
Þ - 2 sin -1 x > 0 Þ - £ sin -1 x < Þ -1 £ x <
2 2 4 2
2
11. f( x) = x + 7 x + k( k - 3) = 0
f(0) < 0 (Q k Î (0 , 3))
Þ a and b are of opposite sign.
æ 1ö æ 1ö
tan -1 a + tan -1 ç ÷ + tan -1 b + tan -1 çç ÷÷ = 0
èaø èb ø
12. f( x) = a + 2 b cos -1 x
D f : [-1, 1]
f( x) is decreasing function.
Þ f( -1) = 1 Þ a + 2 bp = 1
Inverse Trigonometric Functions 345

and f(1) = -1 Þ a = -1
2
æp ö 5p 2
13. Let tan -1 x = t Þ t2 + ç -t÷ =
è2 ø 8
3p -p -p
Þ t= or Þ tan -1 x = Þ x = -1
4 4 4
y

14. –2p
x
–5p/2 –3p/2 –p/2 O p/2 3p/2 2p 5p/2

p
15. 1 £ sin -1 (cos -1 (sin -1 (tan -1 x))) £
2
sin 1 £ cos -1 (sin -1 (tan -1 x)) £ 1
cos(sin 1) ³ sin -1 (tan -1 x) ³ cos 1
sin(cos(sin 1)) ³ tan -1 x ³ sin(cos 1)
tan(sin(cos(sin 1))) ³ x ³ tan(sin(cos 1))
1
16. x + = -2 sin(cos -1 y) Þ x = -1 and y = 0
x
17. tan -1 1 + tan -1 2 + tan -1 3
æ 5 ö
tan -1 1 + p + tan -1 ç ÷=p
è1-6ø
æ -p p ö
18. Let tan -1 x = q , q Î ç , ÷
è 2 2ø
2q + cos -1 cos 2q Þ 2q £ 0
q £ 0 Þ tan -1 x £ 0 Þ x £ 0
y
p/2

5p/2 x
p/2 p 3p/2 2p
19. 0

–p/2

16 ( x 2 + y 2 ) - 48 px + 16 py + 31p 2 = 0
31p 2
x 2 + y 2 - 3 px + py + =0
16
346 Solution of Advanced Problems in Mathematics for JEE

2 2
æ 3p ö æ pö 9p 2
çx- ÷ +çy + ÷ =
è 2 ø è 2ø 16
22. sin -1 (sin 8) = 3 p - 8 = t
tan -1 (tan 8) = 8 - 3p = - t
f (t ) + f ( - t ) = l
2 =l
23. Graphs of y = 3 sin -1 x and y = p(1 - x) are
3p
2
1

–1
1

y = p(1–x)

– 3p
2

Clearly one point of intersection


24. D f : [-1, 1]
p
f( x) max = + 6 at x = 1
2
p
f( x) min = - - 2 at x = -1
2
¥
æ 1ö æ 1ö æ 1 ö æ 1 ö
27. tan -1 ç ÷ + tan -1 ç ÷ + tan -1 ç ÷ +¼ = tan -1 ç
å ÷
2
è3ø è7ø è 13 ø r =1 è r + r + 1ø
¥
= å(tan -1 ( r + 1) - tan -1 ( r))
r =1
é 1ù
1-
1 -1 -1 (1 4) + (2 9) -1 æ 1 ö 1 -1 æ 1 ö 1 -1 ê 4ú
28. cos x = tan = tan ç ÷ = ´ 2 tan ç ÷ = cos ê ú
2 1 - ( 1 4) ´ ( 2 9) è2ø 2 è2ø 2 ê1 + 1ú
êë 4 úû
æ æ 2 ö ö
ç using 2 tan -1 x = cos -1 ç 1 - x ÷ for x ³ 0 ÷ = 1 cos -1 3 .
ç ç1+ x2 ÷ ÷ 2 5
è è ø ø
p p p p
29. tan 2 (sin -1 x) > 1 Þ - < sin -1 x < - or < sin -1 x <
2 4 4 2
æ1+ 2 ´ 4ö -1 æ 1 + 4 ´ 8 ö -1 æ 1 + 8 ´ 16 ö
30. cot -1 ç ÷ + cot ç ÷ + cot ç ÷ +¼
è 4 - 2 ø è 8 - 4 ø è 16 - 8 ø
Inverse Trigonometric Functions 347

= cot -1 (2) - cot -1 ( 4) + cot -1 ( 4) - cot -1 (8) + cot -1 (8) - cot -1 (16) +¼
= cot -1 (2)
p
32. sin -1 (1 + x) is defined for x < 0 and sin -1 x = - cos -1 x " - 1 £ x £ 1.
2
The given equation is sin -1 x + sin -1 (1 + x) = cos -1 x
which can be written as
p p
- cos -1 x + - cos -1 (1 + x) = cos -1 x
2 2
Þ p - cos -1 (1 + x) = 2 cos -1 x
Þ cos -1 ( -1 - x) = 2 p - cos -1 (2 x 2 - 1)
Þ cos -1 ( -1 - x) + cos -1 (2 x 2 - 1) = 2 p
Þ cos -1 ( -1 - x) = cos -1 (2 x 2 - 1) = p
Þ -1 - x = 2 x 2 - 1 = -1
Þ x =0
which implies that the total number of solutions sin -1 x + sin -1 (1 + x) = cos -1 x is only one.
p3
33. (sin -1 x) 3 - (cos -1 x) 3 + (sin -1 x)(cos -1 x)(sin -1 x - cos -1 x) =
16
p3
(sin -1 x - cos -1 x){(sin -1 x) 2 + (cos -1 x) 2 + (2 cos -1 x sin -1 x)} =
16
p3
(sin -1 x - cos -1 x)(sin -1 x + cos -1 x) 2 =
16
p2 p3
(sin -1 x - cos -1 x) =
4 16
p p
2 sin -1 x - =
2 4
3p
2 sin -1 x =
4
3 p
sin -1 x =
8
3x x
x = sin or cos
8 8
æ 1 + x 2 - 1ö
35. f( x) = tan -1 ç ÷
ç x ÷
è ø
1+ x2 -1
=y
x
348 Solution of Advanced Problems in Mathematics for JEE

1 2x
x - ( 1 + x 2 - 1)
2 1+ x2
y¢ =
x2
1+ x2 -1
= > 0 always
x 2( 1 + x 2 )
x®¥ y®1
x ® -¥ y ® -1
-1
tan ( -1 ® 1)
æ p pö
ç - , ÷ - {0}
è 4 4ø
40. cos -1 x + cot -1 x = l " x Î [-1, 1]
é p 7p ù
lÎê ,
ë 4 4 úû
41. x 3 + bx 2 + cx + 1 = 0
f( -1) = b - c < 0
f(0) = 1 > 0
Þ -1 < a < 0
a = -B
B Î(0 , 1)
æ 2 sin B ö
y = -2 tan -1 (cosec B ) - tan -1 ç ÷
è cos 2 B ø
æ 2 cos B ö 2 sin B
= - ç p + tan -1 ÷ - tan -1 = -p
2
è 1 - cosec B ø cos 2 B
p
42. f( x) = + cot -1 {- x}
2
p p
< cot -1 {- x} £
4 2
43. sin -1 (sin 3) + tan -1 (tan 3) + sec -1 (sec 3)
( p - 3) + ( 3 - p ) + 3 = 3
44. (2np , 0) n Î I
45. f( x) = sin -1 ([ x] - 1) + 2 cos -1 ([ x] - 2)
-1 £ [ x] - 1 £ 1 Þ 0 £ [ x] £ 2
-1 £ [ x] - 2 £ 1 Þ 1 £ [ x] £ 3 Þ [ x] = 1 or 2
Inverse Trigonometric Functions 349

Exercise-2 : One or More than One Answer is/are Correct

2. cos -1 x = tan -1 x Þ x Î[0 , 1]


æ 1- x2 ö
tan -1 ç ÷ = tan -1 x
ç x ÷
è ø
Þ x2 = 1- x2 Þ x4 + x2 -1=0
5 -1
x2 =
2
æ 4 2ö æ æ 17 ö ö
3. tan ç cos -1 + tan -1 ÷ = tan ç tan -1 ç ÷ ÷
è 5 3ø è è 6 øø
a = 17 , b = 6

æ 17 ö
5. sin -1 ç x 2 - 6 x + -1
÷ = sin k
è 2 ø
1
where -1 £ k £ 1
17 x
0
y = x2 - 6x + 1
3
2 –
2

p3
6. (sin -1 x - cos -1 x)((sin -1 x) 2 + (cos -1 x) 2 + 2 sin -1 x cos -1 x) =
16
p p p
Þ sin -1 x - cos -1 x = Þ cos -1 x = Þ x = cos
4 8 8

Exercise-3 : Comprehension Type Problems

Paragraph for Question Nos. 1 to 2


1. a = 2p
b = -3
2. a = 0
b=3
350 Solution of Advanced Problems in Mathematics for JEE

Exercise-4 : Matching Type Problems

n
3. (A) 33n = [2 + (n - 1) 2 ] Þ n = 9
2
é p 7p ù
(B) x Î [-1, 1] Þ cos -1 x + cot -1 x Î ê ,
ë 4 4 úû
(C) cos q =|1 + sin q| Þ cos q ³ 0
Sq. both sides,
Þ cos 2 q = 1 + sin 2 q + 2 sin q
sin q = 0 or sin q = -1
Number of solution = 3
(D) a = x( x - 1)
Possible values of a are 6, 12, 20, 30.
4. (A) tan -1 (3) + tan -1 ( -3) = 0 y

(B) 1 + sin x = 2 cos 2 x


Þ 2 sin 2 x + sin x - 1 = 0
(2 sin x - 1)(sin x + 1) = 0
p x
p p 0 p 3p 2p
(C) tan x = - 2 2
4 2
(D) f( x) = x 3 + x 2 + 4 x + 2 sin x
f ¢( x) = 3 x 2 + 2 x + 4 + 2 cos x > 0
and f(0) = 0

Exercise-5 : Subjective Type Problems

1. 5 - 2 p > x 2 - 4 x
x 2 - 4 x + (2 p - 5) < 0
2 - 9 - 2p < x < 2 + 9 - 2p Þ l = 9
B r
2. sin = A
2 IB
A C I
IB = 4 R sin sin r
2 2 B B/2 C
90°–C/2
æ Bö r A C 90°–B/2
sin ç 90° - ÷ = 1 Þ BI 1 = 4 R sin cos r1
è 2 ø BI 1 2 2
A
( II 1 ) 2 = ( BI ) 2 + ( BI 1 ) 2 = 16 R 2 sin 2 …(1)
2 I1
Inverse Trigonometric Functions 351

æ Aö
I 2 I 3 cos ç 90° - ÷ = a (by using pedal triangle)
è 2 ø
A
I 2 I 3 = 4 R cos
2
A
( I 2 I 3 ) 2 = 16 R 2 cos 2 …(2)
2
From (1) & (2) we get l = 16
æ 1ö æ3ö
3. 2 tan -1 ç ÷ - sin -1 ç ÷
5
è ø è 5ø
æ 5 ö æ3ö
tan -1 ç ÷ - sin -1 ç ÷
è 12 ø è 5ø
æ 5 ö æ3ö æ æ3ö æ 5 öö
tan -1 ç ÷ - tan -1 ç ÷ = - ç tan -1 ç ÷ - tan -1 ç ÷ ÷
è 12 ø è4ø è è4ø è 12 ø ø
æ 16 ö æ 63 ö
= - tan -1 ç ÷ = - cos -1 ç ÷
è 63 ø è 65 ø
Þ l = 65
æ 1 ö
¥ ¥ ç ÷
-1 æ 2 ö -1 ç 2 ÷
5. å 2 tan çç
è n 2
+ n + 4
÷÷ =
ø
å
2 tan
ç n2 n ÷
n =0 n =0
ç + + 1÷
è 4 4 ø
æ æ n 1ö n ö
¥ ç ç + ÷- ÷
-1 ç è 2 2ø 2 ÷
= å 2 tan
ç n æ n 1ö ÷
n =0
ç ç + ÷ + 1÷
è 2è2 2ø ø
¥
æ n 1 n ö
= å 2 çè tan -1 æçè 2 + 2 ö÷ø - tan -1 æçè 2 ö÷ø ÷ø
n =0

6. cos -1 (|3 log 26 (cos x) - 7|) = cos -1 (|log 26 (cos x) - 1|)


|3 log 26 (cos x) - 7| =|log 26 (cos x) - 1|
Let log 26 (cos x) = t
|3t - 7| =|t - 1|
Þ t = 3 and t = 2
3 2
Þ cos x = 6 - and 6 -
❑❑❑
352 Solution of Advanced Problems in Mathematics for JEE

26
VECTOR & 3DIMENSIONAL
GEOMETRY

Exercise-1 : Single Choice Problems

1. Perpendicular distance from origin


p
d=
a 2 + b2 + c 2
p2
\ d2 =
a 2 + b2 + c 2
1 ® ®
2. Area of triangle = | a ´ b| = 3
2
® ® p ® ® 12
Þ | a|| b|sin = 6 Þ | a|| b| =
3 3
® ® ® ® p
a × b =| a|| b|cos =2 3
3
® ® ® ® ® ® ® ® ®
4. | c - a|2 = 8 Þ | c|2 - 2 c × a + | a|2 = 8 Þ | c|2 - 2| c| + 1 = 0 Þ | c|2 = 1
® ® ® ®
Also, a ´ b = 2i$ + 2 j$ + k$ Þ | a ´ b| = 3
® ® ® ® ® ® p 1 3
\ |( a ´ b ) ´ c| =| a ´ b|| c|sin = 3 ×1× =
6 2 2
4
5. cos q 1 =
5
4
cos q 2 =
5
Þ cos 2 q 1 + sin 2 q 2 = 1
® ® ® ® p
æ ö
7. l( a ´ b ) × ( a ´ b ) = 4 3 ç abcos = 1÷ Þ b=1
è 3 ø
® ®
l( a 2 b 2 - ( a × b ) 2 ) = 4 3
Vector & 3Dimensional Geometry 353

l( 4 ´ 1 - (1) 2 ) = 4 3
4 3
l=
3
8. x(3i$ + 2 j$ + 4 k) + y(2i$ + 2 k$) + z( 4i$ + 2 j$ + 3 k$) = a( xi$ + yj$ + zk$)
$

Þ (3 - a ) x + 2 y + 4 z = 0
2 x - ay + 2 z = 0
4 x + 2 y + (3 - a )z = 0
For non-trivial solution
3 -a 2 4
2 -a 2 =0
4 2 3 -a
® ® ® ® ® ®
a×a a× b a× c
® ® ® ® ® ® ® ® ®
9. b×a b× b b × c =[a b c ]2
® ® ® ® ® ®
c×a c×b c×c

® ® ® ® ®
10. | c|2 = 4( a ´ b ) 2 + 9 b 2 = 4( a 2 b 2 - ( a × b ) 2 ) + 9 b 2 = 192
® ® ® ® ® ® ® ®
c + 3 b = 2 a ´ b Þ c 2 + 9 b 2 + 6 b × c = 4( a 2 b 2 - ( a × b ) 2 )
- 3
Þ 6 × 4 × 192 cos q = -288 Þ cos q =
2
® ® ® ® ® ® ® ® ® ® ® ®
11. | a - 2 b|2 + | b - 2 c|2 + | c - 2 a|2 = 5a 2 + 5b 2 + 5c 2 - 4( a × b + b × c + c × a )
® ® ® ® ® ® æ -3 ö
= 15 - 4( a × b + b × c + c × a ) £ 15 - 4 ç ÷ = 21
è 2 ø
® ® ® ® ® ® -3
Q a× b+ b× c + c ×a ³
2
® ® p ® ® ® ®
12. 16| a|| b|sin = 3| a|2 + 3| b|2 + 6| a|| b|
2
Þ 3 a 2 - 10 ab + 3 b 2 = 0 Þ (3 a - b)( a - 3 b) = 0
—® —® ® ® ® ® —® —®
Now OC × AB = ( a + b ) × ( b - a ) =| OC || AB|cos q
b2 - a 2 9a 2 - a 2
Þ = cos q = (using b = 3 a)
a 2 + b2 a 2 + b2 9a 2 + a 2
4
\ cos q =
5
354 Solution of Advanced Problems in Mathematics for JEE

4
1-
q 1 - cos q 5 =1
\ tan = =
2 1 + cos q 4 3
1+
5
—® 1 —® —® A
13. AM = ( AB + AC )
2

2 l 3 B C
M
2
14. 3 3 5 =0 Þ l - 3l + 2 = 0
l 2 2
® ® ® ê ® ® ® ® ® ® ú
15. ( a + b - c ) × ê( b + c - a ) ´ ( c + a - b )ú
ë û
® ® ® ® ® ® ® ® ® ® ® ® ® ® ® ® ® ® ® ® ® ®
( a + b - c ) × ( b ´ c + b ´ a + c ´ a - c ´ b - a ´ c + a ´ b ) = 2( a + b - c ) × ( b ´ c + c ´ a )
®®® ®®® ®®®
= 2 ([ a b c ] + [ b c a ]) = 4 [ a b c ]
16. ( a$ ´ b) ´ ( a$ + b) = ( a$ × ( a$ + b)) b - ( b × ( a$ + b)) a$ = (1 + a$ × b)( b$ - a$ )
$ $ $ $ $ $ $
® ® ® —® —® ® —® —®
17. Angle between planes is angle between n 1 and n 2 , where n 1 = AB ´ AC and n 2 = AD ´ AC
® ®
n 1 = -2i$ + 4 j$ - 3 k$, n 1 = 6i$ + 3 j$ - 6 k$
® ® ®
18. a1 = x 1 i$ + y 1 j$ + z 1 k$ , a 2 = x 2 i$ + y 2 j$ + z 2 k$ and a 3 = x 3 i$ + y 3 j$ + z 3 k$ are mutually
perpendicular unit vectors, then
x1 y1 z1
® ® ®
[ a1 a2 a3 ] = x 2 y2 z2 = ±1
x3 y3 z3
22. On solving, Ax = C and Bx = D Q
é 1ù é3 ù ® Ù Ù Ù ,3
)
ê ú ,2
x= 2 x = ê 1ú n=i+j+k (1
ê ú ê ú P
êë3 úû êë2 úû
Plane
P = (1, 2 , 3), Q = (3 , 1, 2)
x -1 y -2 z -3 x+y+z=9
PP ¢ : = = =l
1 1 1
(l + 1, l + 2, l + 3) lies on plane P¢
3l + 6 = 9 Þ l = 1
\ P ¢ = (3 , 4 , 5)

Similarly Q ¢ = ( 5, 3 , 4)
Vector & 3Dimensional Geometry 355

Now check the options.


—®
23. AM = (a - 1) i$ + j$
—®
BM = (a - 1) i$
a -1 1 0
—®
$ $ $
CM = (a - 3) i + 2 j + 2 k are coplanar, then a - 2 0 0 = 0
a -3 2 2
—®
24. Normal vector is parallel to PQ P(1,–2,3)
x1 - 1 y1 + 2 z 1 - 3
= = =l
1 -1 1
Þ x 1 = l + 1, y 1 = -2 - l, z 1 = 3 + l
Mid point of PQ is lie on the plane Q(x1, y1, z1)
2
Þ l=
3
æ 5 - 8 11 ö
Qç , , ÷
è3 3 3 ø
25. |a$ - b$| = 1
1
Þ cos q =
2
3
Volume of parallelopiped = [a$ b$ a$ ´ b$] = sin 2 q =
4
26. Equation of line PQ
x -3 y -7 z -1
= = =l
1 2 -6
Point Q(3 + l , 7 + 2l , 1 - 6l)
If it lies on plane 3 x + 2 y + 11z = 9 , then
25
l=
59
® ® ®
27. V 1 = [ a b c]
® ® ® ® ® ® ® ® ® ® ® ®
V2 =[ a + b - 2 c 3 a -2 b + c a - 4 b + 2 c ] = 15[ a b c]
28. Line represented by x + ay - b = 0, cy + z - d = 0 is parallel to
(i$ + aj$) ´ ( cj$ + k$) = ai$ - j$ + ck$
Line represented by - x + a ¢y + b¢ = 0, c ¢y - z + d ¢ = 0 is parallel to
(i$ - a ¢ j$) ´ ( c ¢ j$ - k$) = a ¢ i$ + j$ + c ¢ k$
356 Solution of Advanced Problems in Mathematics for JEE

If these two lines are perpendicular, then


aa ¢ + cc ¢ = 1
29. Equation of line PQ
®
r = (2i$ - 2 j$ + 3 k$) + m (i$ + 5 j$ + k$)
Þ Co-ordinate of Q(2 + m , 5 m - 2 , 3 + m)
If point Q lies on plane, then
10
m=
27
—® 10 $ 50 $ 10 $
PQ = m i$ + 5 m j$ + m k$ = i + j+ k
27 27 27
® ® ® ® ® ®
30. ( a ´ b) ´ c = a ´ ( b ´ c )
® ® ® ® ® ® ® ® ® ® ® ®
( a × c ) b - ( b × c ) a = ( a × c ) b - ( a × b) c
® ® ® ® ® ®
Þ ( a × b) c = ( b × c ) a y
® (0,1)
31. Let r = xi$ + yj$ (– 7, 0) ( 7, 0)
x
® ® O
r × ( r + 6i$) = 7
Þ x 2 + ( y + 3) 2 = 16
Area of quadrilateral = 8 7 (0,–7)
® ® ® ®
1 |( p - q ) ´ ( r - q )|
33. =4 ®
P(p)
2 1 ® ®
| q ´ r|
2
® ® ®
O
Also, p + k1 q + k 2 r = 0
®
® ® ® Q(q) ®
Þ p = -k1 q - k 2 r = 0 R(r)

Þ k1 + k 2 + 1 = 4
Þ k1 + k 2 = 3
34. Let length, breadth and height of rectangular box be a , b, c respectively.
®
P = ai$ + ck$ Z
(0,0,c)
® R(0,b,c)
R = bj$ + ck$ P
(a,0,c) (0,0,0) (0,b,0)
a ® b c
O = i$ + j$ + k$ Q
2 2 2 x (a,0,0)
—® —®
æa b c ö æa b c ö
| OQ|| OR |cos q = ç i$ + j$ + k$ ÷ × ç i$ - j$ - k$ ÷
è2 2 2 ø è2 2 2 ø
Vector & 3Dimensional Geometry 357

1
Þ cos q = -
3
1
Similarly, cos f = -
3
® ® ® ® ® ® ®
36. r = a (m ´ n ) + b( n ´ l ) + c ( l ´ m )
® ® ® ® ® ® ® ® ®
where [ l m n ] = 4, r × l = 4a, r × m = 4b, r × n = 4c
which imply that
a+ b+ c 1
=
® ® ® ® 4
r ×( l + m + n )
1 ®®® ®®®
37. The volume tetrahedron is given by k = [ a b c ] Þ [ a b c ] = 6 k
6
The volume of parallelepiped is given by
® ® ® ® ® ® ®® ® ® ® ®® ® ® ®
[ a - b b + 2 c 3 a - c ] = [ a b + 2 c 3 a - c ] + [- b b + 2 c 3 a - c ]
®® ® ® ® ® ® ® ®® ® ® ® ® ® ®
= [ a b 3 a - c ] + [ a 2 c 3 a - c ] + [- b b 3 a - c ] + [- b 2 c 3 a - c ]
®® ® ® ® ® ®®® ®®®
= [ a b - c ] + [- b 2 c 3 a ] = - [ a b c ] - 6 [ a b c ]
®®®
= -7[ a b c ]
Volume is 42 k.
38. We know that the equation of the plane passing through the line of intersection of planes
p1 = 0 and p 2 = 0 is
p1 + lp 2 = 0
That is,
( x + 2 y + z - 10) + l (3 x + y - z - 5) = 0 …(1)
Since, this plane passes through the origin (0 , 0 , 0) satisfies this equation. This implies that
( -10) + l( -5) = 0
Þ l = -2
Substituting the value of l in Eq. (1), we get
( x + 2 y + z - 10) - 2 (3 x + y - z - 5) = 0
That is, -5 x + 3 z = 0
Þ 5x - 3z = 0
39. Let the point P( x p , y p , z p ) be the required point.
The distance of the point from x-axis is y 2p + z 2p .

The distance from the point (1, - 1, 2) is


358 Solution of Advanced Problems in Mathematics for JEE

( x p - 1) 2 + ( y p + 1) 2 + ( z p - 2) 2

Þ y 2p + z 2p = ( x p - 1) 2 + ( y p + 1) 2 + ( z p - 2) 2

Þ x 2p - 2 x p + 2 y p - 4 z p + 6 = 0
Therefore, the locus of point P is
x 2 - 2 x + 2 y - 4z + 6 = 0

Exercise-2 : One or More than One Answer is/are Correct

3. Point P on line L1 Ù Ù
i + 7j – 5k
Ù

P(2 + l , 1 + 7l , - 2 - 5l)
(2,1,–2)
Point P on line L 2
P( 4 + r , - 3 + r , - r) Þ l = -1, r = -3 P
(4,–3,0)
Acute angle between L1 and L 2
Ù Ù Ù
13 i+j–k
cos q =
15
Equation of plane containing L1 and L 2 is x + 2 y + 3 z + 2 = 0
4. a$ = b$ + ( b$ ´ c$)
a$ × b$ = 1 and a$ × c$ = b$ × c$
|a$ - b$| =|b$ ´ c$| Þ sin q = 0 (Q q = b$ c$)
|a$ + b$ + c$|2 = 3 + 2( a$ × b$ + b$ × c$ + a$ × c$) = 5 + 4( b$ × c$)
Ù Ù Ù
i – j + mk
5. If these two lines are coplanar, then
Ù Ù Ù
1 -1 m (3i – 2j – 4k)
1 m 2 =0
2 0 5 Ù Ù Ù
Ù Ù Ù
(5i – 2j + k) i + mj + 2k
2
Þ 2m - 5m - 1 = 0
® ® ®
6. i$ ´ [( a - j$) ´ i$] + j$ ´ [( a - k$) ´ j$] + k$ ´ [( a - i$) ´ k$] = 0
®
2 a - (i$ + j$ + k$) = 0 Þ (2 x - 1)i$ + (2 y - 1) j$ + (2 z - 1)k$ = 0
1
Þ x = y =z =
2
® ® ® ® ® ® ® ® ® ® ® ® ® ® ® ® ® ®
7. [ a ´ b c´d e ´ f ] = ( a ´ b ) × [( c ´ d ) ´ ( e ´ f )] = ( c ´ d ) × [( e ´ f ) ´ ( a ´ b )]
® ® ® ® ® ®
= ( e ´ f ) × [( a ´ b ) ´ ( c ´ d )]
® ® ® ® ® ® ® ® ® ®
= ( a ´ b ) × [( c ´ d ) × f ] e - [( c ´ d × e ) ´ f ]
Vector & 3Dimensional Geometry 359

® ® ® ® ® ® ® ® ® ® ® ®
=[ c d f ][ a b e] -[ c d e ][ a b f]
Similarly, solve other 2.
® ® ® ® ® ®
8. 3 ( a - b ) + ( b - c ) + 2 ( c - d ) = 0
—® —®
BC + 2 CD —®
= BA
1+ 2
®
10. b = 2 c$ + la$
®
æ 1ö
| b |2 = 4 + l2 + 4l ç ÷ = 16 Þ l = -4 , 3
è4ø
11. L1 : x = y = z
x -1 y + 1 z
L2 : = =
1 -1 -1
1
Shortest distance =
2
Equation of plane containing line L 2 and parallel to L1
y - z + 1=0
1
Distance of origin from this plane =
2
® ® ® ®
12. r ×( a + b + c ) = 0
®®®
Þ [ a b c ](sin x + cos y + 2) = 0
Þ sin x = -1 and cos y = -1
® ® ® ® ® ® ® ® ® ® ® ® ® ®
13. ( a ´ b ) ´ ( c ´ d ) = [( a ´ b ) × d ] c - ( a ´ b × c ) d = r c + s d
® ® ® ® ® ® ® ®
where r =[a b c ] and s = - [ a b c ] as c and d are non-collinear.
® ® ® ® ® ®
Similarly, h = -[ b c d ] and k = [ a c d]
® ® ®
14. Here, a = i$ + 2 j$, b = 2i$ + aj$ + 10 k$ and g = 12i$ + 20 j$ + ak$
1 2 0
®®®
\ [a b g ] = 2 a 10 = a 2 - 24 a + 240 > 0 , for all a
12 20 a
® ® ®
\ a , b and g are non-coplanar or linearly independent for all a.
Hence, (a, b, c) is the correct answer.
360 Solution of Advanced Problems in Mathematics for JEE

®
19. Let r = xi$ + y$j + zk$
®
If r ´ i$ = $j + k$ Þ - yk$ + z$j = $j + k$
®
Þ r = xi$ - $j + k$
®
If r ´ $j = i$ + k$ Þ xk$ - zi$ = i$ + k$
®
r = i$ + y$j - k$
20. (A) See dot product
(C) y = ln( e -2 + e x )
e y - e -2 = e x
21. ( -3 - 4l , 6 + 3l , 2l) = ( -2 - 4m , 7 + m , m)

Exercise-3 : Comprehension Type Problems

Paragraph for Question Nos. 1 to 3


1. AB = BC (2,0,0)
A

p.v. of H = j$ + r (i$ - j$ + k$) (1,0,1)


D
—® —®
Also, AH × BC = 0 H
(0,1,0) (0,0,2)
Þ [( r - 2)i$ + (1 - r) j$ + rk$] × ( - j$ + 2 k$) = 0 B C

1
Þ r - 1 + 2r = 0 Þ r=
3
i$ 2 j$ k$
2. p.v. of H = + +
3 3 3
2 $ j$ 2 k$
p.v. of centroid = i + +
3 3 3
®
3(p. v .) of centroid - p. v . of H
p.v. of S =
2
1
y coordinate of S =
6
3. Let P º ( a , b, c)
Þ ( a - 2) 2 + b 2 + c 2 = a 2 + ( b - 1) 2 + c 2 = a 2 + b 2 + ( c - 2) 2 = a 2 + b 2 + c 2
æ 1 ö
Þ P = ç 1, , 1÷
è 2 ø
3
PA =
2
Vector & 3Dimensional Geometry 361

Paragraph for Question Nos. 4 to 6


® ® ®
4. PQ = ( b - a ) cos q (where q angle between AB and plane) B(b)
®
® ® ® A(a)
|( b - a ) ´ n|
=
® ® ®
| n| P Q r×n=d
® ® ®
5. Equation of line AP is r = a + l n A(a)
®

® ® ®
For point P( a + l n ) × n = d
® ® ® ®
P r×n=d
d- a×n
l=
®
| n |2

æ® ® ®ö æ ® ® ö
ç d- a×n ÷ ® ç d - a × n ®÷
\ P ç a+ ÷ \ A ¢ is a + 2 ç ® n ÷
®
ç | n|2 ÷ ç | n|2 ÷
è ø è ø
® ® ® ® ® ® ®
b×n -d a×n -d ( b - n) × n
6. Distance =|BQ - AP|= - =
® ® ®
| n| | n| | n|

Paragraph for Question Nos. 7 to 9


7. B(3 + 2l , - 1 - 3l , 2 - l) L1
d r of L 2 < 2l , - 3l + 3, 1 - l >
®
B
L 2 is parallel to plane r × (2i$ + j$ - k$) = 5 A L2
(3,–4,1)
\ 4l - 3l + 3 - 1 + l = 0
2l = -2 Þ l = -1
\ B(1, 2 , 3)
®
So, equation of L 2 is r = (3i$ - 4 j$ + k$) + l(i$ - 3 j$ - k$)
8. Equation of plane contain L1 & L 2 is
x -3 y + 1 z -2
0 3 1 =0
-1 6 2
i . e. ,
( x - 3)(6 - 6) + ( y + 1)(0 + 1) + ( z - 2)(0 + 3) = 0
y + 3z - 5 = 0
9. Any point of L1 is (3 + 2l , - 1 - 3l , 2 - l)
if on plane p, then
362 Solution of Advanced Problems in Mathematics for JEE

2 (3 + 2l) + 1( -1 - 3l) - 1(2 - l) = 5


2l = 2 Þ l = 1
\ Q(5, - 4 , 1)
if on xy plane, then 2 -l =0 Þ l =2
\ R(7 , - 7 , 0)
3 -4 1
1 —® —® —® 1 7
Volume of tetrahedran = [ OA OQ OR ] = 5 -4 1 =
6 6 3
7 -7 0

Paragraph for Question Nos. 10 to 11


Sol. Use crammer rule,
Intersect at a unique point Þ D ¹ 0
Do not have any common point of intersection.
Þ D = 0 and atleast any one of D x , D y , D z is non-zero (condition of no solution)

Paragraph for Question Nos. 12 to 14


® ® ®
Sol. | a | =|b | =| c | = r
® ®
C(c) B(b)
æ ® ®ö
® ça +b÷ ® O
a +ç ÷+c
ç 2 ÷
è ø
PV of E : ®
3 A(a)
® ® ®
® 3 a+ b + 2 c
e=
6
® ® ®
®
a+b+c
PV of G : g =
3
æ ® ® ®ö æ ® ® ö
—® —® ç 3 a + b + 2c ÷ ç a + b ® ÷
12. OE × CD = 0 Þ ç ÷ ×ç - c ÷ =0
ç 6 ÷ ç 2 ÷
è ø è ø
® ® ®
Þ a ×( b - c ) = 0
—® —®
\ OA ^ BC
\ DABC must be isosceles with base BC.
—® —®
\ | AC| =| AB|
Vector & 3Dimensional Geometry 363

—® —®
æ ® ® ® ® ® ® ö æ ® ® ®ö
ç 3 a + b + 2c a + b + c ÷ ç a + b ÷
13. GE × CD = 0 Þ ç - ÷ ×ç - c ÷ =0
ç 6 3 ÷ ç 2 ÷
è ø è ø
® ® ® —® —®
Þ ( a - b ) × c = 0 Þ AB ^ OC
\ ABC must be isosceles with base AB.
\ Circumcentre and centroid lie on median through C.
\ Orthocenter also lie on median through C.
—® —® —® —® —® —®
14. [ AB AC AB ´ AC ] = ( AB AC ) 2
® ® ® ® ® ®
(a ´ b + b ´ c + c ´ a)2
2
—® —® —® —® —® —® ì -1 ® ® ® ® ® ® ü
[ AE AG AE ´ AG ] = ( AE ´ AG ) 2 = í ( a ´ b + b ´ c + c ´ a )ý
î18 þ
1 —® —® 2
= ( AB ´ AC )
324
Paragraph for Question Nos. 15 to 16
15. D(3 , - 1, 2) AB lies along (0 , 1, 2)
CD lies along (3 , - 2 , 0)
Equation of plane containing AB line
x -1 y -1 z -1
0 1 2 = 2 ( x - 1) + 2 ( y - 1) - ( z - 1) = 0
2 -2 0
Containing CD line 2 ( x - 1) + 2 ( y - 1) - ( z - 2) = 0
16. r = (3 , - 1, 2) + d (1, 0 , 0)
Equation of ABC plane is x = 1.
Paragraph for Question Nos. 17 to 18
æ ®ö æ ® ®ö y ®
ç2b ÷ ç3b +2a ÷ B(b)
17. R ç ÷ and Q ç ÷
ç 5 ÷ ç 5 ÷ Q
è ø è ø ®
R A(a)
æ ® ®ö æ ®ö P
ç3b +2a ÷ ® ç2b ÷
mç ÷ l a + 1ç ÷ x
ç 5 ÷ ç 5 ÷ O
è ø è ø
=
m+1 l+1
2m l 3m 2 10
Þ = and = Þm=
5 (m + 1) l + 1 5 (m + 1) 5 (l + 1) 9
364 Solution of Advanced Problems in Mathematics for JEE

1 —® —® 1é 2 ® ® ®ù 3 ® ®
18. Ar ( DOPA) = OP ´ OA = ê (3 b + 2 a ) ´ a ú = ( b ´ a)
2 2 ë 19 û 19
éæ ® ®ö ®ù
1 —® —® —® —® 1 êç 3 b + 2 a ÷ ® 2 ® ® 2bú
Ar ( PQBR ) = OQ ´ OB - OP ´ OR = êç ÷´b - (3 b + 2 a ) ´
2 2 ç 5 ÷ 19 5 ú
êëè ø úû
3 ® ®
= (a ´ b)
19

Exercise-4 : Matching Type Problems

x -1 y + 2 z ®
1. (A) Line = = is along the vector a = -2i$ + 3 j$ - k$ and line
-2 3 -1
® ® ® ®
r = (3i$ - j$ + k$) + t(i$ + j$ + k$) is along the vector b = i$ + j$ + k$. Here a ^ b .
3 - 1 -1 - ( -2) 1 - 0
Also, -2 3 -1 ¹ 0
1 1 1
(B) The direction ratios of the line x - y + 3 z - 4 = 0 = 2 x + y - 3 z + 5 = 0 are
i$ j$ k$
1 -1 2 = i$ + 7 j$ + 3 k$
2 1 3

Hence, the give two lines are parallel.


(C) The given lines are
®
( x = t - 3 , y = 2t + 1, z = -3t - 2) and r = (t + 1) i$ + (2t + 3) j$ + ( -t - 9)k$ ,
x + 3 y -1 z + 2 x -1 y -3 z + 9
or = = and = =
1 -2 -3 1 2 -1
The lines are perpendicular as (1)(1) + ( -2)(2) + ( -3)( -1) = 0
-3 - 1 1 - 3 -2 - ( -9)
Also, 1 -2 -3 =0
1 2 -1
Hence, the lines are intersecting.
® ® 3
(D) The given lines are r = (i$ + 3 j$ - k$) + t(2i$ - j$ - k$) and r = ( -i$ - 2 j$ + 5k$) + s (i$ - 2 j$ + k$).
4
1 - ( -1) 3 - ( -2) -1 - 5
2 -1 -1 = 0
1 -2 3 4
Vector & 3Dimensional Geometry 365

Hence, the lines are coplanar and hence intersecting (as the lines are not parallel).
® ® ® ® ® ® ® ® ® ® ® ®
2. (A) If a , b and c are mutually perpendicular, then [ a ´ b b ´ c c ´ a ] = [ a b c ]2
® ® ®
= (| a|| b|| c|) 2 = 16
® ® ® ® p
(B) Given a and b are two unit vectors, i . e. ,| a| =| b| = 1 and angle between them is .
3
® ®
| a ´ b| p ® ® 3 ® ®
sin q = Þ sin =| a ´ b| ; =| a ´ b|
® ® 3 2
| a|| b|
® ® ® ® ® ®®® ®® ®® ®® ®®
Now [ a b+ a ´ b b] = [ a b b] + [ a a ´ b b] = 0 + [ a a ´ b b]
® ® ® ® ® ® 3
= ( a ´ b ) × ( b ´ a ) = -| a ´ b|2 = -
4
® ® ® ®
(C) If b and c are orthogonal, b × c = 0
® ® ®
Also, it is given that b ´ c = a
® ® ® ® ® ® ® ® ® ® ® ® ® ® ® ® ® ®
Now [ a + b + c a+ b b + c ] =[a a+ b b + c]+[b + c a+ b b + c]
® ® ® ® ® ® ® ® ®
=[a b c ] = a × ( b ´ c ] = a × a =| a|2 = 1
®
(because a is a unit vector)
® ® ®
(D) [ x y a] = 0
® ® ®
Therefore, x , y and a are coplanar.
® ®®
[ x , y b] = 0
® ® ®
Therefore, x , y and b are coplanar.
®®®
Also, [a b c] =0
® ® ®
Therefore, a , b and c are coplanar.
From (i), (ii) and (iii)
® ® ® ® ®®
x , y and c are coplanar. Therefore, [ x , y c ] = 0
366 Solution of Advanced Problems in Mathematics for JEE

Exercise-5 : Subjective Type Problems

1. Line L is the shortest distance line of given lines.


2. [a$ b$ c$] = [b$ ´ c$ c$ ´ a$ a$ ´ b$] = [a$ b$ c$]2
Þ [a$ b$ c$] = 1
( b$ + c$) × ( a$ ´ b$) [a$ b$ c$]
Projection of b$ + c$ on a$ ´ b$ = =
|a$ ´ b$| |a$ ´ b$|
1
3. Let l = m = n =
2
—® ® ® ® ® ® ® ® ® ® ®
4. OC = a 2 ( a + b ) 2 + b 2 ( a ´ b ) 2 + 2ab [ a × ( a ´ b ) + b × ( a ´ b )]
2
2æ 1ö æ 3ö
Þ 1 = a ç 1 + 1 + 2 × 1 × 1 ÷ + b 2 × 1 × 1çç ÷ +0
÷ …(1)
è 2ø è 2 ø
—® —® —® —® p
Also, OB × OC =| OB||
× OC |cos
3
1 1 1
Þ a ×1×1×
+ a ×1 = Þ a= …(2)
2 2 3
8
From (1) and (2), b2 =
9
n +1
® ® æé 0 -1 / 2 ù ö ®
5. v n +1 - v n = çç ê ÷ v0
è ë1 / 2 0 úû ÷ø
® ® é 0 -1 / 2 ù ®
v 2- v 1 = ê v0
ë1 / 2 0 úû
® ® 3 ®
é 0 -1 / 2 ù
v 3- v 2 = ê v0
ë1 / 2 0 úû
® ® n ®
é 0 -1 / 2 ù
v n - v n -1 = ê v0
ë1 / 2 0 úû
Adding all the equations,
® ® ®
v n - v 0 = ( A + A 2 + A 3 +¼¼ A n ) v 0
é 0 -1 / 2 ù ® ®
where A = ê ú Þ v n = ( I + A + A 2 +¼¼ ) v 0
ë1 / 2 0 û
Vector & 3Dimensional Geometry 367

1 2 1 3 1 4
6. Let B=A- A + A - A +¼¼
3 9 27
AB A2 1 3 1 4
=-- + A - A +¼¼
3 3 9 27
æ Aö
çI + ÷B = A
è 3ø
1
B = (3 I + A) -1 A
3
n
æ 1 1 ö 1 1
7. det M n = å çè (2 k + 1)! - (2 k + 2)! ÷ø = 1! - (2n + 2)!
k =0
® ®
8. | a + b| = 3
Þ Squaring both sides
® ® 1
Þ a× b =
2
® ® ® ® ®
c = a + 2 b-3 a ´ b
® ® ® ® 5
Þ a × c =2 & b × c =
2
® ® ® ® ® ®
p =|( a × a ) b - ( b × c ) a|
2
® 5®
p = 2 b- a
2
21
p= Þ [ p] = 2
2
® ® ® ® ® ® ®
9. r = ( a ´ b ) sin x + ( b ´ c ) cos y + 2 ( c ´ a )
® ® ®®®
r × a = [ b c a ]cos y
® ® ®®®
r × b = 2[ c a b ]
® ® ®®®
r × c = sin x [ a b c ]
Þ sin x + cos y + 2 = 0
Þ sin x = -1 and cos y = -1
p
x =- y =p
2
368 Solution of Advanced Problems in Mathematics for JEE

10. New equation of plane : 4 x + 7 y + 4 z + 81 + l(5 x + 3 y + 10 z - 25) = 0


( 4 + 5l) 4 + (7 + 3l) 7 + ( 4 + 10l) 4 = 0
Þ l = -1
Þ Equation of plane : x - 4 y + 6 z - 106 = 0
106
distance = = 212
53
® ® ®
18. w ´ m = n
® ® ® ® ®
n ×( w ´ m ) = n × n = 1

❑❑❑

You might also like